Download as pdf or txt
Download as pdf or txt
You are on page 1of 139

‫بسم هللا الرحمن الرحيم الحمد هلل رب العالمين كما ينبغي لجالل وجهه ولعظيم سلطانه حمدا يليق

بجالله وجماله وكماله حمدا يوافي نعمه ويكافئ أفضاله ونعمائه وآالئه علينا وعلى سائر‬
‫مخلوقاته والصالة والسالم علي سيدنا محمد وعلي ءاله عدد كمال هللا وكما يليق بكماله في كل لمحة ونفس عدد ما في علم هللا صالة دائمة باقية بدوام وبقاء ملك هللا وعلي ءاله وصحبه‬
‫ومن وااله عدد ما ذكره الذاكرون وعدد ما غفل عن ذكره الغافلون في االولين وفي االخرين وفي كل وقت وحين وفي المأل األعلى إلى يوم الدين صالة وسالما يليقان بمقامه وتكون أهال‬
‫أما بعد‬ ‫لجنابه وأداءا لحقه العظيم ينفعنا هللا بها في الدنيا ويوم العرض عليه ولقائه‬

Alrawaji Surgical MCQs; Q & A Notes FOR FREE NOT FOR SALE
UPPER GASTROINTESTINAL TRACT
SALIVARY GLANDS
1. An old age female patient, after abdominal surgery there in parotid enlargement what is the cause?
A. Parotitis(due to parotid duct obstruction) B. Parotid duct stone C. Post intubation trauma D. Hypocalcaemic muscle spasm
2. In which salivary gland, tumours most commonly occur?
A. Parotid gland B. Submandibular glands C. Submental glands D. Sublingual glands
3. A young, aged female previously healthy patient, not smoker, no medical or autoimmune diseases, she has painless left preauricular
deep subcutaneous mass started 4 years ago, not attached to skin but move with muscles of mastication contraction, with change of size
when mouth open or close, what is possible diagnosis?
A. Mucoepidermoid B. Adenoid cystic tumour C. Warthin tumour D. Pleomorphic adenoma E. Sebaceous cyst
4. An old age female with non-tender well defined smooth parotid swelling, diagnosis?
A. Pleomorphic adenoma(the most common benign, 30-50 y, superficial, slow) B. Lymph node C. Mucoepidermoid tumour
5. What is the character of Pleomorphic parotid adenoma?
A. Requires core biopsy before resection B. is adequately treated with enucleation D. Commonly results in facial palsy
C. Commonly undergoes malignant transformation E. Is the most common parotid neoplasm
6. A male patient he has long lasting swelling more than 2 years in front of the ear the swelling adherent to skin is painful with redness
what is diagnosis?
A. Infected sebaceous cyst B. Parotid abscess C. Face lipoma
7. An old age lady, after she underwent an abdominal surgery there in parotid enlargement what is the cause?
A. Parotitis B. Parotid duct stone C. Parotid gland tumour D. Mastication muscles tumours
8. What is the most common cause of facial nerve paralysis?
A. Parotid surgery B. Faciomaxillary injury C. Bells' palsy (70% viral then traumatic 10- 23%) D. Parotid Carcinoma
9. What is the most common complication after parotid surgery or parotidectomy?
A. Frey’s syndrome (late, gustatory, sweating, flushing) B. Facial nerve palsy (early, forehead, eye, lip closure) C. Skin flap anaesthesia
10. an old aged man came with drooling from the corner of the mouth after submandibular gland excision is due to which nerve injury?
A. Injury of the lingual nerve B. Injury of the ansa cervicalis C Injury of the hypoglossal nerve
D. Injury of the marginal mandibular nerve (branch of facial N. emerge from anterior border of lower parotid part)
11. What is the structure that is found at distance of about 15 cm from incisor teeth? 15- 22,5 -40;cm C6, T4 T11
A. Inferior constrictor muscle (cricopharyngeus, 1st. strict 1.4-1.5 cm at C6) B. Diaphragm
C. Aortic and bronchial constriction (aortic arch & Lt main bronchus at T4, 22.5 cm from incisor, 1.5-1.6 width)
D. Lower oesophageal sphincter (at T11, 40cm from incisors...1–1.5 cm in length and 1.5–1.8 cm in width. The least intraabd. length against reflux 1 cm)
12. Regarding the anatomy of the oesophagus, what is true?
A. The cervical oesophagus lies to the right of the midline B. The thoracic Oesophagus is anterior to the aortic arch
C. The left vagus nerve passes posterior to the oesophagus D. The cervical oesophagus is supplied by the inferior thyroid artery
E. The abdominal oesophagus is supplied by the right gastric artery.
13. Which of the following contributes to the arterial supply of the thoracic oesophagus?
A Right gastric artery B Bronchial artery (from thoracic aorta) C. Pulmonary artery D. Innominate artery
14. What is the minimal oesophageal diameter that can allow normal swallowing without any dysphagia or feeling alert?
A. 20-30 mm (normal diameter) B. 25- 35 mm C. 15-20 mm D. 12-13 mm (at least 15mm for symptomatic healing to occurs)
15. At the initiation of swallowing, the pressure at the lower oesophageal sphincter?
A. Remains unchanged B. Decreases and then increases C. Increases and then decreases. D. Increases and then returns baseline.
16. How much is the minimal length of the lower oesophageal sphincter that is required to prevent gastro oesophageal reflux?
A. 1cm (intraabd. sphincter length > 1cm in 95% of cases was healthy) B. 2cm C. 3cm D. 4cm
17. What can affect the lower oesophageal sphincter (LOS) tone? Varies from 10 to 45 mm Hg. in adults
A. It is increased by gastrin (also motilin, pancr polypeptide relaxed by Secretin, CCK, Glucagon) B. It is decreased by metoclopramide
C. It is increased by nicotine D. It increases by chocolate
18. A female patient presented in ER with history for many years of dysphagia, the dysphagia is more for liquids and associated with
history of coughing and sore throat, what’s diagnosis?
A. DOS (Diffuse Oesophageal spasm) B. Dysphagia lusoria (Rt. SCA) C. Dysphagia of cardia (motility disorder for both, mechanical for solid)
D. Oesophageal cancer (solid more)
19. An old aged lady patient who is kyphotic, diabetic, hypertensive, with hypercholesterolaemia, she complained of dysphagia, no history
of GIT diseases before on OGD; no masses, no reflux or oesophagitis but only some indentation of the oesophageal wall, what’s diagnosis?
A. Oesophageal cancer B. Dysphagia of the cardia C. Dysphagia Aortica (old, woman, short, atherosclerotic) D. Dysphagia lusoria
20. What is the treatment of Zenker’s diverticulum?
A. Pharyngo-diverticulectomy B. Diverticulo-myotomy C. Diverticulectomy + Cricopharyngomyotomy (if symptomatic, >2cm)
21. A scenario of patient had a regurgitation of undigested food that eaten for 4 days with very bad smell, with mid chest pain, and
cough, what is the diagnosis?
A. Traction oesophageal diverticulum B. Zenker’s diverticulum (dysphagia 98%, regurg of undigested food sticking) C. Achalasia of cardia
D. Epiphrenic diverticulum
22. A 76 y.o man has chronic dysphagia, that become more worse, with feeling of retrosternal chest pain ,chockin and post pranial cough
with sensation of food sticking, and regurgitation, what is the best before proceeding to surgery?
A. A 24 hrs oesophageal acid monitoring B. Rigid/flexible oesophagoscopy C. Oesophageal manometry D.OGD with biopsy
23. A patient known with symptoms of long-standing GORD, progressive dysphagia over 5 years, manometry done> no achalasia, Barium
swallow > no HH, dilated oesophagus & lower end narrowing with stricture ring and intact mucosal pattern what’s diagnosis?
A. Reflux oesophagitis B. Oesophageal adenocarcinoma C. Hiatus hernia D. Schatzki ring (<13mm symp, >25mm asymptomatic)
24. A case diagnosed as Schatzki ring what the best treatment option?
A. Endoscopic dilatation (Schatzki ring. 4 quadrant biopsies obliterate it) B. Heller’s myotomy C. Laparoscopic myotomy D. Nifedipine
25. Mediastinal granulomas may be associated with which of the following?
A. Epiphrenic oesophageal diverticulum B. Zenker’s diverticulum C. Oesophageal traction diverticulum D. Achalasia
Mediastinal granuloma is more common (comprises 5 to 10 % of mediastinal masses in most surgical series and more benign than fibrosing mediastinitis.
Oesophageal traction diverticula result from granulomatous inflammation of mediastinal LNs. TB, Histoplasmosis, and sarcoidosis are known aetiologies.
26. What is true for the Oesophageal diverticula?
A. Traction diverticula are false diverticula B. Pulsion diverticula are common in the mid oesophagus
C. Pulsion diverticula are usually associated with enlarged lymph nodes D. Myotomy is always indicated for pulsion diverticula
27. A patient complaining of recurrent severe chest pain barium swallow shows Corkscrew oesophagus, what is the diagnosis?
A. Diffuse oesophageal spasm B. Achalasia C. Oesophageal cancer D. Plummer-Vinson syndrome (dysphagia, anaemia, oesoph web)
28. A patient history of achalasia, what complaints to be alert to?
A. Chest infection (aspiration pneumonia) B. Dysphagia (if rapidly progressive) C. Weight loss (SCC, ACA if rapid, significant, unusual)
29. The diagnosis of oesophageal achalasia is confirmed by?
A. Bird’s peak appearance on barium swallow B. Sub atmospheric intraluminal oesophageal pressure on manometry
C. Endoscopic evidence of distal oesophagitis D. Failure of lower Oesophageal sphincter relaxation on manometry
30. In achalasia there is absence of which of the following?
A. Complete relaxation of Lower oesophagus B. Incomplete relaxation of low oesophagus C. Oesophageal peristalsis
31. A case of male patient with signs of Achalasia, small tapering, & proximal oesophageal dilatation, how to manage? Lap Heller’s myotomy
32. A patient presented with dysphagia to liquids more than solid, which is the most appropriate initial investigation?
A. Endoscopy (can cause perforation) B. Barium swallow (initial) C. Manometry (best definitive) D. Biopsy
33. What is the initial investigation of Achalasia? Barium swallow. Definitive diagnostic investigation? Oesophageal manometry
34. What is the Initial treatment of Achalasia? Initial Balloon Dilatation. Best treatment? Heller’s Cardiomyotomy
35. What is the most preferable treatment of Achalasia? Lap cardiomyotomy with dor partial anterior fundoplication)
36. Scenario of dysphagia, LOS Manometric study showed, distal contractile integral (DCI) 400, diagnosis?
A. Diffuse oesophageal spasm. B. Scleroderma. C. Achalasia cardia (DCI > 100 mmHg, > 200 nutcracker
N.B (Normal 15-26 mmHg, DCI 450-8000).. Abnormal (major), if < 450 or >8000 (hypercontractile) and (minor) if <100 =ineffective, if 100-450 = weak
37. Achalasia with Barium study show dilatation, fluid level, what’s your diagnosis? Mega oesophagus = > 10cm diameter, if > 6 severe achalasia
38. A female patient with dysphagia to fluids, barium swallow showed smooth stricture of the distal end of the oesophagus, was a typical
case of achalasia, what is the best treatment?
A. Balloon dilatation (initial if failed >> surgery) B. Oesophagocardiomyotomy (the gold standard; if dilatation failed) C. Nifedipine
39. What is the best long-term management of Achalasia?
A. Nifedipine B. Nitrates C. Heller's myotomy D. Fundoplication
40. The best diagnostic investigation for atypical GORD?
A. 24 hours pH monitoring (atypical as ENT, pulmonary; cough asthma, noncardiac chest pain) B. Manometry C. CXR D. Endoscopy
41. Hiatal hernia with GORD, what’s mechanism of Hernia?
A. Short oesophagus B. LOS incompetence
42. Hiatal hernia with GORD. What’s treatment?
A. Partial fundoplication B. Nissen fundoplication C. Heller seromyotomy repair
43. An obese middle-aged lady with symptoms Of GORD, gastroesophageal junction above level of diaphragm what is the hernia cause?
A. Wide crura B. Decrease gastroesophageal pressure C. Increase intra-abdominal pressure D. Negative Intrathoracic pressure
44. A case of GORD + barium swallow showed GO junction above diaphragm, but the stomach is in place what’s treatment?
A. Heller Cardio myotomy B. Partial fundoplication C. Nissen fundoplication (complete if no contraindications)
45. An old female with dysphagia relieved by a large loud belch, a case of paraoesophageal hernia what is the best investigation?
A. CT abdomen and chest with contrast B. Barium swallow (or upper GI series; the best and definitive) C. Upper endoscopy
46. A 50 y.o obese man with long history of gastric upset, heart burn, dysphagia and early satiety, an upper endoscopy and barium study
done that showed that he has hiatus hernia, his discomfort is relieved well after loud and full belch what type of his hernia?
A. Sliding HH B. Achalasia with GORD C. Paraoesophageal hernia type II (true hernia) D. Rolling hernia type IV
47. A large and symptomatic Paraoesophageal hernia, what is the optimal management? Surgical repair and fundoplication if type 3/4
48. A case with dysphagia, long standing GORD, and chest pain with heavy sensation, all relieved after eructation and belching, cause?
A. Barrett’s oesophagus B. Gastric volvulus C. Achalasia D. Sliding hiatal hernia
49. A patient visited ER with C/O of severe dysphagia and chest pain only relieved with belching, the patient is stable diagnosis?
A. Gastric volvulus (if >180 degree can be complicated with gastric strangulation) B. Barret oesophagus C. Stricture D. Achalasia
50. A patient has paraoesophageal hernia what is the most concern and complication you will fear from?
A. Irreducibility B. Obstruction C. Incarceration D. Perforation (volvulus -> ischaemia -> gangrene -> preforation)
51. What is correct regarding paraoesophageal hernia?
A. is a type I hiatal hernia B. is most common in men <50 years of age
C. May present with chronic anaemia D. is often associated with reflux E. Requires surgery only if symptomatic
52. A female patient with dysphagia and chest pain of 6 months, stomach is up and parallel to the oesophagus, LOS is in place what the
diagnosis is? >> Rolling hiatus hernia; para oesophageal type II complicated with irreducible hernia will not be volvulus except if very big
part of fundus goes up
53. Same scenario of gastric volvulus, what is the optimal management?
A. Anatomical repair (plus antireflux surgery if big defect) B. Fundoplication C. Heller’s cardiomyotomy
54. Hiatal hernia with GOJ above diaphragm and short oesophagus what’s management? Collis gastroplasty (Vertical banded gastroplasty; VBG)
55. During gastric mobilisation for oesophagectomy with gastric tube reconstruction for intrathoracic anastomosis depends on which artery?
A. Short gastric B. Left gastric C. Right gastroepiploic D. Left gastroepiploic
56. A patient with hiatus hernia LOS 2cm above Gastro-oesophageal junction with part of fundus in the hiatus; parallel to the oesophagus,
what type of operation to be done for him?
A. Nissen fundoplication B. Toupet fundoplication C. Watson fundoplication D. Dor fundoplication
57. What does type IV hiatal hernia means?
A. is a sliding hiatus hernia B. is a traumatic diaphragmatic hernia C. is a hernia that contains the stomach
D. It is a hernia that contains parts of the intestine and colon
58. Which of the following is correct regarding Hiatus Hernia?
A. Reflux not seen in paraoesophageal type (mostly in type 1) B. Dysphagia commonest symptom in sliding type
C. Paraoesophageal type is treated medically D. The gastroesophageal junction is intraabdominal in sliding type
59. In caustic oesophageal injury what is correct?
A. Upper endoscopy is contraindicated in the acute phase B. Early induced emesis is helpful in minimising the period of mucosal contact.
C. Alkalis cause coagulative tissue necrosis D. Acids cause severe gastric rather than oesophageal injury. Alkali >> osesophagus. acid stomach
60. A patient with GORD symptoms mainly during leaning forward in prayer activity, what’s the best investigation?
A. Endoscopy B. Manometry C. CT D. Barium meal (hiatus hernia type 1)
61. A patient with GORD symptoms mainly during leaning forward in prayer activity, barium meal was done and showed GOJ above
diaphragm what’s the cause?
A. Short Oesophagus B. Wide crura C. Congenital D. Narrow crura
62. A patient with GORD symptoms mainly during leaning forward in prayer activity, Barium swallow was done and showed GOJ above
diaphragm what’s the best management?
A. Lap fundoplication (small type1 HH <4cm) B. Anatomical repair (if large >5cm & symptomatic) C. Antacids D. Wt. reduction
63. A child ingested a caustic material, with image of barium swallow showing distal segment oesophageal stricture, what’s management?
A. Oesophagectomy B. Endoscopic oesophageal dilatation (after 3-6 weeks after initial injury) C. Stent
64. A Child accidently drank a caustic lye, has redness of lips, mouth, and pharynx, what is the proper management option?
A. Enteral feeding B. Early Oesophagoscopy (3- 48 hr better in first. 24 hr) C. Feeding jejunostomy D. Antibiotics
65. Which of the following is contraindicated in managing corrosive oesophagitis?
A. Gastric lavage (also supine & emesis can add injury) B. Oesophagogastroduodenoscopy C. Corticosteroids D. Tracheostomy
66. A 4-year-old boy presents to the ED after an accidental ingestion of lye. The child is exhibiting stridor and hoarseness, the child is
emergently intubated and resuscitated with IV fluids. An upright chest radiograph shows no evidence of perforation, the patient is
admitted to the intensive care unit, what is the next step in the management? Airway, drugs, OGD if no perforation>> medical ttt, oral liquid
A. Oesophagoscopy B. Oesophagography C. Chest computed tomography scan D. Close observation.
67. An adult girl with a history of unintentional corrosive ingestion that followed by severe oesophageal strictures, now she is following by
regular dilatation for 3 months with feeding jejunostomy, what is the next to do for her? (1st step ABCDE then painkillers, antibiotics, cortisone)
A. Continue endoscopic dilatation (at least 6 m) B. Oesophageal resection C. Stent placement D. Oesophageal bypass
68. A 38-year-old alcoholic male, he has presented to hospital following a 72-hrs., drinking binge. He has vomited excessively. On exam.
He is pale and clammy, tachycardic, hypotensive with rigid abdomen, on chest and upper abdominal CT, pneumomediastinum is found.
He has surgical emphysema tracking into his neck on the Lt., side with lower chest pain radiating to Lt., shoulder diagnosis & ttt?
A. Boerhaave’s syndrome for thoracotomy and oesophageal repair B. Conserve and follow up (Mallory Weiss synd.)
69. Alcoholic old man suddenly vomited and developed chest pain and dysphagia, what diagnosis? Oesophageal perforation
70. A patient after vigorous vomiting has chest pain, neck emphysema and on CXR there is an opacity or fluid in pleural space?
A. Spontaneous Oesophageal rupture (Boerhaave’s syndrome) B. Lung bullae rupture
71. What is the best description of Spontaneous perforation of the oesophagus or Boerhaave’s syndrome?
A. Usually seen in elderly females B. The most common cause of oesophageal perforation C. Usually located in the mid oesophagus
D. Typically presents with sudden onset (of chest pain, dyspnoea, tachycardia, hypotension)
72. A patient while lifting a heavy object developed sudden severe chest pain on CXR pneumomediastinum diagnosis? Boerhaave’s syndrome
73. A patient came to ER after RTA with SOB, on examination the tracheal shifted to the left side, in chest X-ray, the lungs are expanded
and winded mediastinum, what is the diagnosis?
A. Tension pneumothorax B. Cardiac tamponade C. Plural effusion D. Oesophageal Rupture (with tracheal left shifting)
74. What is the most common symptom of oesophageal perforation?
A. Sudden severe chest pain B. Dyspnoea C. Fever D. Emphysema
75. Boerhaave’s syndrome? Scenario, recurrent, attacks of vomiting, chest pain, perforated oesophagus, management? Repair and drainage
76. After difficult endoscopy, there is chest pain, neck pain, crepitus, and erythema?
A. Perforation of the cervical Oesophagus B. Ludwig angina C. Tracheobronchial injury
77. A patient underwent oesophageal dilation for achalasia came after (6 or 12 hours) with chest pain, subcutaneous crepitus and labs
showed WBC 21000, what is your management? Iatrogenic perforation
A. Total Parenteral Nutrition (TPN) B. Stent ( SEMS; if stable, no pain, dyspnoea, leucocytosis or fever) C. Drainage and surgery (if large)
D. Nasogastric Tube (NGT)
78. A patient with two trials of unsuccessful Oesophagoscopy presented in ER with chest pain, the surgeon was worried about perforation,
what is the best investigation for confirmation?
A. Barium swallow B. Oesophagoscopy C. CT D. Water soluble contrast swallow (Diluted Gastrografin)
79. A 60-year-old alcoholic male presents with severe chest pain after repeated vomiting. A chest x-ray shows a small left pleural effusion.
What is the optimal next step in management?
A. Obtain cardiac enzymes and admit to coronary care unit B. Insert nasogastric tube, administer intravenous fluids, and observe
C. Insert nasogastric and left-side chest tube and antibiotics D. Administer Gastrografin swallow test E. Perform upper endoscopy
80. A 45 -year-old-male complains of severe chest pain after a diagnostic upper endoscopy. He has crepitus on palpation of his neck. What
is the next step in management?
A. Repeated upper endoscopy B. Immediate endotracheal intubation C. Gastrografin swallow
D. Administration of aspirin and sublingual nitrite E. Admission to ICU, administration of intravenous narcotics, and observation
81. A case with history of GORD, and chronic epigastric pain, kept on PPI, but not improved on it, what is the management?
A. Barium B. Endoscopy; OGD to check if any Barrette’s change and for biopsy C. Ambulary 24 h monitoring
82. A 23 yrs. old female presented with symptoms of GORD with worsening dysphagia, next step?
A. Endoscopy (to R/O HGD and cancer to diagnose and take biopsies) B. Manometry C. pH monitoring
83. What is the initial test in cases of suspected gastroesophageal reflux; GORD should be?
A. Barium swallow B. Upper endoscopy (96% sensitivity and a 95% specificity) C. Gastric pH monitoring (Confirming & diagnosing GORD)
D. Oesophageal manometry
84. What is the most essential step in management of oesophageal reflux with Barrett’s change?
A. Nissen fundoplication B. Close endoscopic surveillance (to detect if early dysplasia-> cancer) C. Oesophageal resection
D. Medical management of reflux disease
85. Regarding Barrett's oesophagus what is true?
A. is commonly congenital in origin B. is associated with epidermoid carcinoma of the oesophagus
C. is reversed by a successful antireflux surgery D. It is an indication for life-long endoscopic surveillance
86. Barrett’s oesophagus with low grade dysplasia what is management?
A. Surgical resection B. Antacid C. Surveillance every 6-12m D. PPI and rescope every 6-12m
87. A patient starts to have dysphagia to liquids, what is the investigation of choice?
A. Upper Gl endoscopy B. Barium swallow (achalasia >>initial >> Barium. Manometry definitive ) C. CT scan D. X-ray
88. What is the most common used drugs for peptic disease?
A. Long term PPI B. H pylori eradication C. H2 blockers D. Antacids and prokinetics
89. Woman with heartburn and regurgitation. On endoscopy she was found to have distal oesophagitis, what would be the management?
(She’s already on omeprazole 20 mg)
A. Add antacids B. Increase, modify, double the dose and drug (for 8 weeks if failed >> refractory) C. Histamine H2 blockers
90. A male patient, a known case of GORD on PPI, symptoms only mild improvement, after increasing doses for 8 weeks with life style
modifications he still symptomatic. OGD was done and showed oesophagitis, but no Barrett’s, what’s the best important next step?
A. Repeat Endoscopy in 6 months B. Nissen fundoplication C. Oesophageal manometry D. Ambulatory pH monitoring
91. A patient with GORD not responding to medical treatment, which surgery will do?
A. Nissen total fundoplication (if no motility disorders or HGD/Cancer) B. Partial fundoplication C. Dor’s fundoplication
D. Toupet fundoplication E- Belsy fundoplication
92. For GORD cases what is the best investigation, if you plan to do surgery?
A. pH monitoring B. Endoscopy C. Manometry (to R/O motility disorder if +ve for partial fundoplication )
93. GORD case, what is the best investigation 24 hrs., before surgery?
A. pH. Monitoring B. Endoscopy C. Manometry
94. A patient with heart burn, regurgitation, and GORD what’s most sensitive, definitive, gold standard and best to diagnose GORD?
A. Endoscopy B. Ambulatory 24 hrs. pH monitoring C. Manometry
95. What is the adverse effect of long-time use of omeprazole?
A. Diarrhoea is the most common complication B. 10 times risk of osteoporosis C. Tell the patient nothing.
96. A 68-year-old female with diabetes mellitus, coronary artery disease, fibromyalgia, and dyspepsia presents for follow-up. She has been
taking omeprazole (Prilosec) for 10 years. It was started during a hospitalisation, and her symptoms have returned with previous trials of
discontinuation. Which one of the following adverse events is this patient at risk for because of her omeprazole use?
A. Hypermagnesemia B. Urinary tract infections C. Nephrolithiasis D. Hip fractures (due to Osteoporosis)
97. A patient is complaining of dyspepsia for 10 years he developed upper abdominal pain since last 2 months abdominal U.S is free,
patient increased his dose of omeprazole by his own to overcome pain, due to pain, he afraid to eat and lost weight he asked about what is
the most worrisome complication?
A. Loss of weight B. Osteoporosis (high dose of PPI) C. High dose of omeprazole
98. Regarding the Leiomyoma of the oesophagus, what is true?
A. Commonly presents with dysphagia B. is more common in females C. is usually multiple
D is usually diagnosed with endoscopic biopsy E. It is usually located in the lower one-third of the oesophagus Leio =Lower
99. A scenario of patient suffers from small smooth swelling at mid-1/3 of the oesophagus, mostly Leiomyoma what is the best management?
A. Endoscopy with biopsy B. Oesophagectomy C. Enucleation (after oesophagomyotomy) D. Ivor-Lewis oesophageal resection
E. Trans hiatal oesophageal resection F. Endoscopic resection G. Trans hiatal oesophageal resection
100. What is the most prognostic factor for Leiomyoma?
A. Tumour size (larger more malignancy risk, if gastric > 6cm, and for removal) B. Mitotic count or feature C. Associated GORD
101. What is the most common risk factor of oesophageal squamous cell carcinoma; SCC? Achalasia, Smoking, and Alcohol remember ASA
102. What is the most prognostic factor for Oesophageal cancer?
A. Lymph Nodes (in general) B. Depth of invasion (early stage) C. Vascular invasion
103. Which of the following is associated with earlier weight loss than others?
A. Colon cancer B. Prostatic cancer C. Oesophageal cancer D. Hepatoma
104. What is the most common oesophageal cancer regarding the histological type?
A. Adenocarcinoma (USA) B. Squamous cell carcinoma (worldwide, 95% SCC) C. Leiomyosarcoma D. Lymphoma
105. What is the treatment of choice for Barrett's oesophagus with severe high grade dysplasia?
A. Follow-up endoscopy and biopsy B. Oesophagectomy C. Nissen fundoplication D. Proton pump inhibitors
106. A healthy patient presenting with dysphagia with no family history of oesophageal cancer, what’s the strongest risk factor of
oesophageal cancer?
A. Oesophageal stricture B. Barrett's oesophagus (Adenocarcinoma USA) C. GORD (USA) D. Heavy smoking (SCC, worldwide)
107. An increased incidence of adenocarcinoma of the oesophagus is associated with which of the following?
A. Achalasia B. Lye ingestion C. Barrett's oesophagus D. Plummer-Vinson syndrome
108. Symptoms and signs of oesophageal mass or cancer what is the best investigation to diagnose it?
A. EUS B. OGD (initial) C. CT(for staging) D. Upper GI series
109. A patient with wt. loss and dysphagia diagnosed by OGD biopsy as a case of adenocarcinoma, how to know the T stage of cancer?
A. Abdominal US B. Chest, Abdomen & Pelvic CT C. Endoscopic US D. Chest MRI E. OGD guided biopsy
110. An old age patient with symptoms GORD, on diet and lost 6 kg from his body weight, developed worsening of symptoms, and develop
chest infection and postprandial cough, which symptom should take care of it or what is the most alarming symptom?
A. Dysphagia (Barrett’s >> high grade dysplasia >> cancer) B. Recurrent chest infection C. Weight loss D. Postprandial cough
111. What is the most common clinical presentation of oesophageal cancer?
A. Dysphagia (most common, if lumen < 13 mm) B. Weight loss (>50%) C. Odynophagia (20%) D. Anaemia & haematemesis
112. What is the most accurate test to determine the need for neoadjuvant therapy in oesophageal carcinoma?
A. Endoluminal ultrasound B. Chest computed tomography scan C. Bronchoscopy
D. Barium swallow E. Oesophagogastroduodenoscopy
113. Fine-needle aspiration of bilateral upper cervical lymphadenopathy shows squamous cell carcinoma. No primary lesion is found on
clinical examination. What is the most likely source?
A. Lungs B. Oesophagus C. Tongue D. Tonsils E. Nasopharynx
114. Which of the following is correct regarding cervical anastomosis after Oesophagectomy for cancer?
A. Has a lower leak rate than thoracic anastomosis B. Leak is likely to heal spontaneously
C. Has a higher long-term mortality rate than thoracic anastomosis D. Has a lower operative mortality rate than thoracic anastomosis
115. An old patient with a mid oesophageal mass, Coeliac LNs. the patient is well for which surgical procedure?
A. Chemo-radiotherapy B. Stent C. Oesophagectomy D. Chemotherapy
116. A patient mid oesophageal cancer, in muscularis propria length 4 cm, positive coeliac LN, normal functional work up for surgery,
what’s the best management?
A. Trans hiatal oesophagectomy B. Neoadjuvant chemotherapy then surgery C. Palliative stent(if inoperable & dysphagia)
D. Neoadjuvant chemotherapy
117. A case of 70 years old male, with mid oesophageal cancer with no metastasis, what is the management?
A. Stent (only in case of dysphagia) B. Resection (after neoadjuvant CRTH) the primary treatment for upper and mid is surgery after CRT
118. A 70 y.o male with dysphagia and mid oesophagus cancer with metastasis inv no dysphagia, healthy and ready for resection what is
the optimal management? In current TNM staging ; coeliac LNs considard as locoregional disease not distant mets. Although it is of poor prognosis
A. Stent (if dysphagia) B. Resection (If local not coeliac or distant) C. Neoadjuvant chemoradiotherapy then resection
119. A barium swallow image showed lower oesophageal carcinoma, for subtotal oesophagectomy, how much the optimal safety margin?
A. 12cm proximal and 5cm distal B. 5cm upper and 2cm down C. 5cm proximal and 3cm distal
120. A male patient with advanced oesophageal carcinoma. T3N2M0, undergoing stent, 2 hrs. after procedure he developed, Lt chest pain,
absent air entry Lt side, subcutaneous air, what is most probable diagnosis?
A. Pneumothorax B. Iatrogenic oesophageal perforation C. Surgical emphysema
121. For an oesophageal cancer while introducing a stent, perforation occurred, patient developed mediastinitis, air under diaphragm?
A. Tracheostomy / Chest tube B. Conservative C. Primary repair D. Oesophagectomy with end oesophagostomy
122. An old patient with dysphagia, diagnosed as metastatic oesophageal cancer, LN at coeliac artery what’s the best palliation method?
A. Radiotherapy B. Stent; SEMS C. Oesophagectomy D. LASER ablation
123. Which of the following procedures has a high chance of developing pneumonia?
A. Anterior Oesophagectomy B. Lower segment CS C. Reduction of femoral fracture
124. Advanced stage oesophageal cancer, stent inserted, then patient complains of lower chest pain what to do?
A. Conservative (if stent migration) B. 1ry Repair (if stable, contained, no leak) C. Oesophagectomy (as it is query perforation)
125. What is the most significant study to check for oesophageal tumour invasion to the bronchus?
A. Bronchoscopy B. MRI C. Oesophagoscopy D. Chest CT
126. In the previous surgery (oesophagectomy) what is the late postoperative complications?
A. Reactionary haemorrhage B. Short gut syndrome C. Delirium D. Nausea & vomiting (late due to stricture and delayed emptying)
127. What is the absolute contraindication for anti-reflux measures? Oesophageal CA, Unfit for GA & un controllable coagulopathy
A. Motility disorder (do partial fundoplication) B. Barrett’s oesophagus with low grade dysplasia C. Oesophageal cancer D. H Hernia
128. A case with longstanding gastritis & GORD kept on H2 blockers but was not improved so, shifted to omeperazole on which he is very
comfortable for 10 years, what is the most serious adverse effects of long term use of omeperazole other than osteoporosis and fractures?
A. Diarrhoea B. Gastric mucosal atrophy C. Gastric cancer (by reducing gastric acid >> gastrin > cancer risk) D. Malabsorption
129. A middle aged patient with upper GIT bleeding all investigation and coagulation are normal what is most likely diagnosis?
A. Dieulafoy lesion B. Peptic ulcer C. Oesophageal varices D. Angiodysplasia E. Mallory-Weiss tear
130. A patient with upper GI bleeding, there was no evidence of liver disease, peptic ulcer, what is most likely cause? Dieulafoy lesion
131. Where Mallory-Weiss tear is located?
A. In the distal oesophagus B. Anteriorly across the gastroesophageal junction C. Posteriorly across the GOJ
D. On the lesser curve of the cardia E. On the greater curve of the cardia
132. A patient with upper GI bleeding, the bleeding stopped spontaneously, what was the cause of bleeding?
A. Dieulafoy lesion B. Jejunal diverticulosis C. Peptic ulcer D. Mallory-Weiss tear
133. What is the criterion of Gastrointestinal bleeding in Mallory-Weiss syndrome?
A. It is occult in the vast majority of cases B. Can be controlled with balloon tamponade
C. Stops with nonoperative management in most cases D. Caused by mucosal tear in the lower third of the oesophagus
134. A case presented with bleeding per rectum, barium is –ve, colonoscopy showed an erythematous area with protruding dilated
submucosal small vessel is seen in that area of transverse colon what’s most likely diagnosis?
A. Colon cancer B. Dieulafoy lesion C. Aortoenteric fistula
135. An alcoholic male patient, presented with oesophageal mucosal tear post vomiting, no chest pain or haematemesis, management?
A. Thoracotomy and oesophageal repair B. Conservative and follow up (Mallory Weiss synd.)
136. A male patient complains of episode of haematemesis, normal past medical history, labs show mild anaemia, all labs are normal
except elevated urea blood level, what is the diagnosis?
A. Mallory Weiss syndrome (forcible clear vomiting then bleeding) B. Erosive gastritis C. Peptic Ulcer Disease (starting bloody vomitus)
137. A patient with hypertrophied rugal folds of the stomach, some medical treatment was given and failed, asking about the
management? Menetriers, mainstay of treatment are anti-cholinergic drugs, anti-acid suppression and antibiotics against H. Pylori and high protein diet if no
gastrectomy. Menetriers disease (hypoproteinaemic hypertrophic gastropathy and giant hypertrophic gastritis) with giant mucosal folds in the proximal part
of the stomach, diminished acid secretion, and a protein- losing state.
138. After upper endoscopy for long standing gastritis patient, you found a picture of marked hypertrophied and enlarged gastric folds,
superficial punctate erosions and erythema, with full-thickness mucosal biopsy found significant reduction in parietal cells, dilation of
glands, increased eosinophils and plasma cells, oedema, and smooth muscle hyperplasia with foveolar hyperplasia of surface mucous cells
how to manage this case initially?
A. Medical treatment plus H pylori eradication B. Surgical treatment (if interactable) C. Chemotherapeutic D. Radiotherapy
139. A 45 year old patient with bilateral lower limb oedema and epigastric pain for months, on OGD, found foveolar hyperplasia with
erosion, what is the best mode of management? Menetriers
A. Laser ablation B. Medical treatment (PPI, HP eradication and CMV ttt) C. Total gastrectomy (if failed medical treatment)
D. Angioembolisation
140. You have a patient like in the previous scenario of Ménétriers disease who was managed initially with medical therapy but failed with
persistent abdominal pain, weight loss and complicated by gastric outlet obstruction how to manage him?
A. Continue medical and supportive treatment B. Chemoradiotherapy C. Partial gastrectomy D. Total gastrectomy
141. What type of chronic gastritis or gastropathy is due to H pylori? Type B
142. What is correct regarding Helicobacter pylori infection?
A. Common with gastritis B. Likely associated with colitis C. Gram –ve flagellum
143. Which of the following is true regarding the chonically untreated Helicobacter pylori infection?
A. Most common in gastric ulcer than duodenal ulcer B. Cause gastric malignancy
C. Positive in 70-80% in oesophagus and duodenum in normal people
144. Alcoholic patients are presented with severe epistatic pain (all labs, imaging & vitals normal) what is the diagnosis?
A. Acute pancreatitis B. Perforated DU C. Acid peptic disease D. Acute Calculous cholecystitis
145. What is true regarding Helicobacter pylori?
A. Colonisation is highest in childhood B. Eradication does not influence ulcer recurrence
C. It is isolated in up to 90% of duodenal ulcer cases D. Metronidazole achieves eradication as a single agent
146. A 26-year-old male came to ED with epigastric pain relieved by eating and worsening by fasting, which of the following next step in
first investigation?
A. Abdomen US B. Upper Endoscopy(best for diagnosis & biopsies) C. Urea breath test D. Blood antigen for H. Pylori
147. PPI is acting on which gastric cell type of the following?
A. Chief cells B. Antral cells C. Parietal cells D. Enteroendocrine cells
148. Gastric intrinsic factor is secreted from which of the following?
A. Parietal cell B. Chief cells C. Antral G cells D. D cells
149. A patient came to ER with upper GI bleeding, the bleeding stopped spontaneously, BP is 120/80 what is the cause of bleeding?
A. Dieulafoy lesion B. Peptic ulcer disease (if not eroding a vessel) C. Oesophageal varices D. Mallory wiess
150. A 70-year-old male on mechanical ventilation (MV) admitted to the ICU due to intracranial haemorrhage, 7 days later he developed
ground coffee vomitus, what is the diagnosis?
A. Stress gastritis B. Helicobacter pylori gastritis C. Dyspepsia
151. 24 hours after admission to ICU, a postoperative patient has bright red blood through the nasogastric tube, all following shown
efficacy in preventing stress gastritis except?
A. Sucralfate B. Proton pump inhibitors C. Enteral diet D. Histamine-2 (H2) receptor antagonist
152. A patient after endoscopy; gastric or duodenal superficial ulcer due to H-pylori, what’s the priority of treatment?
A. H. Pylori eradication B. Reduce acid reflux C. Endoscopic biopsy of ulcer D. Antacids
153. What is the most common complication of peptic ulcer disease?
A. Bleeding B. Perforation C. Malignancy
154. A patient with bleeding duodenal ulcer, injection of adrenaline by endoscope after 12 hours if rebleeding what to do?
A. Surgery B. Adrenaline (for 2 trials then Surgery/TAE) C. Cautery
155. A patient with stress gastritis, endoscopy failed to stop bleeding due to profuse haemorrhage from all over the gastric mucosa, patient
was shocked, what is the optimal management?
A. Angio embolisation (try first if failed do surgery) B. Surgery (if severe ongoing bleeding, despite of angio) C. Injection sclerotherapy
D. Splenectomy
156. A middle aged patient with history of duodenal ulcer he tried intermittently H. Pylori eradication, today came with epigastric pain
and haematemesis, where upper endoscopy and injection done twice but still bleeding, what is the best next to do for him?
A. Omental patch with oversewing B. TAE for gastroduodenal a. C. Injection sclerotherapy D. Third time endoscopy
157. The gastroduodenal artery is a branch of which of the following?
A. Coeliac axis B. Right gastric artery C. Common hepatic artery D. Right hepatic artery E. Left hepatic artery
158. The superior pancreaticoduodenal artery is a branch of which of the following?
A. The coeliac axis B. The superior mesenteric artery C. The hepatic artery D. The right gastric artery
E. The gastroduodenal artery
159. The blood supply of the thoracic stomach used for oesophageal replacement depends on which vessel?
A. The left gastric artery B. The short gastric vessels C. The right gastroepiploic artery D. The left gastroepiploic artery
160. What is the lateral boundary of a left Paraduodenal hernia?
A. The splenic artery B. The left renal vein C. The superior mesenteric vein D. The inferior mesenteric artery
E. The inferior mesenteric vein
161. A female patient with duodenal ulcer, shocked, received blood, what will you do after resuscitation?
A. Endoscopy and cauterise bleeder B. Surgery and sewing of duodenum C. Laparoscopic repair D. Coeliac angiography
162. A 29 y.o patient presented to ER after abdominal blunt trauma, CT scan showed duodenal intramural haematoma, what is the
optimal management?
A. Laparotomy with evacuation of the haematoma B. Dissection of duodenum C. DL D. Observation and Conservative
163. What is the most frequent complication of giant gastric ulcer? > 2cm
A. Gastric outlet obstruction B. Perforation C. Upper gastrointestinal bleeding D. Gastro enteric fistula
164. A patient with sudden epigastric pain became generalised abdominal pain & tenderness X-ray; air under diaphragm, what’s diagnosis?
A. Gastric or duodenal ulcer perforation B. Appendicular rupture C. Ileal perforation
165. A male patient in his 20s, present with severe epigastric pain for 6 hours, which started suddenly. On examination there is diffuse
tenderness, sluggish bowel sound. Labs: high WBCs, Amylase 300, which of the following best initial diagnostic steps?
A. Erect chest X-ray ( perforated PUD or bowel) B. Abdominal X-ray C. Abdominal CT D. Abdominal US
166. A female patient on treatment for arthritis complaining of epigastric pain and Rt. iliac pain with signs of peritonitis, what is most
likely diagnosis? Perforated peptic ulcer (NSAIDs induced ulcer)
167. The best investigation to differentiate between perforated peptic ulcer and acute pancreatitis?
A. CT with IV contrast (CECT) B. Barium study C. MRI D. Abd US E. Abdominal x -ray erect (initially air
under diaphragm in perforated DU)
168. Which of the following inhibits gastric bicarbonate secretion?
A. Prostaglandins(stimulate) B. Vagal stimulation C. Aspirin (prostaglandin inhib. NSAIDs, atropine acetazolamide) D. Gastrin
169. A male patient KCO peptic ulcer, presented with epigastric pain for 5 days, X-ray air under diaphragm he is stable, no peritonitis
and no fever what is the management plan? It is non-complicated early peptic ulcer perforation
A. PPI and discharge him B. Admission and observation (as patient is stable) C. Exploratory laparotomy (if unstable)
170. A patient who is on NSAIDs for joint pain, presented with sudden severe continuous abdominal pain. He denied any history of
vomiting, constipation, diarrhoea, and or haematemesis. The pain is in the epigastric area with a feeling of nausea. On examination. There
is tenderness and rigidity, what is the diagnosis? NSAID + Epigastric pain + Abdominal rigidity = (Perforated PU)
A. Oesophagitis B. Acute gastritis C. Perforated peptic ulcer D. Boerhaave’s syndrome
171. An ulcer in the lesser curvature, medical treatment failed, need surgery, what is the optimal type of surgery?
A. Billroth I B. Billroth I & gastroplasty C. Antrectomy + Truncal vagotomy
172. What’s the optimal management of a prepyloric ulcer, 3cm diameter, that is indurated and firm?
A. Billroth II B. Truncal vagotomy with antrectomy (with Billroth I; gastroduodenal reconstruction to control ulceration)
173. A patient with a history of gastric ulcer came to ER with bleeding duodenal ulcer management?
A. Truncal vagotomy B. Truncal vagotomy + Pyloroplasty C. Truncal vagotomy + Distal gastrectomy
D. High selective vagotomy (without drainage in refractory or complicated PUD)
174. A scenario of patient with OGD that showed an ulcer in the greater curvature what is the management?
A. Biopsy through Upper endoscopy B. Gastrectomy
175. A picture of gastric ulcer, biopsy was taken showing the cancer is limited to the gastric mucosa, what is the proper management?
A. Wedge resection B. Gastrectomy C. Endoscopic mucosal resection (JSGE guidelines for early gastric CA. T1a differentiated < 2cm diameter)
176. A patient with peptic ulcer, not respond to medical treatment what is the best management for his ulcer?
A. Truncal Vagotomy B. High selective vagotomy (for gastric type 2,3 & duodenal, preserving crow foot for pylorus/antrum in intractable ulcer)
C. Billroth operation
177. What is the cause of anaemia post partial gastrectomy?
A. Vit B12 deficiency (if fundus& body resected >> decrease parietal cells>> decrease intrinsic factor >> Vit B12 malabsorption & deficiency)
B. Bleeding C. Hypovolaemia D. Iron deficiency anaemia (due to malabsorption, less intake, occult bleeding or duodenal bypass)
178. What is the most common complication of gastric operation, Billroth I, II?
A. Dumping syndrome (Early cramps, diarrhoea, palpitation, sweating &hypotension ½ hr. after meal OR Late 1-3 hrs as hypoglycaemic symptoms)
B. Efferent loop syndrome C. Anastomotic leakage D. Internal hernia
179. A patient has history of gastrectomy and reconstruction, 2 hrs., after eating, complained of fainting, improved by feeding what this is?
A. Late dumping (Rebound hypoglycaemia) B. Early dumping (Rebound hypovolaemia within ½ -1 hour) C. Leakage
180. A patient underwent gastrectomy with Roux en Y gastrojejunostomy 3-days ago, NGT normal, drain 150 ml with soft and lax
abdomen, labs are normal, but elevated WBCs what is the cause?
A. Duodenal stump blow out B. Jejunal leakage C. Jejunojejunostomy D. Oesophagojejunal leak
181. A patient underwent gastrectomy and gastrojejunostomy for manag., of gastric bleeding, C/O of bilious vomiting no food, what’s it?
A. Afferent loop syndrome B. Efferent loop syndrome
182. After gastric bypass for bariatric surgery, the patient was well, then developed abdominal pain and distension, and there was vomiting
of bilious content, no fever, and no abdominal tenderness, what is the possible cause?
A. Anastomosis leak B. Afferent loop syndrome C. Efferent loop syndrome >> (post pancreaticoduodenectomy) D. Internal hernia
183. A gastric bypass for cancer, total gastrectomy, and Ryle tube removed at 3rd. day post operatively, oral feeding started at 4th day
after 24 hrs. Abdominal pain, distension, examination pain and tenderness, P 98, T 38.5 drains 100 mL serous fluid what the cause will be?
A. Jejunojejunal leak B. Duodenal blow out (3-4 days, started oral, sudden severe pain, tenderness, fever, tachycardia and shock like state)
C. Afferent loop synd. D. Post gastrectomy syndrome
184. A patient underwent total gastrectomy and oesophagojejunostomy, after drinking, clear or frothy discharge, stable what’s the diagnosis?
A. Duodenal blow out B. Leak from the anastomosis
185. A 38 y.o female patient, with recurrent attacks of epigastric pain during night, relieved by IV hyoscine what is cause?
A. Duodenitis (cause smooth ms relaxation decrease spasm pain relief) B. Pancreatitis C. Cholangitis D. IBS
186. A patient 28 y symptoms of coeliac disease symptoms, which part of intestine is almost affected by coeliac disease?
A. Ileum B. Colon C. Jejunum second half D. Duodenum first part
187. What is the most characteristic symptom in patients with duodenal ulcer?
A. Epigastric pain starting 2-3 hours after meal (after gastric emptying to duodenum) B. Epigastric pain stats immediately after meal
C. Increasing pain on lying supine D. Pain radiates to back
188. What is the duodenal ulcer surgery that has the least risk of recurrence?
A. High selective vagotomy B. Vagotomy and pyloroplasty C. Vagotomy and Antrectomy
189. A patient with severe epigastric pain, which became generalised abdominal pain, CXR shows air under diaphragm, on exploration;
1cm perforation is found in stomach 3 cm proximal to pylorus with 2 cm induration around perforation, he is stable, no sepsis what’s ttt?
A. Billroth II gastrectomy B. Gastro-jejunostomy C. Pyloromyotomy with vagotomy
D. Close the perforation, do toilet, insert drain and ulcer biopsy to R/O malignancy, if big ulcer seems malignant >> wedge excision involve ulcer
190. A patient with sudden severe epigastric pain, with epigastric and right lower quadrant tenderness, what is the initial investigation
that will help you to reach the diagnosis?
A. Chest X ray erect showing the diaphragm B. CT abdomen C. Abdominal US D. Upper endoscopy
191. What is the best operation for management of anterior surface duodenal perforation?
A. Repair with graham omental patch B. Repair with truncal vagotomy C. Repair with selective vagotomy D. Repair with antrectomy
192. A perforated D.U at anterior duodenal wall and laparotomy done, after 24 hrs., there is pus and drainage of about 1.5-litre what’s best?
A. Vagotomy B. Omental patch with IV antibiotics C. Repair with truncal vagotomy D. Gastrojejunostomy
193. A 44 y.o man with past history of H. pylori that eradicated frequently, got 0.5 mm perforated leaking DU, what’s the definitive treatment?
A. Repair + graham patch (or pedicled omental flap; Cellan jones repair if minor non contained)
B. Over or through scope clips (if linear <1cm >>TTSC, if 1-3 cm >> OTSC) C. Under running sutures (if bleeding with vesible bleeder)
D. Billroth II operation (if major non contained, 1st. or proximal 2nd. part. Can do as 1st option duodenoduodenostomy, if faild >>Billroth II)
194. A patient with history of chronic peptic ulcer, presented with severe vomiting, dehydration and on examination there was succussion
splash, what is the first step in management?
A. NGT(gastric outllet obstruction) B. Surgical management(if no relief after 72 hr. >> Vagotomy, antrectomy gastrojejunostomy) C. SEMS
195. How gastric smooth muscle tumours most commonly can be found and diagnosed?
A. Upper gastrointestinal haemorrhage B. Incidental finding on OGD C. Gastric outlet obstruction E. Weight loss
196. A 73-old- male with an alcoholic liver disease previous appendectomy, cachectic, vomiting +ve succession splash what’s diagnosis?
A. Intestinal tumour B. Adhesion C. Ascites D. Gastric outlet obstruction (? Gastric cancer)
197. What is the best investigation to diagnosis GOO and CHPS? OGD Endoscopy for adults & U/S for children
198. A 75-year-old patient, alcoholic, smoker, history of anorexia, nausea, vomiting and weight loss, by examination succussion splash with
distended abdomen, what is the most likely diagnosis?
A. Gastric outlet obstruction; GOO B. Duodenal cancer (the cause of GOO) C. Pancreatitis
199. A patient with PUD on antacid, presented with forceful vomiting that contains food particle, what’s diagnosis? Gastric Outlet obstruction
200. What is the most common histological malignancy in GIT tumours?
A. Adenocarcinoma B. Squamous cell carcinoma C. Lymphoma D. Carcinoid tumours
201. Which of the following if infiltrated is considered early gastric carcinoma?
A. Mucosa and submucosa B. Muscularis mucosa (is the deepest layer of mucosa) C. Muscularis propria (submucosa) D. Serosa
202. Gastric cancer infiltrating muscularis mucosa, how T1 submucosa appear in EUS? 1,3,5 hyperechoic (mucosa, submucosa & adventitia)
A. Second hypoechoic layer B. Third hyperechoic layer C. Fourth appears isoechoic D. Fifth is hypoechoic
203. An image of stomach with a lesion about 1.5 cm, diagnosed as a leiomyoma, what’s the management?
A. Enucleation (if in oesophagus) B. Wedge resection (in stomach) C. Total gastrectomy D. Observation (asymptomatic < 5cm)
204. Regarding Gastrointestinal stromal tumours (GIST), what is correct option?
A. Occur most commonly in the duodenum B. Are almost always malignant C. Often radioresistent (chemosensitive to imatinib)
D. Can be treated adequately with enucleation E. Spread mainly via the lymphatics
205. A patient underwent total gastrectomy and Oesophagojejunostomy, at 4th. day post op. patient has leakage from anastomosis site,
diagnosis confirmed with Gastrografin, no sepsis, drain still showing leak, how you will manage?
A. Conservative B. Omental patch C. Endoscopic stenting D. Redo of anastomosis. Intermediate leak appears 4-7 days after surgery
206. A patient with an exophytic lesion coming from the lesser curvature of stomach, suffering from dyspepsia and fullness, what is the
diagnosis and what is modality of diagnosis?
A. Leiomyosarcoma B. GIST (EUS & OGD guided biopsy) C. Lymphoma (no leiomyoma)
207. An exophytic lesion coming from the lesser curvature of the stomach, seen as a filling defect in radiography, no loss of wait, and there
is gastric upset, in form of dyspepsia and fullness what is most probably diagnosis?
A. Lymphoma B. Gastric carcinoma (Leiomyosarcoma) C. GIST(60% gastric, 80% of all GIT tumours) D. Leiomyoma (oesophagus)
208. A case of gastric GIST with gastric upset, feeling of fullness and weight loss what is the most prognostic factor for GIST?
A. Size B. Mitotic rate, count or feature (if mitotic >5/50 HPF with big size) C. Lymph nodes D. Associated gastritis
209. A picture of endoscopy, showing round gastric swelling, diagnosis, treatment? suspicious EUS features (irregular, ulceration, heterogenous)
A. GIST,Wedge resection (if > 2cm suspicious EUS features) B. EUS surveillance (6-12 m.if <2cm, asympt, favourable features, rounded, regular)
210. What is the most common site of gastrointestinal lymphoma?
A. Small intestine(2nd. in GIT) B. Colon D. Ileocaecal & appendix (3rd. frequent) E. Stomach (the most frequent)
211. What is the most common presentation of gastric lymphoma?
A. Abdominal pain (70%) B. Weight loss (40%) C. Upper gastrointestinal bleeding(rare)
D. Gastric perforation (rare) E. Dyspepsia, nausea and vomiting (30%)
212. What is the most common presentation of gastric cancer?
A. Haematemesis and melaena B. Acute abdominal pain due to perforation C. An epigastric mass
D. Vague abdominal pain and weight loss
213. What is the optimal management of mucosa associated lymphoid tissue (MALT) lymphoma?
A. Chemotherapy B. Total gastrectomy C. Chemoradiation D. Antibiotics (for H Pylori eradication)+- CRTH
214. A case with dyspepsia, epigastric pain with general weakness and weight loss, OGD and Barium meal done that showed increased
thickness of the gastric wall diagnosed as gastric lymphoma, what is the best treatment option for Gastric lymphoma?
A. Radiotherapy B. Chemotherapy R-CHOP (Rituximab, Cyclophosphamide Hydroxy daunomycin Oncovin Prednisone+ Radiation if H. Pylori)
C. Total gastrectomy D. Partial gastrectomy
215. Most important factor for staging "T" component of gastric adenocarcinoma?
A. Endoscopic US B. Transabdominal US C. Gastrografin
216. What is correct regarding gastric adenocarcinoma?
A. The intestinal type is more common in younger patients B. The intestinal type is often associated with peritoneal spread
C. The intestinal type has a worse prognosis than diffuse type D. The intestinal type is often distal in location
217. After tumour resection the specimen examined by histopathologist with microscope he told that, the margin is +ve what means? R1
218. Gastric cancer (adenocarcinoma) was removed completely during surgery but after operation, with histopathologic examination
there was +ve margin because it was seen by microscopy, what that means?
A. R0 (microscopic -ve margin) B. R1(microscopic +ve margin) C. R2 (macroscopic +ve margin) D. R3
219. For pyloric Adenocarcinoma what is the level of lymphadenectomy?
A. D 1(early CA gastric resection+ perigastric LN stations dissection; 1-6) B. D2 (for advanced tumour, stations from 7-12+14)
C. D 3 (D2 + some of distant as para aortic)
220. Gastric cancer with right gastric 3 LNs need resection of what level?
A. D1(1-7) B. D2 (1-12+14) C. D3 D. D1 plus (8a&9)
221. Gastric cancer staging reaches muscularis propria, what is the T stage?
A. T1(lamina propria, muscularis mucosa, or submucosa) B. T2 (muscularis propria) C. T3 (pre serosa) D. T4 (Serosa & surrounding)
222. Where is the most common location of alimentary system schwannoma?
A. Oesophagus B. Stomach (60-70%) C. Rectum D. Tongue (1-12%) E. Colon (next to stomach)
223. A female patient has pyloric carcinoma with Coeliac LN involvement (advanced) with GOO what is management?
A. Total gastrectomy B. Gastrojejunostomy(palliative to relieve obstruction) C. Subtotal gastrectomy
224. A 76 y.o male patient who has long time history of gastric ulcer, that was treated and recurred with usually history of indigestion,
epigastric pain, recently he has weight loss and persistent abdominal pain that is not responding to PPI, OGD done where found about 5
cm fungating mass surrounded by areas of hypertrophied irregular rugal folds, what is this considered per Borrmann classification?
A. Polypoid growth type I > 5mm B. Ulcerative growth type III C. Fungated growth type II D. Infiltrative linitis plastica IV
225. A case of gastric outlet obstruction due to distal gastric cancer, query of pancreatic cancer with gastric and duodenal extension what
is the best treatment?
A. Resection B. Removal with primary closure C. Bypass (gastrojejunostomy open or SEMS) self-expandable metal stents
226. For Ovarian secondary tumour or metastasis/ hyperechogenic ovarian mass where is the primary cancer origin?
A. Stomach B. Colon (next to stomach) C. Liver D. Lungs E. Thyroid
227. A patient developed surgical emphysema neck chest abdomen after ERCP, which organ was injured or perforated?
A. Oesophagus B. Stomach C. Pancreas D. Duodenum
228. A patient has history of gastric carcinoma, resection done with roux en y reconstruction, 4th day started oral, followed by abdominal
pain, so upper GI series done that showed leakage from duodenal stump, what’s management?
A. Reexploration B. Endoscopy C. Drainage (duodenostomy tube from start or abd drain for peritoneal collection) D. Somatostatin
229. A patient with history of gastric cancer, resection done with roux en y reconstruction, 4th day started oral, abd lax, drain showed 150
cm serous fluid, the patient is unstable, pain, fever leucocytosis, what is most likely cause of leakage?
A. Duodenal stump leak B. Oesophagojejunostomy C. Jejunojejunostomy
230. A patient on 3rd. day post pancreatectomy, or pancreatic necrosectomy, frothy discharge came through the drain, what is the cause?
A. Decreased COP B. Decrease vascular resistance C. Decreased vascular permeability D. Inreased vascular permeability
231. A scenario of a patient underwent gastrectomy, 4th., day postoperative there was a 100 mL collection, patient is stable, what is the
source of leakage?
A. Duodenal stump blow out (severe pain & tenderness) B. Oesophago-jejunostomy leak (most fatal &serious) D. Jejunojejunostomy

BARIATRIC SURGERIES
232. A 43 y.o lady diabetic, hypertensive, knee arthritis wt. 125, Height 173, BMI > 40.8, what is the best weight reducing method for her?
A. Medication lower weight B. Bariatric surgeries > BMI more than 35+ C. Ketogenic diet with excercise
233. A patient underwent a sleeve gastrectomy last week, and he was present to the ER today complaining of recurrent vomiting. ABG
show no abnormalities, and vitals are stable, what is the best way to manage this case? Reassure the patient
234. How much percentage of having GORD after sleeve gastrectomy?
A. 5% B. 10% C. 20% D. 30 - 35%
235. Morbidly obese male patient, what is the best pre-op. workup helping to decide the best reduction surgery for him?
A. Barium enema B. CT abdomen C. US D. Upper GI endoscopy (if Lap sleeve gastrectomy but not in RYGB)
236. What is the most effective surgery for weight reduction? Gastric bypass
237. A morbidly obese lady with BMI > 40, DM, HTN, with preoperative OGD a lower oesophageal erosion with ulcers found what is the
most suitable metabolic surgery for her?
A. Laparoscopic sleeve gastrectomy B. Duodenal switch C. ROUX en y gastric bypass D. Gastric banding
238. Obese female patient, uncontrolled diabetic with severe GORD, grade 3, what is the best bariatric surgery for her?
A. Roux en Y gastric bypass B. Gastric sleeve C. Adjustable Gastric Band D. Gastric Balloon
239. What is the most suitable operation for weight reduction of least mortality and least failure?
A. Roux en Y RYGB B. Laparoscopic sleeve gastrectomy C. Others; band, balloons
240. A patient’s wt. 105 kg & height 170 cm went to endocrinologist, barium swallow shows elevated GOJ above diaphragm, management?
A. Nutritional and hormonal ttt B. Lap gastric bypass C. Intragastric balloon D. Lap banding
241. Regarding Duodenal switch; biliopancreatic diversion by Scopinaro's procedure what mostly will be affected?
A. Vit B12 (After GBS; RYGB) B. VIT D
242. A patient with a history of gastric bypass has leak, present with severe pain and hypotension, what’s next step?
A. Endoscopy B. CT(if stable, but unstable; FAST initially) C. Laparotomy (after resuscitation) D. Diagnosis laparoscopy
243. A morbidly obese lady, DM, HTN, gastric band inserted for her she lost weight but for long time, she is eating well and regained
weight, feeling comfortable not feeling as before as some restrictive procedure, she is stable what is the optimal action?
A. Remove the band B. Check and readjust band with 2-5 cc saline C. Insert gastric balloon D. Lap sleeve
244. Post Roux en Y lap gastric bypass, manifestation of SIRS fever, leucocytosis pulse 140, with drain output 60 ml of non-clear fluids
what is the possible cause?
A. Anastomotic leak B. Duodenal blow out C. Pneumonia D. Intestinal injury or fistula E. PE
245. What is the most serious complication of gastric bypass procedures?
A. Hepatic failure B. Anastomotic leak C. Urolithiasis D. Intestinal obstruction (late after internal hernia)
246. A patient came 3 days after Roux-En-Y GBS complains of fever chill, BP 90/60 and left shoulder pain, what’s the best investigation?
A. CT with contrast B. Endoscopy C. Laparoscopy D. Exploratory laparotomy
247. An obese female patient, underwent gastric bypass surgery with picture of leak, severe pain and mild hypotension, what’s management?
A. Endoscopic stenting (SEMS, plastic, clipping, leak type 1,2,3,4) B. US guided aspiration C. Laparotomy (if sepsis, peritonitis)
D. Laparoscopic drainage (with broad spectrum Abs +/- clipping or suturing (if no sepsis or peritonitis) for types II, III & IV leaks
248. A patient has 2 years gastric band for obesity management now, she was presented by vomiting and distention, what’s management?
A. Endoscopic (if stable, failed NOM) B. Laparoscopic C. Laparotomy (unstable pat./deteriorated diffuse peritonitis, faliure of NOM)
D. Non operative management (IV antibiotics, NPO, IVF, nutrition if stable with contained collection >> PC drainage)
249. A 58-year-old woman with severe constipation, presented with colicky central abdominal pain, loss of appetite and malaise. On
admission and investigation, her total colon transit time was found to be 140 hours (see lab results and reports). Double Contrast Ba
Enema: No abnormal findings. Colonoscopy: No abnormal finding. What action should be given? test result normal values free thyroxin
T4 51 50-140 nmol/L Free T3 2.6 2.0-4.0 nmol/L TSH 6.5 0.4-6.5 mU/L?
A. Anorectal manometry B. Increase fibers and fluids C. Repeat thyroid function tests D. Investigate secret self-medications
250. An old age, with chronic constipation, abdominal pain, and colonoscopy done it showed a hyperaemic region, physically free, diagnosis?
A. Diverticulitis B. UC C. Crohn's disease D. Laxative abuse E. Irritable bowel syndrome
251. A patient having air under diaphragm and normal bowel sounds what to do?
A. Emergent exploratory laparotomy B. Gastrografin swallow (no barium no Colonoscopy) C. Perforated PU D. Colonoscopy
252. What is the sequence of return of gastrointestinal motility and bowel sounds after surgery?
A. Intestine, stomach, colon (small in hours, stomach 1-2 days, colon 3-5 days) B. Stomach, intestine, colon C. Colon, intestine, stomach
D. Colon, stomach, intestine E. Stomach, colon, intestine
253. Abdominal distention, oliguria, fever, tachycardia, mild hypotension & intravesical pressure 28 mm Hg., CR-S. 8, what’s management?
A. Colostomy B. Exploration & evacuation (as ACS grade IV) C. Colectomy D. Abdominal CT
254. Scenario of patient admitted to ICU due to acute pancreatitis, later shows anuria, distension, IAP 16, what is diagnosis?
A. Intestinal obstruction B. Acute abdominal compartment syndrome (grade II) (intravesical, can be by direct abd catheter, intragastric)
255. A patient with abdominal compartment syndrome, IAP = 16 mmHg., what is degree of abdomen compartment?
A. I (12-15 mmHg.) B. II (16-20 mmHg.) C. III (21-25 mmHg.+MOF) D. IV(>25 mmHg.) N= 0-5 mmHg, up to 7, late pregnant up to 9
256. Abdominal compartment syndrome, CVP 16, anuria, abdominal pain, pressure through urinary bladder is 28 mmHg. what to do?
A. Laparotomy and laparostomy B. Laparotomy and drainage C. Diuretics
257. A patient post-surgical exploration for pyogenic liver abscess presented with abdominal distension, fever, tachycardia, tachypnea,
oliguria, urinary catheter connected to manometry pressure is 28, what is best management option?
A. Evacuation of collection B. Laparotomy and colostomy C. Laparotomy and laparostomy
258. What is the surest value of increased intraabdominal pressure?
A. 12- 15 mmHg. B. 15-20 mmHg. C. 20-25 mmHg. D. More than 25 mmHg.

SMALL BOWEL
259. A 63-year-old female is referred to the surgical clinic with an iron deficiency anaemia. Her past medical history includes a left hemi
colectomy but no other co-morbidities. At what site is most dietary iron is absorbed?
A. Colon B. Stomach C. Small bowel mainly duodenum (and jejunum) D. Proximal ileum E. Distal ileum
260. A 29-year-old man with abdominal pain, nausea, vomiting, peritonitis, laparotomy findings, perforation in distal ileum 20 cm from
the ileocecal valve, adherent ileum to caecum, plaques whitish in colour, fat stranding, thickened bowels, diagnosis?
A. Actinomycosis B. UC C. Crohn's disease D. TB
261. A case of perforated appendix patient with acute abdomen, laparotomy, normal appendix, terminal ileum shows thick with greenish
white discharge with adhesions, what is most likely diagnosis?
A. Crohn’s disease B. TB C. Actinomycosis D. Typhoid
262. What is the pathophysiology of Crohn’s disease?
A. Mural thickening of intestine (pan intestinal, transmural inflammation, skip areas, perianal fistulas, non caseating granuloma)
B. Granulomatous disease C. Coagulative inflammation
263. A 22 y.o male came to clinic with abdominal pain, diarrhoea and weight loss for 1-month, he gave history of occasional occult bleeding
in stool, what is the most likely diagnosis?
A. Crohn’s disease (usually mild, rarely severe) B. Peptic ulcer C. Ulcerative colitis (moderate bleeding) D. Rectal cancer
264. A case of abdominal pain and diarrhoea colonoscopy showed non-contagious patches in the terminal ileum and colon pathology shows
transmural inflammation what’s is the diagnosis?
A. Ulcerative colitis B. Pseudomembranous colitis C. Crohn’s disease
265. During operation for suspected appendicitis, appendix appear normal, however the terminal ileum is evidently thickened and feels
rubbery, its serosa with grayish white exudate and several loops of apparently normal small intestine are adherent to it, what is the most
likely diagnosis?
A. Crohn’s disease of terminal ilium B. Ulcerative colitis C. Acute ileitis D. Perforated Meckel’s diverticulum
E. Ileocaecal TB F. Actinomycosis ileitis (sulphar yellowish)
266. Features of previous diagnosis (Crohn’s) include all the following except?
A. Mucosal ulcerations separated by normal mucosa B. Most common site is ileum C. Fistula is known complication
D. All cases should be treated surgically (only if strictures, fistulas, abscess formation, neoplasia, obstruction & uncontrollable bleeding)
267. On exploration for acute abdomen, greyish white exudate, thickened rubbery ileum, adherent normal bowel found what’s diagnosis?
A. TB B. Crohn's disease C. Actinomycosis
268. Osteoporosis is one of the systemic involvement manifests of Crohn’s disease, what determines the severity of osteoporosis?
A. Age 20-40
B. Crohn’s involvement (CD will increase bone loss and low density more than non crohn’s due to inflamation, malnutrition and steroid use)
269. What is the pathophysiology of Crohn’s disease? Transmural inflammation progresses to, non-caseating granulomas. Scarring lead to bowel
obstruction and stricture formation.
270. During laparotomy, there is perforated ilium 20 cm proximal to the ileocecal valve, with adhesions between the distal part of ilium
and the colon, wall of the ileum and the mesentery are thickened, diagnosis?
A. Crohn's disease B. TB (rare perforation) C. Actinomycosis D. Ileal perforation
271. What is correct regarding internal fistula that developed due to Crohn’s disease?
A. Most commonly presents with florid sepsis B. Most commonly presents with Crohn’s disease flares up
C. Requires excision of both organs involved for surgical treatment D. Requires surgical intervention as soon as the diagnosis is made
E. Most commonly involves small bowel to small bowel fistulation (enteroenteric)
272. What is the most common cause of spontaneous intestinal fistula? 15-25% spontaneous, 75-85% iatrogenic
A. Radiation injury B. Malignancy C. Crohn's disease (the most common) D. Ulcerative colitis E. Diverticular disease
273. What is the most common enteric fistula occurring due to Crohn’s disease?
A. Entero enteric fistula B. Enterocutaneous fistula C. Enterovesical
D. Enterocolic (Ileosigmoid fistula is 6% in all CD cases, &16-26% of CD cases with internal fistulae) E. Aortoenteric fistula
274. What is the optimal treatment for an Ileosigmoid fistula in Crohn's disease?
A. Closure of the fistula (if technically easy closure) B. Proximal ileostomy C. Proximal ileostomy and closure of fistula
D. Ileocecectomy and sigmoid colectomy E. Ileocaecal resection and closure of the sigmoid defect (if colonic CD & difficult technical closure)
275. What is the main component of urinary stones complicating resection of terminal ileum?
A. Oxalate (ileal dis., resection & bypass + malabsorbed bile &FA -> increase oxalate passive diffusion & absorption of dietary oxalate -> Ca oxalate)
B. Ammonium C. Urate D. Phosphate
276. What is the origin of fistula that is least likely to close spontaneously?
A. Colon B. Oesophagus C. Stomach D. Small intestine (lateral duodenal wall, jejunoileal(high output) &ligament of Treitz)
277. An image during laparoscopic exploration for acute abdomen showing a small intestinal loop perforation with pyogenic membrane
and peritoneal free fluids, no more adhesions no pus collection, what is the optimal management?
A. Turn to laparotomy and open repair B. Laparoscopic repair of the perforation C. Just lavage, peritoneal toilet with swab C/S & drain
D. Conservatively treatment (good nutrition, hydration, treat anaemia, give antibiotics, electrolyte repletion, drain, it will close if no distal obstruct.)
278. A middle-aged female patient has history of instrumental trauma after difficult delivery, felt some faecal incontinence and some faeces
per vagina, diagnosed as rectovaginal fistula, what is the best tool to be used for diagnosis?
A. CT B. Rectal barium enema (or MRI enema with gel instead of contrast, Rectal and intravenous contrast) C. Cystoscopy D. MRI
279. Crohn disease, +ve peritonitis for 4 days with ileal perforation 20 cm away from ileocaecal valve what’s the best management?
A. Resection anastomosis B. Primary repair C. Ileocaecal resection with loop ileostomy D. Right Hemicolectomy
280. A patient with inflammatory colitis, which of the following suggests Crohn’s disease diagnosis?
A. Ileitis B. Rectal sparing (skip regions, non caseating granuloma, perianal complex fistulae) C. Crypt abscess D. Proctitis
281. In contrast to Ulcerative colitis, the Crohn’s disease is characterised by which of the following?
A. Is not associated with increased cancer risk B. May spare the rectum
C. Is more commonly associated with sclerosing cholangitis (UC) D. Is more commonly associated with toxic megacolon (UC)
282. What is the abnormality and diagnosis of transverse colon, with hyperaemic mucosa, multiple ulcers, normal rest of colon & rectum?
A. Ischaemic colitis B. UC C. Crohn’s disease (mostly regional ileitis with rectal sparing) D. Pseudomembranous colitis
283. A female patient pain, history of Crohn’s disease, presented by right iliac fossa pain and fever, and frequent diarrhoea management?
A. Steroids (as relapse or activity) B. Antibiotics C. Azathioprine
284. Crohn’s disease, diagnosed a long time ago, presented with exacerbation and diarrhoea, what is the best drug to give this patient?
A. Steroids B. 5ASA C. Azathioprine D. Biological treatment
285. Which of the following is effective in treating refractory Crohn's fistula?
A. IV Corticosteroids B. Methotrexate C. Infliximab (first line, closure rate of 36%, if failed cyclosporine) D. Azathioprine
286. A patient with a history of 3 months intermittent abdominal pain, loose motions, and recently, has low grade fever, sweaty, malaise,
increasing abdominal pain and arthralgia. CT showed stricture 10 cm from ileocecal valve what’s treatment?
A. Anti-tuberculous treatment B. Balloon dilatation (after remission) C Stricturoplasty D. Resection
E. Cortisone till remission (it is CD relapse or flaring)
287. Tumour necrosis factor alpha antagonist is useful in treating which of the following conditions?
A. DIC B. Septic shock C. Crohn's disease (TNF alpha inhibitors as infliximab, adalimumab) D. Metastatic melanoma
288. A case of Crohn’s disease, on infliximab, came with perianal fistula and discharge, with induration and low-grade fever for 3 weeks.
what is best management?
A. Increase dose of infliximab B. Antibiotics (Cipro + flagyl if infliximab is low increase dose) C. Swab from discharge D. MRI.
289. A previously healthy patient presented with intermittent colicky abdominal pain, loose stool, perineal discharge, and tract connecting
the perineum to the anal canal, what is the best investigation?
A. Colonoscopy (Crohn’s disease, and if malignancy) B. MRI (after confirming the fistula cause) C. Sigmoidoscopy D. Barium enema
290. A stricture 20 cm., from ileocaecal valve with diarrhoea and mild weight loss, no fever, no severe pain, how to manage that stricture?
A. Colonoscopy B. Endoscopic dilatation (for < 5cm & 15 cm from the valve) C. Stricturoplasty (for>5cm and 15cm away/ short bowel)
D. Surgical resection (if small stricture & < 15 cm from ileocaecal valve or if large bowel stricture) E. Cortisone (if mild stricture with flaring up)
291. An IBD patient, underwent resection of the terminal ileum, what is the most important nutrient to be replaced?
A. Bile salts B. B12 (for nerve nutrition and prevent megaloblastic anaemia) C. Fat soluble vitamins D. Fatty acids
292. A case of small bowel obstruction, during laparotomy ileal stricture 10 cm from ileocecal valve found with lymph node enlargement,
with some necrotic like areas at lower ascending colon, what is the most suitable treatment option?
A Right hemicolectomy B Stricturoplasty & nodal biopsy C Ileotransverse anastomosis & biopsy D Biopsy and close abdomen
293. An image of small segment stricture of terminal ileum about 2 to 3 cm from ileocecal valve the patient has manifestations of intestinal
obstruction type of operation?
A. Resection and anastomosis B. Stricturoplasty C. Rt. Hemicolectomy
D. Ileocecal resection & ileostomy (till patient’s general condition improves then ileoascending anastomosis)
294. A 29 y.o girl with Crohn’s disease came to ER with severe abdominal pain, fever, diarrhoea, due to perforated distal ileum, management?
A. Primary repair B. Resection of perforated segment and ileostomy C. Ileocecectomy
295. A female patient with abdominal distension and vomiting O.E; only one impassable stricture 6 cm from ileocecal valve?
A. Stricturoplasty B. Right hemicolectomy C. Resection and anastomosis D. Segmental resection and ileostomy
D. Ileocecal resection & ileoascending anastomosis (if no perforation, peritonitis, not toxic & can tolerate lengthy surgery)
296. A 25 y/o women taking steroid for IBD C/O of abdominal pain and bilious vomiting, O/E her abdomen is distended with tender right
iliac fossa, she had done colonoscopy 2 weeks ago which was normal, contrast barium showed single stricture at terminal ilium 1 cm from
ileocecal valve, most appropriate management?
A. Stricturoplasty B. Right hemicolectomy C. Conservative management D. Segmental resection with ileostomy
E. Ileocecal resection & ileostomy (when patient general condition improves then ileoascending anastomosis)
297. What is the layer responsible for the strength of an intestinal anastomosis?
A. Mucosa (no strength) B. Submucosa (the strength holding layer of bowel) C. Muscularis propria D. Serosa (no strength)
298. Bloody diarrhoea, chronic abdominal pain, patient travelled to many countries, Colonoscopy showed circumferential inflammation
from the rectum to the descending colon, with negative bacterial serology, what is the most likely diagnosis?
A. Crohn's colitis B. Yersinia colitis C. UC (mainly colonic, bloody diarrhoea, -ve C. diff, etc) D. Diverticulitis
299. A case of abdominal pain, fever, GI bleeding and diarrhoea on Colonoscopy there was an erythematous lesion from rectum to caecum
associated with leucocytosis, what’s the cause most likely diagnosis?
A. Typhoid B. Ulcerative colitis C. Crohn’s disease D. Borderline enteritis E. Sigmoid diverticulitis
300. A 30-year-old female patient, she has history of diarrhoea, blood and mucous for 5 years back, her mother has the same history from
long time, most appropriate diagnosis?
A. Diverticulitis B. Crohn’s disease C. Ulcerative colitis D. Colonic cancer
301. A 50 y.o patient with history of Ulcerative colitis for 10 years now he came with more than 8 times with bloody diarrhoea, abdominal
pain, tenderness, and fever by workups found anaemia, leucocytosis, hypoalbuminaemia, increased ESR and CRP diagnosed as severe
acute UC with picture of toxic colitis, what the best management plan?
A. IV and rectal steroids B. IV steroids if failed give infliximab (+ LMWH)-> if failed/deteriorated -> cyclosporine if failed> subtotal colectomy
C. Oral Steroids only D. Oral and rectal steroids E. IV Steroids with Infliximab or Cyclosporine
302. What is the best management of a complicated case of UC with perforation and peritonitis, HB is 8, with tachycardia?
A. Proctocolectomy B. Proctocolectomy with ileoanal pouch C. Colectomy D. Restorative proctocolectomy with IPAA (for elective)
E. Total abdominal colectomy with end Ileostomy (as it is for emergency or urgent indications).(perforation, severe hge, & toxic megacolon)
303. What differences between Crohn’s and UC? CD is transmural, skip region, rectal sparing, perianal abscesses and fistulae, non caseating granulomas
A. Aphthous nodule B. Extraintestinal manifestation C. Non-caseating granuloma (in CD not in UC)
304. A 20 yrs. old girl, presented with chronic abdominal pain, diarrhoea, mucous discharge with blood, wt. loss, +ve family history of
same case diagnosis?
A. Ulcerative colitis (bloody diarrhoea more than CD) B. Crohn’s disease C. Colon cancer D. Diverticular disease.
305. What is the percentage of malignant potential in UC after 20 years? Rule of 2, 8, 18% =10, 20, 30 y risk of malignant transformation for IBD
306. What is the most common extra intestinal manifestation of UC?
A. Arthritis (21%) B. Primary Sclerosing cholangitis (4%) C. Ankylosing spondylitis (2%) D. Erythema nodosum (2-34%)
307. A 27 y.o lady with chronic UC, due to jaundice, ERCP was done, showed intra & extra hepatic bile duct strictures, what’s the diagnosis?
A. Primary sclerosing cholangitis B. Primary biliary cirrhosis C. Choledochal cyst
308. Pyoderma gangrenosum is typically associated with which of the following?
A. Advanced colonic adenocarcinoma B. Crohn’s disease (1-2%) C. Ulcerative colitis (5% to 12%) D. Rectal Villous adenoma
309. What is correct option regarding azathioprine in treatment of IBD?
A. Decrease leucocyte B. Used instead of corticosteroids C. It is onset of action 6-8 weeks D. Can give IV
310. Crohn's disease case, managed by resection anastomosis, post op. developed intestinal fistula (drain output is 1-liter intestinal juice)
but no peritonitis or sepsis, lab showed; metabolic acidosis with low HCO3, what is the cause of acidosis?
A. Intestinal fistula (loss of intestinal juice containing HCO3) B. Hypovolaemic shock C. Septic shock
311. During laparotomy for severe peritonitis, in a neglected patient, an incidental perforated ileum was found, what is the management?
A. Repair B. Ileostomy (till improvement of General .& local condition) C. Resection and anastomosis D. Non-surgical treatment
312. A patient with diarrhoea underwent, colonoscopy where an erosion extended to the splenic flexure was found, what's the management?
A. Mesalamine suppositories B. Mesalamine oral and suppository C. Mesalamine enema D. Mesalamine oral & enema (extensive)
313. A patient with symptoms and signs of bowel obstruction, On the way to CT he has upper GI bleeding, what’s next management step?
A. Upper endoscopy B. Colonoscopy C. Abdomen CT D. Laparotomy
314. A case of intussusception, for adult patient came with intestinal obstruction and explored by surgeon who found the intra operative, a
right sided colonic mass, what should be done pre-operative for this case?
A. Air enema (Pneumatic reduction in children) B. Barium enema (not recommened if query perforation) C. Colonoscopy
D. Abdominopelvic CT (if lead point, surrounding struct. & its extent of invasion.-target like sign) E. Abd. US (in children; doughnut sign)
315. A 20 y.o, with constipation, distended abdomen, generalised abdominal pain, fresh bleeding per rectum, what is most likely diagnosis?
A. PU perforation B. Intussusception C. Dieulafoy lesion D. Haemorrhoids E. Diverticular disease
316. What is true for Intussusception in adults?
A. Idiopathic in most cases B. Usually requires resection (2ry to tumour) C. Often successfully treated with hydrostatic reduction
D. It is seldom recurrent E. Girls more common than boys 3:1
317. A 26 year old present with symptoms and signs of intestinal obstruction, imaging shows intussusception; what is the management?
A. Surgical exploration +/- resection B. Barium enema C. Hydrostatic reduction D. If small bowel do reduction
318. A 26-yrs. old man with epigastric pain and nausea was admitted after a CT of the abdomen revealed finding suggestive of entero-
enteric intussusception. He was treated conservatively, on day 4 he looks ill and has diffuse abdominal pain, what is the next step?
Surgical management if malignancy, ischaemia or peritonitis ruled out after CT and was only small bowel but for large bowel is resection.
319. A 29 y.o man with 5 day history of RLQ pain, fever vomiting, WBC 18000, the pain became diffuse with, rigidity and guarding, he
has now, high liver enzymes, jaundice, and general weakness, abdomen CECT showed diffuse peritonitis with large amount of collection
and filling defect inside the inferior mesenteric vein with submucosal bowel oedema, thickening and target sign, what’s your diagnosis?
A. Hepatitis C infection B. Peritonitis of perforated appendix C. Ruptured liver abscess
D. Secondary portal pyaemia (diverticulitis, appendicitis or necrotising pancreatitis--> septic emboli from ileocolic v--> SMV-->Portal v--> PP)
320. A 24 year old male medically and surgically free, presented with manifestation of intestinal obstruction, suspected to be small bowel
obstruction, what is the most common cause?
A. Hernia (as virgin abdomen) B. Adhesion (if previous surgery--> SMO) C. Meckel’s diverticulum D. Malignancy (if old aged)
321. A 65 y.o lady presented with constipation, distention & abdominal cramps with history of constipation for 2 months, but she believes
the cause is diet change, she has hysterectomy 10 yrs. ago exam; increased bowel sounds with Rt. Iliac fossa tenderness what’s diagnosis?
A. Acute appendicitis B. Adhesive intestinal obstruction C. Sigmoid volvulus D. Small bowel cancer (if virgin abdomen)
322. What is the most common cause of small bowel obstruction during pregnancy?
A. Incarcerated groin hernia B. Adhesions (due to abdominal surgery as CS) C. Gallstone ileus D. Intestinal volvulus
323. A female 20 yrs., history of negative abdominal exploration for acute abdomen 2 months ago, came with vomiting, distention, +ve
thrill, fluid aspiration +ve amylase, what is the management?
A. ERCP B. Laparoscopy C. CT (adhesive bowel obstruction from previous exploration can has high amylase due to peritonitis)
324. During pregnancy, which of the following is correct?
A. Acute appendicitis is more common than in the nonpregnant state B. Intestinal obstruction is most commonly caused by adhesions
C. Laparoscopic cholecystectomy is contraindicated D. Leucocytosis is often indicative of a surgical abdominal pathology.
325. Regarding contrast study for intestinal obstruction?
A. Gastrografin is preferred to barium for studying distal small bowel B. Gastrografin has no therapeutic potential
C. Gastrografin is less hazardous than barium if aspiration occurs D. Gastrografin can cause serious fluid shift (osmolality 6 time as ECF)
E. Barium can convert partial small bowel obstruction into complete obstruction
326. Scenario after appendicectomy, presented with adhesive SBO, was treated with IV fluid, followed vitally and kept on antibiotics, for
48 hrs. but no improvement, what was the supposed initial management to be started with?
A. NG tube (for decompression, NPO, IVF) B. Laparoscopy C. Laparotomy D. Antispasmodics and painkillers (forbidden)
327. A case presented with small bowel obstruction & abdominal distention, tympanic on percussion. CT multiple small bowel dilatation
with transition point at distal small bowel, with mesenteric fat stranding, what is the best management modality?
A. Observation B. Colonoscopy C. Diagnostic laparoscopy D. Exploratory laparotomy (CT signs of start of bowel compromise)
328. A 20-year-old male patient found to have intestinal obstruction, he has history of colonic polyps on colonoscopy, what type of his
polyps that caused the intestinal obstruction?
A. FAP polyps B. Juvenile polyps (infants and children) C. Cowden syndrome (ass. e periorbital warts tricholemmomas, oral fibromas)
D. Peutz-Jeghers Syndrome (the most common polyps causing intussusception and bowel obstruction 42.8-50%, pain 23%, bleeding by ulcers 13.5%)
329. Regarding to small bowel tumours, what is true of the following?
A. Carcinoid common in duodenum B. Adenocarcinoma most frequent in ileum C. Tumour frequency duodenum, jejunum then ileum
330. What is the most common presentation of benign small bowel tumours?
A. Small bowel obstruction B. Gastrointestinal bleeding C. Weight loss
D. Incidental finding on laparotomy E. Intestinal perforation
331. Small bowel obstruction in an elderly female without external hernia or previous surgery is most likely caused by which of the following?
A. Small bowel neoplasm B. Volvulus C. Gallstone ileus D. Abdominal abscess E. Obturator hernia
332. Malignant small bowel neoplasms or small bowel cancer, most commonly presented with which of the following? Pain> Bleeding >Wt. loss
A. Weight loss (late, 24- 44 %) B. Abdominal pain (early 44- 90 %) C. GI bleeding (23-41%) D. Obstruction (22 – 26%)
E. Perforation (6-9%) F. Nausea and vomiting (17 to 64%)
333. What is the most common malignancy of the omentum? Metastases. Leiomyosarcoma is the most common type of primary omental tumour
that originates in smooth muscle, usually in the GI tract, retroperitoneum, and genitourinary tract
334. An elderly patient with central abdominal pain, CT shows small bowel cancer 3x4 cm with calcification, what type of cancer?
A. Schwannoma B. Carcinoma C. Lymphoma D. Carcinoid tumour (as calcification = Carcinoid)
335. Elevation of urinary 5 -hydroxy indole acetic acid, HIAA is diagnostic of which of the following?
A. Pheochromocytoma B. Cushing's disease C. Carcinoid syndrome D. Aldosteronoma
336. From which of the following the highest rate of metastasis occurs in carcinoid tumours?
A. Appendix (90% of all appendiceal tumours are Carcinoid) B. Bronchus D. Stomach
C. Terminal Ileum (60 cm before ileocaecal valve. 90% of carcinoid originated from terminal ileum/appendix)
337. A middle aged male patient with abd., pain, distension & liver percussion note not dull (resonant) what is the optimal diagnostic test?
A. Chest x-ray erect (if perforated DU) B. Chest x-ray supine C. Abdomen x-ray erect (perforated viscus) D. Abd x-ray supine
338. A 25y.o man with history of laparoscopic appendectomy 10 yrs ago, came to ER with sudden abd. pain & vomiting, generalised
tenderness, he is vitally stable, AXR shows multiple air fluid level and dilated loop in certain point, what is next step in management?
A. Diagnostic laparoscopy B. Laparotomy C. Abdomen MDCT(severity of obstruction, transition point, ischaemia, necrosis, perforation)
339. An appendectomy 10 years ago, patient presents with abdominal pain, distention, vomiting for 2 days and absolute constipation for
one day, hyper audible intestinal sounds, conservative treatment done for 72 hours but with no improvement, x ray image (showing
dilated ileal and jejunal loops) what’s management?
A. Exploration+- resection/anastomosis B. Gastrografin follow through C. Give painkillers, laxative and discharge
D. Manage as appendix stumpitis
340. A patient presented by persistent vomiting and abdominal distension for 24 hours, admitted for 2 days under conservative treatment
and did not improve, surgical history of appendectomy, x-ray showing small intestinal obstruction, what is the next step?
A. Exploration (may be band or closed loop) B. Small bowel series C. CT D. Keep NPO, NGT, IVF, Antibiotics & observe
341. A patient admitted in hospital, because partial intestinal obstruction, what is the indication for surgery during observation?
A. Hypotension B. Persistent severe abdominal pain the sun should never rise and set on closed small bowels
342. Interloop infection after abdominal exploration, management? Antibiotics and drainage
343. A woman underwent CS came after 5 days complaining of abdominal pain and tenderness and distension, on imaging she has small
bowel fistula, what are you going to do?
A. Colostomy B. Resection + anastomosis C. Soft diet only (if no distal obstruction, no underlying malignancy, pus, high output>> conserve)
D. Hemicolectomy
344. A patient presented with acute abdomen, exploration done, found a small bowel fistula, where primary repair done, after 4 days,
found red oedematous suture line, with opening one stitch, gush of large amount of small intestinal content, what is the management?
A. Enteral feeding B. Re exploration (distal obstruction >> leakage) C. IV hyperalimentation D. Reassure
345. A Postoperatively, a patient develops distension and abdominal pain, US shows mild ascites, what is the management?
A. Laparotomy B. CT scan
346. A post Hartmann’s procedure patient, urine output is 20ml/h. O2 venous saturation is 25% and jugular venous pulse pressure is
5 cm H2O. What to do?
A. Abdominal CT B. Abdominal US C. Venous duplex D. Exploration
This postoperative patient has oliguria, severe lactic acidosis may be due to sepsis or ischaemia at operation site plus hypovolaemia have to be explored.
To diagnose if compartment syndrome from stump leak or vesicoureteric injury have to do CT, but here the patient is unstable, so we have to do exploration
347. What is the most common source of haematogenous metastasis to the small bowel?
A. Cancer colon rare B. Stomach (also ovarian intraabdominal, directly) C. HCC
D. Melanoma (haematogenous extra abdominal, melanoma then breast then lung)

COLORECTAL AND PERIANAL


1. What is the main energy source for Colonocyte?
A. Glutamine (enterocytes) B. Alanine (Muscles and CNS) C. Short chain fatty acids (Colonocytes) D. Glucose (Brain and CNS)
2. What paralytic ileus is associated with?
A. Hypernatraemia B. Hyperkalaemia C. Hyponatraemia D. Hypokalaemia
3. Paralytic ileus is a complication of which of the following?
A. Cyclophosphamide B. Vinca alkaloids C. Methotrexate D. Cisplatin E. Doxorubicin
4. An child with down syndrome, brought by his mother with abdominal distension, with full sigmoid by faeces, with history of delayed
meconium passage, on PR some distal stenosis but wide above the anal canal with explosive passage of liquid and foul stools What is the
best definitive investigation for Hirschsprung’s disease diagnosis?
A. Full thickness suction rectal biopsy B. Rectal mucosal biopsy C. Abdominal x ray (initial) D. Barium contrast
5. What is most associated with Hirschsprung disease? Trypanosoma cruzi; Chagas disease, also Entaoemeba histolytica
6. Regarding neonatal Hirschsprung's disease what is true?
A. Diagnosis is confirmed by barium enema B. Enterocolitis is the leading cause of death (30%) C. Mainly affects females
D. Shows absent nerve trunks in a ganglionic segment
7. Neutropenic enterocolitis is a complication of which of the following chemotherapeutic agents?
A. Cytarabine (used in acute NLL >>severe pancytopenia, myelosuppression) B. Cyclophosphamide C. Doxorubicin D. Cisplatin
8. An old age woman with scenario of acute embolic mesenteric ischaemia, what is the best management?
A. Exploration B. Anticoagulants C. Transluminal angioplasty D. Resection anastomosis
9. An 80-yrs. old male patient brought to the ER, while he was eating when suddenly started to complain of severe abdominal pain and he
lost his consciousness. The temp was 38.5, RR is 23 /m, pulse is 120/m, Bp 100/70, and examination shows a rigid and tender abdomen.
What is the next step? Exploratory laparotomy, as it is a case of acute mesenteric ischaemia
10. A 47- year-old female presents with iron deficiency anaemia, she has 3rd degree haemorrhoids, which of the following is the most
appropriate thing to do?
A. Colonoscopy ( as screening >40 y with pile or unexplained anaemia) B. Haemorrhoidectomy C. CT scan
11. A 58-year-old female, presents with abdominal pain that is relieved by defecation, no weight loss, her uncle in 60s had colon cancer, all
blood labs normal except mild low HB; 11 (12-15), what is the most appropriate test to diagnose?
A. CT of the abdomen B. Endoscopy; EUS C. Colonoscopy (diagnostic and for biopsy) D. Sigmoidoscopy
12. A 56-year-old man presented to the emergency department with intermittent lower abdominal cramps typically coming three or four
hours after a meal for the last six weeks. His bowel habit had become more constipated than usual, and he was forced to strain hard to
achieve evacuation. On examination, there were no significant clinical findings. Which of the following tests would be most useful to
establish a management plan?
A. FOBT (faecal occult blood test) B. Colonoscopy (R/O CRC/ IC) C. Flexible sigmoidoscopy D. Double barium enema
13. A 72 year old female patient, she has 15 minutes postprandial pain, anorexia, most appropriate diagnosis?
A. Colon cancer B. Chronic mesenteric ischaemia (atherosclerosis of mesenteric ves, wt loss, post prand pain >70% stenosis of coeliac /SMA)
C. Acute ischaemic colitis D. Myocardial infarction
14. An old, aged patient who has complaints of abdominal pain within one hour after eating? Chronic mesenteric ischaemia
15. How to deal with an old age patient with a history of MI, acute abdominal pain and lower GIT bleeding? Sigmoidoscopy
16. An old, aged smoking man he has some abdominal pain and after taking tablet Clodrel his pain improved what was diagnosis?
Mostly he has vasospasm or chronic ischaemic bowel due to small showers from AF so after taking that antiplatelet Clodrel (Aspirin (150 mg) +
Clopidogrel (Plavix 75 mg) he improved or if there was something impending inferior MI and that resolved some small clots, otherwise it contain aspirin
17. A 77 y.o male with HTN, and dyslipidaemia, presented with severe abdominal pain for 4 days associated with bloody diarrhoea, CT
imaging was done showed thickening of descending and sigmoid colon with Thumbprint appearance/ sign, what is possible diagnosis?
A. Pseudo membranous colitis B. Ischaemic colitis (pain, bloody diarrh., thumbprinting sign) C. Acute diverticulitis D. UC
18. A patient presented with abdominal pain, he is vitally stable, imaging showed; increased thickness of sigmoid and thumbprint sign due
to oedema, what is the next step?
A. Colonoscopy B. Diagnostic laparoscopy C. Exploratory laparotomy D. CTA E. Sigmoidoscopy
19. A middle-aged lady presented with 2 days history of severe abdominal pain that only get relieved with morphine, she described, twice
haematemesis and melaena once, how to reach the diagnosis?
A. Haematocrit B. S. Urea C. TLC D. Abdominal angiography; CTA? Acute colonic ischaemia 50% distal to MCA, 15% at origin
20. What is the most common cause of small bowel bleeding in adults?
A. Crohn's disease B. Arteriovenous malformation (Dieulafoy, AV malformation & angioectasia) C. Leiomyosarcoma
D. Meckel’s diverticulum
21. A 65-year-old male, diabetic, HTN and atrial fibrillation, presented with severe abdominal pain on examination there was abdominal
tenderness, his labs showed WBC; 23000 with signs of mesenteric ischaemia, what is the best management plan?
A. MRI B. CTA (if stable) C. Laparoscopy D. Exploratory laparotomy (ischaemia, infarction, gangrene, perforation, peritonitis)
22. An old aged patient, after resection of a pedunculated polyp the results was benign adenoma and patient has no family history of
colonic cancer, what to advice for reduction of colonic cancer?
A. Prophylactic sigmoidectomy B. Prophylactic colectomy C. Annual colonoscopy D. Lifestyle modification
23. A 38 y/o female known case of UC for 6 years, she has perianal lesion, asked, when she should do colonoscopy for cancer screening?
A. If there is mild inflammation B. At time of diagnosis C. After 8 years from UC diagnosis (for dysplasia follow /1-3 y)
D. Annually till or after age of 50 E. Now because she has perianal symptoms
24. A woman complains of pain before menses and resolves in 3rd day, how to diagnose? By clinical symptoms
25. A 45 y.o female came for colonoscopy screening, till her that screening start at 50 of what of the followings?
A. Age B. Low risk 50 C. High risk 10 years younger
26. How to treat a patient who came with RLQ abdominal pain and guarding, he is febrile and very toxic with WBCs 22000?
A. Exploratory laparotomy (unstable no more will be added by imaging) B. CT abd C. Abdomen MRI D. Conserve
27. A 27-year-old pregnant lady came with severe right upper quadrant pain started 5 hours ago with anorexia, vomiting and abdominal
guarding on examination, what is the most probable diagnosis?
A. Perforated peptic ulcer B. Hepatic haemangioma C. Acute appendicitis (the most common surgical abdomen in pregnancy)
D. Intestinal obstruction
28. What is true, regarding incidence of acute appendicitis in pregnancy?
A. Has a higher incidence of wound infection B. It is least common in the third trimester (higher in 2nd., more rupture incidence in 3rd.)
C. Leukocytosis is important in making the diagnosis D. Initial observation is recommended when diagnosis is suspected
29. A 27- year-old obese woman presents with right iliac fossa pain associated with anorexia, nausea, and vomiting. On examination, there
is moderate right iliac fossa tenderness, labs showed: leucocytosis, what is the most appropriate management?
A. Open surgery B. CT C. US D. Diagnostic laparoscopy (in obese lap is superior to open)
30. A 22 y.o lady came with constant pain at Rt. Iliac fossa, vomiting with rebound tenderness, WBCs 12.6, temp 38.2, what’s diagnosis?
A. Salpingitis B. Acute Appendicitis C. UTI/ Ovarian cyst rupture D. PID E. Caecal diverticulitis
31. A 25 yrs. old girl presented with Rt. Iliac fossa colicky pain, anorexia, nausea, no fever no leucocytosis, last menstrual period was 2
weeks ago with little vaginal discharge what is most likely diagnosis?
A. Ectopic pregnancy B. Acute Appendicitis C. Mittelschmerz (mid cyclic ovulation pain) D. Ureteric colic
32. A 24 years old girl with no history of medical or surgical disease, after some outdoor meal she complained from epigastric pain that
became after sometime around the umbilicus with nausea and after pain was only localised to lower right abdomen she vomited twice, no
fever and WBC was just 9.5, she is not stone passer and last period finished 4 days ago, abdomen CT was not confirming appendicitis but
pain was persistent, intraoperatively nothing were abnormal and appendix was also near normal, little catarrh, what is your decision?
A. Do appendectomy even if looks normal (early intramural or serosal can be found) B. Just close and discharge on oral antibiotics
C. Do not do non inflamed appendix D. If you found ovarian cyst fenestrate it
33. A 30 y.o lady with pain around the umbilicus then shifted to the Rt. iliac fossa, PR; severe tenderness, resonant RUQ, what’s diagnosis?
A. Perforated DU (will never cause PR tenderness) B. Perforated acute appendicitis (pelvic adherent to rectum; 80% Bacteroides, 77% E coli)
C. Obturator hernia D. Ruptured ovarian cyst
34. A 32-y.o lady with 24 weeks pregnant came to ER with an acute onset of abdominal pain, fever and vomiting, the pain woke her up in middle
of night with sudden onset of epigastric pain, now diffuse, she has no vaginal bleeding and foetal monitoring with normal vital for the foetus, OE;
the patient has diffuse tenderness with guarding throughout the abdomen, were in epigastric region, pelvic examination is normal ,she has WBCs;
15,000 abdominal X-ray series showed dilated bowel loops but no other findings, what is your next step in management of this patient?
A. Abd U/S B. Pelviabdominal CECT (R/O PU/ viscus or closed loop perforation) C. Admit and observation with serial abd exam
D. Exploratory laparoscopy
35. How long the antibiotic course will be prescribed in cases of non-perforated acute appendicitis?
A. 3-5 days B. 5-7 days C. 7-10 days D. Less than 2 days N.B. acute app less than 2 -- If perforated 7 to 10 d
36. How long the antibiotic course will be prescribed in cases of perforated acute appendicitis?
A. 1-2 days B. 3-5 days C. 5-7 days D. 7-10 days (to be completed after discharge) (intravenous in hospital 2-4 days)
37. A case of an appendicular abscess diagnosed by radiology, what is the most surgical concern?
A. Appendicular perforation (coagulative necrosis) B. High fever/ WBC C. Walled off appendicular mass D. Underlying cancer
38. What is the most accurate tool for diagnosis of appendicitis?
A. US B. Diagnostic laparoscopy C. MDCT scan (the gold standard for acute appendicitis or alternative diagnosis)
39. A 27. y. o patient with appendiceal mass with CT finding of free faecoliths, after non-operative treatment of acute appendicitis,
discharged home with antibiotics, next step in management?
A. No further intervention B. Open appendectomy after 2 weeks C. Laparoscopic appendectomy after 6 weeks (fecolith is indication)
D. Colonoscopy after 6-8 weeks (in elderly to R/O Malignancy)
40. A 58-year-old man with appendicitis was treated conservatively with antibiotics, now he presents with an appendiceal mass with no
collection. How will you manage this case?
A. Interval laparoscopic appendectomy after 12 weeks B. Interval open appendectomy after 12 weeks
C. No further intervention needed D. Colonoscopy after 6 weeks (to R/O underlying malignancy as simple appendicitis not common in his age)
41. What is the best investigation for bleeding Meckel's diverticulum? Technetium scan
42. What is true regarding Meckel’s diverticulum?
A. False diverticulum B. Asymptomatic in most cases C. Commonly presents as gastrointestinal bleeding in adults
D. Commonly presents with intestinal obstruction in children E. Is found in approximately 5% to 10% of people
43. An adult female patient with acute periumbilical pain, anorexia, then localised to Rt. Iliac fossa pain for 2 days with vomiting, PR
tenderness, what is most likely diagnosis?
A. Acute appendicitis B. Mid cycle pain C. Ureteric stone D. Colonic diverticulitis
44. Which of the following medications can be used as prophylaxis in appendectomy?
A. Cephalexin B. Ceftriaxone C. Metronidazole D. Vancomycin E. Ampicillin
45. What is the appropriate agent for prophylaxis in elective Colectomy?
A. Flagyl B. Imipenem C. Vancomycin D. Cefazolin (2gm IV if < 120kg, plus Flagyl 500mg IV) E. Cefotetan
46. During exploration for appendectomy, Caecal diverticula injured, gross spillage, and caecum is massively involved what’s treatment?
A. Ileocecectomy B. Rt. Hemicolectomy C. Primary repair and closure
47. You will do Right hemicolectomy for T1 Caecal cancer, what are the arteries you will ligate?
A. Superior mesenteric artery trunk B. Ileocolic, Rt. colic, main trunk of middle colic
C. Ileocolic, Rt colic, Rt branches of middle colic D. Ileocolic and Rt colic only
48. A 70-year-old patient underwent right hemicolectomy for ascending colon cancer, after ligation of vessels with non-tight ligature by
surgeon that led to severe bleeding which required 10 units of blood transfusion, one day later, acute pulmonary oedema developed with
normal blood tests and electrolytes but low blood urea, what the cause of that event?
A. Massive transfusion effect B. Low ADH secretion C. Congestive heart failure D. ARDS E. None of the above
49. How to minimise surgical site infection, what do you recommend?
A. Give IV antibiotic within 30 min to 1 hr. before surgery B. Preoperative shaving C. Washing peritoneal cavity with betadine
50. A post appendectomy female patient, came with Rt. lower abdominal mild tenderness, normal examination, by CT there is 2x2 cm
collection in retrocaecal region, treatment?
A. Exploring laparotomy B. Percutaneous drainage C. Laparoscopic D. Conservative with antibiotics (stable, asymptomatic)
51. An open appendectomy done, but after 3 days, he has right upper abdominal pain, shoulder pain, fever, leucocytosis, what is next?
A. Laparotomy B. Chest and upper abdomen CT (R/O subphrenic collection/ pneumonia) C. X-Ray
52. A young patient underwent open appendectomy, stayed in hospital for 2 days post operatively and it was uneventful stay, he came
back on the 8th., day with surgical site pain and infection with redness and swelling and tenderness around wound site, with high fever,
shoulder pain and leucocytosis on examination abdomen was rigid with some guarding, what would be your next step?
A. Exploratory laparotomy B. Pelviabdominal CT C. X-Ray D. Oral antibiotics and dressing E. Open drainage
53. A case with history of 2 weeks post appendectomy, came with Rt. hypochondrial pain radiated to the Rt. shoulder what’s optimal test?
A. Abdomen CT (Subphrenic abscess) B. Antibiotics C. Chest CT D. Abdomen x ray erect E. Abdomen US
54. 10 days after appendectomy, right hypochondrial pain radiating to the right shoulder, swinging fever, tenderness over the eighth rib
and lower chest, abdomen soft and lax except mild right hypochondrial tenderness, investigation? CT abdomen (for sub phrenic abscess)
55. A 30-year-old man underwent appendectomy day 6 post-operative complaining of pain at the site of the wound. The wound was red,
swollen, tender, what will you do?
A. Open wound (wound abscess or collection) B. Aspiration C. US D. IV fluid
56. An adult patient with history of 10 days post appendectomy, came to remove the stitches, wound was swollen, red & warm, with
discharge of large amount of pus on removing the stitches with fever and leucocytosis, what is the initial management?
A. Culture/ sensitivity B. Irrigate the with saline C. Pack the wound with betadine-soaked gauze
D. Secondary closure of the wound
57. A post appendectomy patient, came for regular follow up post-surgery, no active complaints, on exam and removal of one stitch serous
discharge from subcutaneous seroma which drains freely from the opening of the wound, no erythema, pain no fever, what is the
appropriate management?
A. Observation/follow up B. Open wound exploration C. Regular wound dressing D. US guided drainage
58. What is the most important gene responsible colonic cancer recurrence?
A. MLH B. APC C. LINC0219 D. KRAS
59. Which one of these gene mutation cancers has no family history (sporadic)?
A. APC (the most critical gene familial and sporadic) B. P53 C. MYH D. K RAS (more in pancreas then colon then lung)
60. A 50 y.o male patient with rectal cancer, his father had rectal cancer, what is the responsible gene for his condition?
A. Proto-oncogene B. Oncogene C. Tumour suppressor gene D. Tumour necrotising factor
61. A patient presented with repeated attacks of diarrhoea with mucous. Colonoscopy revealed, sessile lesion 2x2.5 cm, what’s management?
A. EMR B. Colonoscopic snaring C. Sigmoidectomy D. Proctocolectomy E. Submucus dissection
62. What is the gene mutation responsible for colonic cancer occurrence?
A. APC (tumour suppressor gene, for familial and sporadic, transform normal tissue to dysplastic AFC or early adenoma)
B. KRAS (with BRAF; transform dysplastic AFC or early adenoma to intermediate adenoma >> then by SMAD4 & CDC2 >> late adenomatous polyposis
C. P53 (it transform the late adenoma of FAP to Adenocarcinoma) D. CEA
63. A 23 years old girl on colonoscopy, 100 colorectal polyp found, with positive family history of FAP syndrome, what’s the management?
A. Colectomy B. Mile’s operation (abdominoperineal resection of the rectum APER) C. Lifestyle modification if asymptomatic
64. An adult man with colonic polyps more than 100 polyp, after Colonoscopy, biopsy showed, a benign lesion with normal rest of the
rectum but he has family history of 2 relatives with history of FAP and proctocolectomy done for them, now they are good, what’s manag?
A. Colectomy with ileorectal anastomosis B. Follow up by colonoscopy C. Proctocolectomy with ileal pouch rectal anastomosis
Ileal pouch anal anastomosis is more extensive surgery as compared with ileo-rectal anastomosis and is associated with an increased risk of bleeding and
reduction in fertility in women. Patients with an IRA who subsequently develop severe rectal polyposis will require a secondary proctectomy.
65. A middle aged patient with about 1000 polyps in the colon and rectum, he has family history of similar condition, what’s treatment?
A. Total colectomy B. Polypectomy C. Total proctocolectomy with ileal pouch rectal anastomosis IPRA/IPAA
66. A 19 yrs. old girl with adenomatous sigmoid colon polyp, there is family history for similar case; her brother 23 yrs. & sister 21 years
where colon removed with total proctocolectomy what is the appropriate management for him?
A. Colonoscopy after 6 months B. Total proctocolectomy C. Annual Colonoscopy D. Annual Sigmoidoscopy
67. A patient presented with diarrhoea, with mucous discharge, lower abdominal pain, colonoscopy done show flat adenomatous polyp 2.2
cm what`s the most critical concern?
A. Bleeding B. Intussusception C. Malignancy (polyp 1-2 cm-->10% malignancy risk, > 2cm 50%) D. Infection and ulceration
68. On screening, an adult male patient found to has about 100 colonic polyps with family history of colorectal cancer, responsible gene?
A. Tumour suppressor APC gene B. Mismatch repair gene C. KRAS/MLH1 genes D. P53 gene (last station)
69. What is true regarding Familial adenomatous polyposis syndrome?
A. It is caused by inactivation of a tumour suppressor gene B. Has normal life expectancy after prophylactic colectomy
C. Is associated with increased risk of right-side colon cancer D. Is inherited as an autosomal recessive trait
70. A patient presented with abdominal pain, mass, black lip pigmentation of the mouth & hand and feet, intussusception, family history
of colonic cancer, his father underwent a colonic polyp removal many years ago, what’s the diagnosis? Peutz-Jegher’s syndrome
71. A young girl presented with abdominal pain, family history of FAP what is the best investigation?
A. Abdominal CT B. Colonoscopy C. Abdominal US D. Abdomen MRI
72. Two patients came to you, one of them has a mutation carrier for familial adenomatous polyposis and the other a mutation carrier for
Peutz- Jegher’s syndrome. What is the best screening time for cancer?
Colonoscopy annually continued lifelong for FAP. Colonoscopy at age of 8 every 3 years for Peutz- Jegher’s syndrome.
73. A patient with abdominal wall desmoid tumour, what should be screened for?
A. Lung cancer B. Colonic polyps (10-15% of FAP developed desmoid) C. Breast cancer D. Medullary thyroid carcinoma
74. What is the optimal treatment option of mesenteric desmoid tumour in patient with FAP?
A. Tamoxifen (plus NSAIDs+/- doxorubicin, vinblastine) B. Cortisone C. Radiotherapy (non surgical candidate) D. Methotrexate
75. A patient had Rt. hemicolectomy for FPC 10 yrs. ago, now presented with a 14 cm parietal abd mass at the operation site, what is the
proper management?
A. Triple chemotherapy B. Conservative management C. Wide local excision (desmoid tumour) D. Combined CRT
76. A 70 yrs. old patient present with sudden abdominal tenderness, abdominal U/S and examination are normal, HB% low, what’s initial?
A. Occult stool by FIT(cut-off 10 μg Hb/g faeces to R/O CRC) B. Endoscopy and colonoscopy (screeninig and diagnostic) C. Abdominal CT
77. A 51-year-old female medically & surgically free, with no family history of colon cancer, which colon cancer screening test is appropriate?
A. 5 years Colonoscopy B. Annual faecal occult blood or FIT(if -ve do sigmoidoscopy/5 y & colonoscopy/10 y) C. Annual sigmoidoscopy
78. Which of the following is most likely to cause a false-positive faecal occult blood test?
A. Oral iron therapy B. Coumadin therapy C. Aspirin therapy D. Nonsteroidal anti-inflammatory medications
E. Dietary peroxidases (fruits, vegetables and grains, also upper GI bleeding --> false +ve)
79. Which of the following is most likely to cause a false-negative faecal occult blood test?
A. Oral iron therapy B. Coumadin therapy C. Aspirin therapy D. Iron deficiency anaemia E. Dietary peroxidases
80. A patient with altered bowel habit, spurious diarrhoea, bleeding per rectum; typical history of colonic cancer, image of barium enema
with typical apple core appearance, what is most likely diagnosis?
A. Colonic cancer B. Ischaemic colitis C. Intussusception D. Tuberculous mass E. Ulcerative colitis
81. A male with spurious diarrhoea, weight loss, barium enema contrast cannot pass to the ascending colon, what most likely diagnosis?
A. UC B. Colonic cancer (hepatic flexure) C. Sigmoid volvulus D. Carcinoid tumour
82. A female 56 yrs. old, loss of weight, history of constipation, abdominal x ray show air under diaphragm, what the surgeon will do for
diagnosis? >> CT to rule out colonic cancer with perforation
83. An old age, with abdominal colic, distension, absolute constipation, vomiting, he is pale, with Rt. lower abdominal mass, what’s treatment?
A. Sigmoidectomy B. Ileoanal pouch C. Right hemiolectomy (ascending colon cancer) D. Hartmann’s pouch
84. An image of barium enema typical for cancer colon, with typical apple core appearance, next investigation? Colonoscopy...
Apple core appearance is most frequently associated with constriction of the lumen of the colon by a stenosing annular colorectal carcinoma.
85. What is the gene responsible for Hereditary Nonpolyposis Colonic Cancer; HNPCC (Lynch syndrome)?
A. APC gene B. P 53 C. Cross gene D. MLH gene (DNA mismatch repair genes (MLH1, MSH2, MSH6, PMS2, EPCAM gene)
86. Regarding Hereditary nonpolyposis colorectal cancer; HNPCC what is correct?
A. is an autosomal recessive disorder B. is associated with higher incidence of ovarian cancer
C. Is associated with higher incidence of endometrial cancer D. is mostly left-side colon cancer
87. An old man came with abdominal tenderness, bloody stool, and history of taking amoxicillin for 3 weeks, for UTI, sigmoidoscopy
showed, inflamed, yellowish elevated mucosa with plaque like lesions?
A. Ischaemic colitis B. Pseudomembranous colitis C. Amoebic colitis D. Food poisoning
88. A case of pseudomembranous colitis, no fever or leucocytosis, mild abdominal pain, the patient was given Ciprofloxacin for 14 days for
UTI, what is the first step in his management?
A. IV Clindamycin B. Stop antibiotics C. Start metronidazole oral D. Start Vancomycin
89. What the most organ produces the highest concentration of immunoglobulin A?
A. Bloodstream B. Oral cavity C. Bronchial tree D. Small intestine E. Urogenital tract
90. A male patient on Cipro, for 10 days and develop diarrhoea and diagnosis confirmed membranous colitis what’s treatment? Oral
Fidaxomicin or Vancomycin
91. An adult man after long antibiotic use developed pseudomembranous colitis with diarrhoea, abdominal pain high fever, what is the
optimal treatment option? Stop antibiotic and start oral Fidaxomicin >> if no response or allergy give oral Vancomycin , if allergy oral metronidazole
if fulminent (severe + hypotension, shock, ileus, toxic megacolon >> oral Vancomycin + IV metronidazole, if has paralytic ileus add vancomycin enema.
92. A 32 y.o male after long antibiotic use for two weeks, after severe diverticulitis, he developed pseudomembranous colitis with
diarrhoea, abdominal pain, high fever, antibiotics stopped, flagyl given for 10 days without response, what is the optimal treatment option?
A. IV Vancomycin B. Stop all medications he will be well C. Meropenem D. Oral Fidaxomicin or Vancomycin
93. A known of case ulcerative colitis, developed abdominal pain & distention, image of abdominal x-ray showing hugely dilated
transverse colon, fever, and leucocytosis, was 11000 now became 16000, with tenderness, what’s the most important to concern about?
A. High WBCs (toxic megacolon, perforation, or peritonitis) B. Rebound tenderness C. Fever D.Bloody diarrhoea
94. A 57 yrs. old patient known with Ulcerative Colitis presented with symptoms severe Abdominal pain and distention, nausea, fever, BP
96/56 mmHg, temp: 39 C HR 120 beats/min: WBC 22.500/ per microliter, Hb% 8.9 gm/dl, what is the diagnosis? Toxic Megacolon
95. Colonic distension in toxic megacolon is most prominent in which part of the colon?
A. Caecum B. Ascending colon C. Transverse colon D. Descending colon E. Sigmoid colon
96. An image of (hugely dilated transverse colon) in x ray, he did not say anything more, and asked about, what is the important factor to
determine the need of urgent surgery during the conservative period?
A. Increasing pain B. Bowel obstruction C. Elevated TLC D. The diameter of the dilated colon
97. A patient with diagnosed pseudomembranous colitis is developing a worsening clinical picture and is taken for urgent laparotomy.
What is the appropriate surgical procedure should include?
A. Segmental colectomy and exteriorisation of the ends B. Total abdominal colectomy C. Diverting proximal colostomy
D. Segmental colectomy and primary anastomosis E. Segmental colectomy, primary anastomosis, and diverting colostomy
98. The same previous picture of toxic megacolon, and asked about the commonest cause?
A. Hidden malignancy B. Diverticulitis C. Ulcerative colitis D. Crohn’s disease
99. A patient was on antibiotics, developed diarrhoea, diagnosis done pseudomembranous colitis, with WBC 15500, what is the treatment?
A. I.V. Vancomycin B. Oral Vancomycin C. Ciprofloxacin
100. A patient received antibiotics for treatment of cellulitis for about 13 weeks develop colicky abdominal pain, diarrhoea, fever? Stool
serology >>> stool cytotoxin by ELISA to rule out C. difficile toxins A + B in faeces
101. What is advantage of the Ileostomy through the rectus?
A. Less hernia B. Less prolapse C. Continence
102. A patient with diarrhoea, and gaseous distension, what's the most affected element?
A. K+ B. Mg C. Calcium D. Sodium chloride E. Magnesium salts
103. A patient on antibiotics for treatment of cellulitis for more than 13 weeks, developed colicky abdominal pain, diarrhoea, fever, most
important test to detect the cause?
A. Stool cytotoxin test B. ELISA C. Blood culture
104. A 50 y.o lady came back from Egypt, known case of penicillin allergy, presented to the ER because of painful red lesions on the anterior
shin of the right leg, she received antibiotics, next day developed abdominal pain, fever and diarrhoea, what's the cause of her symptoms?
A. Antibiotic allergy B. Clostridium difficile (pseudomembranous colitis) C. Salmonellosis D. Amoebic dysentery
105. A 45-year-old male smoker had an adenoma removed from his colon. A pathology report shows a benign lesion, what advice should
you give this patient to prevent him from getting colon cancer?
A. Eat a low-fiber diet B. Eat a high-protein diet C. Colonoscopy every year D. Stop smoking and start exercising
106. An elderly male patient underwent colectomy for colonic cancer in which micro metastasis was detected in the lymph nodes, what is
the best explanation?
A. Good prognosis B. Liver metastasis C. It is locally advanced
D. It is sensitive to chemotherapy. (Dukes class C cancer best for chemotherapy)
107. A diagnosed case as early Colonic adenocarcinoma, CT done but cannot stage the cancer what is next for better staging?
A. Colonoscopy B. MRI (for extension, pelvic LNs and liver and T3-4, N) C. EUS (for early T1, T2& T3) D. PET CT
108. What is the best initial investigation for T staging of colonic cancer?
A. EUS (T1/T2; 100%) B. CT (distant metastasis and perirectal than pericolic LN) C. CE MRI (for rectal nodal staging and advanced T colon)
D. PET CT (no role)
109. A Colorectal cancer that invades through the muscularis propria into the sub serosa what this stage is considered?
A. Tl lesion (submucosa, muscularis m , <2cm) B. T2 lesion (muscularis propria 2-4cm) C. T3 lesion (to subserosa 4-6 cm)
D. T4 lesion (invade visceral peritoneum and adjacent structures, > 6 cm)
110. In the TNM staging, what stage II colorectal cancer is defined as what?
A. Tl, NI, M0 B. T2, N0, M0 C. T3, NI, M0 D. T4, N0, M0 (>6cm, invade visceral peritoneum& adjacent structures)
111. A colonic cancer case, with colonic mass about 5 cm, infiltrating pericolic tissues with 5 LN affection, what is TNM stage?
A. Stage I B. Stage II C. Stage III D. Stage IV
112. A 70 yrs. old man with recent constipation for 7 months while doing colonoscopy a colonic mass about 3 cm size was found, what is of
great concern regarding that mass you will be afraid from?
A. Bowel obstruction B. Ulceration and bleeding C. Malignancy D. Bowel perforation
113. An 80-yrs. old male presented with RLQ abdominal pain, laparoscopic appendectomy was done. On gross examination, a mass of
8 mm was detected at the mid appendix. When should you follow up with this patient? Colonoscopy after 6 weeks.
NB >>> CT/MRI is the imaging of choice for staging and determining the extension and liver metastasis.
114. What is the most common cause of large bowel obstruction?
A. Colorectal cancer B. Crohn's colitis C. Diverticulitis D. Adhesions E. Volvulus
115. A male patient came to ER with constipation, abdominal distension, anaemia, and chronic pain in Rt. lower abdomen what’s diagnosis?
A. Caecal volvulus B. Caecal cancer (insidous, pain, anaemia, weakness) C. Appendicular carcinoid D. Carcinoid of caecum
116. An old age male patient, cachectic, with mass in the right lower abdomen, diagnosis? Caecal cancer
Caecal tumours are rare but have insidious onset and cannot easily be diagnosed with barium enema and colonoscopy.
117. An adult male while doing appendectomy, found mass 2.5 cm at base of appendix, mostly Carcinoid, how to manage?
A. Rt. Hemicolectomy B. Appendectomy C. Chemoradiotherapy
118. A post appendectomy patient, found to have 0.5 cm carcinoid tumour in the mesoappendix what’s the plan?
A. Rt. Hemicolectomy B. No further treatment C. Chemotherapy D. Radiotherapy
119. A 45- year-old male patient underwent appendectomy after signs and symptoms of appendicitis. Histopathology report: carcinoma at
the tip of the appendix. What is appropriate management?
A. Observation B. Chemotherapy C. Radiotherapy D. Right hemicolectomy(as carcinoma --> Rt. Hemicol, if carcinoid no LN, no mesoapp.
invasion <2cm observe). If involving the base of the appendix, lymph node involvement, mesoappendiceal invasion or more than 2cm -> Rt. Hemicolectomy
120. An image of small bowel mass, mostly malignant that planned for excision, which inspecting surrounding structures the was a necrotic
swelling of the caecum, what is the optimal surgery for him?
A. Small bowel resection anastomosis B. Right hemicolectomy and ileotransverse anastomosis C. Extended right hemi.
121. History RIF pain and appendectomy done, 1 cm of carcinoid is found at tip of appendix post-surgery, what is the best?
A. Nothing (if away from base or mesoappendix, no LN) B. Right hemicolectomy
122. An old age female nurse, she fell down then developed distention, diagnosis? Caecal Volvulus for right hemicolectomy even if detorted
123. A patient has HTN, DM & Lt. sided hemiplegia. HR is 90, BP is 125/84. On examination, distended abdomen with mild tenderness.
Bowel sounds are decreased. A standing x-ray abdomen is shown, which of the following is the most likely cause of this finding? Caecal
volvulus >>> Whirl, ileocecal twist, transition points, X-marks-the-spot, and split wall have high specificity for Caecal volvulus.
124. Scenario of acute abdominal pain, vomiting & constipation, image of abdominal x ray typical of Caecal volvulus, what concern?
A. Gangrene B. Perforation (due to closed loop, ischaemia, gang --> perforation) C. Obstruction D. Pain
125. A case of acute abdomen with two CT images, there is Caecal volvulus shown in these images, then patient was relieved (spontaneous
detorsion occurred), management?
A. Exploration and resection B. Exploration and fixation C. Observation D. Conserve and discharge
126. A 70 yrs. old, man from nursing home admitted with obstructive symptoms he has Caecal volvulus, responded to conservative
treatment, what the most appropriate next management to do?
A. Hemicolectomy with end ileostomy B. Rt. Hemicolectomy with 1ry anastomosis
C. Caecopexy (if unstable) D. Tube Caecostomy (e infect 40-50%) E. Colonoscopic reduction & decompression (failure rate 80-85%)
127. A 65-year-old male patient underwent right hemicolectomy for Caecal cancer one year ago, presented to ER by abdominal pain and
vomiting 2 days ago, the abdomen is distended but soft lax, what is the appropriate investigation?
A. Small bowel series B. Barium enema C. Colonoscopy D. Laparotomy
128. A male patient with history of right hemicolectomy 3 years ago for Colonic cancer T1a, N0, M0, at present, he complains of right
lower quadrant pain, weight loss and constipation for 3 months, what is the best investigation?
A. Colonoscopy B. Abdominal CECT C. PET D. US E. MRI
129. Cancer colon patient with 50% of wall circumference involvement, colonoscopy show multiple linear deep ulcerations, what’s
regarding plan for resection?
A. Resection for Primary B. Secondary only C. Staging D. Resection to 1ry, 2ry
130. What is the prognostic factor affecting colonic cancer recurrence, which is treated surgically?
A. Stage at presentation B. Lymph nodes C. Extramural tumour deposits D. Presence of distant metastasis
131. What is the indication of neoadjuvant chemotherapy in case of colorectal cancer? Pericolic fat infiltration or mesorectal invasion
132. A 56 y.o lady, weight loss, history of constipation, x ray abdomen showed, air under diaphragm, what the surgeon will do for diagnosis?
A. Abdominal CT with IV contrast B. Colonoscopy C. US D. Barium enema E. Abdominal MRI
133. A 64 y.o male with history of recent bowel habit changes, with abdominal pain, Colonoscopy showed a sigmoid cancer more than 70%
of the lumen, Colonoscope cannot pass more proximal the patient has suspected synchronous lesion what is the best to detect that lesion?
A. CECT B. Colonoscopy C. Barium enema D. CT colonography E. Abdominal MRI
134. A 65 y.o male came to clinic with 3-year history of bleeding per rectum with and without stool, urgency, 3 years ago he had coronary
heart surgery and on antiplatelet, 6 years ago has been treated by radiotherapy for cancer of prostate, what is the cause of his condition?
A. Radiation proctitis B. Colon cancer C. Bleeding due to antiplatelet drug
135. Which large bowel or colorectal part is most commonly affected by Radiation enteritis?
A. The Caecum B. The splenic flexure C. The sigmoid
D. The rectum (75% after pelvic radiotherapy >> rectal symptoms and 20% having chronic proctitis)
136. Rectal flat polyps, 2 cm limited to the mucosa, what is the management? snaring if pedunculated or EMR if sessile
137. An old patient with complete full thickness rectal prolapse, what is the appropriate treatment for him? Altemeier
138. An old diabetic patient with ischaemic heart disease, DM not controlled, with ASA 3, presented with full thickness rectal prolapse,
what is the optimal surgery for him?
A. Posterior mesh rectopexy (Well’s) B. Altemeier operation D. Delorme procedure D. Anterior mesh rectopexy (Marlex; Ripstein)
139. A 40 y.o patient with picture >> with rectal prolapse, soiling his clothes, what’s the optimal management plan for him?
A. Wells; laparoscopic posterior mesh abdominal rectopexy B. Thiersch C. Delorme (perineal, mucosectomy)
D. Altemeier (for old, perineal recto-sigmoidectomy)
140. A 35 y.o male patient with a picture of complete full thickness rectal prolapse, what is the possible cause of that lesion?
A. Congenital pelvic floor weakness (age of 3 y) B. Old age C. Trauma affects the nerve supply of the levator ani muscle
141. An image for rectal prolapsed, what is the most common cause? Chronic constipation, weak floor in old, nerve injury (iatrogenic or traumatic)
142. What is the cause of rectal prolapse in a male patient 40 years old with irritable bowel syndrome?
A. Pelvic floor weakness (if elderly) B. Long term history of diarrhoea or constipation C. Previous injury to anal or hip region
143. A patient was on antibiotics for proctitis, on proctoscopy; rectum was reddish, hyperaemic with yellow fluke seen, best treatment?
A. Ciprofloxacin B. Ceftazidime C. Metronidazole (1st. drug of choice for Giardia lamblia) D. Triclabendazole
144. An image of complete rectal prolapse, what is the most common associated complaint or symptom?
A. Bleeding B. Mucous discharge C. Faecal incontinence (prolapsed mass as a finding or presentation) D. Pain
145. A patient with colonic surgery with anastomosis, presented with leak from site of anastomosis, fever, high TLC management?
Re exploration after resuscitaion+/- antibiotics
146. A 40 year old patient, with rectal prolapse, with chronic constipation, what is the best management?
A. Altemiere B. Delorme C. Rectopexy with sigmoid resection D. Wells rectopexy
147. A rectal foreign body due to inverted mug appear in image you can palpated by PR dilated anus about 6 cm from anal verge and tear
about 50% of circumference, by PR examination you can't feel the FB, how to remove this foreign body?
A. Bimanual judgment B. Anoscopy B. Laparotomy and open rectum C. Laparotomy and squeeze rectum
148. an adult male came with a full thickness rectal tear due to incidental inverted mug injury, he has severe anal bleeding with severe
anal pain, seen in ER who called a Colorectal surgeon, as the patient was stable, he did CT with rigid proctoscopy, how he will repair it?
A. Primary repair with defunctioning colostomy B. Resection anastomosis with colostomy C. Faecal diversion
D. Transanal repair E. Follow up with Barium enema after 5-10 days
149. An old, aged patient with rectal prolapse, what is the procedure? Altemeier procedure (perineal recto-sigmoidectomy)
150. Most common presentation? Mass protruding Most common complaint is faecal incontinence (28 - 88%). Constipation in 15-65%
151. A patient with full thickness rectal prolapse for two years, he has soiling, protruding mass and incontinence, planned for surgical
correction, what is the most common complication of the corrective procedure post-surgery?
A. Incontinence B. Recurrence (Altemeier’s) C. Infection (abdominal/mesh) D. Bleeding (delorme/presacral dissection)
E. Constipation (Ripstein (Marlex) anterior rectopexy)
152. What is the most incapacitating complication and symptoms after rectal prolapse surgery?
A. Constipation B. Bleeding (in presacral dissection & Delorme) C. Infection (abdominal/mesh) D. Recurrence (Altemeier’s)
153. an adult guy after he sat suddenly on wooden F.B, he got a rectal tear 6-7 cm near to half circumference of the rectum, by PR
examination you can feel the tip of wooden foreign body, how to remove this foreign body?
A. Squeeze over the abdomen B. Open the rectum C. Laparotomy and squeeze the intestine D. Let him to strain even if suffers
154. Rectal foreign body, tear involving half rectal circumference, 6-7 cm from anal verge, on laparotomy what you will do?
A. proctectomy and coloanal anastomosis B. Direct repair and Colostomy
155. A patient was found shocked in the floor of the bathroom, profuse PR bleeding, full thickness rectal tear 50% of the circumference,
higher in the rectum there was a foreign body, what could make the patient condition worse?
A. Mucosal rectal tear B. Full thickness rectal tear C. Impacted foreign body
156. A patient was found shocked in bathroom, has rectal injury with foreign body with more than half of the circumference, full
thickness, 6-8 cm from anal verge, exploration was done, and the rectum was opened transversally, what’s the optimal procedure?
A. Primary repair with Colostomy B. Resection anastomosis and colostomy
157. A full thickness rectal tear due to non-palpable rectal FB injury what is the management?
A. Compression of the abdomen B. Perianal extraction C. Laparotomy and squeeze the foreign body
D. Laparotomy and open the rectum to remove the foreign body
158. How much ileostomy output must be kept?
A. 500-700 mL B. 250- 500mL C. 750 – 1000 mL D. 1500 mL
159. A case of permanent ileostomy, what is the most common late complication?
A. Necrosis (early <3 months up to14%) B. Parastomal hernia C. Non-functioning D. Dehydration
160. Dusky ileostomy found 10 days after doing it, what is the optimal management?
A. Hot sponging B. Expectant management C. Refashioning (manage according to finding) D. Closure of ileostomy
161. A patient with colostomy, not functioning good, the cause what is most likely cause?
A. Skeletonisation of the epiploic fat and the mesentery B. Tension on blood supply
C. Early ileus, later hard faeces, or bowel obstruction
162. A loop ileostomy image with dusky red and not functioning, 9 days after fashioning what is the most common cause?
A. Traction on mesentery B. Tight suture to skin C. Frequent manipulation to ascending part D. Dehydration
163. A patient underwent colostomy, he developed mass beside the colostomy show impulse on cough, diagnosis? Parastomal hernia
164. An image shows a child with hanging out ileostomy, which is normal appearance and functioning normally, what is this complication?
A. Prolapse (2 - 22%) B. Necrosis (2-20%) C. Parastomal hernia (14-40%) D. Burst abdomen
165. A patient with hugely prolapsed ileostomy, with oedema of part of it and starting of ischaemic dark patch, what is the management?
A. Refashioning B. Emergent refashioning C. Elective closure of ileostomy D. Expectant treatment (if no ischaemia)
166. A case presented with intestinal obstruction, on laparotomy, you found a mass in the rectosigmoid area what’s next?
A. Radical left hemicolectomy B. Defunctioning Colostomy C. Biopsy and exteriorisation D. Caecostomy
167. An old aged with multimedical problems presented with lower GI bleeding, the endoscopist cannot find any bleeding source, angiography
found vascular abnormality in the transverse colon, what is the most likely diagnosis?
A. Angiodysplasia B. Bleeding Peptic ulcer C. Diverticular disease D. Colorectal cancer
168. An old age patient present with fresh blood per rectum, you did rectal examination and your finger full of blood, but anorectal
colonoscopy was normal. CBC was: HB; 8, what is the investigation that has a high diagnostic value?
A. Upper GI endoscopy B. TC 99 (Radionuclide scanning technetium-99m for Angiodysplasia) C. CT D. US
169. A 65 y.o male with history of chronic constipation, presented by immersive or massive rectal bleeding what is the most likely diagnosis?
A. Angiodysplasia (20%-30%) B. Diverticulosis (33-60%) C. Colorectal cancer (9-12%) D. Rectal ulcer
170. What is the most common cause of lower GI bleeding?
A. Diverticulosis B. Dieulafoy C. Intussusception D. Haemorrhoids E. Rectosigmoid cancer
171. A 40-yr. old with profuse, massive PR bleeding, by colonoscopy hyperaemic and reddish area in transverse colon what is the diagnosis?
A. Aortoenteric fistula (post EVAR into duodenum 3rd., 4th. parts) B. Diverticulitis (33-50%) C. Crohn’s disease D. UC
172. A 67 y.o male with history of chronic constipation, on regular Clodrel for IHD has massive lower GIT bleeding, most common cause?
A. Angiodysplasia B. Diverticulosis C. Colonic cancer D. Peptic ulcer disease E. Haemorrhoids
173. What is the most common cause of massive lower gastrointestinal bleeding in children?
A. Anal fissure B. Intussusception C. Meckel’s diverticulum D. Angiodysplasia
174. Which of the following is the most common site of tubulo-villous adenoma coloured pink? Rectum.
175. Gastrointestinal diverticula do not occur in which of the following?
A. Caecum B. Duodenum C. Rectum (very rare) D. Jejunum E. Ileum
176. Laparotomy findings showed mesenteric jejunal diverticulitis, how to manage?
A. Resection and anastomosis B. Biopsy C. Leave it (if uncomplicated just bowel rest, NGT, IV antibiotics. PC drainage if Abscess)
177. A middle aged patient with infra umbilical midline abdominal pain, persistent pain and leucocytosis, by CT; not acute appendicitis,
exploration done where found ileal diverticulitis with mini perforation surrounded by free peritoneal fluids what to do next?
A. Resection and anastomosis with Abs B. Just take biopsy and fluid for C/S C. Just soft and fibre diet D. Conserve with antibiotics
178. A middle-aged man with pain in LLQ, CT showed contagious abscess, confluent with sigmoid colon what’s cause or pathophysiology?
A. Ruptured diverticular abscess B. Bleeding in abscess C. Obstruction at diverticulum site D. Inflamed mucosa
179. A middle aged patient with history of chronic constipation, he tried all means but not improved, presented to the ER with left iliac
fossa moderate dull aching pain,with WBC 11.800, T 37.9, CT done no collection, just fat stranding and mural thickening, on LIF deep
palpation there is moderate tenderness, what is the optimal management plan?
A. IV antibiotics (if grade 1a,1b with absecss) B. Conservative management C. Oral antibiotics (Hinchey 0 for oral Cipro/flagyl)
180. A 28 y.o, patient medically free presented to the ER with history of abdominal pain, nausea, anorexia and lower quadrant abdominal
pain, he is vitally stable, CT: diverticulitis with fluid 3x4 cm, what is the most appropriate management?
A. Exploratory laparotomy with Sigmoidectomy B. Exploratory laparotomy with easy content C. Diagnostic laparoscopy
D. Conservative IV antibiotics (< 4cc). if > 4cc >> CT guided aspiration if Uncomplicated: Conservative, bowel rest with Oral antibiotics.
If Complicated (Abscess, Obstruction, Perforation): IV antibiotic + CT-guided percutaneous drainage for abscess > 4cc
181. A 45 y.o man has acute sigmoid diverticulitis with peri sigmoidal abscess 6x6 cc, what’s the optimal treatment?
A. Antibiotics B. Percutaneous aspiration (with IV antibiotics & bowel rest as > 4cc) C. Laparotomy D. Laxatives & soft diet
182. A patient suffering from perforated diverticulitis, drainage was done with colostomy, patient received 3rd. generation cephalosporins
before & postoperatively for one week, and the patient developed fever, by investigations intraperitoneal pelvic abscess at day 6 post
operative, what is the most common cause of that?
A. Patient must receive anti-anaerobic B. Should receive 1st generation cephalosporins B. Multiple preop. doses of antibiotics
183. A case of diverticulitis according to C/S kept on Flagyl and Dalacin C, still with Lt. sided abdominal tenderness, fever with repeated
vomiting, and diarrhoea, so underwent exploration surgery and there is severe infection of Lt. side of colon pus was taken for culture,
without antibiotics change, case not improved on medication, what you will do?
A. Stop Flagyl and Dalacin B. Gives 3rd generation cephalosporin C. Stop Flagyl and Dalacin and start Meropenem
184. What is the most common cause of multiple colonic diverticular disease?
A. Congenital (associated with connective tissue disorders as Ehler-Danlos, Marfan's) B. Inflammatory (only if inflamed) C. Malignancy
D. Environmental and lifestyle (Smoking, HTN, dietary, obesity, physical inactivity)
185. A case of huge diverticular abscess you did percutaneous drainage, when will you do the definitive surgery?
A. After 6 weeks (after doing colonoscopy) B. After 8 wks. C. After 12 wks.
186. What is the commonest symptom or presentation of diverticular disease?
A. Bleeding per rectum (in diverticulosis) B. Changes in bowel habits C. Tenesmus D. Dull aching pain (in diverticulitis)
187. A Colovesical fistula due to diverticular disease, urine C/S showed E. coli/& presence of pneumaturia what’s the best diagnostic tool?
A. Pelvi abdominal CT with oral or rectal contrast B. Barium enema (sensitivity 30%) D. Cystoscopy (low sensitivity <50%)
188. What is the diagnostic test of choice for suspected acute sigmoid diverticulitis?
A. Barium enema B. Gastrografin enema C. Abdomen and pelvis CT scan with IV contrast
D. Abdominal ultrasound E. Colonoscopy
189. What is the most common cause of pneumaturia or faecaluria?
A. Ulcerative colitis B. Malignancy C. Radiation enteritis D. Diverticular disease (Colovesical fistula)
190. A patient with dysuria and cloudy urine with bubbling for 2 months, he had history of recurrent left iliac fossa pain for the past 2
years, Colonoscopy done and showed: no diverticulosis or polyp, Cystoscopy done and showed: erythema on the dome of the bladder,
what is the most likely diagnosis?
A. Diverticular diseases (Colovesical fistula) B. Squamous cell bladder cancer C. Transitional cell bladder cancer
191. Regarding the Colovesical fistula what is correct?
A. It is most commonly caused by colon cancer B. It is more common in females than in males
C. It is readily diagnosed with barium enema in most cases D. It requires segmental colectomy and partial cystectomy for treatment
E. Most commonly presents with pneumaturia
192. After anterior resection for sigmoid colonic cancer, he developed post-operative fever, Abdominal CT done: pelvic abscess 6 mL?
A. Percutaneous drainage B. Antibiotics C. Conservative D. Laparotomy evacuation
1. For modified Hinchey I&II; if confirmed abscess by CT if < 4 cc -----> IV antibiotics only, if > 4 cc --- > IV antibiotics for 72 hours and bowel rest if
improved follow up in surgical clinic for colonoscopy after 6-8 weeks, if not improved do CT or US guided aspiration by interventional radiologist.
2. If more than 5cc pus do directly aspiration and antibiotics while he is inpatient and after discharge.
3. In Hinchey IIB with sepsis see; a) If haemodynamically stable but with moderate peritonitis ------> 1ry resection anastomosis.
b) In haemodynamically unstable and fluid responsiveness but with severe peritonitis >> resuscitation and do Hartmann’s
c) If unstable in frail patient with severe peritonitis and septic shock; fluid non-responsive, need vasopressor >>> Damage control surgery; Hartmann’s
procedure and keep in ICU
193. Laparotomy for a patient have acute sigmoid diverticulitis with perforation, Hartmann’s procedure done, sample of the collected pus
sent for culture, the patient was kept on Gentamycin 80 and Metronidazole 500, result no anaerobic E. coli was sensitive. Before surgery,
WBC 18000, CR-S: N, Urea: N. After surgery: WBC 11000. CR-S: high/2 times normal/ Urea high/3 times normal, what’s best option?
A. Laparotomy again B. Stop metronidazole and proceed with gentamicin’s
C. Stop Gentamicin, Metronidazole and give Imipenem D. Gentamicin 180 E. Add Tazobactam IV tds
194. A 52 old patient with 24 hours, bowel obstruction, x ray multiple air -fluid level, during laparotomy mass on recto sigmoid junction?
A Mucosal fistula and colostomy B. Ileostomy C. Hartmann’s procedure D. Left hemicolectomy
Indications of Hartmann's procedure are: 1. Complicated diverticulitis 2. Recurrent diverticulitis being the most common surgical indication.
3. Rectosigmoid cancer, either as emergency as obstruction, perforation, or bleeding or elective; cure, palliation, or anticipation of impending obstruction.
4. Less commonly, the procedure may be done for ischaemia, volvulus, and iatrogenic perforation of the colon during colonoscopy or by a foreign body,
Hinchey's classification: Stage 0, mild clinical diverticulitis (left iliac fossa tenderness, low grade fever and leucocytosis)
Stage I; diverticulitis with paracolic abscess, stage II; Diverticulitis with a more distant abscess (pelvic or retroperitoneal),
Stage III; Diverticulitis with purulent peritonitis, Stage IV; Diverticulitis with faecal peritonitis). Surgery is indicated in about 20-30% of cases of diverticulitis.
195. A middle-aged male patient with lower abdominal pain, PR free, fever, what’s the best investigation? CECT to R/O Sigmoid diverticulitis
196. What is the most common cause of urinary tract infection in colovesical fistula? E coli
197. What is the most common cause of Colovesical fistula?
A. Diverticulitis B. Crohn’s disease (ileocolic/ileoilial) C. Radiation D. Ulcerative colitis E. Colorectal cancer
198. A 56- year-old patient presents with left lower quadrant pain and change in bowel motion frequency. Upon examination, patient has
fever and left lower quadrant tenderness, Imaging showed thickened bowel. What is the diagnosis?
A. Diverticulitis B. Sigmoid mass C. Ruptured ovarian cyst D. Pyelonephritis
199. A patient with history of diverticulosis came with high grade fever, lower abdominal pain, U/S shows 6x6 cc homogenous collection in
the pelvis, what will you do? Percutaneous drainage and antibiotics
200. A patient with history of previous abdominal surgery, while examined him, indentation on the anterior rectal wall, how to manage?
A. CT guided aspiration (if pelvic abscess) B. US guided aspiration C. Trans rectal incision and drainage (rectal abscess)
201. A 62-yrs. old man came to the physician to follow up after colectomy. He was diagnosed as colorectal cancer and underwent total
colectomy, when will he follow up CEA level? Every 3 months for 2 years. Then every 6 months for an additional 3 years.
NB: • Endorectal ultrasound >> every 4 months for 3 years. • Chest abdomen CT every >> 6 - 12 months for 5 years. • Colonoscopy >> every 1y.
202. A 65-year-old heavy smoker male patient, came for a general examination, what is the most important to do screening for him?
A. Osteoporosis B. Colon cancer (USPSTF guidelines; all adults 50-75 y --> CRC screening) C. AAA (as smoker 2nd. after CRC)
In general, USPSTF recommendation for screening of CRC in all adults from 50- 75 years age (strong recommendation grade A) and from 45- 49 (grade B)
and for people from 75-85 years for selected cases only and in general screeining at this age is of low benefit (grade C). No screening for more than 85 years.
1. If the patient has single first degree relative with CRC or advanced adenoma diagnosed after age of 60 >>> start screening at age of 45- 50 and each 10 y
2. If the patient has single first degree relative with CRC or advanced adenoma diagnosed before age of 60 >>> start screening at age of 40 y or 10 y younger
than the affected relative age when diagnosed whichever is earlier and each 5 years
3. If the patient has two first degree relative with CRC or advanced adenoma diagnosed at any age >>> start screening at age of 40 y or 10 y younger than
the youngest affected relative age when diagnosed whichever is earlier and each 5 years
203. What type of polyp has the highest risk of cancer?
A. Villous (if > 1cm= 30% cancer risk. Giant cause severe watery diarrhoea and hypokalaemia) B. Tubular C. Tubulovillous
204. A 55-year-old male with mild abdominal distension and generalised mild weakness, came with diarrhoea and mucous discharge,
electrolyte disturbance in form of hypokalaemia, what is most likely diagnosis?
A. Crohn B. Villous adenoma of the rectum (volume depletion leads to pre-renal failure)-McKittrick-Wheelock syndrome- C. Rectal cancer
205. What is the colonic polyp with the highest malignant potential among the following?
A. 1 cm tubular adenoma B. 4 cm hyperplastic polyp C. 2 cm Villous adenoma D. 2 cm tubulovillous adenoma
206. A middle aged male, with a history of abdominal pain and chronic constipation, underwent colonoscopy where found superficial large
adenoma, what is the management?
A. Endoscopic mucosal resection (for large sessile. Piecemeal resection if >2cm) B. Endoscopic submucosal dissection
C. Transcolonoscopic cauterisaion
207. A 42-yr. male, with rectal bleeding, a biopsy from sigmoid showed "adenoma", what interval you should do colonoscopy screening?
A. 3 - 6 months B. 3 years (if 3-10 adenomas or >1 tub, serrated >10mm) C. 5- 10 years (for low risk, 1-2 tubular adenoma < 10mm)
D. 10 years (for average risk, no adenoma or hyperplastic polyp <10mm) E. 2-3 y (if > 10 adenoma) F. 1y. (serrated polyposis synd.)
208. A 45-year-old female patient prepared for colonoscopy screening, no family history of CRC, how to reduce colonic cancer risk?
A. Lifestyle modification B. Ferrous supplement N.B (Low risk at age of 50- high risk 10 years younger; 35)
209. What is the best investigation tool for detection of obturator lymph node in cancer rectum? MRI
210. A 90 y.o man with multiple comorbidities presented with many cancers in stomach, lung and colon but the original site of the cancer
is unknown, what’s your most appropriate action?
A. Resection of tumours B. Palliative chemotherapy C. Comfort care (watchful waiting)
211. TNM stage for rectal cancer and tumour invades mesorectum, with pararectal LN and secondaries in liver what is worse stage? M1.
212. MRI Staging of cancer rectum reaching the perirectal fat, with lymph node metastasis and liver mets? T4N2M1 according to AJCC 8th
T4 rectal cancers can be T4a or T4b disease. T4a cancers penetrate only visceral peritoneal surface. T4b that directly invade other structures or organs.
213. Staging of rectal cancer not reached the perirectal fat, just subserosal with nearby rectal wall lymph node metastasis? T3N1M0
N1, if spread to 1 to 2 nearby lymph nodes, N1b =2-3 nearby LNs. N1c = tumour deposits in subserosal, mesentry and non peritonealised perirectal tissue
N2a= spread to 4 to 6 nearby lymph nodes, N2b= spread to 7 or more nearby lymph nodes. It has not spread to distant sites
214. A 64-year-old male with history of left sided colonic adenocarcinoma, after 3 years found there are metastatic nodules in the liver,
what is the optimal tool to detect liver metastasis?
A. Triple phase CECT of the liver B. MRI for the liver C. Liver FDG PET scan D. CEA E. Alpha fetoprotein
215. An old, aged man with history of colorectal cancer that treated 3 years age, today he came with right hypochondrial pain and changed
liver enzymes, CECT and MRI done that showed 1 cm liver metastasis and another one 1.5 cm, what is best prognostic size or number?
A. Size is more important B. Site is important C. Both size and number D. Size, location, vascular invasion & number
216. An old aged male patient with colonic cancer that causing large bowel obstruction, intraoperatively, there is a doubt of liver metastatic
deposits, how to confirm that there are liver metastases intra operatively?
A. PET scan of the liver B. Abdominal MRI C. Intra operative FNAC D. Intraoperative US (Sensitivity 94.5%, MRI is 75%)
217. Rectal cancer can’t be assessed if there is mesorectal invasion or not, what you will do to assess?
A. Endorectal US (for T 1-2 N0) B. MRI (mesorectal, locally advanced and CRM ) C. Colonoscopy D. CT
218. An old aged patient, presented by bleeding PR, sense of incomplete evacuation, tenesmus like, PR free, lower abdominal pain diagnosis?
A. Descending colon cancer B. Rectal cancer (the most cancer related tenesmus) C. Diverticulitis D. Amoebic dysentery
219. An elderly man, with asthenia, lethargy, and history of blood streaks in the stool for 2 weeks, labs show low haemoglobin, examination
there is pile, what is the cause?
A. Sigmoid cancer B. Rectal cancer (2ry piles) C. Caecal cancer
220. An ulcerative lesion at rectum, CT was done no +ve finding, how to confirm if there is mesenteric fat invasion?
A. Pelvic MRI (the best for pelvis, mesorectum & rectal CA staging) B. EUS C. Colonoscopy D. Laparoscopic staging
221. Rectal cancer infiltrating the mesorectum, with metastases to inferior mesenteric LNs and to the liver, stage? T4 N2
222. What is the benefit of postoperative radiation without chemotherapy?
A. Improves survival in stage II colon cancer B. Improves local recurrence in stage II colon cancer
C Improves local recurrence in stage III rectal cancer D. Improves local recurrence and survival in stage III rectal cancer
223. A case of rectal cancer invading serosa with + ve LNs more than 4 pelvic lymph node, what is the staging?
A. Stage A B. Stage B C. Stage C1(subserosa+1-4 LNs) D. Stage C2 (extended outside wall + >4 LNs, along supplying vessels)
224. A 42 yrs. old female presented with severe acute abdominal pain not relieved except by morphia, vomited 3 times, and one episode of
bloody diarrhoea, what is the best diagnostic investigation of choice?
A. Abdominal radiology (CECT/ CTA, acute mesenteric ischaemia) B. BUN C. Creatinine kinase D. Colonoscopy
E. MRCP F. Urea G. Cytokine
225. An old age lady with AF complicated by acute mesenteric ischaemia, best investigation? Abdominal radiography; mainly CECT
226. A 42-yr. old female presented with severe abdominal pain relieved only by morphia, vomited 3 times, and one episode of bloody
diarrhoea, how to treat this patient?
A. Coeliac mesenteric anastomosis B. Resection and anastomosis C. Ileal bypass
D. Percutaneous transluminal angioplasty; if no perforation or peritonitis N.B it is mostly a case of ischaemic colitis
227. A 59-yrs. old male presented with haematochezia. Colonoscopy was done, and a polyp was seen at the sigmoid colon where polypectomy
was done and sent for histopathology. The result of histopathology came with; well differentiated adenocarcinoma with free margins from
cancer, what is the next step?
A. Observation, (R0, follow up) B. Sigmoidectomy C. Segmental colectomy D. Fulguration of the polyp site
228. A patient with colorectal cancer started to complain from SOB, what will you order for investigation?
A. CXR to R/O Cannon balls; metastasis to the lungs B. Pulmonary function test C. Ventilation perfusion scan
229. Suspected colon cancer what is the best investigation for screening after positive faecal immunochemistry test?
A. Recto sigmoidoscopy B. Abdominopelvic CT C. EUS D. Colonoscopy (for CRC)
230. An old age patient is complaining of abdominal pain, recent constipation with mid sigmoid colon cancer; what’s best management?
A. Anterior resection B. Resection & colorectal anastomosis. C. Abdominoperineal D. Hartman procedure
231. Rectal adenocarcinoma 5 cm away from the levator ani, another case 11cm from anal verge, invading the mesorectum, it is moderately
differentiated, what is the indication of neoadjuvant chemoradiotherapy?
A. Mesorectal invasion B. 5cm from levator ani C. Adenocarcinoma D. Moderately differentiated
232. A patient underwent sigmoidoscopy found obstructing big sigmoid lesion, that resected, and histology report revealed: Sigmoid
Adenocarcinoma with free margin with no invasion to mucosa, what is the next step?
A. Abdominal CT scan (for staging, as in obstruction manage then stage) B. Sigmoidectomy C. Pelvic MRI D.PET scan
233. A 63 y.o man has history of recent bowel habit changes, in form of chronic constipation for the last 6 months with weight loss,
Colonoscopy was done for him where found a 5 cm mass at 20 cm from the anal verge, surgeon decided to do excision, which artery will be
ligated during surgery?
A. Left colic and middle colic B. Left colic and left branch of middle colic C. Sigmoidal branch of inferior mesenteric artery
D. Right and left colic E. Inferior mesenteric
234. An old aged patient who has slowly progressive abdominal pain, distension, nausea and constipation for 4 days, on exam. tympanic
abdomen, tenderness, on plain abdomen xray coffee bean appearance, and on CT, whirl sign and bird beak, what is most likely diagnosis?
A. Caecal volvulus B. Sigmoid volvulus (for rectal tube decompress. tube sigmodostomy or resection, anastomosis or double barrel colostomy)
C. Toxic megacolon D. Intussusception
235. A 70-year-old male with huge abdominal distension, severe pain and absolute constipation with image of sigmoid volvulus what is the
best imaging modality to diagnose it?
A. Sigmoidoscopy (diagnostic & therapeutic) B. Abd x-ray (initial) C. Abdomen CECT (best) 100 % sensitivity and >90 % specificity)
236. A 46y.o female patient, she is kept on antipsychotic drugs, C/O of abdominal pain, manifestation of intestinal obstruction, diagnosis?
A. Acute intestinal pseudo obstruction B. Volvulus C. Small bowel cancer D. Acute sigmoid diverticulitis
237. A bedridden patient in infirmary; home nursing, taking sometimes tranquillizers noticed after falling from the bed with intestinal
obstruction, what is more likely diagnosis?
A. Sigmoid volvulus B. Rectal cancer C. Pseudo obstruction (Ogilvie syndrome after tranquillizers) D. Faecal impaction
238. An old man with sigmoid volvulus, he is haemodynamically stable, soft lax abdomen, no fever or leucocytosis what is the next initial
appropriate management?
A. Flatus tube reduction (initially if asymp., stable) B. Sigmoid resection anastomosis (failure of detorsion, no peritonitis or isch. bowel, viable ends)
C. Sigmoid resection with Hartmann’s (if unsuccessful detorsion with ischaemic bowel, gross peritoneal contamination or unstable patient)
239. What is the best management for an unstable patient with severe rectal bleeding due to left sided Colonic angiodysplasia?
A. Left hemicolectomy B. Angio-embolisation (unstable) C. Endoscopic laser ablation (if Stable)
240. A patient diagnosed with tubular sigmoid colon tumour, which was resected completely with free margins, what is the next step?
A. Follow up 3-6 months (R0) B. Sigmoidoscopy C. Segmental colectomy D. Fulguration of the tumour
241. An old patient with sudden, abdominal pain, and weight loss, radiologically found pneumoperitoneum and Sigmoid colonic perforation
was diagnosed, what is most likely cause?
A. Ischaemic colitis B. Inflammatory bowel disease C. Perforated typhoid ulcer D. Rectosigmoid cancer
242. An old male patient was admitted as a case of large intestinal obstruction. He underwent rigid sigmoidoscopy that showed a mass in
the sigmoid region. A biopsy was taken, and result came as sigmoid colon adenocarcinoma, what is the best next step?
A. Colonoscopy B. Abdomen CT C. MRI pelvis D. Sigmoidectomy
243. Colonic perforation in a patient with acquired immunodeficiency syndrome is most likely due to which of the followings?
A. Clostridium difficile B. Cytomegalovirus (mostly, retinitis, colitis or oesophagitis) C. Bacteroides D. Salmonella typhi
244. A 54 y.o male patient with abdominal distension, late vomiting, PR free, with barium enema showed, narrowing of the lumen with
apple core sign and some pelvic LN enlargement, CT no abnormalities, what is diagnosis?
A. Crohn’s disease B. Sigmoid cancer (if early, not perforated or complicated can be missed by CT) C. Diverticulitis D. UC
245. Which is best initial investigation for a sigmoid ulcer with ascending colon polyps?
A. MRI B. CT C. EUS (for staging) D. Barium enema E. Colonoscopy (diagnostic and biopsy)
246. Male patient with rectosigmoid cancer started to develop SOB for 2 weeks, what is the cause?
A. Lung metastasis (rectal higher than colonic, also PE can occur) B. Aspiration C. Chemotherapy induced weakness D. COPD
247. Endorectal U/S how many layers will be detected?
A. 2 B. 3 C. 4 D.6 E. 5 (1, 3, 5 = hyperechoic, 2 and 4 =hypoechoic)
248. What is the optimal treatment of Proctalgia fugax?
A. Amitriptyline B. Salbutamol inhaler C. Puborectalis release D. Biofeedback (nothing needed just reassurance)
249. Regarding gastrointestinal bleeding in children, what is correct?
A. Anal fissure is the leading cause B. Clear nasogastric aspirate rules out upper GI bleeding
C. Meckel's diverticulum is seldom cause of massive bleeding D. Bleeding is the most common presentation of intussusception
250. A middle-aged female patient with history of chronic constipation with acute anal pain with defecation, pain is not throbbing & blood
on towel, nothing prolapsed from anus, no history of anal problems before, and otherwise she is medically free, what might be diagnosis?
A. Anal fissure B. Prolapsed internal piles C. Submucus abscess D. Proctalgia fugax
251. A patient complains of anal pain and perianal itching that is more severe at night, no history of soiling of the cloths, no fever, diagnosis?
A. Anal fissure B. Anal fistula C. Partial rectal prolapse D. Prolapsed internal piles (internal piles painless but can bleed)
252. A patient presents with defecation pain and an itchy anus (pruritus) after defecation, as well as painful defecation with streaks of
blood in his stool, what is the most likely diagnosis?
A. Abscess B. Anal Fissure C. Thrombosed piles D. Fistula E. Haemorrhoids (not usually causing pain)
253. Which of the following surgical operations is commonly complicated by faecal incontinence?
A. Lt. Lat. partial sphincterotomy B. Haemorrhoidectomy C. Fissurectomy D. Lt. Lateral complete sphincterotomy
254. A 25-year-old man, medically free, presented to the ER with severe anal pain during defecation and passage of amount of fresh blood
after defecation. Physical examination confirmed the presence of posterior anal fissure, what is the most appropriate?
A. Examination under anaesthesia (if anal spasm and cannot examine or diagnose as a fissure ) B. Lateral internal sphincterotomy
C. Chemical sphincterotomy with diltiazem (the best for acute) Ca channel blocker as GTN oint. D. Botulinum injection
255. A patient with anal fissure not responding to drug treatment, what procedure is most suitable for his condition?
A. Lateral internal sphincterotomy B. Anal curettage (no benefit) C. Lateral external sphincterotomy D. Local Botulinum inject.
E. Posterior internal sphincterotomy >> because of poor blood supply it can cause delayed healing causing burr hole like anus, so it is not done
256. What is the most common cause of anterior anal fissure or tear in females?
A. Normal or difficult vaginal delivery B. Constipation C. Body building and weight lift D. Caesarian section
257. After internal sphincterotomy for anal fissure, the patient developed incontinence to gas and soft stool, PR showed injured external
anal sphincter, how to diagnose the structural abnormality of the anal canal or what is the best investigation for incontinence?
A. EUS (internal sphincter injury) B. Manometry C. Pudendal nerve terminal motor latency D. Sigmoidoscopy
E. Pelvic MRI (especially of external sphincter; injury, atrophy, discriminate tear from scar)
258. An 82 bedridden old man, came to clinic complaining of perianal pain, with visible mucosal protrusion during examination, diagnosis?
A. Haemorrhoids B. Anal cancer C. Anal Fissure D. Perianal abscess
259. What is the proper management of first-degree piles, where no prolapse, pain or bleeding?
A. High fiber diet (for asymptomatic I.II, III; conservative) B. Rubber band ligation (healthy sympt. I, II,III)
C. Sclerotherapy(sympt, I.II, III, coagulopathy, antiplatelets, immunocompromised, portal HTN) D. Haemorrhoidectomy (failed others & grade IV)
260. A patient with bleeding per rectum, proctoscopy bulging mucosa at three sites, with spontaneous retraction upon removal of the
proctoscopy, what is the optimal management?
A. Stool softener and bulk forming diet (for asymptomatic 1st.,2nd.& 3rd. degrees) B. Haemorrhoidectomy (for 3rd. and 4th degrees)
nd
C. Sclerotherapy (2 . degree alternative to RBL if coagulopathy) D. Rubber band ligation (the best but not for coagulopathy)
261. A patient with rectal bleeding at 5 and 7 O’clock, that reduces spontaneously, sclerotherapy done, what type of haemorrhoid treated?
A. Internal (2nd. degree if coagulopathy or RBL is contraindicated) B. External C. Prolapse D. Thrombosed
263. What is the degree of pile for a patient, who has only prolapsed piles which are only manually reducible but not spontaneously?
A. First degree B. Second degree C. Third degree D. Fourth degree
264. A 28 y.o female came to the clinic complaining of on and off painless rectal bleeding, she had anal swelling that reduced spontaneously,
Anoscopy showed haemorrhoids on 3,7,11 o'clock with active bleeding. Labs showed anaemia, haemoglobin was 8.8, what is the most
appropriate management?
A. Follow up B. Conservative treatment C. Rubber band ligation D. Haemorrhoidectomy
265. A protruded swelling from the anus while doing proctoscopy with intact mucosa, no bleeding, no pain fistula or fissure, reduces
spontaneously after removal of scope, management?
A. Rubber band ligation (symptomatic healthy with 2nd degree) B. Conservative B. Haemorrhoidectomy D. Sclerotherapy
266. A 29 y. o man came to clinic with itching and protrusion of small mass through anus associated with minimal bleeding, increase when
go to GYM and he used to reduce it manually. What is the grade of this haemorrhoid?
A. II B. III C. IV D. Rectal prolapse
267. You have a patient who has anal swelling and bleeding; swelling is protruded not reduced except manually what’s the management?
A. Haemorrhoidectomy (3rd. degree prolapsed piles) B. Rubber band ligation C. Sclerotherapy D. Laxative
268. What is the origin of the inferior rectal artery?
A. Anterior division of internal iliac artery(from int. pudendal. of IEA) B. Post division of internal iliac a. C. Inferior mesenteric a.
269. What is the management of bleeding third-degree piles that most of the time prolapsed and reduced only manually?
A. Haemorrhoidectomy B. Sclerotherapy C. Conservative D. Cryotherapy E. Rubber band ligation
Acutely thrombosed external haemorrhoids can be treated by office-based excision if the patient presents within first 3 days of symptoms. if > 3 for excision
Extensive or bleeding thrombosed external haemorrhoids may require surgical excision in the operating room.
270. What is the most common complication after haemorrhoidectomy? Bleeding, stenosis, stricture, post-op. urine retention
271. What is the most common cause of management-related anal stenosis?
A. Sclerotherapy B. Haemorrhoidectomy C. Haemorrhoidectomy with rectal mucosa resection
272. A male patient with 2 years post haemorrhoidectomy, came to surgical clinic presented with difficulty of defaecation what’s cause?
A. Post haemorrhoidectomy stenosis D. Missed cancer C. Acute on top of chronic fissure D. Thrombosed piles
273. A patient underwent haemorrhoid surgery, came with pelvirectal abscess, what was the procedure done and complicated with that?
A. Photocoagulation B. RBL(bleeding, thrombosed ext. piles) C. HAL RAR(lower complication, higher recurrence)
D. Cryotherapy(pain, necrosis, incontinence) E. Stapled haemorrhoidopexy (PPH)(retroperitoneal sepsis Rectovaginal fistula incontinence)
274. A patient with perianal pruritus at night, no soiling of clothes, no history of bleeding, pain or fever, what’s the most likely diagnosis?
A. Anal fissure B. Internal haemorrhoids C. Perianal fistula D. Proctalgia fugax (normal exam, psychogenic)
275. A young female with a vesicular rash around the anus, which is painful with ulcerted masses with some pustules, what’s the diagnosis?
A. Condylomata lata B. Condylomata acuminate C. Anogenital herpes (HSV-2) D. Candida
276. A 32-year-old patient came after 3 days with perianal pain, swelling and tenderness but no fever or throbbing, what is the diagnosis?
A. Anal fissure B. Prolapse pile C. Haematoma D. Abscess
277. History of haemorrhoidectomy 3 weeks ago, presents with tachycardia only, what's the modality of highest diagnostic value?
A. D Dimer B. CTA (if bleeding or PE) C. Sigmoidoscopy D. ECG and CXR
278. An adult patient has history of anal discharge on examination; internal opening 2 cm from anal verge, no external opening, MRI was
done, PR examined palpable swelling, collection intra anal at the posterolateral aspect 2 cm from anal verge what`s the abscess type?
A. Pelvi-rectal (supra levator 4%) B. Perianal (60%) C. Ischiorectal (20-25%) D. Intersphincteric abscess (5%)
279. Scenario of MRI for anal fistula without external opening, what is the most likely diagnosis?
A. Suprasphincteric fistula (5%) B. Trans sphincteric fistula (25%) C. Extrasphincteric fistula (1%) D. Intersphincteric fistula (70%)
280. Rectal fistula without external perianal opening. CT showed pelvic collection in the posterior aspect, what’s the most likely diagnosis?
A. Pelvi-rectal fistula B. Intersphincteric abscess C. Ischiorectal abscess D. Intersphincteric fistula with lower rectal/ pelvic abscess
281. A patient with throbbing anal pain, fever with no soaking of cloths. Colonoscopy free, pain with DRE?
A. Intersphincteric abscess (anal glands in this space) B. Pile C. Fissure D. Submucus abscess (will be seen by colonoscopy as swelling)
282. A case of a patient complaining of chronic anal discharge, by inspection there’s no external opening, MRI was done (image shows a
track from 2 o’clock to 6 o’clock) about 5 cm away from anal orifice, what is the diagnosis?
A. Ischiorectal fistula B. Levator ani abscess C. Perineal abscess D. Intersphincteric abscess E. Prolapsed piles
283. History of DM, after passage of hard stool, throbbing pain felt in the gluteal region with swelling and fever, what’s the cause?
A. Ischiorectal abscess (penetrate the external sphincter into the ischiorectal space ) B. Gluteal abscess C. Sacral PN abscess
284. A middle-aged male patient found to have anal fistula on 7 o’clock, where is the most common internal opening of that anal fistula?
A. Left lateral B. Middle posterior C. Middle anterior D. High perineal
285. Regarding the complex anal fistulas due to perianal Crohn's disease, what is the correct choice among the following?
A. Typically occurs late during the disease B. Fistulas are usually multiple C. Faecal diversion is usually curative
D. Lesions are typically located posteriorly E. Granulomas are seldom found on biopsy
286. A perianal discharge, there was a history of perianal abscess drained twice; before 2 yrs., and again 6 months ago, now, by DRE there
is a cord like structure, by proctoscopy; internal opening at the dentate line, no swelling, no fever or leucocytosis what’s the diagnosis?
A. Perianal fistula B. Ischiorectal fistula C. Intersphincteric fistula D. Supralevator abscess
287. Anal fistula at 6 O’clock with visible remnant of chronic posterior anal fissure, what is the type of fistula?
Low anal fistula (fistulating fissure)
288. Regarding to a fistula with discharge, located posterior to anal sphincter what is your management?
A. Fistulogram B. Fistulotomy and marsupialisation (Trans sphincteric fistula) C. Lateral internal sphincterotomy D. MRI
289. A male patient presented with one-month diarrhoea, has also colicky abdominal pain, anal fistula with the internal opening above
dentate line what’s the best investigation?
A. CT B. MRI Fistulogram C. Colonoscopy (as suspicious CD) D. Barium enema E. CT enteroclysis
290. Regarding the anal fistulas, which of the following is true?
A. Anal fistula is an abnormal connection between anal canal and perianal skin C. Setons have increased risk of incontinence
B. Antibiotics alone sufficient in ttt of anorectal abscess D. Traumatic fistulae are usually intersphincteric
291. A middle aged patient complaining of internal cloth soiling, sigmoidoscopy was normal, PR mucosal bulge on the left side at the tip of
the finger and indentation in the midline posterior, two images of pelvic MRI (mass on the left side at the levator ani), no external opening,
what is the optimal treatment plan?
A. Seton B. Lay open from inside C. Sealing by glue D. LIFT
292. MRI showing abscess within the levator plate, no external or internal opening, but MRI showing track descending 3 cm till midline at
6 O’clock, mostly diagnosis?
A. Pelvirectal fistula B. Subcutaneous fistula C. Submucus fistula D. Low perianal fistula
293. What is the most common presentation of anal canal carcinoma after or next to bleeding? (bleeding is 45%)
A. Discharge B. Mass C. Pain (30%) D. Pruritus (if Bowen's or Paget diseases in anal SCC) E. Rectal prolapse
294. A female patient with sacral sinus with discharge, bimanual examination discharge caseous material, what’s diagnosis?
A. Pilonidal sinus B. Anal dermoid C. Retro rectal dermoid D. Coccygeal teratoma
295. Regarding Sacrococcygeal teratoma what is true?
A. Usually malignant B. More common in males C. Requires complete excision of the coccyx
D. Diagnosis is ruled out if calcification is absent on radiography
296. On per rectal examination, a mobile mass 2 cm, located 3 cm from anal verge, T1 N0 M0, what do you do for it?
A. Abdominoperineal B. Anterior resection C. Local anal excision D. Radiotherapy
297. What is the most likely cause of perianal skin lesion in a patient with leukaemia?
A. HPV B. Adenocarcinoma C. Squamous cell carcinoma D. Teratoma
298. A homosexual patient, presented with ulcerative mass on the anus, painful, rectal examination cannot be done due to pain and
stenosis, what is the diagnosis?
A. Chronic anal fissure B. Anal canal carcinoma C. Rectal carcinoma D. Anogenital herpes
299. A patient with leukaemia, presented with painful swelling coming from anus, itching and bleeding, what is the most likely diagnosis?
A. Sentinel pile B. Anal carcinoma C. Condylomata accuminata D. Thrombosed external piles
300. A homosexual male patient, with history of HPV infection, has history of leukaemia presented with features of perianal ulcerating
mass, with crustations underwent surgical excision of the tumour, specimen showing dysplasia, margin was involved; not free what will
you do, and what is the diagnosis? (Anal cancer) wide local perianal excision with control of margin
A. Re excision with free margin B. Radiotherapy C. Wide local excision and flap
301. A patient known case of leukaemia on chemotherapy treatment with anal mass mostly Condyloma accuminata, he underwent excision,
which was not complete with involved positive margin, what is the best option?
A. Radiotherapy B. Re excision with safety margin C. Increase chemotherapy dose D. Wide local excision with grafting
302. An ulcer 12 cm from anal verge, polyp in the ascending colon, colonoscopy done, what is the best investigation modality for staging?
A. MRI (it is rectal, and MRI is the best for rectal, pelvic LNs and liver) B. CT C. EUS D. PET CT
303. A patient with hard ulcerative mass 11 cm from the anal verge and multiple polyps in the ascending colon, first modality for staging?
A. Abdomen CT (higher than T1- T2) B. Pelvi abdominal MRI (the best for rectal) C. EUS (for T1, T2) D. Colonoscopy
304. A case of rectal carcinoma that about 12 cm. from anal verge, so surgery decided to him after neoadjuvant chemotherapy what is
surgery suitable for him?
A. APR B. Low anterior resection (upper, mid 3rd.) C. Procto sigmoidectomy D. Resection with Hartmann’s procedure
305. How much is the desired minimal distal margin of resected rectal cancer?
A. 1 cm B. 2 cm C. 3 cm D. 4 cm E. 5 cm
306. A 30 y.o female with longitudinal; tender tear on the 9 o'clock lithotomy position with bright bleeding per rectum, what is factor
making you to do biopsy?
A. Site B. Size C. Bleeding D. Recurrence
307. A 56-year-old man presents with symptoms of absolute constipation and rectal bleeding. On examination he has a distended abdomen which
is soft. A digital rectal examination reveals an empty rectum. A conventional abdominal x-ray shows a dilated colon with associated dilatation of
the small bowel, what is the most appropriate investigation?
A. CT enteroclysis B. Colonoscopy C. Barium contrast enema under fluoroscopy
D. CT Colonography E. Conventional CT scan Bowel preparation in the setting of large bowel obstruction is contraindicated and dangerous.
308. A scenario of patient suffering from bleeding per rectum PR show ulcerative mass 4cm from anal verge, management?
A. Colonoscopy with biopsy B. CT C. MRI
309. Anal mass, about 5 cm away from anal verge, most important thing in treatment? Do pelvic MRI for LN & EUS for endorectal staging &
mesorectal invasion, ttt. Chemoradiotherapy if locally advanced and not reach to para aortic LN as Nigro’s protocol then Abdominoperineal resection
310. What is the indication of neoadjuvant chemotherapy in case of rectal cancer 5 cm from the anal verge, no lymph node, and high-grade
adenocarcinoma?
A. Mesorectal invasion B. Near sphincter C. 5cm from anal verge D. High grade tumour
311. An old, aged patient with rectal cancer with invasion to mesorectum, how to manage?
A. Neoadjuvant chemotherapy B. Adjuvant chemoradiation C. Trans anal excision D. APER E. LAR
312. Rectal cancer 5 cm from the anal verge, no LNs what is best next?
A. APER for rectal cancer when the cancer is located close to the anus B. Neoadjuvant chemoradiation(Negro)
C. LAR when the cancer is located well above the anus
313. Rectal cancer 4 cm from anal margin what is the best for nodal staging?
A. CT abd & pelvis B. Endoanal U/S C. MRI D. PET
314. For anal carcinoma, what is the main treatment?
A. Radiotherapy B. Chemoradiotherapy C. Surgery
315. What is correct regarding preoperative radiotherapy for rectal adenocarcinoma?
A. Improves survival B. Increases postoperative morbidity C. Down-stages tumour in up to 50% of cases
D. is less effective than postoperative radiation E. Reduce risk of local recurrence and improve outcomes in rectal cancer
316. A patient presented with severe perianal pain & perianal swelling, examination showed prolapsed mucosa with ulceration and necrosis,
PR was impossible what is the most likely diagnosis?
A. Prolapsed piles B. Anal canal carcinoma C. Perianal abscess
317. What is the most common presentation of anal canal cancer? BPM
A. Pain (35%) B. Mass (30%) C. Discharge D. Bleeding (in 45%) E. Pruritus ani
318. What is the optimal treatment of Cloacogenic carcinoma of anal canal?
A. Local excision(if away from anal verge) B. APR C. Chemotherapy alone D. Radiotherapy alone E. Chemoradiation
319. Circumferential mass at anus, pathologist told you, it is squamous cell carcinoma, what’s management?
A. Radiation B. Chemoradiation C. Staging and reassess
320. A 56 yrs. male with constipation, bleeding per rectum, cachexia, has circumference anal verge adenocarcinoma, management?
A. Defunctioning colostomy B. APR with permanent colostomy C. Radiotherapy D. Chemoradiotherapy therapy then surgery
Nigro protocol is RTH+ 5FU+ (Mitomycin C; at start of radiotherapy) OR Mitomycin + Capecitabine) then sphincter preserving surgery as anterior resection.
321. What is the appropriate management of 3 cm squamous cell carcinoma of the anal canal?
A. Chemotherapy B. APR C. Chemotherapy + Radiation (even T1-2 N0 M0, < basement invasion) E. Radiotherapy + local excision
322. A histopathological report with a result of high-grade intraepithelial lesion from an anal lesion what might be that pathology?
A. BCC B. Sentinel pile C. Hypertrophied anal papilla D. SCC (mostly due to HPV16 & 18 in 80-85% of cases)
323. A patient complaining of abdominal mass, on the anal canal biopsy showed human papilloma virus, what is most likely diagnosis?
A. Paget's disease B. Bowen disease (AIN, precursor of anal SCC ) C. Adenocarcinoma D. Anal SCC
HEPATOBILIARY SURGERY
1. Regarding the hepatic caudate lobe, what is correct? The caudate lobe; Segment I
A. Drains directly into the inferior vena cava B. Represents segment IV C. Is supplied by the left portal vein only
D. Is supplied by the right portal vein only E. Lies to the right of the inferior vena cava
2. Anatomic and segmental Liver division depend on? or liver is divided into Liver segments by? 8 segments by hepatic V, or 4 Sectors by PV
A. Hepatic vein (Couinaud French classification) B. Portal vein (for sectors as American) C. Hepatic artery D. Biliary divisions
3. A 27 y.o female patient exposed to trauma after motor vehicle accidents with right hypochondrial, and lower chest pain, yesterday she
missed herself until next day presented with severe RUQ pain, U.S. normal, possible organ to be injured?
A. Spleen (if back left trauma) B. Liver (if lower rib fracture) C. Stomach (if full) D. Intestine E. Gallbladder
4. A child while riding bicycle, developed blunt handlebar trauma to his abdomen, what is the most organ to be injured?
A. Spleen B. Liver (in 10-16 y; liver edge subcostally 1-2cm unprotected) C. Mesentery (mostly in Seatbelt) D. Kidney
5. A 14 year old female was complaining of epigastric abdominal pain for one month that is affected her life, on examination, she had
multiple bruises in abdomen when asked about it, and she said it appeared after she fell over her bicycle 18 days back, what is best next?
A. Abdominal CT (as she is stable must see any internal injury) B. Abdominal US C. Laparotomy D. Diagnostic laparoscopy
6. A middle-aged patient with right sided deep chest stab wound at mid clavicular line and over the 8th rib the patient is stable, and U/S is
free what is the management?
A. Laparotomy B. Chest and upper abdomen CT (the initial) C. Conservative D. Laparoscopy (if positive CT findings or if unstable)
7. What is the possible injury in a patient who was involved in RTA, with ecchymosis in the mid abdomen at the seat belt site?
A. Spleen (posterior left) B. Liver (anterior right) C. Large Intestine D. Mesentery
8. Seat belt injury, what is the most common injured soft non bowel structure?
A. Duodenum (if associating chance fracture) B. Spleen (common solid left) C. Liver (common solid right) D. Mesentery
9. A female patient, got a high velocity accident with seat belt sign, on x-ray has chance fracture, what will be associated with this fracture?
A. Duodenal perforation (2nd.to 4th. parts as fixed) B. Gastric perforation C. Jejunum perforation D. Vena cava perforation
10. Seat belt in the chest and abdomen, mostly cause injury to which of the following structures? hollow viscus (fixed), mesentery, liver, spleen
A. Retroperitoneum B. Pancreas C. Hollow viscus (more for proximal jejunum & distal ileum as being fixed) D. Rim of pelvis
11. RTA speed of 130 km/hr., wearing seat belt, presented to the ER, with abdominal wall and chest abrasions, what’s the initial investigation?
A. eFAST US (initial) B. Abdomen CT (definitive if +ve eFAST or unstable, severe trauma) C. Abdominal x ray D. MRI
12. What is the most appropriate initial method to diagnose small bowel injury in a conscious stable trauma patient with seatbelt injury?
A. Diagnostic peritoneal lavage (now replaced by CT plus US) B. FAST (Unstable unconscious) C. Multidetector helical CT scan
D. Serial abdominal examination (Patients with high risk mechanisms with equivocal signs on initial CT scan) E. Plain abdominal film
13. A middle aged post RTA trauma patient, with RUQ pain radiating to shoulders, vitals: P:108, BP: 100/70, what’s initial management?
A. US (initial) B. Abdomen CT (the best) C. DPL (if inconclusive FAST) D. Diagnostic laparoscopy (if unstable)
14. A patient with RUQ pain, fever, by imaging there is liver cyst, 40 mL with aspirated chocolate pus, what’s the responsible organism?
A. Entamoeba histolytica B. Hydatid disease C. Liver cirrhosis D. HCC
15. What is the investigation of choice for amoebic liver abscess? Serology - ELISA for Entamoeba histolytica
16. A case came from Mexico has throbbing RUQ pain, high fever & cough on US, a liver hypoechoic cystic mass was found what diagnosis?
A. Amoebic liver abscess (Mexico is amoeba endemic) B. Hydatid cyst (very rare in mexico) C. Pyogenic liver abscess D. HCC
17. What is the parasitic infestation that cause acquired megacolon: acquired Hirschsprung?
A. Trypanosoma cruzi >> Chagas dis. Also, Entamoeba histolytica) B. Treponema pallidum C. Ascaris D. Trichuris
18. Anchovy Sauce pus (chocolate brown pus) is the clinical feature of which of the following?
A. Amoebic liver abscess B. Pyogenic liver abscess C. Peritoneal abscess D. Hydatid liver
19. A 27-year-old patient with RUQ mass, fever, severe throbbing pain, aspirate show anchovy sauce pus what is the possible diagnosis?
A. Hydatid disease B. Amoebic liver abscess C. Pyogenic liver abscess D. Liver cirrhosis
20. For the previous case of amoebic liver abscess, which diagnostic imaging can be used? (N.B not the best)
A. CECT (better than US) B. MRI C. US (initial) D. Technetium-99m liver scanning E. Gallium scanning
F. Liver Angiography (DD abscess/vascular lesion) G. Abdominal x-ray, CXR H. All above answers I. All except E and I.
21. What is the best serological diagnosis of amoebic liver abscess?
A. Indirect haemagglutination (IHA) test B. ELISA C. Enzyme immunoassay (EIA, or ELISA; the best) D. PCR
E. Stool Antigen detection F. All are correct
22. Regarding an amoebic liver abscess what is the best line of treatment?
A. Surgical drainage is usually required B. Negative stool testing for amoebiasis rules out the disease
C. It should be drained percutaneously under computed tomography guidance
D. Treated by Metronidazole (750mg, tds, PO for 10 d or Tinidazole 2gm/d for 5 d) Nitazoxanide (500 mg twice a day for 10 days) can be used
23. Which of the following indicates failure of medical treatment?
A. High risk of abscess rupture, as defined by cavity size greater than 5 cm.
B. Left lobe liver abscess, which is associated with higher mortality and frequency of peritoneal leak or rupture into the pericardium.
C. Failure to observe a clinical medical response to therapy within 5-7 days
D. Cannot differentiate amoebic from pyogenic liver abscess E. All the above.
24. A 29 y.o man visited Mexico 7 months ago, he came to ER with abdominal pain, fever & amoebic liver abscess 10x15 cm how to treat?
A. Percutaneous aspiration B. Percutaneous catheter drainage (if > 5cc or failed medical ttt) C. Metronidazole (if < 5cc, initially for 7-10d)
D. Ciprofloxacin 500mg BD E. Surgical drainage (if lt. lobe abscess or multiple or superficial, rupture risk >10cm or ruptured, ? pyogenic)
After a course of IV Metronidazole (for tissue amoeba) for 3-5 days, if fever subsided continue Flagyl till 10 days followed by Paromomycin 25-35mg/kg/d tds
for 7 days (for luminal intestinal amoeba) or Diloxanide furoate 500 mg tds for 10 days to eradicate amoeba from the intestine, otherwise catheter drainage
25. A patient with right upper abdominal pain, palpable mass, she has pets in the house, no fever or leucocytosis, liver cyst on imaging, it
was case of hydatid cyst, how to reach the diagnosis?
A. ELISA for Echinococcus granulosus (hydatid cyst) sensitivity 80% B. Aspiration and C/S of the cyst content C. ELISA for E. histolytica
26. A picture showing mass in right hypochondrium reaching umbilicus, there is multiple daughter cycts (hydatid liver cysts), investigation.
Hydatid liver cyst? ELISA. CT scanning has an accuracy of 98% and the sensitivity to demonstrate the daughter cysts. It is the best test for DD, liver
hydatid/amoebic or pyogenic. ERCP is both diagnostic and therapeutic in patients with intrabiliary rupture of hydatid cysts.
27. After you surgically evacuated large liver multiloculated abscess, you found a nodule in the mesentery (there is a picture), what will you do?
A. Leave it, and request postoperative pan CT B. Biopsy C. Resection and anastomosis to the segment D. Intra-operative US
28. A patient with liver cyst with peripheral calcification, what’s the diagnosis and treatment?
A. Hydatid cyst (conservative if small & asymptomatic) B. Pyogenic abscess C. Amoebic Liver Abscess D. Metastasis
29. A picture of too huge echinococcosis; hydatid cyst about 15 cm, no fever, jaundice or pain what most appropriate initial step?
A. Albendazole (then definitive) B. Surgical deroofing (>10cm, definitive CE2, CE3b) C. PAIR(CE1 or CE3a) D. Liver resection
Initially but not definitively; Albendazole is the best but must be followed by the definitive management as PAIR or surgery. Drug alone is not effective
Cysts < 5cm or deep inaccessible give Albendazole 2. Cysts > 5cm ->> Albendazole with PAIR 3. Cysts > 10 cm-->> Albendazole 4- 30 days before Surgery
30. A scenario about hydatid cyst measuring 10x13 cm in right lobe of the liver with multiple daughter cysts and multiple septated cysts,
what is the most appropriate definitive management?
A. Surgical deroofing of cyst B. Percutaneous aspiration C. Right hepatectomy D. Albendazole 15mg/kg/24h in 2 doses
1. Medical or conservative treatment for Small cysts < 5cm, asymptomatic, uncomplicated (biliary peritonitis, biliary communication or bilomas) give
Albendazole 3-6 months. Contraindications of medical treatment include early pregnancy, bone marrow suppression, chronic liver disease, large cysts
2. PAIR for > 5cm, type I, II Gharbi (CE1, CE3a) and III if not honeycomb (septated), multiple cysts in segments I, II and III, infected cysts, cysts without
biliary communication, cysts in patient with high risk or unfit for surgery or GA, if cysts recurred after surgery, or patient refused surgery
3. Surgical deroofing or marsipulisation for infected, large size > 10 cm with multiple daughter cysts, superficially located single that is vulnerable for
rupture by PAIR or spontaneously, if with bile ducts communication.
31. What is the layer of excision or what to excise in case of hydatid liver cyst in surgical deroofing?
A. Laminated layer and adventitia B. Germinal layer and adventitia C. Adventitia and capsule
D. Germinal and laminated layers N.B the ectocyst or adventitia is fibrous capsule without active parasite so, no benefit from its removal
32. A 62 year old male with left hypochondrial pain, normal colonoscopy, few pus and blood cells in stool what is the most likely diagnosis?
A. Ischaemic colitis B. Bacillary dysentery (Shigella dysenteriae, pain, fever bloody diarrhoea) C. Crohn’s disease D. Diverticulitis
Preferred regimens Ciprofloxacin 500 mg PO /12 hr for 3 days or 2 gm once oral or Azithromycin 500 mg /12-24 hrs for 3 days or IV Ceftriaxone 2-4 g once
33. A 49 y.o diabetic patient with RUQ pain, fever, jaundice, image of CT with large liver cyst with enhanced wall, what is the diagnosis?
A. Pyogenic liver abscess (80%) B. Amoebic liver abscess (10%) C. Fungal abscess (10%) D Hydatid cyst D. Necrosed HCC
34. Large pyogenic liver abscess with CT image denoting some pus collection about 10x10 cc, what is the best management?
A. Antibiotic only(<3cm) B. Surgery (if > 10 cc, ruptured, peritonitis and failed /inaappropriate PCD, if underlying pathology requires surgery)
C. Percutaneous aspiration (if < 5ml)
D. CT guided percutaneous catheter drainage as pigtail (even >5-10 cm, keep till <10ml /d, WBC normal, for 7 days)
35. Which of the following drug use can be associated with Seizures?
A. Aztreonam B. Flagyl C. Clindamycin D. Ciprofloxacillin E. Imipenem-Cilastatin
36. A patient visited ER with severe epigastric pain, fever and loss of liver dullness, mild leucocytosis, stable patient, what’s likely diagnosis?
A. Chronic DU B. Ruptured gas containing liver abscess C. Chilaiditi syndrome D. Perforated viscus or diverticulitis
37. What is the most common cause of pyogenic liver abscess?
A. Ascending cholangitis (50- 60% biliary dis.) B. Intra-abdominal sepsis C. Perforated Appendicitis (10%) D. Diverticulitis
38. A patient with gallstone disease presented with RUQ tenderness, fever, rigors, yellow sclera, and high bilirubin what’s diagnosis?
A. Acute cholecystitis C. Cholangitis (Charcot’s triad) C. Hepatitis D. GB Mucocele E. Acute pancreatitis
39. Charcot’s triad is specific for which of the following? Charcot’s triad, fever(90%), abdominal pain in (70%) and jaundice in (60%)
A. Cholangitis (50-75% of cholangitis have Charcot’s) B. Appendicitis C. Pancreatitis D. Acute cholecystitis
40. Which is the commonest organism in case of cholangitis?
A. E. Coli (25%-50%) B. Staphylococcus C. Klebsiella species (15%-20%) D. Streptococcus pneumonia
41. A patient had cholecystectomy 2 yrs. ago, came with RUQ pain, fever with rigor and jaundice, what’s is the expected diagnosis?
A. Missed stone B. Gastroduodenitis C. Ascending cholangitis (Charcot’s triad) D. Pancreatitis
42. A trauma patient post RTA with pulse 120, BP 110/60, mild right hypochondrial pain, CT showing large liver laceration, what to do?
A. Discharge B. Admit with close observation C. Exploration D. Repeat CT after 6 hrs.
43. A 50-yr. old, with IHD and DM, admitted to ICU with severe pneumonia and was treated with antibiotics, after 3 days of admission, he
developed hypotension and was treated with hydration and inotropes. On admission labs were normal, after 3 days LFT became abnormal,
total bilirubin 20 & very high AST and ALT (1000), mild increase in LDH, US: unremarkable, what is diagnosis?
A. Ischaemic hepatitis (hypoperfusion & ischaemia) B. Intravascular haemolysis C. ICU related jaundice D. Cholecystitis
44. A 24 y.o patient with abdominal discomfort and right upper abdominal pain, no history of gallstone disease or outdoor meals, he
developed anorexia, vomiting and became dehydrated and hypotensive, on investigation; ALT 1000, AST 550, Amylase normal, Lipase
normal, what is the most likely diagnosis?
A. Pancreatitis B. Cholecystitis C. Hepatitis (ischaemic?) D. Gastroenteritis
45. What is the most reliable method to help determine the cause of ascites? The serum-ascites albumin gradient (SAAG)
46. How to calculate the serum-ascites albumin gradient? Serum albumin concentration minus ascitic albumin concentration SAC- AAC
1. More than 1.1 gm/dl see if ascitic protein <2.5 gm/dl it is either due to Cirrhosis, late Budd Chiari syndrome or massive liver metastasis , if > 2.5 gm/dl
the cause is either CHF, constrictive pericarditis early Budd Chiari syndrome, IVC obstruction or sinusoidal obstruction syndrome
2. If <1.1 gm/dl it is mostly due to biliary leak, nephrotic syndrome, pancreatitis, peritoneal carcinomatosis or TB
47. How much albumin should be given for every liter of ascites removed after large-volume paracentesis (>5 L)? 6 - 8 gm of albumin/L of
removed ascitic fluid so, 30- 40 gm /5 liters
48. A patient with liver cell failure and ascites, underwent paracentesis and become shocked, what is the optimal fluid resuscitation?
A. Saline B. Ringer’s lactate C. Plasma D. Salt (or sodium) free albumin E. Dextrose 20%
49. What is the liver disease that can cause thrombocytopenia with coagulopathy?
A. Liver haemangioma B. Liver cirrhosis with cell failure C. Hepatocellular carcinoma D. PSC
50. What is the most common malignancy associated with chylous ascites?
A. Spontaneous bacterial peritonitis B. Ruptured ovarian cyst C. Lymphoma (up to 50%) D. Iatrogenic/ surgical trauma (8%)
51. What is the hepatic mass that is characterised by increased platelets consumption?
A. Haemangioma (mostly < 5cm, Rt lobe) B. HCC C. FNH D. Liver metastatic deposits
52. A patient with hepatic benign lesion diagnosed with liver haemangioma, decided to be managed conservatively, what should be advised?
A. Avoid trauma contact and sport trauma B. Loss weight C. Smoking cessation
53. What is correct regarding hepatic haemangioma?
A. It is the most common benign hepatic tumour B. is diagnosed with percutaneous needle biopsy
C. is associated with alpha-fetoprotein level D. should be resected as soon as diagnosed
54. What is the most common benign hepatic lesion?
A. Haemangioma (cavernous) B. Adenoma C. Focal nodular hyperplasia D. Hamartoma
55. A 40-year-old female has a 4-cm haemangioma in the right lobe of the liver on computed tomography scan, she is asymptomatic, what
is the appropriate action that should be done?
A. FNAC B. Arrangement for elective resection C. No further action (as asymptomatic < 5cm) D. Angiographic embolisation
56. A 40 year-old man with no previous history of medical or surgical diseases, no symptoms, on initial US examination, you found a liver
mass, and after liver MRI there is smooth, homogenous mass about 6x6 cm with hyperintense on T2 weighted images, what’s optimal next?
A. CECT B. FNAC C. If normal AFP, follow up with CE MRI/CECT D. US (initial) E. Laparoscopy
57. A central scar in a hepatic lesion is characteristic of which of the following?
A. Focal nodular hyperplasia (typical form; firm, solitary lesion with a well-defined margin with central scar) B. Hepatic adenoma C. HCC
C. Haemangioma D. Hamartoma
58. What is true regarding the Hepatic focal nodular hyperplasia?
A. Usually occurs in women of reproductive age (female up to 90% at 35-50 years) B. is related to oral contraceptive use
C. Presents with abdominal pain in most cases D. Carries the risk of spontaneous rupture
59. What is the most common aetiological factor for hepatocellular carcinoma worldwide?
A. HCV B. Hepatitis B Virus (50% >250 million carriers in the world) C. Alcoholic cirrhosis D. NASH (20-30%) E. Aflatoxin
60. A patient with right hypochondrial pain and low-grade fever, deranged liver enzymes and mild jaundice, what diagnosis might be?
A. Liver abscess (high grade fever) B. Viral hepatitis C. Cholangitis (high with rigor) D. Cholecystitis (LFT mostly normal)
61. A 55-year-old female with liver cirrhosis and ascites, with history of multiple paracentesis 2 years ago, now came with abdominal pain
by investigation there is 3 cm Lt lobe mass with increase vascularity in arterial phase what is this mass?
A. Haemangioma B. HCC C. Metastasis D. Liver adenoma
62. A 60 y.o male patient presented with 3 months history of RUQ pain on abdominal US, hypoechoic lesion with some mixed echogenicity,
on Gadolinum enhanced MRI, hypervascular liver lesion with densely enhancement on arterial phase what’s diagnosis?
A. Hepatic haemangioma B. HCC C. Hepatic adenoma D. Metastatic nodule E. FNH
1. If liver dis. >> hep markers & AFP if high >20ng/ml >> CECT, MRI or FNAB exclude HCC. If mass > 2cm >> RBC scintigraphy, if haemangioma conserve,
if inconclusive do CE MRI, if haemangioma/ FNH no need follow up, if difficult to diff. HCC from adenoma >>Tru cut, FNAC if inconclusive do laparoscopy
2. if no liver disease or if liver disease with mass < 2cm >>> enhanced MRI or triphasic CT then procede for biopsy or laparoscopy.
63. What is the management of that liver mass of HCC?
A. Surgical resection (if solitary, resectable < 2cm, , no hepatic vascular invasion, Child A, no portal HTN, good LFT, physical fit score 0; BCLC stage 0)
B. Chemoembolisation (TACE, for localised HCC mass not candidate for resection, transplant or ablation Child A,B, no spread)
C. Liver transplant (early-stage non-resectable HCC < 5cm, if nodules < 3 and all < 3cm if multiple tumour)
D. Chemotherapy (Sorafenib or Lenvatinib for Child Class A; or systemic chemotherapy, if not candidate for transplantation)
64. An old age patient, with cirrhosis, ascites, U/S shows mass in liver about 3 cm, to do staging for this case what’s best investigation?
A. Endoscopy C. Laparoscopy (better if add laparoscopic US, can diagnose radiologically missed lesion and take biopsy if CT is not conclusive)
D. Abdominal CT (next or initial) E. PET CT Scan (extra abdominal staging) F. MRI (for hepatic and local extension)
65. An old age patient with liver cirrhosis, ascites, liver mass 3 cm. which investigation for liver staging is initially recommended?
A. Multiphasic CECT(initial) B. Pelvic MRI C. Laparoscopic US (most sensitive for mets) D. Whole body PET scan (for metastasis)
66. What is the treatment of choice of ascites?
A. Portosystemic shunt B. Peritoneal shunt C. TIPS (For refractory ascites) D. Paracetesis (< 5 L, tense, refractory, symptomatic)
E. Conservative (mild, asymptomatic not infected no hepatorenal syndrome ) restrict salt < 5g salt/d = 2g sodium or 20-30mEq/d and spironolactone
67. What is the most common indication for liver transplant in adults? If no HCC;>> (ALD, 40% the first), (NASH, 30%) & HCV 20%
A. HCC B. Metastasis C. Fatty liver in diabetic D. Alcoholic cirrhosis with HCV (in HCC, the HCV is 40%, NASH 25%, ALD 15%)
68. What are biliary complications that are most commonly related to in liver transplantation?
A. Adequacy of hepatic venous outflow B. Adequacy of portal venous flow C. Adequacy of hepatic arterial flow
D. Length of the donor's CBD E. Anastomotic and non-anastomotic biliary strictures, leak and duct stones, mostly anastomotic
69. A patient with congestive heart failure, lower abdominal pain and vomiting, with moderate ascites, ascitic fluid examination show low
albumin, normal WBC, how to manage?
A. 2nd., generation cephalosporin B. Paracentesis (> 5L, can be/2wks, add 5g albumin/liter above 5) C. Exploratory laparotomy
70. Chronic liver disease, ascites, low albumin, high INR, high bilirubin, normal enzymes, hepatic mass 3 cm, how to determine severity of
liver disease and how to assess his condition?
A. MELD score (from 6 to 40, and is a measure of how severe a patient's liver disease is (modified; serum Na, total bilirubin, creatinine, and INR)
B. NAFLD fibrosis score (Age, hyperglycaemia, BMI, platelet count, albumin level, and AST/ALT ratio) <1.455= no fibrosis, > 0.675= advanced fibrosis
C. Child Turcotte Pugh score (ascites, bilirubin, albumin, PT/INR encephalopathy, A: 5-6, B: 7-9, C: 10-15). Survival: A = 100%; B = 80%; C = 45%.
71. A patient with history of HCV, cirrhosis and refractory ascitis then HCC develped, liver resction is planned, the hepatic surgeons
asked the interventional radiologist for preoperative portal vein embolisation, to increase future liver remnant whose first did PVE?
A. Glisson B. Gores C. Belghiti D. Kinoshita (1st. one described it) E. Makuuchi (1st. one did it)
72. A patient with liver cirrhosis and has ascites, frequent tapping in ER, presented with 4 cm liver mass, all labs within normal, CT done
and showed enhancement in arterial phase, what’s management?
A. Orthotopic Liver transplantation (HCC on top of HBV ) B. Mass resection/ Lobe hepatectomy C. Fulguration D. Chemotherapy
73. Regarding hepatic encephalopathy, which is incorrect?
A. Ammonia production can be reduced by neomycin B. Lactulose is useful for ammonia reduction
C. Protein diet should increase in management (protein will increase ammonia--> encephalopathy) D. Hypokalaemia is common imbalance
74. A female patient with multiple liver masses, she is a KCO hepatitis C, what is your further management?
A. Liver biopsy (if CT inconclusive) B. Abdominal US (initial) C. Abdominal CT (the best non invasive)
75. A male patient, CT abdomen shows hepatocellular carcinoma, what next work up investigation?
A. CT brain B. MRI abdomen and pelvis C. PET scan (to check for early cancer & extrahepatic metastasis) D. Bone scan
76. An old age male alcoholic presented with melaena for 2 days, what’s the cause?
A. Bleeding duodenal ulcer B. Bleeding gastric ulcer C. Oesophageal varices
77. Sinistral portal hypertension is most commonly due to which of the following?
A. Hypercoagulable states B. Schistosomiasis C. Alcoholism
D. Chronic pancreatitis (in 31. 8%-53. 8%), are associated with splenic V thrombosis--> left sided portal HTN)
78. A patient has no signs or symptoms of cirrhosis and all labs normal, but he was presented with haematemesis, Endoscopy done showed
gastric fundus varices, imaging enlarged spleen (no thrombus) what is the cause of haematemesis?
A. Gastritis B. Mallory Weiss syndrome C. Left portal hypertension (splenic vein thrombosis)
79. An alcoholic old male patient with liver cirrhosis, ascites, portal hypertension and variceal bleeding what is the optimal management?
A. TIPS (for portal HTN consequence, refractory ascites, refractory variceal bleeding, hepatorenal synd., not in portal V thromb, infection, tumour)
B. TIPS and transplantation C. Oesophageal sclerotherapy D. Porto caval shunt,
80. A 43 year old alcoholic patient has liver cirrhosis, ascites, oesophageal varices and rectal haemorrhoids secondary to portal hypertension,
what is the most common complication that can occurs due to the procedure that will be done as a management?
A. Recurrence of symptoms (if stent dysfunction) B. Uncontrollable bleeding (rectal) C. Arrhythmia(serious) D. Encephalopathy (30-50%)
81. 49-year-old alcoholic patient with liver cirrhosis, portal hypertension, ascites, oesophageal varices and bleeding haemorrhoids, what is
the most dreaded complication if you did haemorrhoidectomy without or before doing TIPS procedure?
A. Recurrent symptoms B. Severe uncontrolled bleeding (untreated cause) C. Incontinence D. Diverticulosis coli
82. A patient with two episodes of upper gastrointestinal bleeding, O/E abdominal distension (ascites), what is the diagnosis?
A. Peptic ulcer B. Oesophageal varices (due to cirrhosis and portal HTN) C. Dieulafoy lesion D. Duodenal ulcer
83. An alcoholic old male patient with 2 days history of melaena, what is the most likely diagnosis?
A. Bleeding duodenal ulcer B. Bleeding gastric ulcer C. Oesophageal varices (alcoholic cirrhosis)
84. What improves mortality in liver cirrhosis with oesophageal varices that is complicated with variceal bleeding?
A. Octreotide B. Prophylactic IV Ceftriaxone (decrease bacterial infection, rebleeding and death) C. Blood transfusion
85. All the following are true in management of variceal bleeding except? Fentanyl, lorazepam, oxazepam, Sevoflurane & Propofol are allowed
A. Terlipressin may be used before confirmed diagnosis B. Recurrent bleeding is prevented by surgery
C. Child C patient should undergo transplant evaluation D. Sedatives should be given to reduce anxiety (can precipitate hepatic coma)
86. A complicated cirrhotic patient came with portal hypertension, and upper GI bleeding due to varices diagnosed by upper endoscopy,
what to give after resuscitation?
A. Somatostatin B. Terlipressin (after) C. Octreotide (before) Octreotide: Before the endoscopy. Terlipressin: After resuscitation
IV octreotide IV bolus of 50 µg followed by a drip of 50 µg (reduce portal pressure). Terlipressin (alternative): 2 mg q4h IV for 24 to 48 hours, then 1 mg q4h
EBL if bleeding recurs/portal pressure> 12mmHg. TIPS: Second-line therapy if the above methods fail.
87. What’s the optimal treatment for bleeding gastric varices in chronic pancreatitis with left portal hypertension with failure of other plans?
A. Distal pancreatectomy B. Splenorenal shunt C. Porto caval shunt D. Splenectomy (or splenic a. embolisation if impossible)
E. Trans jugular intrahepatic portosystemic shunt procedure
1. If no SpVT inject thrombin or fibrin glue or cyanoacrylate-based compound for the treatment of acute gastric variceal bleeding or EVL is recommended
2. TIPS and BRTO: If acute gastric variceal bleeding is not controlled with endoscopic intervention, then TIPS or BRTO (up tp 97% success)
88. What is true regarding Somatostatin? GH inhibitor
A. It is produced by antral G cells B. It has a half-life of 30 minutes C. It increases portal blood flow
D. It inhibits gastric motility (also inhibits endocrine, exocrine, pancreatic, and pituitary secretions modify memory formation in the CNS)
89. Which of the following is a known action of Secretin?
A. Stimulates gastrin secretion B. Stimulates pancreatic enzyme secretion C. Inhibits intestinal motility
D. Stimulates gastric acid secretion
Secretin: regulate gastric acid, pancreatic bicarbonate, and has osmoregulation, it stimulate bile and bicarbonate secretion in the duodenum, pancreatic
and biliary ducts as well gastric pepsin secretion. Also, secretin inhibits gastric acid secretion, upper small intestinal motility and LOS pressure.
90. Which of the following inhibits intestinal motility?
A. Serotonin (increase gut motility) B. Cholecystokinin (increase gut motility) C. Epinephrine (inhibit gastric motility)
D. Motilin (increase gut motility) E. Secretin
91. What is true regarding the Cholecystokinin; CCK?
A. Stimulates the sphincter of Oddi B. It is secreted by the antral mucosa C. Stimulates gastric emptying
D. Stimulates pancreatic enzyme secretion
92. For a patient with Gastric bleeding after epinephrine injected, rebleeding 24h, what is the best treatment option? Endoscopic reinjection
93. A patient with upper GI severe bleeding, endoscopy done, after resuscitation failed, patient received 2 blood units, what’s next step?
A. Capsule endoscopy B. Another endoscopy and injection C. Laparotomy& ligation D. Tagged RBC
94. A patient presented with haematemesis, during OGD, there was an ulcer with visible clot at its base at night, and the patient developed
haematemesis again, came to ER with hypotension BP 80/60, what’s plan of management after resuscitation?
A. Ligate and suturing artery B. Endoscopy and reinjection C. Start octreotide D. Start Antacids.
95. A patient with history of PUD, came to ER with haematemesis, UGI endoscopy showed an ulcer in the posterior wall of the first part of
the duodenum with bleeding vessel and haematoma in the floor, bleeding stopped, but 2 days later haematemesis recurred, what to do?
A. Repeat endoscopy B. Suture (ligating) the vessel C. Octreotide D. Surgical exploration
96. A patient with bleeding duodenal ulcer, injection of adrenaline by endoscopy after 12 hours if rebleeding what to do?
A. Surgery B. Adrenaline (for 2 trials then Surgery/TAE) C. Cautery D. Rubber band ligation
97. A patient presented by massive haematemesis, BP 90/40, Hb ↓, what is the best management?
A. Endoscopy & cauterisation B. Explore, duodenotomy and oversewing of bleeder
98. A middle aged patient with haematemesis due to alcoholic cirrhosis, OGD that showed grade 2 varices, what is the best management?
A. Sclerotherapy B. Devascularisation C. Total shunt D. Endoscopic variceal ligation (better with vasoactive octreotide)
99. Scenario of patient undergoes an upper endoscopy that revealed grade 2 varices what is the best treatment?
A. Sclerotherapy B. Devascularisation C. Rubber band ligation (if combined with octreotide in VH2 will be superior to either alone)

BILIARY SYSTEM
1. What is the least variation in intrahepatic biliary ductal system?
A. 4 B. 3 C. 5 D. 8
The branching pattern of intrahepatic ducts was atypical in 37% of cases. The two most common variations were drainage of the right posterior sectoral
duct; RPSD into the left hepatic duct; LHD (11%) and triple confluence of the right anterior sectoral duct; RASD, RPSD and LHD (10%)
2. What is the second most common site that CBD opens into gastrointestinal tract? Never open in first part and in 3rd part about 1.5%. Renard
A. 1st. part of the duodenum B. 2nd. part of duodenum (the first most common in 75%) C. 3rd. part D. Inferior duodenal flexure 19.5%)
3. Regarding the the common bile ducts what is correct?
A. Supraduodenal is most vascularised B. Retropancreatic, and hilar excellant blood supply C. CBD supplied by LHA
5. Bilirubin is characterised by which of the following?
A. Most of direct bilirubin is a result of haemolysis B. Indirect bilirubin is a result of biliary tree obstruction
C. Most of bilirubin in serum is indirect and carried by albumin D. Unconjugated bilirubin is direct and water soluble
6. What is the most accurate and reliable investigation in obstructive jaundice?
A. Direct bilirubin (if no available GGT/ALP) B. Alkaline phosphatase C. GGT (the most specific) D. AST D. ALT
7. What is the most common cause of biliary colic?
A. Choledocholithiasis B. Gallbladder stone impaction in cystic or CBD C. Gallbladder sludge D. Biliary dyskinesia
8. A patient with history of pancreatitis months ago, now referred to surgery clinic for mild abdominal pain what’s initial investigation?
A. US (initially to R/O gallstones, pseudocyst) B. CT (the best for ch. pancreatitis) C. Endoscopy D. ERCP
9. An obese patient, DM, Sickle Cell Disease, what type of GB stones he might has?
A. Cholesterol (Obesity, pregnancy) B. Pigmented (for only haemolytis without DM) C. Mixed (due to DM & haemolysis)
10. KUB was done and incidentally gallstones were found, what is the next step?
A. Lap chole. B. Arrange for ERCP C. US (to see if stones, size, inflammation, CBD) D. Laparotomy E. Cholecystectomy
11. A 43 y.o lady with history of epigastric pain on PPI as OTC not regular during investigation US was done and showed gallbladder
filling defect like stone about 2-3 mm in diameter, what is the best option among the following?
A. Laparoscopic cholecystectomy B. Conservative with PPI& Rowachol C. Repeat the ultrasound after fasting 8 hours
12. What is the most accurate investigation for gall bladder stone?
A. US (best for GB and CBD diameter) B. CT (for CBD stones) C. HIDA (if US inconclusive and still symptomatic)
13. A middle-aged female patient, with RUQ pain for 12 hours vomiting, US, showed non-thickened G.B or thin-walled distended gall
bladder, with multiple gallstones, no fever or leucocytosis, CBD is obscured but no jaundice what is most likely diagnosis?
A. Pancreatitis B. GB Empyema C. Acute cholecystitis D. GB Mucocele E. Liver abscess F. GB cancer
14. A 42 y.o obese lady with long history of fatty dyspepsia, presented to ER before 4 weeks with biliary pain that was relieved by IVF and
painkillers, today came again but with with sudden RUQ pain, jaundice, no fever or leucocytosis what’s the optimal diagnostic test?
A. US (to check CBD diameter and liver parenchyma) B. Biliary system Ultra sonogram C. ERCP D. CT
15. An obese fertile female patient, with history of jaundice for 1 week with anorexia and abdominal pain, examination showed right
upper quadrant tenderness, no history of medication or diseases in the past, what is initial step?
A. MRCP B. Abdominal US C. CT D. Biopsy
16. Asymptomatic gallbladder polyp 4mm what is the optimal management?
A. Expectant treatment (except if sessile) B. Lap. Chole.(if > 1cm, in >60 y age, and if associated gallstones) C. Extended cholecystectomy
17. Healthy patient done a routine checkup, imaging revealed a polyp in the gall bladder measuring 0.6 cm, what's next step?
A. ERCP B. Surgical consultant C. Liver function D. Urgent cholecystectomy
18. A 55 y.o male patient, with recurrent attacks of UQ pain, US done, and found no stones but 0.7 mm polyp in the fundus of gall
bladder, what’s management? > 2cm usually malignant, 1-2 cm possibly malignant
A. Conservative treatment (for non-symptomatic polyp) B. Lap cholecystectomy (as symptomatic even 0.5 cm is indication) C. Open chol.
D. Extended open cholecystectomy E. Expectant treatment till reach 1.5 cm
19. A patient with right RUQ pain referred to the Rt. shoulder with dyspepsia, T38.5, U/S shows thickened GB wall, what’s diagnosis?
A. Acute cholecystitis B. Acute pancreatitis C. Peptic ulcer D. Gallbladder empyema
20. A 43 y.o lady no history of past medical diseases apart from she was obese and rapidly lost weight, recently she has history of severe
RUQ pain 3 days ago that relieved by painkillers and IVF again today she visited the ER with severe RHQ pain fever 38.8, normal liver
enzymes, total bilirubin 9, direct 5, indirect 4.5, and amylase 100 with WBC 18.000, she was admitted to the ward by diagnosis as?
A. Acute pancreatitis B. Acute cholecystitis (grade II,WBC >18, tender palpable mass >72h) C. Acute hepatitis D. Appendicitis
21. What is true regarding to acute cholecystitis?
A. 75% associated with stone B. Positive with organism (E. Coli) C. HIDA scan normal even with acute cholecystitis
22. What is the most common organism isolated from bile?
A. Escherichia coli B. Klebsiella C. Staphylococcus aureus D. Bacteroides E. Proteus
23. A pregnant lady everything was normal, but she complains of itching and had elevated liver enzyme, and ALP, what is the diagnosis?
A. Viral hepatitis B. Pregnancy cholestasis (causing obstructive jaundice) C. Cholecystitis D. Cirrhotic liver
24. What is true regarding the physiological changes in pregnancy?
A. Increased gallbladder contractility B. Elevated ALP (may rise tripple normal, marker of placental dysfunct, eclampsia, acute fatty liver)
D. Direct hyperbilirubinemia C. Elevated alanine aminotransferase (AST)/ aspartate aminotransferase (ALT) E. Indirect bilirubinemia
25. What is correct regarding gallstones diagnosed during pregnancy?
A. May resolve spontaneously after delivery B. Are the most common cause of jaundice during pregnancy
C. Are seldom the cause of recurrent acute cholecystitis during the same pregnancy D. Are most commonly pigment stones
26. When cholecystectomy can be done in a pregnant? 2nd Trimester, when laparoscopic cholecystectomy is indicated, it should be performed
either in the second trimester or early in the third
26. A case with history of gallstone disease in the US, signs of chronic cholecystitis. GB image what do you suspect wall thickness to be?
A. Thick wall (fibrotic) B. Dilated cystic duct C. Thin walled D. Normal thickness
27. Same picture, patient with acute cholecystitis that was initially managed conservatively, medical treatment failed and urgent lap chole
was done, dense omental adhesions to the gall bladder were adhesiolysed, what indicates chronicity?
A. Adhesions B. Thick oedematous wall (Acute) C. Sub serosal haemorrhage
D. Thickened, fibrotic wall (serosal surface adherent to surrounding with cholelithiasis, no pericholecystic fluid and +ve Murphy’s sign)
28. A 70 y old patient, with history of gallstones, presented to ER with picture of small bowel obstruction, x-ray showed, a stone in the
terminal ileum, pneumobilia, with US; no stones inside the GB, what to do?
A. Enterolithotomy B. Enterostomy and cholecystectomy C. Crushing of the stone.
29. What is the best treatment option in case of gallstone ileus? If patient stable and fit: Enterolithotomy, Cholecystectomy & fistula closure
30. An old frail patient with gallstone ileus; bowel obstruction, x-ray pneumobilia, what’s treatment? Enterolithotomy with stone removal)
Enterolithotomy remains the mainstay of operative treatment especially for old, with comorbidities that cannot tolerate lengthy operation.
A one-stage cholecystectomy and repair of fistula is justified only in selected patients in good general condition, no comorbidities and adequately
stabilised preoperatively. Specific criteria for a one-stage procedure remain to be established.
A two-stage surgery is an option for patients with persistent symptomatology after enterolithotomy surgery. Large prospective studies of laparoscopic and
endoscopic-guided procedures are expected.
31. What is the most common biliary-enteric fistula connection?
A. Gallbladder and stomach (0-13.3) B. Gallbladder and duodenum (2nd. part, 32- 96.5%) C. Common bile duct and jejunum
D. Gallbladder and jejunum (up to 2%) E. Common bile duct and ileum F. Gallbladder and transverse colon (0-11%)
Most frequent fistula occurs between the Gallbladder and the duodenum (32.5-96.5%), then CBD and duodenum, Choledochoduodenal (0- 13.4%),
Cholecystogastric (0- 13.3%), Cholecystocolic (0- 10.9%) Cholecysto jejunal and ileal each up to 2.5%.
32. What is the most common site of obstruction in gall stone ileus?
A. Terminal ilium (diameter 18.7 mm) B. Duodenum (24.8mm) C. Jejunum (24.5 mm) D. Colonic
Most frequent fistula occurs between the gallbladder and the duodenum. The mean diameter of the duodenum to be 24.8mm, jejunum to be 24.5mm,
proximal ileum to be 19.5mm, distal ileum to be 18.9 mm and terminal ileum to be 18.7 mm.
33. A 39 year old lady; known case of gallstones for 15 years presented with abdominal pain, fatigue and there was air in biliary system,
no fever or jaundice, what is the diagnosis?
A. Gallstone ileus (Cholecystoduodenal fistula) B. Acute Cholecystitis C. Acute pancreatitis D. Cholangitis
34. A 40-y. o lady with fatty dyspepsia or RUQ, US showed thick-walled gallbladder with no gallstones, what’s treatment plan?
A. Laparoscopic cholecystectomy B. Conservative treatment C. Cholecystostomy
35. Intraoperative picture during laparoscopic cholecystectomy, inability to define the anatomy of Calot's triangle, next?
A. Convert to open B. Fundus first dissection C. Do cholecystectomy D. Intraoperative cholangiogram
36. An 85 yr. old female with symptoms of a calcular cholecystitis by US no stones, thick walled GB, with pericholecystic collection high
WBCS, no comorbidities fit for GA, how to manage?
A. Lap cholecystectomy(old age, controlled DM not contraindication) B. Open chole C. Cholecystostomy D. Medical treatment
37. During lap Chole, picture of haemorrhagic GB, with adhesions, and, normal anatomy was lost, what will you do?
A. Partial cholecystectomy B. Open cholecystectomy C. Continue laparoscopic cholecystectomy
38. What is the single most common abdominal operation for elderly people?
A. Adhesiolysis for small bowel obstruction B. Closure of perforated duodenal ulcer
C. Resection of colon cancer D. Cholecystectomy E. Appendectomy
39. A 65-y/o diabetic male presents with RUQ pain, nausea, and vomiting. T 39°C, P 115, BP 88/60 mmHg; Spo2 is 92% on 2-L nasal
cannula. OE; markedly positive Murphy’s sign. WBC; is 19,000/mm3 and a normal total bilirubin. US shows a gallbladder wall of 7 mm,
with some air in its wall, pericholecystic fluid, gallstones, and a CBD 4 mm. She was admitted ICU, IV fluids and vasopressors are started,
and IV antibiotics are administered, which is the most appropriate surgical management?
A. Percutaneous cholecystostomy B. Immediate cholecystectomy C. Just give IV antibiotics for one week
D. Cholecystostomy then cholecystectomy in 3 months (emphysematous grade 3 cholecystitis hypotension, altered mental, and renal finctions)
40. A case of severe acute calcular cholecystitis in diabetic patient where perforation occurred, and patient was presented with biliary
peritonitis with high fever, WBC 20000, blood sugar is 11 mmol/L, what is the most common site of gallbladder perforation?
A. Body B. Neck C. Hartman’s pouch D. Fundus (the most common due to poor blood supply mostly subacute with abscess)
41. What is the best investigation for perforated gall bladder?
A. ERCP B. MRCP C. US D. Abdomen and pelvic CT
42. A 43 y.o female with symptomatic gallstone disease, no history of medical diseases, presented with acute cholecystitis for early lap
chole, during laparoscopy due to inflammation and adhesions, some bleeding occurred, how much HB. threshold for blood transfusion?
A. 5-6 gm/dl B. 7-8 gm/dl C. 8-10 gm/dl D. 10-12 gm/dl E. More than 12 gm/dl
Perioperative red blood cell transfusion criteria 1) less than 6gm/dl probably will benefits 2) from 6-8gm/dl transfusion unlikely to be of beneft in the
absence of bleeding or impending surgery 3) more than 8gm/dl no indication for transfusion in the absence of other risk factors (Bailey’s 2023).
In postoperative surgical patients, transfusion should be considered at haemoglobin concentration of 8 g/dl and with clinical symptoms (chest pain,
orthostatic hypotension, or tachycardia unresponsive to fluid resuscitation). The threshold of blood transfusion should be a haemoglobin 7gm/dl in adults
and most children strong recommendation A.
43. Asymptomatic gallbladder stone and patient has history of enteric fever, best treatment?
A. Cholecystectomy (to eradicate typhoid reservoir as a septic focus) B. Conservative C. Anti-typhoid treatment
44. A frail old aged male patient in ICU he is diabetic and ischaemic heart disease presented with gallbladder, Mucocele with severe pain
or GB empyema or acute severe a calculous cholecystitis, he is unfit for surgery and GA, what is the best management?
A. Cholecystostomy tube or per cutaneous cholecystostomy B. Lap chole C. Open chole D. Interval cholecystectomy
Complications of percutaneous cholecystostomy include the following: Bile peritonitis, Haemobilia, GB perforation, Vagal effect due to catheter placement.
45. A patient who had an episode of Calculus cholecystitis recovered from the attack now came to hospital and asked about what’s next?
A. ERCP B. Delayed Cholecystectomy (interval lap chole. after 6-8 weeks from pain onset) C. Early lap chole (first 72 hr up to 7 days)
46. A post burn patient presented with RUQ pain dilated gallbladder thick wall and peri cystic fluid what is the management?
A. Cholecystostomy tube (If critically ill burn patient with fever and high LFT) B. Cholecystectomy (if fit for surgery & GA) C. Antibiotics
47. A 44 yrs. old male patient post lap. Chole. 4 days ago, has right upper quadrant pain presented in ER with fever, U/S reveals bile
collection in gallbladder bed and hepatorenal pouch, MRCP shows cystic duct stump leakage, what is the definitive management?
A. ERCP with stent B. Reoperation and stump ligation C. Surgical drainage D. US guided aspiration
D. Admission and IV antibiotics
48. A difficult laparoscopic cholecystectomy, converted to open cholecystectomy, cystic duct leak, drain output increased from 100 mL in
the first postoperative day to 600 mL after few days, management? ERCP and stent if reverse decreased and patient is stable just expectant ttt
49. A 40-y. o female with chronic calcular cholecystitis; CCC, preoperative US showing pericholecystic fluid, thickening of gall bladder
wall, during laparoscopic Cholecystectomy the GB anatomy lost, what’s the most appropriate next step?
A. Continue laparoscopic cholecystectomy B. Partial lap chole (fundus first) C. Open complete chole D. Open partial chole.
50. A 90 yrs. old lady, DM, critically ill with multiple comorbidities not fit for GA with acute cholecystitis, what’s management?
A. Open cholecystectomy B. Medical treatment C. Lap. Cholecystectomy (if no comorbidities & if fit) D. Cholecystostomy
51. A 50 y.o lady, diabetic, not controlling her blood sugar, presented to ER with RUQ pain, fever, sugar 9 mmol, US showed signs of
acute calcular cholecystitis with dilated CBD, only indirect bil &ALP are high, what is the optimal management?
A. ERCP if free do lap chole before discharge B. MRCP C. Do lap chole directly D. Delayed lap chole
52. What is the most common source of bile leakage for post cholecystectomy surgery?
A. Common bile duct B. Common hepatic ducts C. Cystic duct stump D. Gallbladder bed E. Porta hepatis
53. A patient underwent lap chole, POD day 2, drain show 100 cc bile, abdomen is soft and lax, no fever or leucocytosis what’s next initial?
A. Abdominal US (for amount & CBD diameter) B. MRCP (if no drain to confirm leak)
C. ERCP and long stent insertion (if daily >200cc, or less but persistant or associated with peritonnitis) D. HIDA scan (active leak or leak site)
E. Expectant; monitor the bile amount, clinical and lab status
54. Post cholecystectomy abdominal pain and distribution US confirm presences of free subhepatic bile, what to do?
A. Percutaneous aspiration B. Laparotomy C. Drainage catheter (if continuous and pyogenic)
55. During laparoscopic cholecystectomy, iatrogenic longitudinal CBD tear of about 4 cm occurred, what is optimal management?
A. Primary closure with suture B. ERCP and stent via the rent C. Clip of the rent and drain D. Closure over T- tube
56. A patient underwent laparoscopic cholecystectomy on POD 2, drain shows 100 cc bile, the abdomen is soft & lax, what’s next?
A. MRCP B. ERCP C. HIDA scan D. US if CBD stone do ERCP with stent if needed) if choice as follow up is right
57. What is the accepted amount of bile drainage via T tube or drain per day? Up to 500 ml
A. 0-100 mL B. 100-250 mL C. 200-500 mL N.B. (T Tube up to 500, ileostomy 1200 and Colostomy 500-700mL)
58. A patient underwent lap chole with post-operative day 1 you noticed bile leak in drain, what is the best treatment?
A. ERCP with stent B. Monitor the amount, labs and vital signs C. Re laparotomy D. MRCP/US
59. What is the most common cause of common bile duct stricture? Incidental injury
60. Regarding Strasberg Bismuth classification for biliary ductal injury, which type will appear normal on ERCP?
A. Type A B. Type B C. Type C (Rt. posterior sectoral) D. Type D E. Type E 2
61. An elderly male presented with painless obstructive jaundice and weight loss, diagnostic laparoscopy revealed GB masses and dilated
CBD, an image showing peritoneal seeding, which of the following is most appropriate management?
A. Lap chole B. Open chole C. ROUX en y hepaticojejunostomy(palliative)
62. During laparoscopic cholecystectomy, surgeon completely transected CBD just above the duodenum, how to manage this complication?
A. Choledochojejunostomy B. Choledochoduodenostomy C. Repair over T-tube D. Hepaticojejunostomy
63. A post difficult lap chole with obscured CBD, the surgeon struggled with dissection but CBD injury above the duodenum, how to manage?
A. Choledochoduodenostomy B. Choledochojejunostomy C. Repair over T-tube D. Hepaticojejunostomy
64. Bile found in peritoneum, diagnosed as cystic duct leakage that occurred post cholecystectomy, what’s the best management?
A. T Tube drainage B. ERCP with stent if high output C. Primary repair
65. A 41 y.o female patient known case of gallstone disease presented with jaundice and epigastric pain, increased alkaline phosphatase,
bilirubin, SGOT, SGPT, and GGT, the condition more than 7 weeks, what is the most common cause?
A. Pancreatic tumour (head cancer) B. Choledocholithiasis C. Cholangiocarcinoma D. Chronic calcular cholecystitis
66. A patient with history of Cholelithiasis, today present with abdominal pain after fatty meal, on US: multiple gallbladder stones, thick
wall, CBD 12 mm, what’s next?
A. ERCP (if jaundiced) B. MRCP (if no jaundice) C. Lap cholecystectomy D. Open cholecystectomy
67. A 44 y. o lady, six months post cholecystectomy, MRCP image show a filling defect in CBD, what’s mostly the diagnosis?
A. Missed floating CBD stone B. Hepatic duct structure C. CBD injury D. Cholangiocarcinoma
68. A middle-aged female patient with history of Lap chole, 2 years ago, came to ER with jaundice what is the most probable cause?
A. Retained stone (< 2 y= 2ry. If > 2y -> 1ry) B. CBD stricture (after 1m- 2 y if mild injury) C. Cholangiocarcinoma
D. Pancreatic head cancer
69. A patient with an uneventful laparoscopic cholecystectomy 6 months ago, presented with flactuant jaundice for 1 month with MRCP
showing dilated CBD and two filling defects, otherwise he is normal, what is diagnosis?
A. Cholangiocarcinoma B. Stricture C. Missed CBD stone (then impacted) D. Cholangiocarcinoma E. CBD worms
70. In choledocholithiasis which of the following parameters decrease?
A. Alkaline phosphatase B. Clotting factor X (it’s vitamin K-dependent, cause A, D, E, K deficiency) C. Prothrombin time D. INR
71. Right upper quadrant pain, fever, jaundice, similar attack with previous diagnosis of calcular Cholecystitis, what is the diagnosis?
A. Acute Cholecystitis B. Chronic Cholecystitis C. Acute Cholangitis (due to CBD stone obstruction)
72. A patient with upper abdominal pain with impacted CBD stone, ERCP was done but failed to remove the stone, he still in upper
abdominal pain and mild tenderness what is the cause of pain?
A. Bleeding B. Cholangitis due to CBD stone C. Duodenal perforation D. Acute on chronic cholecystitis
E. Post ERCP Pancreatitis (if recently high lipase/amylase and no pain before)
73. A female patient presented with RUQ pain, jaundice, labs, all high: Dbil, Tbil, ALP, Temp, 38.6 what’s the next step?
A. MRCP B. ERCP (cholangitis) C. Abdominal CT D. IV antibiotics and IVF
75. Gallstone disease, high fever, Rt. hypochondrial and epigastric pain, tenderness, deep jaundice, dilated CBD by US, she received
antibiotics for 3 days but with no improvement of pain or jaundice, what is the best management plan?
A. ERCP for drainage+/- stent+ IV broad spectrum antibiotics B. MRCP C. CT D. CBD exploration
76. A female patient with gallstones presented to ER with right hypochondrial pain, fever, chills, jaundice, diagnosed as cholangitis, after
IV fluids what is your management?
A. IV antibiotics and ERCP+/- stent B. Observation C. Pain killers and avoid fatty meal D. Choledochotomy
77. 10 days post Choledochotomy; no bile drained through T tube, US done, no stone then MRCP done that showed filling defect in CBD
what to do?
A. Clamp T tube 24 hrs., then remove it B. Remove T tube C. ERCP and biopsy, brush cytology +/- Biliary duct drainage
78. After Lap cholecystectomy 2-3 weeks ago, the patient came with jaundice, bile passing from port site, US done; CBD is 1.1 cm there is
an MRCP image, CBD dilated there is shadow what is best next option?
A. Observation B. ERCP with stone extraction C. Follow up US or CT D. Diagnostic laparoscopy
79. A female patient with no history of jaundice, during lap cholecystectomy, you found wide cystic duct & CBD, what you will do?
A. Cholecystectomy with US B. Per (intra)-operative cholangiography C. Open cholecystectomy & palpate CBD D. Intraoperative US
80. An intraoperative Cholangiography discovered a missed stone in CBD, how to manage?
A. Choledochoscopy with extraction B. ERCP C. MRCP D. Leave it will pass down by time
81. A 43 year old lady with history of gallstone disease for 7 years, she visited the ER with picture of calcular obstructive jaundice, where
ERCP done, but due to markedly impacted big stone at the CBD end, the endoscopist failed to extract the stone and patient still not
improved, clinically or laboratorily, with high bilirubin direct and total, LFT, ALP, what is the best option to solve his problem?
A. Do another ERCP B. Choledochojejunostomy C. Hepaticojejunostomy D. Choledochotomy with T tube drainage
82. A patient underwent CBD exploration, image of post-operative T tube cholangiogram with dilated CBD, contrast is seen in second
part of the duodenum, pancreatic duct and common channel also apparent, what is the diagnosis?
A. Choledochal cyst (as dilatation without distal obstruction or stones ? Todani’s IA) B. CBD stricture C. Long common channel
83. What is the best operative approach to choledochal cyst surgery?
A. Cystoduodenostomy B. Cystojejunostomy C. Roux-en-Y Cystojejunostomy D. Cyst excision & hepaticojejunostomy (type 1, 4)
84. Biliary tree injury, T tube inserted, sub hepatic drain removed, and T tube clamped ↑↑bilirubin 80, what’s the cause of jaundice?
A. Distal CBD stone (ERCP with stent) B. Biliary injury C. CBD injury E. Slipped cystic duct clip
85. A patient with impacted CBD stone, he underwent ERCP, sphincterotomy and stone extraction done, bleeding happened with
haematemesis once, 2 days later, patient presented with repeated attacks of haematemesis and melaena, what is the cause of rebleeding?
A. Pancreatic injury (pancreatitis) B. Duodenal leak C. Active bleeder (for OGD / TAE) D. Erosive gastritis
86. An old patient had percutaneous cholecystostomy, presented after 24 hrs., by upper GI bleeding, what’s most important investigation?
A. CT angiography (major haemobilia will do IR embolisation) if minor haemobilia do fluroscopy B. Endoscopy(initial if low suspicion)
D. Abdominal CT D. MRCP E. EUS
87. After a difficult ERCP ended by stent in CBD end for a patient with calcular obstructive jaundice due to impacted CBD stone, the
patient complained at night of right hypochondrial pain, jaundice and he said he passed coffee ground vomitus with dark stool, at day 2,
his labs were, WBC 11.3, HB before was 12.5 and now is 11.2, T bil 30, D bil is 10, AST, 50, ALT 56, Amylase 128, lipase 94 U/L he is
tachycardic but BP 110/76, OGD done where found no injury but some clotted blood in D2, what’s your diagnosis?
A. Duodenal injury B. Acute pancreatitis C. Haemobilia (bleeding, pain, jaundice Quincke's triad) D. Stress ulcer E. Bilhaemia
88. An ERCP done for impacted distal end CBD stone, bleeding from the ampulla due to difficult removal, cauterised but at night, the
patient was in acute epigastric pain, subcutaneous and chest wall emphysema, high amylase 225 and subcostal air in x-ray what is diagnosis?
A. Duodenal bleeding B. Pancreatic injury C. Duodenal perforation (pain, emphysema, gas under diaphragm) D. Impacted stone
89. A patient developed surgical emphysema, of the neck, chest and abdomen after ERCP, which organ was injured or perforated?
A. Oesophagus B. Stomach C. Pancreas D. Duodenum E. CBD
90. What is the most common cause of haemobilia? Iatrogenic trauma to the liver and biliary tree.
91. What is the first test to rule out haemobilia? Oesophagogastroduodenoscopy. OGD. If inconclusive or negative no ulcer or injury -->> CTA
92. What is the first-line therapy for haemobilia? Angiography embolisation
93. A female patient during ERCP for stone extraction, failed due to perforation, and she developed haematemesis and melaena and mild
tenderness BP 80/50, how to manage?
A. Endoscopy repeat B. Laparoscopy C. Coeliac angiography embolisation D. Laparotomy
94. Same last scenario and asked about the cause of epigastric tenderness?
A. Duodenal perforation (sudden severe pain after recovery) B. Pancreatic injury (if at night with high lipase/ amylase) C. Stone impaction
95. An elderly man progressive jaundice, deep yellow sclera, dark urine, pale stool, distended GB with stones, dilated CBD, investigation?
A. CT B. ERCP (Periampullary or pancreatic head cancer for diagnosis and biopsy) C. US D. MRCP
96. What is the management of post CBD injury stricture?
A. Choledochoduodenostomy with stent B. Choledochoduodenostomy without stent C. ERCP with stent
D. End to end anastomosis (hepatico jejunostomy)
97. An old aged male patient with picture of painless, progressive obstructive jaundice, itching, anorexia and weight loss, what is the best
modality for hepatobiliary pancreatic tumour staging?
A. Abdominal CECT(chest, abd cavity mets) B. MRCP C. EUS (diagnostic, biopsy& staging) D. ERCP(limited in staging)
98. What is the most common cause of bile duct cancer or Cholangiocarcinoma?
A. Primary Sclerosing Cholangitis; (also chronic UC, choledochal cysts, and infection with chinase liver fluke) B. Choledochal cyst
C. Stone of bile duct
99. A patient known case of ulcerative colitis underwent MRCP and showed intra and extra hepatic duct strictures, what’s the diagnosis?
A. Cirrhosis B. 1ry. biliary cholangitis (previously; PB Cirrhosis) C. Primary sclerosing cholangitis (more with UC & Cholangiocarcinoma)
100. A young aged female patient suddenly developed jaundice and fatigue, she had high ALP, and high bilirubin, splenomegaly and
hepatomegaly; US no finding, MRCP multiple foci of stricture/ dilatation of CBD which is the best initial next?
A. Liver biopsy B. Antinuclear antibody (not specific) C. Repeat US D. Colonoscopy (to detect UC that caused the PSC)
N.B 5-10 % of patients with PSC are associated with UC
101. A 40 yrs. old female patient with hypochondrial pain, U/S suggested GB neoplasm, what is the management?
A. Open cholecystectomy B. Lap chole (contraindicated as port seeding) C. Chemoradiotherapy D. Hepaticojejunostomy
102. What is the optimal management of Gallbladder with adenocarcinoma in mucosa only? T1a = invade lamina propria; part of mucosa
A. Cholecystectomy only (T1a, epithelium + lamina propria = mucosa) B. Wedge resection of liver C. Extended cholecystectomy
103. Gallbladder with squamous cell carcinoma, invading muscular layer but did not reach the serosa, what is the management?
A. Coeliac L.N biopsy B. Extended Cholecystectomy (for T1b N0 lesion= muscular layer) GB+2cm of liver +LNs C. Liver resection
104. What is the most accepted treatment option for gall bladder cancer with cancer spread beyond the serosa to the peritoneum?
A. Cholecystectomy only (Stage T1a) B. Cholecystectomy with adjuvant CTH C. Radical cholecystectomy and LN dissection (T2)
D. Resection of segment 4 quadrant (T3)
105. An elderly lady 65 y. old with complaints of right hypochondrial pain, investigations showed; no stones in distal CBD, laparoscopic
exploration was done with attached 3 images of dilated GB /peritoneal nodules) what’s management?
A. En bloc GB resection B. Cholecystectomy C. Palliative Cholecystectomy (unresectable gallbladder cancer) drainage, stents, bypass
106. A 60 y. o diabetic lady, she is KCO gallstone disease, came to ER with severe RUQ pain, anorexia, jaundice and general weakness, no
fever, after abdominal CECT found, distended not tender gallbladder, which is full of pus with peritoneal nodules what’s the optimal
management plan?
A. Cholecystojejunostomy B. En block resection of the mass C. Cholecystostomy and biopsy D. Palliative cholecystectomy
107. A 43 years old lady, KCO big sized gallstones for 17 years, with history of fatty dyspepsia, H pylori infection that is treated but
frequently recurring, there was recent history of vague abdominal pain, nausea, vomiting, indigestion that misdiagnosed as spastic colon,
but recently she has general weakness, loss of appetite, weight loss, jaundice, US distended GB, with wall calcification with hepatomegaly,
MRCP no CBD stones or filling defects but there are hepatoduodenal L.Ns with peritoneal nodules what is the most likely diagnosis?
A. Gallbladder adenocarcinoma (advanced) B. Klatskin tumour (Contracted gallbladder) C. Acute emphysematous cholecystitis
D. Pancreatic head cancer (dilated intra and extrahepatic biliary radicles with deep jaundice)
108. An elderly patient with RUQ pain, fever, nausea, O/E positive RUQ tenderness, U/S showed GB stone, dilated intra & extra hepatic
biliary tree, weight loss, malaise, fatigue and night sweat with parameters of obstructive jaundice, what is the most probably diagnosis?
A. Distal cholangiocarcinoma(both extra/ intra) B. Klatskin tumour(only intra) C. Gastric cancer D. GB malignancy
109. Cholangiocarcinoma with deep jaundice and right hypochondrial pain, how to diagnose? CECT, MRCP, MRI and EUS
110. A patient admitted with biliary pain then improved, you did abdominal US and found, multiple gallstones with dilatation of
intrahepatic bile ducts only, what you will do for him?
A. Cholecystectomy before discharge B. ERCP C. MRCP(? Klatskin can be caused by Gallstones) D. EUS E. Abd. US
111. A 60 year old patient with gallbladder stone as suggested by U/S and just dilated CBD without stones, what is the best investigation?
A. ERCP B. MRCP(if no stones R/O periampullary/ pancreatic head CA, then ERCP/EUS for biopsy) C. CT D. PTC
112. What is the most common site of biliary tract cancer or Cholangiocarcinoma or biliary carcinoma?
A. Intrahepatic biliary tree B. Confluence of biliary tree (hilum) C. Extra hepatic bile duct
D. Gallbladder E. Cystic duct F. Distal part of CBD
N.B. Confluence of biliary tract = perihilar = bifurcation of CHD = Klatskin tumour 40-60% of biliary tract cancers
113. A case diagnosed as Cholangiocarcinoma with dilatation intrahepatic biliary radicals where is tumour site?
A. Hilum (Klatskin tumour 60 to 70% of extrahepatic cancers) B. CBD end of or Ampulla of Vater (both intra and extrahepatic dilatation)
114. What is the most appropriate diagnostic investigation for diagnosis of Klatskin tumour? MRCP or alternatively; MDCT-- US is only initial
MR cholangiography is the best test for the diagnosis of the primary tumour and to evaluate biliary extension. It has a sensitivity of 86%–100%.
115. An old, aged patient with weakness, weight loss, deep jaundice, pruritus, fever and abdominal pain, in the US; intrahepatic biliary
radicle dilatation with normal CBD, what is the next step?
A. MRCP (Klatskin tumour) B. ERCP C. PTC D. CT
116. In anaerobic glycolysis in malignant cells what is the imaging modality to detect cancer cells and metastasis and follow up for treatment
response and if recurrence occurred?
A. Brain MRI B. Abdominal CT C. Sestamibi scan D. PET CT scan of liver
117. What is the optimal management plan for a case of periampullary carcinoma?
A. Chemoradiotherapy B. Hepaticojejunostomy C. Pancreatectomy D. Pylorus preserving pancreaticoduodenectomy; PPP
118. During CO2 pneumoperitoneum what will occurs?
A. The mean arterial pressure increases B. The mean arterial pressure decreases
C. The systemic vascular resistance increases D. The systemic vascular resistance decreases E. The cardiac output increases
119. What is the haemodynamic consequence of carbon dioxide pneumoperitoneum?
A. Decrease in cardiac index B. Decrease in systemic vascular resistance C. Decrease in mean arterial pressure
D. Increase in cardiac preload
120. During laparoscopy and surgeon just started the procedure, patient become hypotensive 84/50 what’s the cause?
A. Cold gas B. Increase preload C. Rapid inflation of the abdomen CO2 insufflation can lead to marked hypotension, low COP, even death
121. Hypotension develops after pneumoperitoneum and trocar placement for Lap cholecystectomy, what is the next action to be done?
A. Convert to open cholecystectomy B. Deflate the abdomen C. Give intravenous fluids
D. Place the patient in head down position E. Check for bowel injury
PANCREAS AND SPLEEN
PANCREAS
1. The highest bicarbonate concentration is found in which secretions?
A. Saliva B. Gastric secretions C. Biliary secretion D. Pancreatic secretion E. Small intestine
2. The best investigation, the most sensitive in diagnosis and management of pancreatic divisum?
A. CT B. US C. ERCP (diagnostic and therapeutic but invasive) D. MRCP (diagnostic non-invasive)
E. S MRCP (Secretin enhanced MRCP diagnostic only but the best and non-invasive)
3. What is the best diagnostic tool that can show pancreatic ducts’ anatomy and pathology?
A. EUS B. Pancreas protocol abdomen CT (the initial) C. MRI / S-MRCP (effective, safe non invasive) D. Abdominal US
4. A 32 year old male patient with chronic abdominal pain, nausea, postprandial fullness and non-bilious vomiting with history of PUD
treated with PPI, diagnosed by gastroenterologist as an annular pancreas how to manage? Child 1st. duod.duod 2nd.duo.jej. /adult duojej 2nd.Gastjej
A. Increase PPI dose B. Gastrojejunostomy in infants C. Duodenojejunostomy in adult D. Choledochoenterostomy
Upper GI series or CT, if inconclusive then MRCP, if MRCP/CT suspected periampullary malignancy>> ERCP for drainage and biopsy
5. An old age male patient, suffering from upper abdominal pain, CT: dilated distal 2/3 of pancreatic duct, MRCP shows stone at the
junction between proximal 1/3 & distal 2/3 of pancreatic duct what is the best, definitive management?
A. Distal pancreatectomy B. Duodeno pancreatectomy C. ERCP & stent (if small <5 mm. ERCP and extract. if large ESWL, ERCP extract)
6. A patient with abdominal trauma, presented with generalised abdominal pain, on examination, patient had guarding, ecchymosis in the
flanks, high amylase, CT done showing pancreatic tear but there is no duct injury, what will you do?
A. Observation B. External drainage C. Pancreatectomy D. Pancreatorrhaphy
7. What is the lab which increases initially, in case of acute pancreatitis, or what is true from the following choices?
A. Amylase (start within 1st. 12 hrs then declines in 4-8 d) B. Lipase (the best and specific remain 8-14 d) C. BUN D. Trypsin
8. A middle aged patient with severe epigastric pain for 6 hours and clinically you have doubt of having acute pancreatitis, what is the
most suitable investigation to diagnose as pancreatitis?
A. Amylase blood level B. Lipase blood level (normal 0-160 U/L or 2.67 ukat/L) C. Abdominal CT D. Abdominal US
9. Clinical case of acute pancreatitis, what is the better test for diagnosis?
A. Amylase B. Lipase(the most sensitive, specific, stay up to 14 d. 4 times active than amylase) C. ALP D. High LFT & bilirubin
1. Lipase or amylase level is three times the upper normal. 2. Abdominal pain is consistent with pancreatitis. 3. imaging is consistent with acute pancreatitis.
10. Which of the following pancreatic enzymes is secreted in an active form?
A. Lipase B. Phospholipase A C. Trypsin D. Elastase
11. A 60 y.o male who is chronic alcoholic for 20 years presented with acute sudden severe epigastric pain, for 12 hours, he has no history
of gastritis or gallstone disease, his Amylase, Lipase, WBCs, CRP and LDH all are high, what is the best imaging investigation of choice?
A. CECT (the best for acute pancreatitis, presence & extent of pancreatic necrosis) B. MRCP C. Abdominal US D. ERCP
12. A 30 y.o lady with history of gallstones presented with epigastric pain for 6 days radiating to back, and vomiting, on examination,
diffuse distension and epigastric tenderness, labs; Lipase and Amylase were high, CXR showed left sided pleural effusion, what is the
investigation of choice for her?
A. Abdominal CECT (72 hr passed So, R/O acute severe pancreatitis) B. US C. Analysis of the fluid D. ERCP
13. A 65 year old diabetic female patient, on oral hypoglycaemic drugs, presented with severe upper abdominal pain, increased amylase
and lipase (acute pancreatitis), what is the most likely cause?
A. Drug induced B. Fever C. Alcohol D. Gallstones (DM increase biliary disease two folds increase cholesterol &triglyceride,)
14. A Saudi lady citizen admitted to the hospital with a picture of acute biliary pancreatitis, she has fatty dyspepsia for long time and taking
only PPI, what is the most common cause of pancreatitis in Saudia arabia and gulf region?
A. Alcoholic B. Gall stone disease (due to obesity& bariatric surgery) C. High cholesterol D. Haemolytic diseases
15. What is the pathophysiology of acute pancreatitis with GB stone or biliary pancreatitis?
A. Bile reflux B. Bile retention
C. Proenzyme activation & secretion (oedema increases pancreatic duct pressure prematurely activates trypsinogen to trypsin>> autodigestion & inflam.)
16. A middle aged male, involved in RTA, got severe trauma with abdominal bruises around umbilicus, CT haematoma around 2nd. part
of the duodenum, amylase 600 total, direct bilirubin is high indirect shows mild elevation also, BP is 100/60, what is the diagnosis?
A. Biliary injury B. Pancreatic injury > Traumatic pancreatitis C. Duodenal injury D. Duodenal obstruction
17. While doing ERCP for missed stone, bleeding at the ampulla of Vater 2 nd. day post procedure, the patient presented with acute
epigastric pain, with high lipase and amylase what is the cause of pain?
A. Duodenal bleeding B. Pancreatic injury >> Pancreatitis C. Duodenal injury D. Missed stone
The most common causes of post-ERCP pain are acute pancreatitis and duodenal perforation
18. A 39 y.o, female patient, KCO gallstone disease, C/O abdominal pain, amylase 450 and WBC is 35000, what is most likely diagnosis?
A. Acute necrotising pancreatitis B. Diverticular abscess C. Acute Appendicitis D. Pelvic inflammatory disease
Clostridium difficile infection is common in necrotising pancreatitis and negatively impacts morbidity and disease recovery. Predictors of mortality include
age > 70 years, severe leukocytosis or leucopenia (white blood cell count, ≥ 35,000/μL or < 4000/μL) or bandaemia (neutrophil bands, ≥ 10%), cardiorespiratory
failure, thrombocytopaenia (platelet count < 150 × 100/mm3), coagulopathy (international normalised ratio > 2.0), and renal insufficiency (BUN > 40 mg/dL)
19. What does Ranson’s criteria score mean? 11 parameters in alcoholic all less than that for biliary (10 parameters) either on admision or at 48 hr
A. Severe pancreatitis B. Mortality rate < 5%.
Biliary P: age >70, WBC >18, glucose >220, AST >250, LDH >400, Ca.<2mmol, HCT fall >10, BUN rise>2mg/dl Fl. Seq >4L BD >5mEq/L (OLDER WISE)
Alcoholic age >55, WBC >16, glucose >200, AST>250, LDH >350, Ca.<2mmol, HCT fall>10, BUN rise >5mg/dl BD > 4 mEq/L Fl. Seq>6L) PaO2 < 60 mmHg
Score interpretation 0 to 2 points(mild): Mortality 0% to 3%. 3 to 4 points(moderate): 15% 5 to 6 points(moderately severe): 40% 7 to 11: nearly 100%
20. What is the degree of a case with acute pancreatitis: age (55 years), hypoxia <60 (low), amylase (> 2000) diagnosis? Severe pancreatitis
21. A 60 y.o man presented with acute pancreatitis, clinical and laboratory data of Ranson score are given: low pO2, WBCs 21.000, ALT,
ALP are high, LDH 450, AST 244, glucose 11 mmol/L (normal up to 5.5), very high amylase, what’s diagnosis?
A. Severe acute pancreatitis ?alcoholic, mort. >50% B. High amylase indicates worse prognosis C. Mortality rate < 5% D. For ERCP
22. What is the option that prevents and decreases the incidence of infection or necrosis of in acute pancreatitis?
A. Enteral feeding B. Antibiotic C. Puncture aspiration D. Necroscopy
Treatment consists of aggressive intravenous fluid resuscitation, pain control, and institution of enteral nutrition as early as possible
23. An old patient, with DM, gallstones, high triglyceride, necrotising pancreatitis, peritoneal septic fluid +ve E coli, what is best next?
A. Exploration, debridement B. Antibiotic C. Percutaneous drainage with IV antibiotics according to C/S
24. A 35 year old lady admitted with acute pancreatitis and treated conservative then failed medical treatment, what is the most sure
indication of surgery?
A. Pancreatic abscess B. Pancreatic necrosis (worse > abscess) C. Pancreatic necrosis with abscess formation D. High fever & WBC
The mortality rate associated with pancreatitis 20%, pancreatic abscess is generally less than infected necrosis, largely related to sepsis and multiorgan failure
25. Which of the following is true regarding necrotising pancreatitis?
A. Increased lipolysis B. Hypoglycaemia C. Decreased gluconeogenesis
26. An ICU lady with necrotising biliary pancreatitis & superinfection with pneumococci as seen in her C/S what’s the useful follow up test?
A. Urine culture for her B. Serum creatinine (sign of organ failure; renal) C. ECG
27. A patient with necrotising pancreatitis, has severe epigastric pain, admitted to the ICU, developed hypoxia and suddenly seen by ICU
doctor with anxiety and irritation BP 100/65, P 130, RR 36, WBC 15000, what is the cause of that problem?
A. Pancreatic abscess B. ARDS; (one 3rd. of severe P. develop ARDS --> 60% death in 1st. wk) C. Atypical pneumonia D. Septic shock
28. Which of the following is included in Ranson criteria? Age, WBC, HCT, Glucose, AST, LDH, Ca, BUN, PaO2
A. WBCs B. ALT C. Amylase D. Lipase
29. How much mortality rate in a case of severe acute pancreatitis? More than 50%-100% if points more than 7-11 criteria. 5-6 points = 40%
30. Which of the following has the most worse prognostic value in acute pancreatitis?
A. High Amylase B. High Lipase C. High AST D. Low PO2 (Hypoxia after ARDS, means organ (respiratory) failure >death)
31. How much the IV fluids maintenance for middle aged female with no comorbidities diagnosed with severe acute pancreatitis?
A. 1ml/kg/h (if mild pancreatitis, stable as a maintenance but if moderate or severe 1.5ml/k/h) B. 5ml/kg/h C.10ml/kg/h
D. 5-10ml/kg/h (in severe pancreatitis in haemodynamically stable patient first day fluids 5-10 ml/kg/h or 250-500/h with > 3rd. to be given in 12 to 24/h
If unstable 500-1000 mL in 10-30 min., then 10-20ml/kg/h can be decreased according to his status)
32. What is the best fluid replacement and resuscitation in haemorrhagic pancreatitis with BP 85/50, P 120, oliguria, although of IVF?
A. Ringer B. Saline C. Blood (if unstable and shocked) D. Albumin
33. A 90 kg patient with acute haemorrhagic pancreatitis in the ICU, urine output 30 mL/h, shocked, what is the best fluid resuscitation?
A. Ringer lactate (if no haemorrhagic shock not if hypercalcaemic as RL =3mEq/L of Ca) B. NaCl 0.9% C. DNS D. Blood
34. An old age alcoholic male patient, came to ER with severe epigastric pain, referred to the back, given pain killers and PPI then
abdominal CT and blood workups done that showed, pancreatic calcification, duct dilation, with very high Amylase, what’s the diagnosis?
A. Chronic pancreatitis B. Acute on top of chronic pancreatitis C. Necrotising pancreatitis
35. A case with acute pancreatitis, admitted to ICU, increased creatinine BP. 80/30 diagnosis? (Septic shock) organ dysfunction (renal)
36. Another patient with infected pancreatic necrosis also was in ICU and after 3 days he looked violet colour, agitated and confused, what
is the cause of these changes? Septic shock complicated with ARDS +/- respiratory failure violet looks as cyanotic due to hypoxaemia
37. A patient presented with severe epigastric pain radiating to the back, labs are acceptable, he has past history of repeated epigastric
pain, 2 months ago. In social history he is long time drinking alcohol. what's the most likely diagnosis?
A. MI B. Perforated chronic peptic ulcer C. Chronic pancreatitis (alcohol induced)
38. What is the most important investigation in the case of chronic pancreatitis?
A. S-MRCP B. CECT (better in acute P sp. 98-100%) C. ERCP (Invasive, expensive, induce Cholangitis/pancreatitis) D. MRI/MRCP
39. An alcoholic, acute pancreatitis 3 months ago, there is still epigastric pain, on follow up there is epigastric cyst 11 x 15 cm, management?
A. Internal drainage (if sterile) B. Conservative treatment (if asymptomatic < 4cm in tail) C. External drainage (if infected, purulent)
40. Post pancreaticoduodenectomy for traumatic injury of the pancreas and duodenum, drain output is frothy secretions; capillary refill is
4 sec., what is the cause of shock?
A. Dilatation of peripheral blood vessels >> Septic shock B. Decrease GFR C. Decrease peripheral vascular resistance
41. What is the most common cause of massive bleeding in chronic pancreatitis?
A. Pseudo aneurysm (up to 60%) or pseudocyst rupture 6-30% & erosion into vessel) B. Mycotic aneurysm C. AV fistula D.Splenic VT
42. Which of the following abscesses, even if small size requires surgical drainage?
A. Amoebic liver abscess B. Peri diverticular abscess C. Appendiceal Abscess D. Pancreatic abscess (infected pseudocyst)
43. What is the most common cause of chronic pancreatitis?
A. Acute pancreatitis B. Alcoholism (60% of all chronic Pancreatitis) C. Hyperlipidaemia D. Gallstones (for acute pancreatitis)
44. A 36-y. o alcoholic man, presented by severe epigastric pain, jaundice, 3 attacks of black stool and fluid thrill what is correct diagnosis?
A. Chronic DU B. Cancer stomach C. Chronic pancreatitis with pseudocyst
45. Right UQ pain, recurrent steatorrhoea, diabetic, alcoholic, with image of CBD and multiple pancreatic duct dilatations, what’s diagnosis?
A. CBD stone B. Pancreatic carcinoma C. Chronic pancreatitis D. Pancreatic cyst
46. What is the most important investigation in the case of chronic pancreatitis? MRCP or S MRCP >> the best noninvasive sensitive imaging
47. What is the best management of pancreatico pleural fistula with duct disruption and stenosis? MRCP is the diagnostic modality of choice
A. Distal pancreatectomy B. Pancreaticojejunostomy C. Cystogastrostomy D. Thoracotomy and decortication
E. Tube thoracostomy & octreotide (if normal duct no stricture) F. ERCP with pancreatic duct stent (head, body ductal disruption & stricture)
G. Distal pancreatectomy & pancreaticojejunostomy (if complete pancreatic duct obstruction & disruption in the pancreatic tail and after failure of ERCP)
48. A 60 y.o male patient, who is chronic alcoholic for 25 years, presented to ER with vague mass palpable in the epigastric region
associated with weight loss, he has history of two months epigastric pain after a severe attack that time which improved after hospital
based management, now he is stable no fever or leucocytosis, what is the best optimal and sensitive imaging test for his present condition?
A. Abdominal CECT (in acute fluid collections & Acute pseudocyst) B. ERCP (not used) C. EUS D. Abdominal US
E. Contrast enhanced MRI/ MRCP (as it is chronic pseudo cyst with debris)
49. A patient had pancreatitis 6 weeks ago, now complain of epigastric pain and tenderness, a mass was felt in the epigastrium and on US
revealed mass with thick wall and fluid, WBC high, what is the diagnosis?
A. Pancreatic pseudocyst (post chronic pancreatitis) B. Pancreatic Abscess(infected WON) C. Walled off necrosis D. Pancreatic CA
1. Before pancreatic necrosis = not infected cysts:
< 4 wks.: Acute fluid collection 4-6 wks. + Homogenous fluid: Acute pseudocyst. > 6wks: Chronic pseudocyst
2. After the pancreatic necrosis = infected:
< 4w =Acute necrotic collection > 4w = Wall-Off necrosis
Management Non-infected pseudocyst >>> Internal drainage
Infected pseudocyst: External drainage + Antibiotics
Acute fluid collection: Observation
Wall off necrosis: percutaneous drainage
50. A case of symptomatic pancreatic pseudocyst, 18 cm x 24 cm with history of acute pancreatitis before 6 weeks, how to manage?
A. Percutaneous drainage B. Endoscopic drainage C. Surgical drainage (drainage if symptomatic, rapid enalrgement, systemic illness)
51. An image given for a large pancreatic pseudocyst that is close to stomach with compression symptoms, what’s the symptomatic ttt?
A. Observation(if AFC < 4 wk) B. External drainage (if infected) C. Internal drainage (cystogastrostomy) D. Surgical drainage
52. A case of pancreatitis 5 weeks ago, now she has epigastric tenderness and cannot tolerate food with vomiting each time and fever. By
US, you found large cyst about 12x10 cm mass with thick wall and fluid inside with debris, which is heterogenous and not liquefied. Labs:
Amylase 346, WBC 15000, what is the diagnosis?
A. Pancreatic Pseudocyst(no high fever or WBC) B. Pancreatic Abscess(WBC is higher) C. Infected walled-off pancreatic necrosis
53. What is the most common indication for surgery in chronic pancreatitis?
A. Jaundice B. Persistent pain C. Pseudocyst D. Gastric outlet obstruction E. Endocrine deficiency
F. if > 6 cm size G. After 12weeks
54. What is the most common symptoms or common presentation associated with chronic pancreatitis?
A. Weight loss B. Pain C. Fever (it is sign but 76%) D. Tachycardia(sign 65%) E. Vomiting F. Hypocalcaemia
55. Scenario of pancreatectomy, post-operative, he became febrile, hypotensive, what is the cause of hypotension?
A. Increase capillary permeability (early due to hypovolaemia or late due to sepsis) B. Decrease GFR
56. An alcoholic man, with chronic pancreatitis, abdominal pain, weight loss, diarrhoea, by CT multiple strictures and dilatations of the
pancreatic duct, multiple stones in the distal pancreatic duct found, what the optimal management?
A. Distal pancreatectomy B. Lateral pancreatojejunostomy (modified Puestow procedure)
C. Total pancreatectomy D. Pancreaticoduodenectomy E. Distal pancreatectomy & jejunostomy
57. What is the appropriate management of chronic pancreatitis with pancreatic ductectasia?
A. Pancreaticoduodenectomy B. Distal Pancreaticojejunostomy C. Longitudinal pancreaticojejunostomy
D. Near total pancreatectomy E. Sphincteroplasty
58. An alcoholic patient with multiple stones in distal part of pancreas, and there is chain of lakes in main pancreatic duct?
A. Distal pancreatectomy B. Longitudinal pancreatico-jejunostomy C. External drainage
59. A 61-year-old smoker presented in clinic with jaundice, abdominal pain and weight loss, on examination, fluid was heard with shaking
the abdomen and palpable GB and non-tender, what is the gold standard investigation? Fluid with shaking = pancreatic head CA caused GOO
A. US (initial if > 3cm >95%) B. Abdomen & pelvis helical CECT pancreatic protocol C. ERCP (if obstructive put stent not before CT)
D. Biopsy (if CT diagnosis in doubt and resectability is uncertain therapeutic intervention is needed)
60. A case of patient with painless right UQ swelling, distended GB, intra / extra hepatic dilated, weight loss with +ve Courvoisier sign,
diagnosis? Cancer of the head of pancreas
61. A male patient 50 y old, alcoholic, presented with abdominal pain, weight loss, palpable gallbladder scenario suggestive of malignancy.
Liver enzymes acceptable except for significantly high ALP, what is most likely diagnosis?
A. Periampullary tumour/ Pancreatic head cancer (abd pain 80%, jaundice 56%, wt loss 80%) B. Hepatocellular CA C. CBD stone
62. For the previous case of malignant obstructive jaundice due to pancreatic head what’s the best investigation for diagnosis?
A. CECT pancreatic protocol (diagnostic >2cm & staging) B. MRI (best diagnostic than CT) C. PET CT D. EUS+FNA (definitive)
63. A 64-year-old patient with progressive jaundice, weight loss and vague abdominal pain. Upon examination gall bladder was palpable
what’s the most likely diagnosis and staging?
A. Periampullary cancer/ distal cholangiocarcinoma /pancreatic head cancer (EUS+FNA) B. Klatskin tumour C. Colon cancer
64. A case of Periampullary tumour, what is the best modality for staging?
A. Endoluminal US B. CT C. ERCP D. MRCP
65. A case presentation with hyperinsulinaemic hypoglycaemia, blood glucose is 39mg/dl, insulin is 38pmol/L, C peptide 207pmol/L, on
OGD nothing is abnormal but found a mass like in the pancreatic head about 2x2 cm, what is the most initial appropriate test to define
and localise that pathological mass? Insulinoma
A. Abdominal MRI B. Triple-phase spiral (helical) CT C. EUS (the best if CT failed) D. ERCP E. Laparoscopic US
66. What is the correct regarding Insulinomas?
A. Are often multiple B. Are mostly benign (90%) rule of 90 (benign, solitary <2cm, intrapancreatic) C. Commonly located in head
D. Are a common component of multiple endocrine neoplasia type I (MEN I)
Insulinomas constituted most cases (55%), followed by gastrinomas (36%), VIPomas (vasoactive intestinal polypeptide tumour) (5%), & Glucagonoma (3%)
Whipple triad: episodic hypoglycaemia, CNS dysfunction temporally related to hypoglycaemia and dramatic reversal of CNS effects by glucose intake
67. A case with features of insulinoma, Whipple triad, the patient underwent CT but was not confirmatory for its diagnosis or not detected
by CT, what is the other option or best investigation for diagnosis?
A. EUS (if US, CT, MRI failed) B. MRI (initial) C. SRS (if US, CT, MRI failed to localise)
D. Laparoscopic US with palpation (if all failed to localise)
Non invasive imaging procedures (CT), (MRI), US, Invasive ; selective DSA, EUS biopsy, portal vein, and arterial-venous blood sampling to determine
insulin respectively. Insulinomas, are hard to localise, angiography detects approximately 70% if > 5cm, laparoscopic US, with palpation is the best test.
68. What is the optimal management for a 2-cm mass in the head of the pancreas with hypoglycaemia and high insulin levels?
A. Total pancreatectomy B. Whipple resection C. Local excision (insulinoma) D. Streptozotocin administration
1. Enucleation for small (<2 cm), not abutting the pancreatic duct, anterior or posterior surfaces in the neck, body, or tail with a tissue plane bet tumour& duct
2. Insulinoma >2 cm and/or abutting the pancreatic duct or involve CBD/pancreatic duct, without clear plane between the tumour pancreas, for resection
3. Malignant insulinomas; 10% Peripancreatic lymph node dissection should be performed
4. In the presence of liver metastases, resection of the primary tumour and accessible metastases is advocated if it can be done safely.
5. Unresectable, medications can be employed to minimise hypoglycaemia and the progression of disease.
69. Hyperinsulinism in a newborn is most likely caused by which of the following?
A. Nesidioblastosis (non insulinoma hypoglycaemic hyperinsulinaemia ) B. Glycogen storage disease C. Benign insulinoma
D. Malignant insulinoma
70. What is the most common site of Gastrinoma in pancreas? Gas H
A. Head B. Body C. Neck D. Tail
71. Which of the following is likely to be multiple?
A. Gastrinoma (duod gastrinoma<1cm multiple. 60-80% solitary, 20-40% multiple, 60-90% malignant, 70-80% sporadic) B. Insulinoma (10% multiple)
C. Somatostatinoma D. Glucagonoma(mostly malignant, solitary)
N.B. Insulinoma (body and tail), Gastrinoma (head and uncinate process), and Glucagonoma (tail or body)
Abdominal pain (in 75 %) Diarrhoea (73 %) weight loss (17 %) and gastrointestinal bleeding (25 %). Gastrin >150—Gas-->1000—ZES. Secretin >200 ZES
72. A 50 year old male patient with history of severe epigastric pain, bilious vomiting and diarrhoea, not responding to usual lines of
treatment, US is free, on OGD found oesophagitis, with free stomach and duodenum and by CT; hypoechoic mass 2x3 cm of the head
of pancreas what is most likely diagnosis?
A. Glucagonoma B. Somatostatinoma C. Gastrinoma D. Insulinoma E. VIPoma
Over 90% of Gastrinomas are found in the gastrinoma triangle bounded by the third part of the duodenum, the neck of the pancreas and the porta-hepatis
73. A child with GI symptoms and positive provocative secretin test, what is the diagnosis?
A. VIPoma B. Gastrinoma (>150 gast. >1000 ZES, SRS for localisation) C. Glucagonoma
74. A case with one month epigastric pain, diarrhoea, multiple gastroduodenal ulcers with upper GI bleeding, gastrin 120, diagnosed as
Zollinger Ellison syndrome not responding to PPI, what is the lab test to confirm it as a ZES?
A. Fasting Gastrin blood level (suspected: 1000pg/mL) B. VIP C. Secretin stimulation test (as lab and SRS as imaging)
D. Somatostatin receptor scan E. Endoluminal US
75. What is the investigation of choice Gastrinoma localisation?
A. MRI/CT(initial) B. SPECT CT(better than SRS) C. SRS (Somatostatin receptor scintigraphy, sens 80-100%, spec, >90%) D. OGD
76. A case presented with watery diarrhoea, hypokalaemia, hypochloraemia, and acidosis are features of which of following diseases?
A. Insulinoma B. Gastrinoma C. Vasoactive intestinal peptide producing tumour D. Glucagonoma
77. A 2-3 cm pancreatic tumour with hypokalaemia or watery diarrhoea, hypochlorhydria and pancreatic mass what’s most likely diagnosis?
VIPoma; (Werner-Morrison or WDHA syndrome) syndrome of profuse watery diarrhoea, hypotension, abdominal pain and hypochlorhydria)
78. Among the pancreatic neuroendocrine neoplasms which one is known as pancreatic cholera?
A. Somatostatinoma B. Glucagonoma C. VIPoma D. Gastrinoma E. Insulinoma
79. Which of the following is poorly absorbed in achlorhydria?causes Vit B12 deficiency, Pernicious anaemia, long-term PPIs, H. pylori, VIPomas
A. Proteins B. Fats C. Bile salts D. Vitamin B 12 E. Vitamin D
80. What is the most malignant neuroendocrine tumour with metastasis at start of presentation?
A. Insulinoma B. Gastrinoma C. Glucagonoma D. Somatostatinoma (75% malig.(55-75% pancreatic (36% head, 32% tail,14%body)
Pentad of DM, GBstone, wt loss, steatorrhoea and diarrhoea, and hypochlorhydria. Symptoms hypoglycaemia, abdominal pain, anorexia, nausea, & vomiting
81. What are characteristics of Somatostatinoma? Rare 1 in 40 million, < 5% of pancreatic NETs
A. Mild diabetes, skin rash, glossitis B. Ulcer diathesis, diarrhoea
C. Mild diabetes, diarrhoea, gallstones (plus weight loss, steatorrhea & achlorhydria) D. Diarrhoea, hypokalaemia, hypochloraemia
82. What is the most common site of Glucagonoma? 50 to 80 % are metastatic at diagnosis
A. Head of pancreas B. Tail C. Neck D. Body and tail; distal pancreas (alpha cell location; site of glucagon secretion)
83. What Glucagonoma is associated with? Necrolytic migratory erythema, diabetes mellitus, severe weight loss, glossitis, stomatitis, angular
cheilitis, diarrhoea, deep venous thrombosis, anaemia and hypoaminoacidaemia. Gastrinomas are found in 1 in 2 million / year, VIPomas found in
1 in 10 million/year, and glucagonomas are found in 1 in 20 million/ year
84. Necrolytic migratory erythema is a characteristic feature of which of the following?
A. Gastrinoma B. Insulinoma C. Glucagonoma D. Vipoma E. Somatostatinoma
85. Which serum level requires assessment if necrolytic erythematous rash in a diabetic patient have developed?
A. Insulin B. Somatostatin C. Cortisol D. Gastrin E. Glucagon (N is <50 pg/mL, above 1000 pg/mL >> glucagonoma)
86. A patient has mild diabetes, skin rash, and glossitis which of the following is the most likely diagnosis?
A. Somatostatinoma B. Gastrinoma C. Glucagonoma (necrolytic ME; 70-80%, VTE;40%, wt. loss, 60-80 , DM, 40%, diarrhoea 18%)
87. What is the most characteristic metabolic abnormality in glucagonoma?
A. Hypoglycaemia B. Hypo amino acidaemia C. Hypocholesterolaemia D. Hypercholesterolaemia
88. What is the most characteristic of malignancy in a cystic pancreatic neoplasm?
A. Size > 6 cm (>3cm has 3 folds risk) B. Wall calcification C. Multiple loculations D. Dense vascularity
Findings associated with malignant transformation in MCNs include:
• Larger size (5 cm or larger in one series) • A thickened or irregular cyst wall • An internal solid component or mass
• Possible calcification of the cyst wall (Uptodate) Lesion size greater than 3 cm: carries a threefold increase in the malignancy risk. Presence of mural
nodules carries an eightfold increase in the malignancy risk. Coarse calcification
Features associated with pancreatic cyst malignancy include invasion of adjacent structures, nodal and distant metastases, larger lesion size (cyst ≥4 cm),
thick irregular walls, enhancing soft-tissue components or mural nodules, mucinous type peripheral calcifications, and pancreatic duct dilatation
89. What is the characteristic or the most important investigation of mucinous cystadenoma of the pancreas after FNA of the cyst fluid?
A. High CEA in cyst fluid B. High amylase C. High glucagon D. High glycogen level
90. A patient presented with jaundice, with history of pancreatic cyst which was aspirated, and the fluid examined and showed low amylase
and high glycogen level, what’s the type of cyst?
A. Serous cyst (low in CEA, Amylase and viscosity) B. Mucinous cyst (without intraductal neoplasm has high viscosity, high CEA but low Amylase)
C. Pancreatic pseudo cyst (high CA19-9 and amylase with lower in viscosity and CEA)
D. IPMN (high CEA, Amylase & viscosity) )
CEA normally <5ng/ml if CEA level more than 200 or 800 ng/ml it is sure mucinous. Amylase less than 250 IU/L is specific for non-pseudocysts
91. A pancreatic cyst, with low amylase, high glycogen in cyst fluid, what is most likely? Serous cystadenoma
92. Aspiration of pancreatic sac reveals fluid rich in, glycogen, sugar or carbohydrates what is the diagnosis?
A. Pseudocyst B. Mucinous cystadenocarcinoma C. Serous cystadenoma (if mets or invasion to spleen, liver etc >>cystadenocarcinoma)
93. What is the likelihood of malignancy in cystic pancreatic neoplasm?
A. High glycogen content B. Mucin content (Mucinous cystic neoplasm) C. Sunburst sign on CTscan D. History of pancreatitis
94. A case with cyst in main pancreatic duct, small cysts or papilloma in distal pancreatic duct branches, what is management?
A. ERCP and stent B. Distal pancreatectomy C. Total pancreatectomy (if IPMN? papilloma) D. Pancreatojejunostomy
Spleen-preserving distal pancreatectomy is reserved for chronic pancreatitis, proven benign pancreatic lesions, cysts, neuroendocrine tumours, and trauma
contraindicated for malignant lesions of the distal pancreas. Distal pancreatosplenectomy with lymphadenectomy is indicated for locally resectable adenocar.
95. Intraductal papillary mucinous neoplasm ERCP picture, what is sign is characteristic to IPMNs on ERCP?
A. Fish eye sign B. Fish mouth
Contrast-enhanced CT and MRI have been shown to have comparable diagnostic performance for the diagnosis of malignant IPMN.
• High-risk stigmata; enhancing solid component > 5 mm (strongest association with malignant IPMN), main duct ≥10 mm or obstructive jaundice.
• Worrisome features; cyst ≥ 3 cm, thickened and enhancing cyst wall, enhancing mural nodule < 5 mm, main pancreatic duct 5-9 mm, lymphadenopathy
96. With cannulation of ampulla of Vater during ERCP, you find mucin coming from the ampulla, what is the diagnosis?
A. Mucinous cystadenocarcinoma B. Intraductal papillary mucinous neoplasm. IPMNs C. Mucinous cystadenoma
97. A 35-yrs. female patient presented to the ER with a 2 day history of epigastric pain. 2 weeks ago, she had acute biliary pancreatitis. CT
was done and shows 4 cm peripancreatic fluid. There is no necrotic tissue, what is your next step? Conservative management as before 4 wks.
Internal drainage > if not resolved spontaneously and in uncomplicated pseudo cysts requiring intervention.
98. A patient presented by CT with pancreatic head lesion 3 cm with multiple papillary projections in dilatations in main and branch
pancreatic ducts, what is the optimal management?
A. Distal pancreatectomy (if body& tail) B. Total pancreatectomy C. Pancreaticoduodenectomy (Whipple’s procedure)
99. A patient with recurrent epigastric pain, imaging showed multiple cystic lesions and papillary projections in in the body and tail, like
IPMN, with nodular changes, ALP 500, what is the best management?
A. Pancreaticoduodenectomy(if head) B. Pancreaticojejunostomy C. Distal pancreatectomy(as body & tail) D. ERCP & Stent
NCCN recommends neoadjuvant chemotherapy for resectable and borderline resectable disease.
• Pancreatoduodenectomy (Whipple procedure) is the treatment of choice for tumours of the pancreatic head.
• Distal pancreatectomy, including the resection of the body and the tail of the pancreas and the spleen, is the option for tumours in the body or tail.
• Standard lymphadenectomy should involve the removal of ≥15 lymph nodes to allow adequate pathologic staging. No extended lymphadenectomy
• Postoperative gemcitabine or 5-FU chemotherapy is recommended. No chemoradiation should be given to patients after surgery.
100. Regarding the pancreatic tumour location what is most commonly resectable pancreatic tumours?
A. Pancreatic tail B. Pancreatic Head(head has better survival as early presentation by jaundice) C. Pancreatic body D. Pancreatic neck
101. A patient with pancreatic cancer, what is the best pain killer due to severe epigastric pain?
A. NSAIDs B. Paracetamol C. Tramadol D. Morphia E. Hyoscine bromide
SPLEEN
1. What is the most common site for accessory splenic tissue?
A. Gastro splenic ligament B. Gastrocolic ligament C. Splenic hilum D. Splenocolic ligament E. The pelvis
2. Accessory spleens are most commonly found in which of the following?
A. Idiopathic thrombocytopenic purpura B. Thrombotic thrombocytopenic purpura C. Schistosomiasis
D. Hereditary spherocytosis
3. An adult after RTA, got abdominal trauma with left hypochondrial pain radiated to left shoulder, tenderness and ecchymosis, vital
signs, BP 100/70, what’s best investigation?
A. US (initial, for laceration, blood and haematoma) B. CT (the best modality of choice for splenic trauma) C. DPL D. Laparotomy
4. If a trauma to the left lower chest, what is the most likely organ is vulnerable to injury?
A. Spleen B. Intestine C. Liver D. Stomach
5. A 12- year-old exposed to a blunt trauma to his abdomen and later presented with generalised abdominal pain. Imaging of the spleen
showed a 7 mm haematoma and 4 cm tear (grade 3), the patient is haemodynamically stable, what is your management?
A. Splenectomy B. Spleen preserving surgery C. Conservative
If patient is stable with no deterioration on follow up, so the management is conservative. If stable patients (Grade 1-3) we suggest non-operative management
6. In the case of Warshaw technique, what is the surgeon is preserving?
A. Distal pancreas B. Splenic vessels C. Spleen D. Gastric vessels
7. A post abdominal trauma child brought after ER with, Splenic tear 3cm, his BP is 90/60, given 1000 ml fluid IV but his BP still 90/60 what to do?
A. Splenectomy B. Observation & conservative management (grade II) C. Reimplantation D. Splenic a. Embolisation
8. A child with 2cm splenic laceration and perisplenic fluid, how to manage?
A. NOM (Non-operative management include observation and embolisation) B. Laparotomy with splenectomy C. Laparotomy/ splenorrhaphy
9. Trauma patient with splenic laceration 2cm with fluid collection, what is your management?
A. No surgical intervention B. Splenectomy C. Preserved surgery
American Association for the Surgery of Trauma Splenic Injury AAST graded splenic injuries to 5 grades. Grade I <10% subcapsular haematoma or
<1 cm laceration. Grade II 10%–50% subcapsular haematoma or <5 cm intraparenchymal haematoma or 1–3 cm laceration without trabecular vascular
involvement. Grade III >50% subcapsular haematoma >5 cm intraparenchymal haematoma or >3 cm laceration or trabecular vascular involvement
Grade IV Laceration involving vasculature with >25% devascularisation Grade V Shattered spleen Hilar laceration with splenic devascularisation
10. An 18 year male came with stab wound injury next to umbilicus with small opening clean and bleeding stopped, vitally stable with normal lab
results, CT scan report showed: small splenic laceration with minimum fluid surrounding it, what is the best definitive management?
A. Wound closure (observation: NPO 24h, IVF, serial examination, Hb) B. Antibiotics C. Splenectomy D. Exploratory laparotomy
11. Splenic injury with 3 cm laceration, which grade?
A. I (<1 cm) B. II (1-3 cm) C. III (>3 cm) D. IV (Laceration of segmental or hilar vessels >> major devascularisation (> 25% of spleen )
12. Splenic injury in a child and haemoglobin level is 11.0, what to do?
A. Conservative B. Give haemoglobin C. Splenectomy D. Splenorrhaphy
13. A post RTA patient has no ribs, shoulder or clavicular fractures only left sided back contusion with isolated left shoulder pain?
A. Shoulder dislocation B. Liver injury C. Splenic rupture D. Cervical spondylosis
This is called Kehr sign shoulder pain due to diaphragmatic irritation by splenic rupture in absence of shoulder or clavicular fracture
14. RTA victim, with multiple traumas; free fluid in abdomen with query splenic rupture he is vitally stable, what is the management?
A. Thoracotomy B. Laparotomy (if unstable even after resuscitation) C. Abdominal CT scan (to see degree & other associted injuries)
15. A patient was involved in RTA, presented with pulse 120 BP 80/40, he received 2 L of fluids then BP 70/40, abdomen is distended, and
CT showed a splenic laceration, what to do for him?
A. DPL B. Lap exploration C. Laparotomy (after ABCDE (unstable, FAST, may need DPL) D. Abdominopelvic CT
16. A trauma patient has a Glasgow Coma Scale score of 13, blood pressure 80/40; widened mediastinum on chest x-ray, and bloody
peritoneal tap., what is the next step in management?
A. Obtain head computed tomography scan B. Perform thoracotomy C. Perform arch angiography
D. Monitor intracranial pressure E. Perform laparotomy (to R/O thoracic aorta rupture+/- splenic injury)
Surgery is indicated in patients who cannot be adequately observed (due to limited resources or other injuries), are unlikely to tolerate a significant episode
of hypotension, and those who fail nonsurgical management (ie, observation, embolisation)
17. A 9-y.o girl came to ER complaining of fatigue and lethargy, physical examinations she is pale and has hepatomegaly and splenomegaly
6 cm below costal margin, low HB 5gm/dl, sickle cell positive she had same attack 2 times before what's your best management?
A. Regular blood transfusions B. Corticosteroids C. Hydroxyurea D. Splenectomy
The most frequent indications of non traumatic splenectomy is sickle cell disease with recurrent acute sequestration crises (a serious complication of SCA and
considered the second leading cause of death after infection in the first decade of life), severe haemolysis secondary to hypersplenism and splenic abscess
18. An RTA victim, with polytrauma, he has free fluid in the abdomen and splenic lacerations, there is a doubt of associating thoracic
aortic rupture what is the treatment plan?
A. Thoracotomy (if isolated thoracic aortic injury) B. Laparotomy (if can be extended) C. CT scan (if stable with inconclusive FAST)
D. Left thoracoabdominal incision
19. How to diagnose post-traumatic splenic rupture if the patient is stable and mild abdominal pain?
A. Abdominal CT B. Abdomen MRI C. Abdominal US (for unstable)
CT scan with intravenous contrast is the gold standard in haemodynamically stable or stabilised trauma patients (Good Recommendation 1A)
E-FAST is effective and rapid to detect free fluid (Good Recommendation, 1A)
N.B. Blunt + Stable = CT ..............Blunt + unstable = FAST, Penetrating + Stable = CT.......... Penetrating + Unstable = Exploration
Indications of direct exploratory laparotomy: Blunt + Unstable + Positive FAST, Blunt or penetration + Positive CT (Even if stable), Peritonitis, Evisceration
and Rectal/ gastric / Gastroduodenal bleeding
20. When should give pneumococcal vaccination post urgent splenectomy?
A. 2 weeks after splenectomy (if emergency) B. 2- 3 weeks prior elective op. (if elective splenectomy). C. Nothing is needed
21. A 30-year-old patient presented with stool mixed with blood for 2 weeks, upper OGD and lower colonoscopy negative, by examination
palpable spleen, what is the diagnosis?
A. Typhoid ulcer (most typhoid cases has splenomegally) B. Diverticulosis C. Duodenal ulcer D. Meckel diverticulosis
22. A middle aged patient with a history of 15 days fever, last 3 days he got acute abdomen & feature of peritonitis, what’s diagnosis?
A. Perforated typhoid ulcer B. Acute appendicitis C. Acute pancreatitis D. Ischaemic colitis
23. A female patient with splenomegaly diagnosed as Felty’s syndrome what are true finding combinations?
A. Peritonitis + + Rheumatoid B. Splenomegaly + Rheumatoid + Pancreatitis
C. Splenomegaly + Neutropenia + Peritonitis D. Neutropenia + Splenomegaly + Rheumatoid SANTA =felty
Felty’s syndrome (FS) is a rare extra-articular manifestation of seropositive rheumatoid arthritis (RA) characterised by RA, neutropenia, and splenomegaly.
A mnemonic for the basic components of Felty syndrome is SANTA; Splenomegaly, Anaemia, Neutropenia, Thrombocytopenia Rheumatoid Arthritis
24. A case of immune thrombocytopenic purpura; ITP, patient underwent splenectomy but still platelet 50,000 what is optimal treatment?
A. Steroids (1st. line if severe bleeding, if no splenectomy do it as 2nd line) B. Azathioprine; Imuran C. Immunoglobulin (in acute bleeding)
D. Conservative treatment (platelets >30000, no bleeding)
25. What is true regarding the immune thrombocytopenic purpura?
A. It is caused by antiplatelet immunoglobulin G originating in spleen B. It is associated with splenomegaly
C. It is associated with prolonged prothrombin time D. Splenectomy is required for most paediatric cases
E. Occurs with a male to female ratio of 3: I
26. A middle aged female developed profuse bleeding after tooth extraction, she mentioned past history of bleeding tendency, all bleeding
profiles were normal apart from platelets 40.000?
A. ITP B. TTP C. Thalassemia D. Gerstmann syndrome E. DIC
27. An old patient, with head trauma came to the emergency department, where the emergency doctor found, LL petechiae with labs;
Platelet count of 65,000 and prolonged bleeding time, normal clotting time with normal INR, what is your diagnosis?
A. DIC B. ITP (petechiae or ecchymoses if platelets1 0000-30000.. some have major bleeding) C. TTP
28. A normal-sized spleen is found in which of the following diseases?
A. Idiopathic thrombocytopenic purpura B. Myelodysplasia C. Gaucher's disease D. Thalassaemia E. Schistosomiasis
29. A patient had a major surgery which required 15 blood units, he afterwards started bleeding from NGT, ETT, what is diagnosis?
A. Thrombocytopenia (DIC) B. Transfusion reaction C. Haemolytic reaction
30. What is the component of blood transfusion responsible for immunosuppression?
A. Red cells B. Immunoglobulins C. White blood cells D. Platelets
31. What is the time for platelet transfusion during splenectomy for idiopathic thrombocytopenic purpura?
A. On making the incision B. After ligation of the splenic artery C. On induction of anaesthesia D. After splenectomy
32. When Idiopathic thrombocytopenic purpura is most likely to respond to splenectomy?
A. The spleen is enlarged B. The patient is a female C. The disease is chronic D. If the disease is steroid responsive
33. A patient with ITP underwent splenectomy, after 1-year platelet decreased again, what is the possible cause?
A. Incomplete spleen resection B. Presence of accessory spleen (sometimes relapse can occurs in 20-30%)
34. What is the most sensitive indicator of increased haemolysis with hypersplenism?
A. Reticulocyte count B. Bilirubin level C. Hemoglobin level D. Haptoglobin level (N 50-220 mg/dL critical value: < 40 mg/dL)
35. Which of the following is a defect in the haemoglobin chain that responds to splenectomy?
A. Immune thrombocytopenic purpura B. hereditary spherocytosis C. Thalassaemia D. Glucose-6-phosphate deficiency
36. Thalassaemia case >> iron chelation >>now pancytopenia and blood transfusion 300 ml/kg/year change to another iron biochemical
chelation) what is management option?
A. Observation B. Splenectomy (the clues: Pancytopenia + repeated transfusion)
37. A 35 yrs. old man had splenectomy post trauma, received post splenectomy vaccination after operation, what to do next?
A. Give antibiotics for 2-3 yrs. B. No further ttt C. Give prophylactic preop dose D. Give just 1 month antibiotics postop.
38. What is true regarding overwhelming post splenectomy sepsis; OPSS?
A. Commonly occurs after splenectomy for trauma B. Does not occur if accessory spleens are present
C. Can be fatal within hours of onset (due to fulminant septic shock in 24-48 hr mortality rate of up to 50%) D. Most common in elderly patients
E. Most fatal cases occur 10 to 15 years after Splenectomy (first 3 years post splenectomy)
39. A child with splenomegaly, jaundice, dark urine and +ve history of both parents have blood dyscrasias, Hb 9.2, what’s best treatment?
A. Splenectomy (hereditary spherocytosis) B. Corticosteroids C. Packed RBCs transfusion D. Whole blood transfusion
40. A 60 yrs. old man came to the clinic because he is vomiting blood. He was diagnosed recently with pancreatitis. The patient was
stabilised, and endoscopy was done and revealed gastric fundus bleeding with dilated veins, sclerotherapy was done. Duplex abdominal
ultrasound showed splenic vein thrombosis, what is your management?
A. Anticoagulants for SVT B. Sclerotherapy and band ligation C. Splenectomy (splenic art. embolisation if surgically unfit) D. TIPS
41. A patient presented with left upper quadrant pain, with irritation of the diaphragm. BP 100/70, tachycardia, how to investigate?
Ultrasound can rule out left subphrenic abscess, splenic rupture, or infarction, left pericolic abscess or splenic flexture tumour, or rare Tietze's syndrome
42. What is true for Hairy cell leukaemia?
A. Can be cured with splenectomy B. Is an aggressive form of leukaemia C. It is a T-cell leukaemia
D. Death is usually related to infectious complications (in > 50%) due to pancytopenia with neutrophil <1000) may require Cladribine & pentostatin
43. A case of leukaemia has generalised lymphadenopathy and hepatosplenomegaly what is the best diagnosis of lymphadenopathy?
A. FNAC B. LN excision biopsy (Cytogenetic studies for Philadelphia chromosome to DD between myeloproliferative& myelodysplastic syndrome)
44. A case of female patient with myeloproliferative disease, presented with severe left hypochondrial pain referred to the shoulder, blood
film normal, what will you do?
A. Lap splenectomy B. Open splenectomy C. Endovascular stent D. Conserve (for asymptomatic or mild, cases)
45. What is the principal side effect of ganciclovir therapy?
A. Bone marrow suppression (severe) B. Elevated liver enzymes C. Acute pancreatitis D. Acute renal failure
46. A patient with myeloproliferative disease, has left hypochondrial pain and referred to the back, what’s the pathophysiology of pain?
A. Haemorrhage B. Infection C. Aneurysm D. Splenic infarction
47. Myeloproliferative disease with huge splenomegaly with severe Lt. Hypochondrial pain what’s the next best step for diagnosis?
A. Laparoscopy B. MRI C. CECT (for inflammatory, infarction, cyst, calcification, LNs, perisplenitis, abscess) D. Bone marrow biopsy
E. Splenectomy F. Observation G. EUS (with FNA is confirmative especially in malignant splenic dis.)
48. Postoperative morbidity after splenectomy for haematologic diseases is highest for which?haematologic malignancies >50% suffer complications
A. Idiopathic thrombocytopenic purpura B. Thalassaemia C. Myeloid dysplasia (haematologic cancer as leukaemia) D. Sickle cell dis.
49. Myeloid metaplasia is characterised by which of the following?
A. It is a disease of young females B. It is rarely associated with splenomegaly
C. Results in extra medullary haematopoiesis D. Results in increased bone marrow megakaryocytes
50. The risk of overwhelming post splenectomy sepsis is highest for patients requiring splenectomy for which of the following?
A. Thalassaemia B. Trauma C. Immune thrombocytopenic purpura D. Hereditary spherocytosis
Greater risk is associated with splenectomy for blood diseases as sickle cell anaemia, thalassaemia and tumours more than splenectomy due to trauma
51. A female patient, she is anaemic, pale, and jaundiced, with splenomegaly, target cells in blood smear what’s most likely diagnosis?
A. Thalassemia B. Spherocytosis C. Sickle cell
52. Target cell in blood smear, in which disease? Thalassemia & (hepatic disease with jaundice, haemoglobin C disorders, and the post splenectomy)
53. A child, splenomegaly 4 cm below costal margin, both father and mother have blood disease, jaundice, reticular cells, what’s diagnosis?
A. Sickle cell anaemia B. ITP C. Thalassaemia D. Hereditary spherocytosis (Coombs testing -ve, reticulocytosis, Spherocytes+ve)
54. A patient with spherocytosis sent to surgeon for splenectomy, what investigation you should do before doing surgery?
A. Fragility test B. US to rule out gall stones (to avoid CBD stone) A. CT scan
55. A young female patient present with abdominal pain vomiting and fever. On examination there was splenomegaly and mild jaundice,
US revealed pigmented stones which of the following is the most likely diagnosis?
A. Thalassemia B. Hereditary spherocytosis C. G6PD deficiency D. Paroxysmal nocturnal haemoglobinuria
56. What is known side effect of Sulphamethoxazole? Kernicterus diarrhoea, vomiting, electrolyte imbalance, fungal overgrowth, headache, nausea,
skin reactions. Contra-indications Acute porphyrias. stop if blood disorders (including leucopenia, thrombocytopenia, megaloblastic anaemia, eosinophilia)
ENDOCRINE AND BREAST SURGERY

ENDOCRINE
THYROID, PARATHYROID GLANDS AND NECK SWELLINGS
1. A patient with midline firm solid neck nodule, moves with deglutition or swallowing, which of the following tools confirm the diagnosis?
A. US (initial) B. TSH C. CECT(the preferred imaging in TGDC for surrounding structures) D. FNAC (the best confirmative if > 1cm)
E. Radioisotope scintigraphy scan (uptake only by thyroid tissue used in doubt of thyroid tissue I123 is better)
2. A child with midline neck swelling that moves with protrusion of the tongue, what is the most common site of thyroglossal duct cyst?
A. Below hyoid bone (infrahyoid 50-65%, at hyoid 15-59%, suprahyoid 20-25%) B. Overlying thyroid cartilage C. Submental triangle
Lingual: 2%, Suprasternal: 5–10%
3. What is the most common midline single neck swelling?
A. Pharyngeal pouch B. Dermoid cyst C. Laryngocele D. Thyroglossal cyst
4. A central mass compressing the trachea with symptoms of difficulty swallowing, cough when lying down, what is the diagnosis?
A. Thymoma B. Lymphoma C. Goitre (confirm by US, TSH, Radioisotope scintigraphy scan & FNAC)
5. A young child with thyroglossal cyst that is firm but not tender how you exclude as if ectopic thyroid tissue?
A. FNAC B. Excisional biopsy C. Thyroid US D. Nuclear scintigraphy with TC 99m or I123 E. EUS
6. A 78-year-old female, midline swelling in the neck for more than 15 years, diagnosed by US as MNG, the swelling does not move with
tongue protrusion but moves with deglutition only, patient refused surgery, what the fate of that swelling?
A. Infection B. Laryngeal compression C. Spontaneous resolution D. Malignant transformation (up to 35 % TNG, MNG, Graves,)
7. A patient with a tender, fluctuated and red central neck cyst, no fever or throbbing pain, what is the best treatment?
A. Sistrunk B. Incision and drainage C. Aspiration (for culture as AFB/mycobacterial culture) D. Broad spectrum antibiotics IV/oral
8. A child, with midline neck mass, move with deglutition and tongue protrusion, mostly infected thyroglossal with abscess, what to do?
A. Excision B. I & D C. Broad spectrum antibiotic (Cefazolin and Clindamycin 10-14 days) D. Sistrunk op.
9. An image for A 57 yrs. old female with thyroglossal cyst appeared 2 months ago, move with deglutition and tongue protrusion, patient
refused surgery, what is the fate of this swelling? Complications, infection, abscess, sinus, if ectopic thyroid tissues and malignancy of that ectopic
A. Infection (TGDC less common in old) B. Malignancy C. Laryngeal compression D. Spontaneous Resolution
10. A patient with diagnosis of thyroglossal duct cyst. how to treat?
A. Sistrunk operation (cyst + all duct tissues & central 3rd. of hyoid bone) B. Follow up C. Needle aspiration D. I & D
11. All the following are superficial neck swellings except which of the following?
A. Branchial cyst B. Sebaceous cyst C. Lipoma D. Neurofibromatosis
12. A female patient presented with neck swelling, she is completely asymptomatic, neck US was done revealed a solid mass of 7 mm with
regular margins, which of the following is the most appropriate next step?
A. FNA B. Thyroid scintigraphy C. Check TSH (to rule out it is toxic thyroid nodule if normal do thyroid scan R/O cold) D. CT scan
13. A patient complaining of dysphagia with an image of radionuclide scan, showing part of oropharynx what is most probable diagnosis?
A. Ectopic thyroid (10% lateral neck) B. Lymph node C. Lingual thyroid (90% of ectopic are lingual) D. Thyroglossal cyst
14. How to rule out an ectopic thyroid tissue in thyroglossal cyst or the best diagnostic test for ectopic thyroid tissue in thyroglossal cyst?
A. Neck CT B. Neck US C. FNAC D. Radionuclide imaging or thyroid scintigraphy scan by radioisotope I-123
For hot nodule do radioactive iodine ablation or surgery. Cold nodule >> FNAC to rule out malignancy, if suspicious benign-> lobectomy or T. thyroidectomy
15. A child 12 yrs. there is bilateral neck swellings mostly LNs how to reach diagnosis?
A. Neck CT B. Biopsy of both sides and histopathology C. FNA of one side
16. A 47 y.o patient with neck swelling, US revealed a mass on right side 2x1cm and left side 3x2 cm, from which one FNA should be taken?
A. Left nodule B. Largest nodule C. Both nodules (not < 1cm size) D. Right nodule
17. A patient presented with thyroid nodule more than 1.3 cm, normal TSH, T3 and T4, what is the the best confirmatory investigation?
A. MRI B. US C. CT D. FNAC (>1cm do FNAC)
18. A hard, fixed neck mass, with progressive hoarseness of voice, appeared 10 wks., ago, what is the best diagnostic investigation?
A. FNAC (? anaplastic CA) B. Core biopsy C. Isthmectomy (if compression manifestation)
19. A 25-year-old recently married wife with investigation for fertility found TSH high, T4 normal she wants to get pregnant?
A. Give thyroxin now (hypothyroid) B. Give thyroxine during pregnancy C. Follow Up D. Proceed to pregnancy without any ttt.
20. A patient with diffuse painless thyroid gland enlargement with T3, T4 N, high TSH and +ve thyroid antibodies, what’s the treatment?
A. Thyroidectomy B. Radioactive iodine C. Thyroxine (Hashimotos?) D. Antithyroid drugs
21. Respiratory distress associated with goiter while lying supine, is mostly caused by which of the following?
A. Recurrent laryngeal nerve palsy B. Malignant tracheal invasion C. Retrosternal goiter extension D. Hge. in large goiter
22. A 36 y.o male C/O difficulty of breathing and swallowing for 4 months x-ray: midline mass compressing trachea, what’s most likely?
A. Thymoma B. Lymphoma C. Lung cancer D. Thyroid goiter (especially if RSE)
23. A patient with diffuse goiter, exophthalmos, and lab showed elevated TSH and low T3 and T4, what is the initial pathophysiology of
the condition (exophthalmos)?
A. Inhibitory anti TSH Abs B. Stimulatory anti TSH Abs C. T lymphocytes sensitisation D. B lymphocytes
24. A case of hypothyroidism (high TSH, normal T3, T4) and proptosis, what is the initial cause of proptosis in hypothyroidism?
A. Stimulatory anti TSH B. Inhibitory anti TSH C. T lymphocytes C. B lymphocytes.
25. A patient with a history of with history of Hodgkin’s disease treated by radiofrequency ablation, he is at risk for hypothyroidism.
He came for a check-up and his labs show a TSH of 5 (N; 0.5 - 5.0) T4 5mcg/dl. What will you do to establish a diagnosis of hypothyroid?
A. Repeat TSH in 4 weeks B. Measure T4 levels C. Thyroid scan D. Thyroid US
26. A female patient with history of MNG for 10 years, underwent subtotal thyroidectomy then, she came presented with recurrence of
goiter with normal T3, T4 and elevated TSH, what is the cause of recurrence?
A. Hypothyroidism (no levothyroxin taken) B. Thyroxin replacement therapy C. Recurred thyroid D. Malignancy
27. What is the most accurate test to detect subclinical hypothyroidism?
A. Radioactive iodine uptake B. TSH level (mild rise with normal, T3, T4) C. Total thyroxine level D. Free thyroxine level
28. A para 1 or 3, has not been able to get pregnant again for 3 years, no history of surgery or neck radiation before, she has symptoms
such as fatigue, weight gain, constipation, labs showed: High TSH, normal FSH, LH, very high Prolactin, what is the diagnosis?
A. Hyperprolactinemia B. Hypothyroidism (1ry) C. Vitamin D deficiency and hypocalcaemia
In primary hypothyroidism, serum total T4, free T4, and free T3 levels are low, whereas the TSH level is elevated.
In secondary or central hypothyroidism (due to surgery, irradiation, drugs), the TSH level, free T4 index, and free T3 level all are low.
When emergency surgery is required in patients with severe hypothyroidism, IV T4 (200–250 μg L-thyroxine) should be given before surgery to prevent
stress- induced Myxoedema coma.(Myxoedema coma: hypothermia, confusion, hypotension, bradycardia, hyponatraemia, hypoglycaemia, & hypoventilation)
29. A patient known hypothyroidism, post operatively, she developed confusion, hypoxia, hypercapnia, hypotension (Myxoedema coma),
what is the most urgent management? Airway ETT+O2---levothyroxin & T3-----dexamethazone
A. O2 therapy B. Cortisone (3rd. step after airway and levothyroxin as dexamethazone 2-4 mg bd or Hydrocortisone at 50-100 tds, dexa is better)
C. IV Levothyroxine (after airway management 300-600 mcg then 50-100)
D. Intubation and mechanical ventilation with O2 therapy& continuous cardiac monitoring ABCDE
30. What is the most related sign to thyrotoxicosis from the following?
A. Obesity B. Atrial fibrillation (10-20% in adults) C. Low energy level D. Loss of hair at lateral part of eyebrows
31. Which of the following medications improve malignant exophthalmos or Grave’s induced orbitopathy? IV cortisone/ teprotumumab;Tepezza
A. Antithyroid drugs B. IV Steroids/oral prednisolone (as exophthalmos here is due to antibodies not high T3,T4 ) C. Thyroidectomy
32. A patient with grave’s disease and eye symptoms, what will increase eye symptoms in grave’s disease?
A. High free T3&4 B. Male C. Smoking (current is worsing orbitopathy) D. Ex smoker have more risk as current smoker
33. In the case of Grave’s disease, which antibodies are responsible?
A. Anti TSH releasing antibodies B. Anti TSH antibodies C. Antithyroglobin antibodies D. Antipyroxidase abs
34. Grave’s disease is caused by? TSH receptor IgG stimulatory antibodies. Graves' disease mediated by TSH receptor stimulation Abs.
Antibodies to the TSH receptor may be stimulatory (eg, in patients with Graves' disease) and cause hyperthyroidism, or inhibitory (eg, in chronic
autoimmune thyroiditis) and cause hypothyroidism.
35. What is the pathophysiology of exophthalmos in grave’s disease?
A. Activation of B-lymphocyte B. Activation of T-lymphocyte C. TSH stimulating antibody D. Thyroid inhibitory Abs.(hypothyroidism)
36. A male patient with grave’s disease after thyroidectomy, he got fever, tachycardia, tachypnoea, hypertension dx?
A. Thyroid storm B. Septicaemia C. Infection
N.B hypotension can occur in later stages with shock. Thyrotoxic crisis; storm is an acute exacerbation of hyperthyroidism. It occurs if a thyrotoxic
patient has been inadequately prepared for thyroidectomy
37. Near total thyroidectomy done for toxic MNG and then it recurred labs showed, low TSH, high T3 but T4 normal what the cause?
A. Hypothyroidism B. Hyperthyroidism (toxicity from remaining tissue) C. Suppression T Cell D. Autonomous nodule
Near‐total thyroidectomy is an operation that involves the surgical removal of both thyroid lobes except for a small amount of thyroid tissue <1g on each side.
In total thyroidectomy all macroscopic thyroid tissue will be removed without leaving any tissues beyond.
Subtotal thyroidectomy, removal of thyroid tissues except 4-7 gm on each side. Up to 60 % of patients can maintain adequate thyroid function without thyroid
hormone supplementation when there is a four- to seven-gram remnant. Hartley-Dunhill procedure (unilateral total thyroidectomy with contralateral subtotal
thyroidectomy). For patients with toxic multinodular goiter (MNG), total thyroidectomy should be performed, if near total will cause hyperthyroidism again
38. A patient with history of toxic nodular goiter kept on antithyroid drugs but he is not taking it regularly, presented by palpitations and
tachycardia, his TSH 0.2, what is his management?
A. Total thyroidectomy B. Near total thyroidectomy C. Antithyroid drugs regularly for 18 months if no remission>> subtotal thyroidectomy
39. What is the management of Graves’ disease with exophthalmos that is not responding to Antithyroid drugs for the past 10 months?
A. Subtotal thyroidectomy (after 18 ms of ATD failure) B. Near total thyroidectomy C. Radio iodine therapy
C. Continue antithyroid medication for 12-18 months
40. A pregnant woman in the 2nd. trimester: 14 weeks pregnancy, she has tremors, palpitation, right neck swelling, most likely treatment?
A. Thyroidectomy (Methimazole intolerance, allergy or agranulocytosis) B. Antithyroid drugs (PTU the least teratogenic) C. Radioiodine
41. What is the treatment of toxic goiter in paediatric patients?
A. Radioactive iodine (not recommended will cause hypothyroid, not for < 5 y age) B. Surgery(for large, Graves, unable to use ATD, Malignancy)
C. Antithyroid drugs (1st. line, Methimazole or Propylthiouracil for 12 to 18 months)
42. A pregnant lady in 28 wks pregnancy who is KCO hyperthyroidism kept on 15 mg carbimazole came with symptoms of hyperthyroidism,
labs: low TSH, high T3 & T4, what's the most appropriate management?
A. Switch to Propylthiouracil; PTU (she is 3rd. trimester) B. Increase carbimazole dose to 20mg C. Referral for thyroidectomy (in 2nd.)
Anti-thyroid drugs are the treatment of choice for hyperthyroidism in pregnancy. The lowest effective dose should be used. Treatment should be monitored
with FT4 and TSH. These should be measured every 2-4 weeks initially and then 4-6 weekly once thyroid hormone levels are stabilised. Propylthiouracil is
the best as others like methimazole and carbamazole are teratogenic especially in first trimester. Surgery if antithyroid cannot be tolerated, allergy or
agranulocytosis occurred, and it is safest in the second trimester. Iodine ablation is absolutely contraindicated in pregnancy
43. A patient with neck pain & hyperthyroidism with upper respiratory tract infection, what’s appropriate treatment?
A. Propranolol (to treat tachycardia and tremor? subacute thyroiditis) B. PTU C. Thyroid scan D. Methimazole
Only give propranolol to relieve symptoms it may be subacute thyroiditis or de Quervain’s disease usually occurs after viral upper RTI
44. Rt. thyroid nodule, FNAC, show follicular atypia, what’s the best management?
A. Total thyroidectomy(in Bethesda V&VI, 75 & 99% malig.) B. Rt. Lobectomy (Bethesda III, atypia of undetermined significance 10-30% malig.)
C. Near total thyroidectomy
45. A picture of hyperthyroidism with US showing right solid nodule 2 cm and thyroid scan showing picture of toxic multinodular goiter,
what is the most appropriate management?
A. Subtotal thyroidectomy B. Iodine Radio-ablation C. Right thyroidectomy D. Near total thyroidectomy/or Total
46. A patient with diffuse multinodular goitre with thyrotoxicosis and exophthalmos, what is the optimal treatment?
A. Subtotal thyroidectomy B. Total thyroidectomy C. Near total thyroidectomy D. Iodine ablation
47. What is the most common thyroid cancer?
A. Papillary (80%) B. Follicular (10%) C. Anaplastic (1-2%) D. Medullary (5-10%)
48. A 32 y.o woman has right neck lump on examination showed enlarged cervical lymph node with normal thyroid gland, percutaneous
biopsy taken from the lump, histopathology report: normal follicular thyroid cells?
A. Ectopic thyroid B. Follicular thyroid cancer C. Papillary thyroid cancer (Follicular variants; 40%) D. Thyroglossal cyst
49. A 36-year-old patient with left neck mass, 2x2cm in posterior angle of mandible US: normal thyroid, left large LN with cystic
component FNA? All smears show follicular thyroid, what is the diagnosis?
A. Metastatic thyroid cancer (mostly papillary thyroid cancer) B. Aberrant thyroid C. Ectopic thyroid D. Thyroglossal cyst
50. What is the marker with prognostic significance in cases with presence of psammoma bodies?
A. Alpha -fetoprotein B. Human chorionic gonadotropin C. Alkaline phosphatase of hepatic origin
D. Testosterone level E. Serum lactate dehydrogenase
51. For histopathological examination results came with marked psammoma bodies in which of the following it is present?
Medullary thyroid cancer B. Cervical cancer C. Papillary thyroid cancer D. Testicular seminoma
Psammoma bodies are well circumscribed, laminated concentric calcified structures, name is derived from the Greek word for sand because they resemble
grains of sand found in a diverse group of tumours which include Papillary thyroid carcinoma, papillary serous carcinoma of the endometrium, melanotic
schwannoma (psammomatous variety), meningioma mesothelioma, serous cystadenocarcinoma of the ovary, adenocarcinoma of lung
52. A patient underwent thyroidectomy for papillary thyroid cancer, he is on normal diet, came with generalised seizures, what’s cause?
A. Cerebral metastasis B. Hypocalcaemia (removed parathyroids) C. Hyponatraemia
53. Regarding Hurthle cell thyroid carcinoma what is true among the following? follicular thyroid cell “derived” cancer not follicular varient CA
A. Is adequately treated with thyroid lobectomy (total thyroidectomy) B. Is seldom bilateral C. Often metastasises to the cervical LNs
D. It is not related to previous neck radiation
54. Regarding Hurthle cell thyroid carcinoma, what is correct? 75% of the tumour consists of Hurthle cells
A. Can be readily diagnosed with fine-needle aspiration (cannot DD adenoma & carcinoma) B. Is often associated with previous neck radiation
C. Is more aggressive than follicular thyroid carcinoma D. Typically shows avid I 131 uptakes
Hurthle cell tumours can be benign in the form of Hurthle cell adenomas. Malignant Hurthle cell carcinomas are characterised by capsular invasion, vascular
invasion, infiltration of the thyroid gland and/or distant lymph nodes, and metastatic spread
55. Which of the following results of thyroid fine-needle aspiration indicates surgical treatment?
A. Nodule that disappeared on aspiration B. Anaplastic thyroid cells
C. Clumps of follicular cells (follicular thyroid adenoma/carcinoma lobectomy and isthmusectomy / if cancer ->> thyroidectomy + Radioiodine ablation)
D. Lymphoma (R-CHOP [Rituximab, cyclophosphamide, doxorubicin, vincristine, prednisone FOR 3 weeks+/- radiotherapy]
56. A patient with thyroid mass measuring 2x3 cm on one lobe, patient is asymptomatic, TSH and T4 are normal, Bethesda IV, what’s
next step in management?
A. Hemithyroidectomy (Hemithyroidectomy = lobectomy. malignancy risk 25-40%) B. Nodule excision C. Total thyroidectomy
57. What is true for Follicular thyroid carcinoma?
A. is the most common thyroid cancer B. is readily diagnosed with fine-needle aspiration C. is commonly multifocal
D. Spreads via haematogenous route
Distant metastases from follicular thyroid carcinoma are mainly haematogenous and are commonly observed in the lungs and bones.
58. A patient with hot thyroid nodule measuring 3x3 cm was removed by thyroid lobectomy, histopathology showed 8mm papillary thyroid
cancer, what is the most appropriate management?
A. Completion thyroidectomy B. Follow up 3 months (< 1cm follow up. > 1cm completion thyroidectomy) C. RAI D. Chemotherapy
In general, if less than 1 cm do follow up if larger than 1 cm do completion thyroidectomy
59. A 33 y.o pregnant lady about 14 weeks she has painless thyroid mass with scan showed as cold nodule, what is the next best option?
A. Thyroxine oral B. PTU C. Total thyroidectomy if FNAC showed malignant D. Hemithyroidectomy (if FNAC showed BIRAD IV)
Thyroid nodule with low TSH >> US >> hypoechoic >> Thyroid scan>> if cold >> FNAC if suspicious or benign hemithyroidectomy if malig>> thyroidectomy
60. Recurrence of thyroid carcinoma after definitive treatment is best detected by which test?
A. Ultrasound of the neck B. Thyroid-stimulating hormone level measurement C. Thyroglobulin serum measurement
D. Computed tomography scan of the neck E. Triiodothyronine/thyroxine measurement
61. A 55-year-old patient was diagnosed with autoimmune thyroiditis, with a history of progressive enlargement of the right lobe of the
thyroid. FNA report: malignant cells, which of the following is most likely the type of thyroid malignancy?
A. Papillary B. Medullary C. Anaplastic D. Lymphoma
N.B If history of Hashimoto, think about both Papillary + Lymphoma Find out the other DD --> If rapidly progressive, compressive symptoms,
fever so, it’s Lymphoma (in 70% due to Hashimotos disease) if not so it is just papillary

62. All the following are true except?


A. The most common cause of 1ry hyperparathyroidism is adenoma B. Stensen’s duct open opposite 2nd upper molar tooth
C. RLN injury during thyroidectomy can be permeant D. Thyroid malignant lymphoma should be treated surgically (R-CHOP+ RTH)
63. What is the most common thyroid cancer associating the Hashimoto thyroiditis?
A. Papillary B. Follicular C. Medullary D. Lymphoma
Thyroid lymphoma is frequently accompanied by Hashimoto’s thyroiditis, and it may be difficult to differentiate the two entities histologically. Also, PTC
64. A 60-y.o patient came with a history of Hashimoto’s disease who developed a huge goiter, biopsy of the thyroid gland was taken which
showed malignant thyroid cancer, which type of carcinoma could this be?
A. Anaplastic B. Papillary C. Medullary thyroid cancer D. Thyroid lymphoma (enlarge rapid, compressive symptoms, fever)
65. What is the optimal management for stage II thyroid lymphoma?
A. Total thyroidectomy B. Total thyroidectomy and radical neck dissection C. Total thyroidectomy and chemotherapy
D. Chemoradiation (R-CHOP+RTH) E. Cervical radiation
Treatment for large-cell lymphoma is CHOP (cyclophosphamide, doxorubicin hydroxydaunorubicin], vincristine, and prednisone) regimen or rituximab+RTH
66. A patient with hard thyroid (Riedel’s thyroiditis), management? Initially: Corticosteroids.
If no response after steroids? Do decompressive Isthmectomy Medical ttt for inflammatory component of Riedel thyroiditis traditionally consists of
Corticosteroids and Tamoxifen. Other agents such as Mycophenolate and Rituximab have also been used. Thyroid hormone supplementation/replacement
is used to treat the associated hypothyroidism.
67. What is the best treatment for Riedle thyroiditis with compression manifestations and dyspnoea? Isthmectomy and/or tracheostomy
68. A tracheostomy-related tracheoinnominate fistula is best managed by which of following?
A. Division of innominate artery and ligation of both fistula ends B. Division of innominate artery and vein graft
C. Division of innominate artery and polytetrafluoroethylene graft D. Primary repair of the innominate artery
The open surgical technique involves median sternotomy or a variation of it such as a collar incision with a partial sternotomy, ligation, and division of
the innominate artery. Ligation without division of the innominate artery should not be performed as the artery can re-fistulise.
69. Thyroid cancer case had mixed component solid and cystic, total size 2x3 cm, solid 2 cm, what best modalities to reach the diagnosis?
A. Core biopsy from solid (if solid 1-1.5 or mixed >1.5 – 2 cm & 0.5 in high risky) B. FNA aspiration from cyst C. Core needle biopsy
Presence of suspicious US features, we perform FNA biopsy of mixed nodules ≥1.5 to 2 cm, or if the solid component > 1 to 1.5 cm (0.5 cm in high-risk patients)
70. Which thyroid cancer secretes calcitonin?
A. Medullary B. Follicular C. Anaplastic D. Papillary
71. A 32 y.o male with neck mass 2x3 cm, his labs showed high calcitonin, what is the most likely diagnosis?
A. Papillary B. Anaplastic C. Medullary D. Lymphoma
72. A patient with neck mass and family history of parathyroid cancer investigation: Congo red positive what is likely diagnosis?
A. Medullary cancer (for stromal amyloid deposition) B. Parathyroid adenoma C. Anaplastic thyroid cancer
73. A patient with hypothyroidism symptoms, TSH high and T4 normal what is the most likely diagnosis? (Hashimoto thyroiditis)
74. What is true regarding medullary thyroid cancer? Core biopsy is more sensitive than FNA for diagnosis & immunohistochemical studies
A. More common than papillary thyroid cancer B. Tend to metastasise by lymphatics
C. Genetic survey may lead to prophylactic total thyroidectomy
Primary thyroid lymphoma affects the thyroid gland first, followed by spread to the lymph nodes and other organs later. Secondary thyroid lymphoma
affects lymph nodes and other organs first followed by spread to the thyroid later.
75. Multiple endocrine neoplasia is associated with germline mutation of which of the following?
A. P53 gene B. RET proto-oncogene C. N-myc gene D. APC gene
76. A 27 y.o male with painless midline neck mass, he has his father at age of 63, died due to similar neck mass this boy has his mass
increased more in the last 3 months recently developed; hoarseness, dysphagia, and respiratory difficulty what’s most likely diagnosis?
A. Anapastic carcinoma B. Hurthle cell CA C. Medullary thyroid cancer (sporadic 75%, MEN2 in 25%) D. Papillary thyroid cancer
FNAB as the procedure of choice in solid thyroid nodules if >0.5 cm in size, ≥1 cm if suspicious US features, > 1.5 if not suspicious. Total thyroidectomy &
bilateral central neck dissection (level VI) for all patients with medullary thyroid carcinoma, whose tumour is ≥1 cm or who have bilateral thyroid disease.
77. What is the hallmark of multiple endocrine neoplasia type 2 syndromes?
A. Hyperparathyroidism B. Phaeochromocytoma C. Medullary thyroid carcinoma N.B MTC: congo red, calcitonin, amyloid tissue, familial
D. Pituitary adenoma E. Neural gangliomas
78. A patient with huge colloid goiter with pressure manifestations like dyspnoea and dysphagia, patient is euthyroid, management?
A. Anti-thyroid drugs B. Radioactive iodine C. Subtotal thyroidectomy (to relieve compressive symptoms)
79. A patient with thyroid mass, about 2 cm in one lobe, FNA shows amyloid substance, what is the optimal for treatment?
A. Lobectomy with ipsilateral neck dissection B. Lobectomy with radiation
C. Total thyroidectomy with modified neck dissection (Medullary thyroid cancer)
D. Total thyroidectomy with radiation (if + ve surgical margins, mediastinal extension or if metastatic as a palliative )
Amyloid tissue is characteristic for medullary thyroid carcinoma. The primary and mainstay treatment for medullary thyroid carcinoma is extensive and
meticulous surgical resection for loco-regional disease and is the only current means of cure for MTC. Positive surgical margins or mediastinal extension
may be an indication for adjuvant radiotherapy, and external beam radiotherapy (EBRT) may provide a palliative controlling bony metastases symptoms.
80. What is the optimal management of medullary thyroid cancer in multiple endocrine neoplasia type 2 (MEN 2)?
A. Subtotal thyroidectomy B. Total thyroidectomy ± Radical neck dissection C. Hemithyroidectomy
D. Radioactive iodine E. Radiotherapy F. Thyroid lobectomy and cervical lymphadenectomy
1. MTC confined to the neck with no evidence of cervical LN on preoperative US, especially if large intrathyroidal MTC, with calcitonin is >20 pg/ml;
total thyroidectomy with prophylactic bilateral dissection of the central lymph node compartments is the preferred initial treatment and prophylactic
dissection of uninvolved contralateral lateral neck compartments for serum calcitonin >200 pg/mL 2. Evidence of cervical lymph node involvement; total
thyroidectomy with bilateral central compartment dissection and dissection of the involved lateral neck compartment(s) is the preferred initial treatment
81. A patient showed rapidly growing thyroid mass in 10 weeks, FNA showed it is malignant, what’s the type of malignancy?
A. Follicular B. Papillary C. Medullary D. Anaplastic (US >> FNAC with staging by CECT and brain MRI)
The initial radiological tumour staging should include cross-sectional imaging in particular, CT with contrast (or MRI) of the neck, chest, abdomen, and
pelvis and, if available, FDG PET/CT. Contrast-enhanced imaging of the brain (MRI preferred) should also be performed, if clinically indicated.
82. An old aged patient, suggestive of anaplastic thyroid, symptoms and signs of dysphagia, no dyspnoea, what is the next diagnostic test?
A. FNAC (if US already done to detect anaplastic giant multinucleated cyst) B. Open biopsy C. U/S (if not done before it is the initial)
D. Neck CT (to see extent, local invasions, metastasis, for staging and treatment plan)
83. A 76 yrs. old male patient, since the last 6 months, he has a neck mass, hard, fixed, with hoarseness of voice, what is the best next?
A. FNAC (after initial US if palpable >1cm; confirmative of anaplastic) B. Isthmectomy C. Neck CT D. Core biopsy
All solitary or dominant nodules ≥ 1cm in diameter should be evaluated by FNAC. There is variance in the size of the nodule as an indication for FNAB, as
follows: if > 0.5 cm size, >1 cm, in the absence of suspicious sonographic features, >1 cm if suspicious sonographic features are present; >1.5 cm if no
suspicious sonographic features are present (NCCN).
84. What is the best treatment for anaplastic thyroid cancer? Palliative thyroidectomy for unresectable with compressive symptoms, palliative
chemo XRT and total thyroidectomy for resectable lesion. Treatment of anaplastic thyroid carcinoma is mostly palliative.
NCCN guidelines recommend attempting total thyroidectomy in patients with resectable disease. The ATA guidelines strongly recommend surgical
resection for patients with confined (stage IVA/IVB) ATC in whom R0/R1 resection is anticipated. Radical resection (eg, laryngectomy, tracheal resections,
oesophageal resections, major vascular or mediastinal resections) is generally not recommended given the poor prognosis of ATC and should be considered
only very selectively after thorough discussion by the multidisciplinary team.
85. What is the rate of axonal regeneration after nerve injury?
A. l cm/month B. 1mm/day (1-3mm/d) C. 1mm/week D. l cm/week
86. Which of the following is preserved in radical neck dissection?
A. Internal jugular vein B. Sternomastoid muscle C. Spinal accessory nerve D. Submandibular salivary gland
E. Posterior belly of the digastric muscle
87. A post thyroid surgery patient developed loss of high-pitched sound, which nerve is affected?
A. Superior laryngeal nerve injury (External branch of SLN) B. RLN C. Both
88. Hard woody thyroid nodule with compression manifestations and history of hypothyroidism episodes and jaundice, best diagnosis?
A. FNA (Cytology) B. Radionuclide C. Thyroid function D. Isthmectomy or open biopsy
Hypothyroidism can cause a cholestatic condition with a 50% decrease in bile flow and in bilirubin maximum biliary excretion and with an increased
proportion of conjugated bilirubin in liver and plasma. Cholestasis induced by hypothyroidism. Only medullary thyroid cancer can metastasise to liver,
lungs, and bones and are multiple in involved organs and if invaded recurrent laryngeal can cause hoarseness and dysphagia.
89. In the case of woody neck what is the best investigation?
A. TSH B. Biopsy C. MRI N.B. Biopsy to DD simple fibrosis, infection or malignancy
90. Woody neck scenario patient presented by hoarseness of voice what is the next step?
A. Decompressive thyroid isthmectomy and for tissue diagnosis B. Total thyroidectomy C. Core biopsy
91. A patient with a known case of hypothyroidism presented by thyroid lump, with hoarseness of voice and dysphagia, O/E: woody firm
fixed thyroid swelling, what is the best investigation?
A. FNAC B. Open biopsy C. TSH D. Iodine isotope
92. An old age male patient did total thyroidectomy he developed swelling in the neck with inspiratory stridor and shortness of breath in
the ward, how you will manage?
A. Thoracotomy B. Surgical exploration and haematoma evacuation C. Drainage D. ECG, CXR and O2 mask
If suspected or diagnosed haematoma in the operating room, must evacuate it rapidly never conserve it, if discovered in the ward do bedside evacuation
and flush the drain, see if disappeared monitor his vital and advise him if any complain to call urgently if recur after bedside evacuation, shift fastly to OR
We recommend using the SCOOP approach (skin exposure; cut sutures; open skin; open muscles (superficial and deep layers and pack wound).
93. A post left thyroid lobectomy patient, complaining of severe shortness of breath after surgery and pointing to her neck what to do?
A. Take her back to surgical theater B. Bedside evacuation of haematoma C. Nasal cannulation
94. Post thyroidectomy fever, hypertension and tachycardia, what is the diagnosis?
A. Thyroid storm (fever, tachycardia, hypertension, & neurological and GI symptoms ) B. SIRS C. Pneumonia D. Wound infection
95. What is the effect of cutting the recurrent laryngeal nerve unilaterally during thyroidectomy?
A. Both cords will be away from midline B. Ipsilateral cord paralysis will be midline with voice hoarseness
C. Both will be midline with low pitched voice D. Bilateral RLN injury is less serious than unilateral
Unilateral RLN …… Dyspnoea on exertion and severe dysphonia to complete paralytic aphonia, and, frequently, aspiration of food and drink into the trachea
Unilateral Ext. branch of superior laryngeal N & RLN…. Hoarseness of voice Bilateral RLN …. Stridor and aphonia
Bilateral Ext & RLN …… Aphonia Injury of external laryngeal nerve …… loss of high-pitched sound.
96. A patient post thyroidectomy developed aspiration and hoarseness, which nerve injured?
A. Superior laryngeal nerve B. Recurrent laryngeal nerve
Hoarseness or aspiration = RLN. The patient will not be able to sing high notes or speak for long = Superior laryngeal
97. What is the most affected nerve in patent ductus arteriosus (PDA) surgery?
A. Phrenic Nerve B. Vagus Nerve C. Left Recurrent laryngeal Nerve D. Cardiac plexus
98. An old age patient with long standing large multinodular goiter, he underwent surgery, after the operation there was hoarseness of
voice, indirect laryngoscopy done in the clinic, and there was no vesible pathology of vocal cords movement, what is the cause?
A. Unilateral partial RLN injury B. Bilateral partial RLN injury C. Tracheomalacia D. External laryngeal nerve injury
Dyspnoea although all intact this means he has tracheomalacia due to loss of support of trachea by goiter
Hoarsness of voice this means he has Ext. LN injury leading to loss of cricothyroid muscle as a tensor
99. A female patient previously healthy, she has with bone pain, flank pain, asthenia, high PTH, high level of Ca, what is the diagnosis?
A. Primary hyperparathyroidism (bones, stones, abdominal moans, and psychic groans due to hypercalcaemia) B. Parathyroid adenoma
C. Secondary hyperparathyroidism (due to chronic renal failure & vit D deficiency>> hypocalcaemia) D. Tertiary HPT(post 2ry e hypercalcaemia)
100. A patient with flank pain, bone pain, with lateral side neck hard mass, with hypercalcaemia, what is the most likely diagnosis?
A. Parathyroid Carcinoma B. Metastasis from anaplastic carcinoma of thyroid C. PTH adenoma
D. Primary hyperparathyroidism E. Malignant thyroid nodule
101. A patient complaining of bone pain, depression, loss of memory, fatigability, what is the most likely cause?
A. High serum Ca and normal PTH (1ry HPT) B. Decrease Ca urine level C. Decrease serum phosphate level
Primary hyperparathyroidism is diagnosed based upon levels of blood calcium and parathyroid hormone. In most people with PHPT, both levels are higher
than normal. Occasionally, a person may have an elevated calcium level and a normal or minimally elevated PTH level
102. What is the most common affected gland in MEN1 syndrome?
A. Parathyroid B. Thyroid C. Adrenal D. Pancreas
103. What are the most common and predominant parathyroid pathology in MEN I syndrome?
A. Parathyroid adenoma B. Parathyroid carcinoma C. Parathyroid hyperplasia D. Parathyroid hypertrophy
Parathyroid hyperplasia, pituitary adenoma (prolactinoma) and pancreatic neuro endocrine tumours
(e.g., Zollinger-Ellison syndrome, Insulinomas, VIPomas, Glucagonomas) PPP
MEN2 A = 2M, 1P Parathyroid tumour, Phaeochromocytoma and Medullary thyroid carcinoma. B = 1P 2M Phaeochromocytoma, Mucosal
neuromas/Marfanoid habitus, Medullary thyroid Carcinoma
104. In multiple endocrine neoplasia type I, what is true?
A. Almost all patients have parathyroid hyperplasia B. Almost all patients have pancreatic endocrine tumour
C. Almost all patients have pituitary adenoma D. All patients have hyperparathyroidism, pancreatic, and pituitary lesions
105. What is the most common cause of hypercalcaemic crisis?
A. Sarcoidosis B. 1ry hyperparathyroidism C. 2ry hyperparathyroidism D. Malignancy E. Renal failure
Hypercalcaemia is most commonly results from malignancy or primary hyperparathyroidism, Other, less common causes of elevated calcium include
increased intake or absorption, granulomatous disease, immobilisation, and thiazide diuretic use. Hypercalcaemic crisis does not have an exact definition,
although marked elevation of serum calcium, usually more than 14 mg/dL (3.49 mmol/L) is associated with acute signs and symptoms of hypercalcaemia
as nausea, vomiting, constipation, polyurea, polydipsia, weakness, muscle and joint pain, mental changes, confusion, headache, depression up to coma in
severe hypercalcaemia. Calcium is maintained within a narrow range from 8.5 to 10.5 mg/dl (4.3 to 5.3 mEq/L or 2.2 to 2.7 mmol/L). Normal values and
reference ranges may vary among laboratories as much as 0.5 mg/dl. Each 1mg/dL= 0.25 mmol/L.
106. Which is the FDA approved drug to treat malignancy related hypercalcaemia?
A. Denosumab (Xgeva; treating osteoporosis) B. Recombinant PTH C. Radiotherapy D. Over hydration with NaCl
The FDA has approved the use of gallium nitrate for the treatment of cancer-related hypercalcaemia that is resistant to hydration. The drug is marketed
under the trade name Ganite by Genta Incorporated. Denosumab is a monoclonal antibody that specifically binds to RANKL, which is transmembrane
protein essential for formation, function, and survival of osteoclasts that responsible for bone resorption. It is indicated for hypercalcaemia of malignancy
refractory to Bisphosphonate therapy
107. What is the initial step in management of a hypercalcaemic crisis as intravenous drug?
A. Calcitonin B. Normal saline hydration and forced calciuresis C. Furosemide D. Mithramycin
In ionised calcium assay with a normal range of 4.8 to 5.6 mg/dL (1.2 to 1.4 mmol/L), mild, moderate, and severe hypercalcaemia may be defined as follows:
Mild: Ionised calcium 5.6 to 8 mg/dL (1.4 to 2 mmol/L). they should avoid thiazide, lithium, volume depletion, high calcium diet or supplements and vit D
Moderate: Ionised calcium 8 to 10 mg/dL (2 to 2.5 mmol/L). if sudden rise must be treated with treated saline hydration and bisphosphonates
Severe Ionised calcium 10 to 12 mg/dL (2.5 to 3 mmol/L). Initial therapy of severe hypercalcaemia includes the simultaneous administration of intravenous
(IV) isotonic saline, subcutaneous calcitonin, and a bisphosphonate (typically, IV Zoledronic acid). The administration of calcitonin plus saline hydration
should result in substantial reduction in serum calcium concentrations within 12 to 48 hours. The bisphosphonate will be effective by the second to fourth
day and provide a more sustained effect, thereby maintaining control of the hypercalcaemia.
108. What is the most common cause of hypercalcaemia in hospitalised patients?
A. Primary hyperparathyroidism (for outpatient) B. Metastatic carcinoma(hospitalised) C. Sarcoidosis D. Immobility
Malignancy remains the most common cause of hypercalcaemia in hospitalised patients. Primary hyperparathyroidism is the most common cause of
hypercalcemia in the outpatient setting.
109. A patient underwent left lower parathyroidectomy for primary hyperparathyroidism (adenoma). He presented 4 months later with
depressed mood and fatigability. Both parathyroid hormone and calcium were high, what is the most common cause?
A. Parathyroid hyperplasia (>6 months recurrent) B. Missed adenoma (< 6months) C. New adenoma D. Parathyroid cancer
Less than 6 months: Missed adenoma not resected on initial surgery After 6 months: Parathyroid hyperplasia (recurrent)
110. Fracture, x-ray shows multiple and recurrent saddle stones, all labs are normal except PTH was very high, what else you must do?
>> Sestamibi scan is a procedure in nuclear medicine which is performed to localise parathyroid adenoma, which causes Hyperparathyroidism.
111. A scenario of hyperparathyroidism, what’s preoperative investigation? Sestamibi scan for localisation
112. What is the best method for localisation of parathyroid nodule before surgery?
A. MRI scan(2nd. line) B. Sestamibi scan (90%) C. Ultrasound the 1st. line (93%) D. CT scan if 4 dimentional >> 93.7%
113. What is the best method of parathyroid gland localisation intraoperatively when surgeon cannot detect it normally?
A. Intraoperative USG ( preop.) B. CT scanning C. Sestamibi scan (preop.)
D. Indocyanine green fluorescence (95% detection) or and parathyroid autofluorescence
114. From which pharyngeal pouch, the inferior parathyroid glands originate?
A. First pharyngeal pouch B. Second pharyngeal pouch C. Third pharyngeal pouch D. 4th. pharyngeal pouch (superior)
Embryologically, the parathyroid glands derive from the endoderm of the third and fourth pharyngeal pouches. The third pharyngeal pouch gives rise to
the inferior parathyroid glands, while the superior parathyroids arise from the fourth pharyngeal pouch. Due to their relatively long course of descent,
the final location of the inferior glands is more variable.

ADRENAL
115. What is the most potent stimulus for antidiuretic hormone secretion?
A. Hypovolaemia B. Hyponatraemia C. Hyperkalaemia D. Raised serum osmolarity
ADH or Arginine Vasopressin, secretion can be stimulated by several mechanisms. The two prominent stimuli are changes in plasma osmolality and changes
in blood pressure or blood volume. Other triggers for AVP secretion include emetic stimuli, hypoglycaemia, pain, thermic stresses, hypoxia, hypercapnia,
acidosis, and angiotensin II stimulation
116. In the elderly what is true change?
A. There is decreased insensible water loss B. There is increased antidiuretic hormone response
C. There is increased volume of distribution of water-soluble medications D. There is decreased volume of distribution of lipid-soluble medications
117. An old patient who was exposed to head trauma with traumatic brain injury, he is comatosed admitted to ICU, and after doing full
investigations found that he has polyuria and vasopressin or ADH was low about 0.1 (N 1-10ng/dl) what is the possible diagnosis?
A. Inappropriate ADH secretion (increased (ADH) B. ADH deficiency (Central DI) C. Nephrogenic diabetes insipidus
Normal values for ADH can range from 1 to 5 pg/mL (0.9 to 4.6 pmol/L). Neuroendocrine dysfunction has been recognised because of traumatic brain
injury and consists of both anterior and posterior pituitary insufficiency; water and electrolyte abnormalities. The acute head trauma can lead (directly or
indirectly) to dysfunction of the hypothalamic neurons secreting ADH or of the posterior pituitary gland causing post-traumatic DI
118. Case about a child with increased water consumption, going frequently to the toilet, Urine and serum osmolality not mentioned, no
mention of any electrolyte, with no response to water deprivation test, no other complaints?
A. Central DI B. Peripheral DI C. SIADH
DDAVP (desmopressin) can help differentiate the two diseases: administration results in improvement of urine osmolality in the central, but not in the
peripheral; nephrogenic. The treatment of DI includes a low-salt, low-protein diet, diuretics, and NSAIDs supplemented by DDAVP in central.
119. Where is the site of action of antidiuretic hormone?
A. Collecting ducts B. Glomeruli C. Proximal tubules D. Distal tubules E. Loop of Henle
120. Where most renal absorption of sodium take place?
A. The proximal tubules B. The loop of Henle C. The distal tubules D. The collecting ducts
121. A case of post head trauma, diabetes insipidus, polydipsia and polyuria, what is the cause? Loss of or decreased anti-diuretic hormone
122. Which of the following is consistent with syndrome of inappropriate antidiuretic hormone (SIADH)?
A. Hypovolaemia B. Increased urine sodium C. Hypernatraemia D. Plasma hyperosmolality E. Excessive diuresis
In SIADH, the serum sodium is low, with low serum osmolality, urine sodium is high, with high urine osmolality, low urine output (no diuresis). Criteria of
diagnosing SIADH are low serum or plasma osmolality< 275 mOsm/kg and increased urine osmolality > 100 mOsm/kg with totally hypervolaemia or
euvolaemia except is associated with other problem as vomiting, diarrhoea or diuretic therapy. Mnemonic for SIADH: SIADH Sodium Is Always Down
Here, mnemonic of symptoms: FLAT fatigue, lethergy, anorexia, thirst, mnemonic of drug causes:SIADH cannot void=SSRI, Indomethacin(analgesics)
antidepressants, diuretics as thiazide, haloperidol, cyclophsphamide and carpamazepine vincristine.
123. A 75-year-old type 2 DM undergoes a bowel resection for colonic cancer. He is well prior to the operation with well-controlled DM and
no other underlying condition. The operation was successful, and the patient is given postoperative insulin and IV dextrose. 2 days post-
operative he became agitated Na; low, K; low, Cr; low. Serum osm: low, Urine osm: normal which of the following is the most likely cause?
A. Water overload or intoxication (as normal urine osmolality) B. Addison's disease C. Diabetic nephropathy D. SIADH
124. An 18-month baby girl with asymmetrical breast enlargement, other examination normal, what you should do?
A. Brain MRI B. Abd CT C. Pelvic CT D. Pelvic US (to R/O peripheral precocious puberty as ovarian Leydig-Sertoli cell cancers)
125. Which one of the following is a physiologic change of aging?
A. Increased total body water B. Decreased antidiuretic hormone secretion ADH C. Decreased aldosterone secretion
D. Exaggerated thirst response E. Decreased atrial natriuretic peptide (ANP)
126. What is the first hormone that will be secreted from kidney as a response for renal hypoperfusion = hypovolaemia?
A. Renin (renal hypoperf -> renin-> angiotensinogen to angiotensin I, by ACE-> angiotensin II-> vasocon & increase BP and express both aldosterone & ADH)
B. Aldosterone C. Norepinephrine D. Cortisone
127. Angiotensin-converting enzyme is produced by which of the following?
A. Type I pneumocytes B. Type Il pneumocytes C. Hepatocytes D. Juxtaglomerular cells
E. Vascular endothelial cells
128. In renal artery stenosis, angiotensin-converting enzyme inhibitors will result in which of the following?
A. Decrease glomerular filtration rate B. Increase glomerular filtration rate C. Result in dilation of the afferent arterioles
D. Result in constriction of the afferent arterioles E. Result in constriction of the efferent arterioles
129. What is the most common cause of renovascular hypertension?
A. Aneurysm of the renal artery B. Fibromuscular hyperplasia C. Renal artery calcinosis D. Renal artery atheroma
130. An elderly taking multiple anti-hypertensive medications, but still have persistent high BP, US showed asymmetry in kidneys sizes
what’s the cause?
A. Renal artery stenosis B. Adult PCK C. Pheochromocytoma induced HTN
131. An old patient who has long standing HTN not controlled although many drug treatment lines, he underwent, ultrasonography done,
that showed small left kidney then renal arteriography that showed renal artery stenosis, what is next investigation to do?
A. Renal CTA B. Renal MRA C. Renal biopsy D. Rertrograde pyelography E. Balloon angioplasty
The gold standard for diagnosing renal artery stenosis is renal arteriography. However, a variety of less invasive tests are being employed for evaluation for
testing purposes. The choice of test should be based upon institutional expertise and patient factors.
132. What is the site of action of aldosterone?
A. Proximal renal tubule B. Distal renal tubule C. Collecting ducts D. Glomeruli E. Loop of Henle
133. A post operative case with pulmonary oedema what is the cause of oedema? Antidiuretic hormone; ADH
134. What is the primary regulator of aldosterone secretion?
A. Angiotensin II B. Serum K level C. Adrenocorticotropic hormone D. Prostaglandins
135. An RTA patient with marked bleeding and hypovolaemia, with decrease of renal blood flow, which hormone will be secreted in
response to renal hypoperfusion?
A. ADH B. Steroids C. Catecholamines C. Renin (the first one secreted due to renal hypoperfusion)
D. Aldosterone (2ry to renin angiotensin system, to reabsorb sodium, leading to the water reabsorption >> increase blood volume) aided by ADH that
lead to water reabsorption)
136. What is true for Aldosterone action?
A. Stimulates sodium reabsorption in proximal renal tubules B. Stimulates Sodium reabsorption in distal renal tubules
C. Stimulates potassium reabsorption in proximal renal tubules D. Stimulates potassium reabsorption in distal renal tubules
137. A 55-yrs. patient, with Cushing's syndrome is admitted for adrenalectomy, which drugs should be given? Pre-op corticosteroids
Cushing = Cortisone excess so have to decrease it. Addison’s = Cortisone loss have to give i.e CU= cut AD=add
138. A Cushing syndrome patient with proven right adrenal adenoma on MRI for resection, what is the preoperative drug to be given?
A. Hydrocortisone B. Fludrocortisone C. Methylprednisolone
139. A patient with polydipsia, polyuria, hyperglycaemia and hypertension, labs with high cortisol level, what is diagnosis?
A. Hypoadrenalism (Addison's disease) B. Hypercortisolim (Cushing's syndrome /disease) C. Hyperaldosteronism (Conn’s synd.)
140. A 30 y.o female patient with history of ulcerative colitis which relapsed with about 10 times bloody diarrhoea, fever and severe
abdominal pain, she was kept in the ICU after query fulminant UC, she was taking cortisone treatment for some adrenal disease, after she
was admitted to ICU, developed septic shock with labs showed hyponatraemia, hypoglycaemia, hyperkalaemia, acute renal injury, non-
anion gap metabolic acidosis, and eosinophilia, what is the most important drug to be given for saving that patient?
A. Fludrocortisone 0.1 mg orally B. Dexamethasone C. Hydrocortisone 100mg IV bolus then 50, 6 hourly D. NaCl with calcium
Emergency measures Establish IV access with a large-bore needle. Draw blood for immediate serum electrolytes and glucose and routine measurement of
plasma cortisol and ACTH. Do not wait for laboratory results. Infuse 2 to 3 liters of isotonic saline or 5% dextrose in isotonic saline as quickly as possible.
Frequent haemodynamic monitoring and measurement of serum electrolytes should be performed to avoid iatrogenic fluid overload.
Give hydrocortisone (100 mg IV bolus), followed by 50 mg IV every 6 hours (or 200 mg/24 hours as a continuous IV infusion for the first 24 hours).
If hydrocortisone is unavailable, alternatives include methylprednisolone and dexamethasone. Saline must be administered if dexamethasone is given
instead of hydrocortisone. Taper parenteral glucocorticoid over 1 to 3 days, if precipitating or complicating illness permits, to oral glucocorticoid
maintenance dose. For patients with primary adrenal insufficiency, begin mineralocorticoid replacement with fludrocortisone, 0.1 mg by mouth daily, when
saline infusion is stopped.
141. Regarding the adrenal gland, what is correct?
A. The adrenal cortex does not have nerve supply B. The adrenal medulla is supplied by postganglionic adrenergic fibers
C. The right adrenal vein drains into the renal vein D. The left adrenal vein drains into the inferior vena cava
142. A patient with MEN2, has medullary thyroid carcinoma, what is the most common associated diseases?
A. Phaeochromocytoma B. Hyperparathyroidism C. ZES D. Marfanoid habitus
143. An ICU patient developed hypotension, hyponatraemia, hypoglycaemia and hyperkalaemia, what is the most important drug given to
save the patient?
A. Hydrocortisone (addison’s disease) B. Fludrocortisone C. Calcium D. NaCl
144. A patient presented with central obesity, striae, and buffalo hump; what is the best investigation?
A. CT brain B. Urine cortisone Cush= cortisol high= remove cortisol source addison adrenal insufficiency = so add cortisone
145. A 52-year-old female known case of DM and knee osteoarthritis, has been using NSAID to relieve her pain, she developed HTN, Na
normal, K low borderline, what do you think the cause of HTN?
A. Essential HTN B. Phaeochromocytoma C. NSAIDs induced HTN D. Primary hyperaldosteronism
146. A patient with dyspepsia, polyuria, hypernatraemia, hypertension and hypokalaemia, what diagnosis?
A. Primary hyperaldosteronism (Aldosterone excess of hyperaldosteronism = Conn’s syndrome in 33% or bilateral adrenal hyperplasia in 66%)
B. 2ry. hyperaldosteronism
Surgery is the preferred treatment in primary hyperaldosteronism caused by unilateral disease. Laparoscopic adrenalectomy is preferred as it is associated
with fewer complications and a shorter hospital stay than open adrenalectomy. Complete adrenalectomy is preferred to partial adrenalectomy due to
greater efficacy and resolution of symptoms. For nonsurgical candidates, mineralocorticoid receptor antagonists (MRA) were preferred as medical therapy.
147. From the following what is the characteristic of primary hyperaldosteronism?
A. Hyperkalaemia B. Hyper reninism C. Hypertension (hypernatraemia, hypokalaemia) D. Hyperplasia of zona reticularis
148. A female patient with history of previous breast cancer 5 years ago, abdominal CT showed mass 5 cm at upper of the kidney, next?
A. CT guided FNA biopsy B. Hormonal assay (metanephrines aldosterone/renin ratio dexamethasone suppression, sex hormones before FNAC)
C. Laparotomy and excision D. MIBG scan
149. Aldosteronoma or Primary hyperaldosteronism is associated with which of the following?
A. Hypertension, hypokalaemia, high aldosterone and high renin levels B. Hypertension, hyperkalaemia, high aldosterone and low renin levels
C. Hypertension, hyperkalemia, high aldosterone and high renin levels
D. Hypertension, hypokalaemia, high aldosterone and low renin levels, mild hypernatraemia and hypomagneaemia
150. What is the most common site of ectopic pheochromocytoma?
A. Lower pole of the kidney B. Para-aortic tissue C. Mediastinum D. Pelvis
151. Hypertension in cases of extra-adrenal pheochromocytoma is caused by which of the following?
A. Combined epinephrine & norepinephrine B. Pure epinephrine C. Pure norepinephrine D. Renin, epinephrine, & norepinephrine
152. Which of the following is least appropriate or not indicated for evaluating adrenal incidentaloma?
A. Magnetic resonance imaging B. 24-hour urinary catecholamine testing C. 24-hour urinary cortisol testing
D. Low dexamethasone suppression test E. Fine-needle aspiration F. Serum potassium
FNAC is not accurate in differentiating between primary adrenal cancer and benign tumours. It is important to rule out pheochromocytoma before
performing FNAC. Every patient with an adrenal incidentaloma undergoes careful assessment, including clinical examination, for symptoms and signs of
adrenal hormone excess. All patients with adrenal incidentalomas undergo a 1-mg overnight dexamethasone suppression test to exclude cortisol excess.
Pheochromocytoma be excluded by measurement of plasma-free metanephrines or urinary fractionated metanephrines. In patients with concomitant
hypertension or unexplained hypokalaemia, the aldosterone/renin ratio is used to exclude primary aldosteronism.
153. A patient with abdominal pain, mass in the adrenal 5 cm CT show hypodense and lipid tissue, normal hormonal labs what is next?
A. Reassurance (myelolipoma 4-6 cm) B. Image guided biopsy (if adrenocortical metastasis) C. Adrenalectomy
154. What is the indication for fine-needle aspiration biopsy of an adrenal incidentaloma?
A. Suspected pheochromocytoma B. Suspected Aldosteronoma C. Suspected adrenal carcinoma
D. Suspected adrenal metastases
155. What is correct regarding Clonidine?
A. An alpha1 agonist B. An alpha1 antagonist C. An alpha -agonist(analgesic, sedative and anxiolytic, anti HTN, migraine& menopausal flush)
B. An alpha1 and 2 agonists E. An alpha 2-antagonist
156. In the preoperative preparation of pheochromocytoma, what medications are given in order?
A. Diuretic and then B blockers B. Alpha blockers and then B-blockers C. B blockers and then alpha blockers
C. Diuretics and then Alpha -blockers
Preoperative preparation includes the use of alpha-adrenergic antagonists, beta-adrenergic antagonists with or without other antihypertensive agents,
fluid therapy as well as insulin therapy for hyperglycaemia if required
157. A 46-year-old male patient, with severe headache, visited a clinic where discovered a recent HTN, full labs and abdomen CT done
that showed an 8 cm adrenal mass, what other investigations to do?
A. 5 HIAA B. 24 hrs. Ca in urine C. Sestamibi scan D. 24-hour urinary Catecholamines and Metanephrines
158. A patient with hypertension, hypernatraemia, hypokalaemia, CT showed suprarenal hypoattenuated 3.5cm mass, what’s diagnosis?
A. Cushing B. Adrenal tumour (aldosteronoma)
159. A patient developed hypernatraemia, hypokalaemia, hypertension, CT showed 6 cm irregular suprarenal mass of hyperattenuation
with calcification and adherent to surrounding structures what is the most likely, diagnosis?
A. Adrenocortical Carcinoma (large, hyperattenuated, inhomogenous, irregular with calcification) B. Renal tumour C. Phaeochromocytoma

Endocrine pancreas
160. At morning of day of surgery, the patient has FBS 130 mg/dl (7.2 mmol/L). how to manage?
A. Postpone the operation B. SC insulin C. Do surgery at the end of the list D. IV insulin, sliding scale
161. A patient on metformin you admit him, what type of insulin you will give him as an inpatient?
A. Sliding insulin SC B. IV Insulin +long-acting insulin C. Insulin BID D. Fixed rate IV insulin
162. What is the most common systemic complication of diabetes mellitus?
A. Blindness (9%) B. Cardiovascular (HTN up to 70%) C. Renal disease (20%) D. Deafness E. Neuropathy (10-18%)
163. For Phosphorus what is correct?
A. Is a major extracellular anion B. Is passively absorbed from the gastrointestinal tract
C. Deficiency may result in insulin resistance D. Deficiency is rare in hospitalised patients
164. What is the hormone that indirectly affects the glucose blood level, causing hyperglycaemia, increase insulin resistance in tissues?
A. Cortisone B. Thyroxine C. Glucagon D. Growth hormones
165. Which hormone causes hyperglycaemia, directly antagonising insulin action and increase insulin resistance?
A. Thyroxin B. Glucagon C. Cortisol
Glucagon is a direct and powerful hormone which is insulin antagonist. Others work indirectly. Glucagon and epinephrine, prevent hypoglycaemia by increasing
gluconeogenesis and glycogenolysis and decreasing glucose uptake and consumption from peripheral tissues during fasting.
166. A type 1 diabetic, with abdominal pain, vomiting, high glucose, urinary glucose (+3) and ketone (+3) in urine, what is the best treatment?
A. Insulin B. IV Fluids (DKA 1st. h 1-1.5 L NaCl 0.9% then if <200 --> DNS) C. Antibiotics D. Oral hypoglycemic drugs
167. A 45 years, type 2 diabetic patient, generalised abdominal pain, vomiting, high glucose, negative urine glucose and ketone, management?
A. Insulin B. IV Fluids (manage HHS as DKA) C. Antibiotics
168. A trauma case, he was presented to ER with diabetic ketoacidosis, O/E peritonitis, lab; ketoacidosis, what’s best initial management?
A. Fluid replacement B. Laparotomy C. Antibiotics D. Oral hypoglycaemic
The best initial treatment for a case of DKA is IV fluids
169. Well known case of uncontrolled type 1 DM, was presented to ER with drowsiness and abdominal pain, labs were high sugar 400mg/dl,
pH 7.05, ++ ketone in urine, what is your management?
A. 10 units insulin+400 cc dextrose B. 0.1 unit/kg insulin subcutaneous C.NaHco3 D. One liter normal saline bolus
The current American Diabetes Association guidance on the management of DKA recommends using 0.9% sodium chloride initially as a 15–20 mL/Kg
bolus for haemodynamic resuscitation and then 250–500 mL/h of fluid until glucose is normalised and then 150–250 mL/h until DKA resolution
170. A diabetic female patient with history of DVT 25 years ago, and underwent excision of malignant melanoma of 2 yrs. back, today she
presented with painful swelling of the leg showing ugly limb with crustations, what is the diagnosis?
A. Haemosiderin B. Necrobiosis lipoidica diabeticorum
Necrobiosis lipoidica is a rare, chronic granulomatous disease of the skin. Skin involvement usually begins as red-brown or violaceous papules, plaques, or
nodules and rapidly progresses to yellow-brown, atrophic, telangiectatic plaques. The lower legs, especially the shins of the tibia, are by far the most
common sites of involvement. It occurs in association with diabetes mellitus but can occur in non-diabetic patients. Ulceration is a common complication,
occurring in 10 to 20 % of patients. Topical or intralesional administration of corticosteroids is often used as initial therapy.
171. A diabetic female patient presented with large leg swelling with hyperpigmentation, and crustation, what’s most likely diagnosis?
A. Lipodermatosclerosis B. Haemosiderin deposition C. Group A streptococcal infection

BREAST SURGERY
1. What is the prominent L.N drainage to the breast?
A. Axillary (75%) B. Pectoral C. Internal mammary D. Supraclavicular
About 75% of lymph from the breasts drains into the axillary lymph nodes, making them important in the diagnosis and staging of breast cancer
Axillary lymph nodes according to pectoralis minor muscle are Level I: lateral and inferior to the muscle. Level II: posterior to the muscle.
Level III (infraclavicular): medial and superior to the muscle
2. Which one of the following is correct regarding level 2 axillary lymph nodes?
A. Lie anterior to the pectoralis minor muscle B. Lie inferior the lower edge of pectoralis minor
C. Lie posterior to pectoralis minor D. Lie superomedial to pectoralis minor
3. A 24-year-old lady presents with a hard, mobile, well-circumscribed painless left breast mass that has been increasing in size from the
past few months, not related to her menstrual cycle, what is the most likely diagnosis?
A. Fat cyst (traumatic fat necrosis) B. Fibroadenoma C. Fibrocystic changes D. Intraductal papi lloma (difficult to palpate)
4. A 53 year old lady with pendulous breast, she has history of trauma to the breast 8 months ago with firm to hard 5 cm mass, above
nipple, in upper outer quadrant, not tender, dimples over it, or peau d’orange, no axillary lymph nodes, what is most likely diagnosis?
A. Haematoma B. Traumatic fat necrosis C. Breast cancer (no trauma, firm to hard, peau d’orange, inverted nipple, CNB confirm)
5. A breast feeder, has unilateral tender breast mass; involving and attached to the areola with many small cysts, what’s likely diagnosis?
A. Ductectasia (another DD is fibrocystic changes) B. Intraductal papilloma C. Fibroadenoma D. Mammary Paget’s disease
Antibiotic therapy may be tried, the most appropriate agents being Co-Amoxiclav or Flucloxacillin and Metronidazole.
Surgery is often the only option cure of this notoriously difficult condition; this consists of excision of all of the major ducts (Hadfield’s operation)
6. A previously lacating lady, has bilateral greenish nipple discharge; bilateral mammography and US showed duct dilatation what's next?
A. MRI B. Biopsy C. Follow up (Ductectasia) D. Complete excision
7. A 37 y.o lady with cystic breast mass, after needle aspiration, greenish fluid found, the mass completely disappeared, what’s the next?
A. Antibiotic B. Cytopathology (the initial even if not disappeared, if clear non bloody, no need for other workups) C. Mammography
D. Observation E. Tru cut F. Surgery
8. A middle aged lady with breast cyst after needle aspiration, clear fluid came but the swelling didnot completely resolve, what’s the next?
A. Mammography (initial after cytology) B. Fine needle C. Core biopsy (the best for tissue diagnosis) D. Breast US
If a mass disappears after aspiration of non bloody fluid, yellow, green or serous no further workup is necessary. If didnot disappear completely or bloody,
do mammography and biopsy, if not bloody, send the aspirate for cytological examination to R/O malignant cells, then to mammography if older than 35 or
even 30 if suspicious hereditary breast cancer, if mammography and cytological examination both normal so the cummulative negative predictive value for
breast cancer is 100%, if aspiration not satisfactory send after mammography to take biopsy for more confirmation or ruling out underlying breast cancer
9. What is the most common disease in lactating breast?
A. Bacterial (lactational) mastitis B. Abscess C. Fibrocystic disease D. Mondor’s disease E. Breast cancer
10. Image showed right inner lower quadrant breast mass 3x3 cm size, erythematous, of 2 days history, no LN detected, what’s best next?
A. Antibiotics B. I&D C. US guided aspiration D. Mammography
11. Lactating women present with right breast pain for 6 days on examination, hot tender swelling lateral to the right areola, with no skin
changes, no fever the patient started to take flucloxacillin. Ultrasound showed cystic lesion, thickened content, what next?
A. I&D (if thin, ischaemic or gang. skin >5cm, superficial, nonlactating, underling mass) B. Repeated aspiration (if intact skin <5cm, deep, lactating)
C. Excisional biopsy D. Continue antibiotics
12. A 30- years old breastfeeding female, presented with painful lump in her right breast O/E: redness, hotness, tenderness, no lymph
node enlargement, temperature, 37.1, she started antibiotics for a period, US report showed: thick cyst with fluid collection, diagnosed as
an abscess or cyst for clinical correlation, what is the most likely diagnosis?
A. Abscess (pain, responded to Abx, signs of inflam.) B. Cyst C. Fibrocystic disease D. Galactocele
13. A 21 y.o female with painless, mobile and firm breast mass, 2 cm size, CNB shows fibroadenoma, what is the best management?
A. Follow up B. Excision biopsy C. Danazol D. Surgical excision
E. Metformin never affects it (20% decrease its size)
14. A 32 year female, with small breast mass 1x1.5 cm, proved and confirmed clinically, mammographically and CNB as simple breast
fibroadenoma, she was afraid and too much worry, due to her mother died due to breast cancer at age 43 what is optimal management?
A. Reassurance and follow up (if increased in size do US /CNB) B. CNB and excision C. Prophylactic mastectomy
15. A 25 y.o lady has breast lump 3 cm, confirmed by US to be fibroadenoma, she is worried about it, and asked to be excised what’s next?
A. Excision after CNB (if >2cm, rapid growth, suspicious, patient request) B. Mammogram C. CNB D. Self-examination monthly
16. A 19 y.o female, with rounded, firm, mobile breast mass, 3 x 2 cm, on US and CNB; showed fibroadenoma, what’s management?
A. Excision (if >2cm, rapid growth, suspected malig, recurrent, phylloid, painful, for cosmetic, requested removal) B. Mammography
C. Give tamoxifen D. Follow up (if small, painless, slow growth)
17. What is true about Fibroadenoma?
A. Rare before age of 40 B. Most common cause of breast mass in 4th and 5th decades
C. Its size decreases with pregnancy D. Recurrence after excision of large lesions is possible
18. In TNM classification and staging of breast cancer, regarding to fibroadenoma what is incorrect?
A. Mostly benign B. Mostly mobile C. Diagnosis by mammogram D. May be huge E. Premalignant lesion
19. Which of the following is least appropriate when evaluating a 14-year-old girl with a breast lump?
A. Ultrasound B. Clinical follow-up C. Mammography (not< 35 except if familial Br. CA) D. Fine-needle aspiration
20. What is true regarding the fibrocystic changes of the breast?
A. Disappears after menopause B. Premalignant C. Need surgery
21. Regarding the Nipple discharge what is true?
A. Likely to be benign if spontaneous B. Likely to be malignant if bloody and unilateral
C. If ductogram is normal no need for further evaluation D. It is usually benign condition
22. A female CO of non-compressible breast mass, bilateral green nipple discharge, mammogram show BIRAD 2, what’s management?
A. Duct excision B. Reassure and follow up BI-RADS 2: benign probability of malignancy is 0%. BI-RADS 3 benign in 98% . 4A 2-9% malig
23. A 34-year female with breast fibroadenoma with atypia and hyperplasia, what is the cancer risk for that?
A. Her age B. Atypia (Atypical fibroadenoma; ill defined, irregular, heterogeneous, microcalcification, clefts) C. Size D. Hyperplasia
24. What is the most common cause of nipple discharge in non-lactating women?
A. Prolactinoma B. Hypothyroidism C. Breast cancer D. Fibrocystic disease/Ductectasia E. Intraductal papilloma
Pathological benign causes mostly unilateral and non-milky discharge as intraductal papilloma in 52%-57% of cases, mammary Ductectasia, fibrocystic
changes or breast abscess. Pathological malignant as ductal carcinoma in situ DCIS in 5- 15%. The most common cause of these discharges is intraductal
papillomas, but fibrocystic disease, advanced duct ectasia, cancer of the breast, and vascular engorgement in near-term pregnancy can also be causative.
25. A 35. y.o lady presents with a left nipple bloody discharge, by imaging it was suggestive of intraductal Papilloma, what to do next?
A. Complete affected duct excision B. Observation C. Mastectomy D. image-guided biopsy
Treatment of intraductal papilloma involves surgical excision and complete removal of the tumour. This is due to the possibility of upgrading to atypical
ductal hyperplasia or DCIS upon excision. Surgical excision, in the form of lumpectomy with complete removal of the papilloma, is recommended.
26. A female patient with breast mass 1 cm by mammogram, stellate mass with irregular border, microcalcifications and distortion of
surrounding structures, by true cut biopsy there is hyperplastic tissue, what is the optimal management?
A. Excisional biopsy by wire localisation preoperative (ADH to exclude upgrade/presence of Br. CA) B. MRM C. Simple mastectomy
27. A 55 yrs. old female, with 1 cm breast mass seen by US, what to do next?
A. FNAC B. Core biopsy C. Mammogram D. MRI
28. A woman with a left retro areolar hard breast mass, with serous discharge for 9 months, U/S show irregular shape, she underwent
mammogram and it showed: irregular, indistinct or speculated mass with suspicious microcalcifications and axillary LN involvement,
BI-RAD V, what is the next step?
A. Excisional biopsy B. Core biopsy +/- surgical excision C. Modified radical mastectomy D. Breast MRI
For benign tumours do excisional biopsy but in query malignant do core needle biopsy. BIRAD V is highly suggestive of malignancy, >95% of cases.
29. A 33 years old, female patient, presented with breast mass in upper outer quadrant with non- cyclic pain, next or initial investigation?
A. CNB (best investigation after doing mammography) B. Mammography (initial) C. MRI breast D. Lumpectomy
30. A case of Breast mass denoting carcinoma from its features with skin infiltrations and palpable LN imaging done, what to do now?
A. Excisional biopsy B. Core needle biopsy before definitive treatment C. FNAC
If there are malignant features -> Core needle biopsy. If there are benign features -> Excisional biopsy
31. A case of 38 yr. old lady with firm breast mass 4x5 cm, in the lateral upper quadrant, with palpable LN, what you will do initially?
A. Mammogram B. MRI (if mammography negative) C. True cut biopsy (after mammography) D. Chest CT
32. A 35 years old with breast mass 4x5 cm with palpable axillary LN initially you did mammogram, was suspicious what is best next?
A. Mammogram B. MRI C. Tru cut biopsy or CNB
33. A female patient with history of breast-feeding stopped for 6 months presented with breast mass, FNA shows whitish fluid, diagnosis?
A. Cancer B. Galactoceles C. Breast abscess
34. A 34-year-old female has lump in the upper outer quadrant of left breast get tender in the days before her period sensitive to touch
and uncomfortable, 2 cm in size might feel firm smooth rubbery or hard but no LN involvement what is the most likely diagnosis?
A. Complex fibroadenoma B. Breast abscess C. Fibrocystic changes (benign, 20-50 y HRT, pain, mass, cyst, nodularity) D. Duct papilloma
35. What is the next appropriate management of the previous case (Fibrocystic changes)?
A. Observe, lifestyle modification and painkillers/ Tamoxifen, danazol B. US guided aspiration C. Incision and drainage D. WLE
36. You did fibroadenoma excision for one of your patients and send it to histopathology then histopathology reported: lobular carcinoma
in situ with free margins. What is your management? LCIS is multicentric in 60-80% of patients and bilateral in 20-60% age 49 and 50 years
A. Mastectomy B. Chemo best observation C. Radiotherapy D. Follow up (R0)
37. Which of the following regarding Lobular carcinoma in situ is correct?
A. It is mostly found in premenopausal women B. It usually presents as a breast lump C. It is precancerous lesion
D. It has characteristic calcification pattern on mammography E. It is associated cancer is lobular in nature
38. Which has the best prognosis of all breast cancers breast cancers?
A. Mucinous B. Paget C. Tubular N.B. Mnemonic; TCLM (good prognosis tubular then colloid then lobular then medullary)
39. A 49 y.o lady on OCP for 13 years her period started at age of 14 her mother, died due to breast cancer, what is main risk for cancer?
A. Menarche at 14 B. Contraceptive pill C. First live born after 30 D. History of mother with breast cancer
40. What is the most common cause of cancer-related death in females?
A. Breast cancer B. Colon cancer C. Ovarian cancer D. Pancreatic cancer E. Lung cancer
Breast cancer was the most common cancer worldwide among women. Breast cancer is the most and first leading cause of death for women in the world,
For all patients, males and females; the most common causes of cancer death in 2020 were: lung, colon and rectum, liver, then stomach and breast.
41. What is the most common presentation of ductal carcinoma in situ?
A. Breast pain B. Breast lump (no palpable mass) C. Nipple discharge D. Pleomorphic microcalcification (fine-linear branching)
42. Which of the following is associated with malignant breast lesions?
A. Hyper echogenicity B. Widely scattered (will be clusterd) C. Micro calcifications (< 0.5 mm heterogenous) D. Smooth margins
Typical features characteristic of invasive malignant carcinoma include evident mass, micro-calcification, architectural distortion or asymmetric density.
43. A 47- y- old female single with positive family history of breast cancer, routine mammogram showed bilateral increased density and
glandular pattern. Core needle biopsy showed atypical ductal hyperplasia, what is the appropriate management?
A. Wire surgical excision B. Simple mastectomy C. Lumpectomy
Factors associated with a lower risk of upgrade, including no mass lesion, removal of at least 50 % of the calcifications seen mammographically, no necrosis,
and ADH involving only one or two terminal duct lobular units. For women with ADH who met these criteria, upgrade rates were only 3 to 5 %
44. A 44 year old female patient, with unilateral bleeding per nipple, cytological examination showed malignant cells what’s diagnosis?
A. Intraductal carcinoma B. Intraductal papilloma C. Mammary Ductectasia
45. A middle-aged female presented with bloody nipple discharge and a 3 cm breast lump, which was excised, histopathological
examination, done and revealed intraductal carcinoma, margins were involved and not completely free, what is the best next step?
A. Re-excision of the previous site B. Radiotherapy C. Chemotherapy D. Radical mastectomy
46. A 46 y.o lady with bloody nipple discharge, after US, and mammography, 3cm mass with clinically and radiologically negative axilla
diagnosed as DCIS, the patient asked for conserving her breast, after complete excision, histopathologist concluded that it is invasive
ductal carcinoma with free and negative margin, what the best next option?
A. Re excision of more margin B. Adjuvant radiotherapy for BCT(no RTH before BCS) C. Preoperative chemoradiotherapy D. MRM
Management strategies for DCIS include surgery, then radiation therapy, and adjuvant endocrine therapy (if ER positive cancers).
47. Which of the following is considered early Breast cancer?
A. Stage I (IA, IB) B. Stage II (if IIA) C. Stage III D. Stage IV N.B early stage if < 5cm no > 3 LN spread as IA, IB, IIA
48. A female patient has a breast mass 7 cm in upper outer quadrant infiltrating chest wall, she desires breast conserving surgery, which is the
management plan for her? BCT) is standard for T1-T2 tumours, breast cancers < 5 cm; lumpectomy, wide local excision, Quadrantectomy, segmentectomy
A. Chemo radio B. Chemo then WLE C. Radio then WLE D. Chemo radio then WLE
49. What is the absolute contraindication to breast-conserving surgery for breast cancer?
A. Large tumour B. Tumour of high-grade C. Early pregnancy D. Retroareolar tumour E. Clinical axillary LNs
Absolute Contraindications: First-trimester pregnancy (radiation therapy is contraindicated during pregnancy. BCT is reasonable for some 2nd. and many
3rd. trimester patients who can receive radiation after delivery). Multicentric disease, diffuse suspicious or malignant appearing microcalcifications,
Extensive ductal carcinoma in situ, inflammatory breast cancer and tumours for which clear margins are unobtainable with lumpectomy with favourable
cosmetic results. Relative Contraindications Prior radiation therapy to chest wall or breast, Tumours greater than 5 cm, Large tumour size relative to
breast size, Women with a known or suspected genetic predisposition to breast cancer (i.e., BRCA1, BRCA2), Active connective tissue or collagen disease
involving the skin (i.e., scleroderma) and Known or suspected Li-Fraumeni syndrome (p53 mutation)
50. An epileptic postpartum lady experienced status epilepticus before so, it was kept by her neurology physician on phenobarbital as a
prophylactic of seizures, now she is planning to breastfeed her baby, what is the best option while she is currently using phenobarbital?
A. Stop breastfeeding and give artificial B. Feed after 8 hours of taking C. Feed it normally D. Start weaning
51. What is the appropriate management of radiation mastitis?
A. Local heat application B. Danazol therapy C. Therapeutic breast massage D. Simple mastectomy
E. Pentoxifylline therapy (Trental plus vitamin E it is effective in reversing radiation induced effects)
52. Which of the following is contraindicated during pregnancy for treating breast cancer?
A. Methotrexate (teratogenic and can terminate the pregnancy) B. 5-Fluorouracil C. Doxorubicin D. Cyclophosphamide
53. What definitive diagnosis of inflammatory breast cancer is provided by?
A. Unique mammographic appearance B. finding tumour emboli in dermal lymphatics
C. Finding extensive inflammatory cell infiltration of the tumour (T4d with dermal lymphatic invasion) D. Ultrasound cavitation
E. Elevated white cell count, fever, axillary lymphadenopathy
54. The diagnosis of inflammatory breast cancer is confirmed by which of the following?
A. Mammography (not specific due to thickening and oedema) B. Fine-needle aspiration C. Ultrasound D. Skin punch biopsy
55. What is the optimal approach to inflammatory breast carcinoma treatment?
A. Total mastectomy B. Modified radical mastectomy C. Lumpectomy and radiotherapy D. Chemotherapy
E. Chemotherapy, modified radical mastectomy, and radiotherapy
56. What is the optimal management of a T4 breast cancer?
A. Modified radical mastectomy and radiation B. Chemo radiation only C. Simple mastectomy and radiation
D. Chemotherapy followed by mastectomy and radiation
57. What is the initial management of T4 invasive lobular breast carcinoma?
A. Tamoxifen therapy B. Modified radical mastectomy C. Radiation therapy to the breast and ipsilateral axilla
D. Neoadjuvant chemotherapy
58. Li-Fraumeni syndrome shows increased incidence of which of the following? SALB (Sarcoma, Adrenal cancer, Leukaemia& Breast cancer)
A. Colon cancer B. Ovarian cancer C. Lung cancer D. Breast cancer (also Sarcoma, , Leukaemia, and Adrenal cancers)
59. A middle-aged lady patient with mass in UOQ in breast not fixed to skin or muscle, mass 1.5 cm biopsy was taken; revealed infiltrative
duct carcinoma, no mets, clinically & SLNB; negative axillary LN, what’s treatment? SLNB:T1,T2, DCIS, mass on mammo palpable >2.5 cm, <55y
A. Wide local excision with sentinel LN B. MRM C. Simple mastectomy D. Lumpectomy (therapeutic/diagnostic)
60. A 34-year-old lady with a retro areolar breast mass 3x4 cm mass with nipple retraction, US was done, what is the next to do?
A FNA B. Mammogram (minimum is 35 unless hereditary breast cancer) C. MRI D Core needle biopsy (under mammo, US, MRI guide)
61. A 46 y.o lady with hard, irregular, retro areolar left breast mass 3 x 4 cm with serous discharge, what is the most probable diagnosis?
A. Ductectasia B. Breast cancer C. Fibrocystic disease D. Traumatic fat necrosis E. Fibroadenoma
62. A 54 y.o lady with right breast retro areolar mass, with dimpling skin, firm in palpation, ipsilateral serous nipple discharge, no
palpable axillary lymph nodes, what is most likely diagnosis?
A. Intraductal papilloma B. Ductectasia C. Traumatic fat necrosis D. Breast Cancer
63. An old lady with retroareolar breast mass, with dimpling skin, firm consistancy, serous nipple discharge, no axillary LNs, what diagnosis?
A. Breast cancer B. Mammary Ductectasia (not written as atypical Ductectasia) C. Fibrocystic disease
64. In the management of locally advanced breast cancer, what, Neoadjuvant CTH does indicate?
A. Surgery then chemotherapy B. Chemotherapy then surgery
65. Breast mass in upper inner quadrant aspiration reveals malignant cells, management?
A. Radical mastectomy B. MRM (inner quadrants has bad prgnosis than outer) C. Radical mastectomy, CTH D. Radical + both CRH,
66. After mastectomy, winging of the scapula results from injury to:
A. Medial pectoral nerve B. Lateral pectoral nerve C. Long thoracic nerve (scapula winging) D. Thoracodorsal N(latissimus paralysis)
E. Intercostobrachial N. (paresthesias and dull, aching, or burning pain of the upper arm, shoulder, and axilla)
67. Regarding to the long thoracic nerve, what is correct?
A. It is purely motor B. Provides sensation to the medial wall of the axilla C. Its injury results in weakness of arm abduction
D. Provides sensation to the medial aspect of the upper arm E. Supplies motor innervation to the latissimus dorsi muscle
68. What is correct regarding the thoracodorsal nerve?
A. It is purely motor B. supplies motor innervation to the serratus anterior muscle
C. injury during axillary dissection will lead to winged scapula D. injury will result in loss of sensation on the medial surface of upper arm
69. What is treatment for confirmed Paget’s disease of breast?
A. MRM B. MRM then radiotherapy or BCS with adjuvant RTH C. Chemoradiotherapy D. Chemo then MRM then RTH
70. A patient with locally advanced breast cancer will be given neoadjuvant chemotherapy then MRM or BCS, after how long the optimal
time to do surgery after the last chemotherapy dose without any adverse effects on wound healing?
A. After two weeks B. After 6 weeks C. After 8 weeks
D. After 4 weeks (3 weeks in genetic mutation and 5 wks if discontinue chemotherapy due to side effects)
71. A female swimmer underwent modified mastectomy and breast reconstruction which type of flap will interfere with swimming?
Latissimus dorsi flap
72. An old aged lady with breast mass and enlarged axillary pathological LNs., what is the next step in management?
A. Core needle biopsy B. Excisional biopsy (if superficial & accessible, otherwise image guided aspiration) C. MRM D. Chest& bone CT
73. A woman presents with a long-standing 4 cm irregular breast mass, which is growing in size, with enlarged axillary LNs and thickened
skin overlying the mass. US showed hypoechoic lesion, Mammogram with an ill-defined mass with calcifications, what is the next step?
A. Core biopsy B. Excisional biopsy C. Re assess after 6 months D. FNAC E. Both breast MRI
74. A 55 yrs. female with breast ulceration, scaling, oozing, no mass on examination, Mammogram show no mass what is the optimal next?
A. Dermatological consultation B. F/U within 6-month C. Nipple biopsy D. Local corticosteroids
A punch biopsy or a core biopsy including nipple skin is the initial procedure of choice and should be performed under local anaesthetic in the outpatient
clinic Core biopsy through the skin is an alternative.
75. A 46 yrs. old lady smoker for 18 yrs. concerned about breast cancer, what is the appropriate screening test for her?
A. Mammogram B. Pulmonary function and CXR C. Breast and axilla US D. Breast and chest MRI
76. Breast mass BIRAD V on US and enlarged pathological LNs. What is the next step in management?
A. Core needle biopsy B. Excisional biopsy C. MRM D. CT scan of bone and chest
77. What is treatment vertebral metastasis, in a locally advanced breast cancer with HER +ve.?
A. Mastectomy + vertebral radiotherapy B. Mastectomy + vertebral chemotherapy C. Mastectomy + chemotherapy
78. Factor VIII related antigen is a marker of which of the following?
A. Kaposi's sarcoma B. Melanoma C. Post mastectomy angiosarcoma D. Desmoid tumours E. Merkel cell carcinoma
79. What is the most prognostic factor determining breat cancer recurrence post full therapy?
A. Number of lymph nodes B. Positive oestrogen / progesterone receptors
C. Grade, size of the breast mass (> 5cm & number of LNs >4, +ve margin, lymphovascular invasion, high grade of the tumour)
80. For Male breast cancer what is correct?
A. Has a peak incidence at the age of 40 years B. It is typically hormonally dependent
C. Typically presents with bloody nipple discharge D. Is seldom an indication for tamoxifen therapy
81. Male breast cancer most commonly presents with which of the following?
A. Mastodynia B. Bleeding per nipple C. Breast mass only D. Breast mass and bleeding per nipple
E. Breast mass with ulceration
82. For Male breast cancer what is true?
A. Is associated with the BRCA-I gene mutation B. Can only be of ductal origin C. Is seldom hormone receptor positive
D. Develops at a much younger age than female breast cancer
83. For male breast cancer which of the following is not correct?
A. Less than 2% of all cases of breast cancers B. Exocrine or endocrine oestrogen exposure can predispose to it
C. Most commonly it is infiltrating duct carcinoma D. Most commonly it is infiltrating lobular carcinoma
84. Male with biopsy proved breast cancer, no family history of breast cancer, what is the type of gene mutation?
A. P53 B. BRCA1 C. BRCA2 D. New mutation
85. How much incidence of hereditary breast cancer with BRCA1 & BRCA2 gene mutation? Both 70%. / BRCA1:60-90%, BRCA2:50-60%
About 5-10% of all breast cancers are hereditary; BRCA1 and BRCA2 mutations account for up to 70% of hereditary breast cancers.
86. Female patient underwent modified radical mastectomy, developed oedema in the upper limb, management?
A. Limb elevation and analgesia B. Elastic stocking C. Incision and drainage
87. What is the commonest site of bone metastasis due to metastatic breast cancer?
A. Shaft of femur B. Spine (lumber) C. Chest wall D. Skull bone
88. What is the best pain management option due to Hip metastasis caused by metastatic breast cancer?
A. Radiotherapy B. Opioid C. Bisphosphonate's infusion (if no response with it or Denosumab or interactable pain >> local RTH)
89. A female patient with history lumpectomy for phyllodes tumour after recurrence, what is the optimal line of surgical treatment?
A. Lumpectomy + radio B. Lumpectomy + Chemo C. Simple mastectomy (breast + skin ellipse nipple areola complex no pectoralis +-LNs)
90. What does it mean in histopathologically written report for breast swelling; large nucleus and nucleoli in breast biopsy and what’s
management? It means that is breast cancer for Simple mastectomy
91. A middle-aged female patient with breast mass for 4 years, it was small in the last 2 years now it becomes increasing in size to 15cm.
On US it showed unicystic or multicystic with echoic shadow, what’s plan of treatment?
A. Radical mastectomy (no LNs) B. Simple mastectomy (Malignant phyllodes tumour ) C. Chemotherapy D. Radiotherapy
92. Female with swelling 3 cm with peau‘d Orange with supra clavicular L.N, no evidence of distant metastasis, what is this stage?
A. T3 N0 M0 B. T3 N1 M0 C. T4b N3c M0 D. T4 Na1 M1
Tis Paget’s disease with no invasive or DCIS. T1 is tumour ≤ 2 cm in greatest dimension (T1a if > 10 mm but ≤ 5mm, T1b if > 5 mm but ≤ 10 mm, T1c if >
10 mm but ≤ 20 mm). T2 2cm - < 5cm. T3 if > 5cm in the greatest dimension. T4a if there is extension to chest wall, not including only pectoralis muscle
adherence/invasion. T4b if there is ulceration and/or ipsilateral satellite nodules and/or oedema (including peau d’orange) of the skin, but not IBC.
T4c if a and b together and T4d is inflammatory breast cancer, IBC.
N either cN1: metastasis to movable ipsilateral level I, II axillary LNs. N2a if metastases in ipsilateral clinically fixed or matted level I, II axillary LNs.
(N2 b); if ipsilateral internal mammary nodes in the absence of clinically evident axillary LNs metastases.
N3 a if metastases in ipsilateral infraclavicular (level III axillary) LNs, with or without level I, II LNs involvement
(N3b); if ipsilateral internal mammary LNs with level I, II axillary lymph node metastasis
(N3c); metastases in ipsilateral supraclavicular LNs with or without axillary or internal mammary LNs.
M0 if no distant metastasis, cM1 if distant metastases detected by clinical and radiographic means, pathological; pM1; if any histologically proven
metastases in distant organs; or if in non-regional nodes, metastases > 0.2 mm.
93. A female patient with 11x12 cm breast mass, examinations showed no palpable lymph Nodes, Core needle biopsy done and showed
malignant phyllodes tumour, what’s the next appropriate step?
A. Wide local excision (WLE) A. PET scan B. Chest CT without contrast C. Simple Mastectomy
94. What is the initial site of distant metastasis in breast cancer?
A. Lungs B. Liver C. Bones D. Brain
95. Which tumour not producing gynaecomastia
A. Lung B. Bone C. Prostate D. Testis
96. What is the least tumour cause Gynaecomastia?
A. Colon B. Lung C. Prostate D. Gallbladder E. Testes F. Renal G. Bone
NB if both (gallbladder and bone) came choose GB tumours as it never happens with it, but bone tumours can rarely associated with gynaecomastia
97. Gynaecomastia is a side effect of which of the given choices?
A. Ketoconazole B. Amphotericin B C. Fluconazole D. Miconazole
TRAUMA AND CRITICAL CARE SURGERY
1. HEAD, NECK, TRAUMAS & NEUROSURGERY
1. A mother brought her 3 months baby to clinic concerning his large head and poor feeding followed by vomiting. on examination
scalp veins were dilated, and eyes were forcibly deviated downward, what is the cause of this condition?
A. CSF overproduction B. Obstruction of CSF flow C. Decreased CSF absorption D. All the above (hydrocephalus)
2. A 57-year-old patient travelled to Mexico and returned after a short while, he had suffered from a stroke long time back where treated
and became well then, he developed CNS manifestations and CT done and this (picture) was found, what’s the diagnosis?
A. Cerebral stroke (recurred) B. Hydatid cyst in the brain (hydatid very rare in Mexico) C. Amoebic brain abscess (rare)
3. Regarding brain tumours, what is correct?
A. The golden investigation is brain CT B. Size rather than the site affects symptoms
C. Prognosis is better in females more than males D. Prednisone is important in management brain tumours
4. Brain CT scan done for a patient with multiple hyperdense small lesions with history of treated melanoma, now what diagnosis?
A. Brain metastases by melanoma B. Hydatid cyst of the brain C. Cerebral stroke
5. What is the cause explaining congenital torticollis? ischaemia or trauma > fibrosis & contracture of sternomastoid ms.> deformity
6. Explosion with neck injury, surgical exploration was done; oesophageal & proximal subclavian artery injury found, what’s treatment?
A. Repair of both B. Repair artery & oesophagostomy C. Ligation artery & repair oesophagus
D. Subclavian artery ligation & oesophagostomy (no primary oesophageal repair as not clean, may be septic due to explosion)
In an unstable patient, the subclavian artery can be ligated, or a temporary shunt can be placed. Divert the oesophageal contents till become stable & clean
7. A middle aged patient was extracted from area of explosion with multiple injuries and oesoophageal with internal carotid artery injury,
he was fully resuscitated, given tetanus toxoid and antibiotics on admission and now he is in operating room, what’s the best optimal plan?
A. Repair both ICA and oesophagus B. Repair carotid artery with oesophagostomy C. Ligation of the artery with oesophagostomy
D. Repair the artery with oesophagectomy
8. An RTA patient with forehead trauma, he was conscious alert but suddenly deteriorated, what is the cause of his sudden deterioration?
A. Subdural haemorrhage B. Epidural (extradural) haematoma C. Subarachnoid haemorrhage D. Intracerebral hge.
9. A child with a ball hit his head, lost consciousness, then regained his consciousness, then deteriorated again, diagnosis? Extra dural Hge.
10. An elderly patient came with severe headache to ER then he suddenly deteriorated with decreased level of consciousness, CT done
showed a haematoma, what is the best management?
A. Admit to ICU B. Give mannitol C. Evacuate it
Diagnosis is: Epidural haematoma, initial: IV mannitol and head elevation, the best: Evacuation
11. A case with history of blood coming from ear with perforation of the tympanic membrane, what is the cause?
A. Basal skull B. Subarachnoid haemorrhage C. Mastoid bone trauma
If temporal bone fracture especially if longitudinal that can affect the mastoid process and injure the external auditory meatus due to lateral or temporal
force causing drum rupture and bloody otorrhoea, associated with conductive hearing loss due to dislocation of the auditory ossicles, with CSF otorrhoea
12. What is the finding commonly seen in basal skull fracture with jugular foreman damage and structures compression?
A. Can’t abduct eye B. Loss of mastication C. Loss of sensation over zygoma D. Ipsilateral vocal cord paralysis
Involvement of the glossopharyngeal nerve leads to the following: loss of sensation to the posterior ipsilateral aspect of the tongue, reduced secretions from
the ipsilateral parotid gland, and loss of the ipsilateral gag reflex with ipsilateral vocal cord paralysis.
13. A patient with increase intracranial pressure, next or initial step of management till the neurosurgeon oncall arrives hospital?
A. Elevate the head of the bed (35- 40-degrees) B. Mannitol C. Trephine opening
14. RTA, scalp laceration, fascial and femoral fractures and at 3 rd. day became unconscious what the cause of loss of consciousness?
A. 2ry. haemorrhage (subdural haematoma < 72 hr. acute SDH) B. Fat embolism C. Electrolyte disturbance D. SAH
15. An old age, fell on his head 2 weeks ago, presented with disturbed level of consciousness, CT concave frontal lesion, what’s diagnosis?
A. Axonal brain injury B. Coup & Countercoup injury C. Subacute subdural haematoma
Acute subdural haemorrhage starts minutes after trauma up to 72 hours, Subacute from 3-7 days up to 15 days and can extend to 20 days. after 3 weeks of
head trauma is considered chronic
16. A patient was involved in RTA, presented with face laceration, on the 2 nd. day, he developed confusion & restlessness, was stable but
became more unwell with some deterioration, what’s diagnosis?
A. Delayed concussion (if no deterioration) B. 2ry Cerebral Hge. C. Hypovolaemic shock D. Extradural haemorrhage
17. A 65-year-old male patient, came to neurosurgery clinic for follow up after head trauma two weeks ago, with pressure manifestation,
in the form of numbness of upper limb, with mouth deviation, referred to ER for Brain CT, and the finding was: subacute subdural
haematoma, all vitals stable, GCS: 15 how to manage?
A. Admission and work up B. Craniotomy C. Burr hole D. Conservative at home
Recent articles state that burr hole drainage is a superior technique compared to twist drill craniostomy and craniotomy, due to a lower incidence of
recurrence and morbidity. Symptomatic subacute/chronic subdural haematomas are often treated via one or more burr holes as the blood clot has liquefied
and can be washed out more easily. The compressed brain can take some time to re-expand, and subdural collections may re-accumulate.
18. A child 11 years old came with his father, that child has fits after history of head trauma 2 weeks ago, what is the best management?
A. Conservative B. Burr hole C. Follow up with serial Brain CT D. Mannitol infusion
19. History of two months head trauma, presented with vomiting, tiredness, affected conscious level. CT Subdural haematoma, treatment?
A. Serial CT B. Burr hole (symptomatic chronic for burr holes) C. Craniotomy
20. A CT report showed a concave dense lesion in the right temporal area? Ruptured Subdural haematoma
21. An old, aged male was working for long time as a Shepherd, stopped more than six months ago, he was walking and fell on his head
with no major issues, after two weeks of fall he was brought by ambulance to ER where the patient was unable to stand up or walk with
general weakness no loss of consciousness or severe headache with CT image for two hypodense brain lesion what is most likely cause?
A. Acute subdural haematoma B. Chronic subdural haematoma D. Extradural hge. C. Subacute subdural haematoma
22. An old age male patient fell on his head in the bathroom 1 week back, he presented with neurological manifestation, brain CT was
done showing concave lesion, what is the most likely diagnosis?
A. Concussion B. Subacute subdural haematoma C. Extradural haematoma D. Traumatic subarachnoid hge.
N.B. Extradural >> biconvex, subdural >>> concave or crescent shape and unilateral subarachnoid haematoma is biconcave.
23. A middle-aged patient was in involved in RTA, he has traumatic right maxilla fracture, he was haemodynamically stable, after two
weeks of RTA, he fell with lost consciousness, brain CT showed a picture of isodense with little hypodense around it what is diagnosis?
A. Subacute subdural haematoma B. Delayed post concussion C. Brain tumour D. Venous sinus thrombosis
Patients sustaining facial fractures are at risk for accompanying traumatic intracranial haematomas, which are a major cause of morbidity and mortality.
24. An 87-y old man is brought to the ER because of headache, nausea, and vomiting for 24 hours, two weeks ago, he fell in the bathroom
and his head hit the bathroom ground. His GCS 7/15. On examination, decreased muscle power in the right leg and arm. Plain head CT
showed a high density, 13 mm concentric collection. The patient was intubated and resuscitated, what is your next step in management?
A. IV mannitol B. Admit ICU and observe C. Craniotomy and surgical evacuation D. Insert intracranial monitor
Guidelines recommend Craniotomy and surgical evacuation for patient with acute and symptomatic subacute SDH who have clot more than 10mm or
midline shift more than 5 mm regardless of GCS score, in a case like this with low GCS and coma no benefit at all from mannitol or any conservative methods
25. A patient with intracranial haemorrhage with increasing CNS manifestations, management? Craniotomy
26. An old patient came with subdural haematoma with signs of lateralisation imaging revealed 13 mm shifting. his GCS 7/15 then was
intubated and resuscitated, what to do next?
A. IV Mannitol B. Admit ICU and observe C. Craniotomy
27. An ischaemic stroke with haemorrhagic transformation, developed papilloedema, what would you do? Give mannitol to decrease ICP.
If haematoma is > 10mm or midline shift > 5mm do Craniotomy
28. Trauma patient, currently well, during the lucid interval was reported by paramedics, now he is deteriorating, most likely diagnosis?
A. Epidural haematoma Epi =extra B. Subdural haematoma C. Base of skull fracture NB. Epidural haematoma = lucid interval
29. An 18-year-old man with head injury with right eye proptosis with loss of light reflex, he is spontaneously respirated, no available
neurosurgeon and the nearest hospital with specialist is about 2 hrs distance from your hospital, what is the best to do for this patient?
A. Immediate transfer B. Craniotomy in casualty C. IV mannitol to decrease ICP (as a diuretic; Na < 155 mEq/L) D. Burr hole
As a rule, first to do is head elevation then mannitol IV if there is specialist, he can do Craniotomy if acute/subacute or burr hole if chronic. Craniotomy
and Burr hole is most often done by neurosurgeon but is there is a well-trained general surgeon can only do a Burr hole.
30. A middle aged man involved in MVC, came to ER with inability to speak to any stimulus, opens his eyes to pain, withdraws his
forearm to pain, how much his GCS?
A. 9 B.8 C.10 D.7
31. A post RTA with head trauma, open his eyes to painful stimulus, withdraws to pain, and responds to questions with wrong answers, GCS?
A. 6 B. 7 C. 9 D. 10 (open to pain=2, withdraw hand away from prick = 4, wrong, inappropriate answers, =4. So, GCS 10)
Opens to painful stimuli =2, withdrawing to pain =4, respond to question wrongly means confused = 4 so E+M+V =10
32. Cyclist came to ER unconscious, opened his eyes on verbal orders, confused, withdrew to pain, what’s his GCS?
Open to verbal orders = 3, withdraw to pain = 4, respond to question wrongly means confused = 4 so E+M+V =11
Visual response if spontaneous eye opening =4, for speech = 3, for pain =2, no response = 1
Verbal response if fully oriented to time, place persons =5 if only confused = 4, inappropriate words = 3, incomprehensible =2, no response =1
Motor response if obey all commands = 6, if localises pain = 5, if withdraws from pain = 4, if abnormal flexion (decorticate response) =3 ,if abnormal
extension (decerebrate act) = 2 if no response =1 the least score is 3 the best is 15-13, moderate 12- 9, worse is <9-8 then for intubation
33. An RTA case with speaking wrong answers, opens eyes to pin prick, extend his arm to stimulus, hypoxic, disturbed conscious level for?
A. Nasopharyngeal airway B. Orotracheal intubation (GCS 8) C. Tracheostomy D. Oropharyngeal airway
34. A patient with a head injury opens his eyes and withdraws his arm only to painful stimuli. He is making incomprehensible sounds.
How much is his Glasgow Coma Scale score?
A. 12 B. 10 C. 8 (open and withdraw to pain = 2+2, inappropriate, confused = 4. So, GCS 8) D. 6
35. A patient came to ER unconscious, open his eyes on verbal orders, confused, localise pain, what is GCS? 12
Open eyes to verbal order =3, Respond to question in confused manner = 4, Localise pain =5 So, E+M+V = 12
36. A head trauma patient who is confused, opens his eyes to sound and he is localising pain what is his GCS degree? 12
A. Minimal B. Mild (13-15) C. Moderate (9- 12) V=4, E=3, M=5 =12 D. Severe (<8)
37. A case of head trauma, he is in deep coma for 5 days, what is the initial way of feeding at the current time?
A. NGT B. Gastrostomy C. Peripheral line D. Central line E. Nothing per oral, only IV fluids
Enteral tube feeding: ETF as NGT can be used in unconscious patients, starting enteral nutrition too early is not nutritionally beneficial, and TPN with
20% glucose fed through a tube is recommended as adequate nutrition for these patients, but not be employed for longer than 10 days.
38. A patient with permanent basal ganglia damage cannot be fed by NGT, how can take permanent nutrition?
A. NGT B. Gastrostomy (initially if tolerated) C. Jejunostomy (if Gastrostomy intolerated) D. TPN
Three evidence-based guidelines were identified, all of which recommend the use of gastric feeding tubes as the first choice of treatment, and the use of
post-pyloric jejunostomy feeding tubes if gastric feeding is poorly tolerated. In general gastrostomy is more preferred to jejunostomy if the patient has
functioning stomach and can tolerate gastric enteral feeding, it is less invasive with lower risk of complication if compared with jejunostomy.
39. Transient ischaemic attack; TIA with atrial fibrillation on irregular medication, what to give him/her?
A. Aspirin B. Heparin C. Warfarin INR 3-4 D. Warfarin INR 2-3
INR goal for warfarin is 2.0 to 3.0 and the dose of aspirin is 81 to 325 mg. Warfarin is used for long-term therapy in patients with AF to prevent a
thrombus from forming in the left atrium.
40. A patient came with symptoms of TIA, what is the next step? Initially Carotid duplex >>> Best is MRA
41. A 75-year-old lady with recurrent attacks of loss of consciousness with blurred vision, no history of chronic illness next investigation?
A. Duplex US B. Magnitic resonance angio; MRA (Amarausis fugax) C. CT Angiography; CTA (if MRA unavailable or contraindicated)
Amaurosis fugax is a medical emergency as it may herald a devastating hemispheric or basilar stroke or permanent visual loss, it refers to a transient loss
of vision in one or both eyes mostly due to vascular cause, it is a harbinger of an imminent stroke, it is a result of an occlusion or stenosis of the internal
carotid artery circulation. Thromboembolism originating from the carotid circulation, as well as hypoperfusion caused by the stenosis of this circulation,
are the underlying mechanisms. The ocular ischaemic syndrome results from chronic hypoperfusion due to unilateral or bilateral carotid artery occlusion
42. A scenario of Transient ischaemic attack; TIA/ amaurosis fugax what is the best investigation?
A. MR Angiography B. Doppler US C. CT Brain D. CTA
AHA/ASA recommend MRI with diffusion-weighted imaging/apparent diffusion coefficient sequences as the preferred diagnostic study in patients with
suspected TIA/AF. AHA/ASA further recommends [Class 1, level of evidence, B] that subjects with suspected TIA should preferably undergo brain MRI
within 24 hours of initial presentation. Head CT can be considered if MRI is unavailable or contraindicated
43. A 55-y. old female who had an episode of visual loss for 20 min., it came back, what is your diagnosis?
A. Transient ischaemic attack (TIAs last only 2min. to < 1 hr, majority resolve in <15 min, completely in < 24 hrs) B. Multiple sclerosis C. Brain mets
44. A patient presented with signs and symptoms of stroke, what is the next step in management?
A. Plain brain CT B. MRI C. EEG
Noncontrast CT scanning is the most commonly used form of neuroimaging in the acute evaluation of patients with apparent acute stroke.
A lumbar puncture is required to rule out meningitis or subarachnoid haemorrhage when the CT scan is negative, but the clinical suspicion remains high.
Multimodal CT imaging, CT angiography and CT perfusion to NC CT has the potential to identify large vessel occlusions and areas of salvageable tissue.
45. What is the minimum cerebral perfusion pressure that can maintain the cerebral blood flow and prevent cerebral ischaemia?
A. 20-30 mmHg. B. 55 - 60 mmHg. C. 80-85 mmHg. D. 120 mmHg.
The relationship between MAP and CPP drives resuscitation guidelines to recommend maintaining a MAP greater than or equal to 65 mm Hg.
Assuming a normal ICP, this threshold should guarantee a CPP of 55 to 60, the minimum needed to prevent cerebral ischaemic injury.
The normal cerebral perfusion pressure CPP ranges from 60-80 mmHg. Cerebral perfusion pressure equals the mean arterial blood pressure minus
intracranial pressure so, CPP = MAP-ICP, and the normal adult ICP is defined as 5 to 15 mm Hg (7.5–20 cm H2O). More than 15 is pathological, ICP from
20 to 30 mm Hg represent mild intracranial hypertension, ICP greater than 20 to 25 mm Hg. require treatment in most circumstances and sustained ICP
values of greater than 40 mm Hg indicate severe, life-threatening intracranial hypertension.
46. A case of stroke and complaint of weakness in the arm, leg, and face, where’s the damage?
A. Middle cerebral artery B. Anterior cerebral artery C. Mid basilar artery
The middle cerebral artery; MCA is by far the largest cerebral artery, the larger of the two main terminal branches of internal carotid artery, the second is
anterior cerebral artery, and MCA is the vessel most affected by cerebrovascular accident (CVA). The middle cerebral artery supplies most of the outer
convex brain surface, nearly all the basal ganglia, and the posterior and anterior internal capsules.
47. A bedridden patient present with confusion and agitation, low BP and high HR, ECG show sinus tachycardia, what is the diagnosis?
A. Stroke, CVA B. MI C. PE D. Brain tumour E. Extradural haemorrhage
48. Fracture of the skull base with Raccoon eyes then the patient is decompensated and now for intubation, what type of airway care to do?
A. Orotracheal B. Nasotracheal C. Oxygen nasal canula D. Laryngeal mask airway
Orotracheal intubation is generally the preferred and commonly used method as it is easier, quicker no need for GA or theatre admission, is the commonest
used, especially during emergent intubations, and less painful so it is better in this case.
49. An ICU patient with high output fistula on TPN, doctor asked the nurse to give the patient 2U PRBCs, the nurses stopped TPN and gave
him the 2 blood units on same line of TPN, after 2 hours later patient become comatosed, what is the cause of the electrolyte imbalance?
A. Hypoglycaemia B. Hypocalcaemia C. Hypokalaemia D. Hyponatraemia
One of the adverse effects with TPN use if sudden stoppage of TPN infusion for some time is development of hypoglycaemia
50. An old, aged patient was admitted to ICU with diagnosis of subarachnoid haemorrhage, but the initial CT in A/E was normal, what
would you do next?
A. Lumbar puncture (to rule out CT missed subarachnoid hge.) B. MRI C. Brain CT with contrast
51. What is the most common cause of subarachnoid haemorrahage in general?
A. Ruptured cerebral aneurysm B. Head trauma (40-60% of TBI have SAH) C. Electrolye imbalance D. Malignant HTN
Trauma is the commonest cause of subarachnoid haemorrhage. Rupture of an intracranial aneurysm is the second most common cause of subarachnoid
haemorrhage and is distinguished from traumatic SAH by history and the distribution of blood on a CT scan.
52. What is the most common cause of non traumatic subarachnoid haemorrhage?
A. Cerebral neoplasm B. Head injury C. Ruptured berry’s aneurysm (80%) D. Amyloid angiopathy E. Intracranial HTN
53. A CT scan image for left sided main carotid artery; MCA, ischaemic stroke and presented within 2 hrs., what is management now?
A. Thrombolytics (door to treatment 60 min, maximum time to give thromolytics 3- 4.5 hr. of sympt. onset) B. Heparin C. Warfarin
54. A young patient with altered mental status. CT brain showed parieto-temporal hyperdense lesion what’s the most likely diagnosis?
A. Herpes encephalitis B. Meningoencephalitis C. Brain abscess (CT; double rim sign, outer hypodense and inner hyperdense)
55. Which facial bone fracture is associated with diplopia on upward gaze or vertical movements of the eyes?
A. Inferior orbital wall or orbital floor fracture B. Medial orbital wall C. Lateral orbital wall
56. Wrestler, has punched over by blow to the lateral aspect of his face, suspected fracture infraorbital bone, or floor of the orbit what you
suspect to occur?
A. Unable to look to upper gaze (only limitation) B. Nasal bloody discharge C. Nasal watery discharge
D. Diplopia (due to entrapment of the perimuscular tissue surrounding the inferior rectus ms. in the fracture site)
57. What is the commonest site for trigeminal nerve injury in face trauma?
A. Temporal bone B. Zygomatic arch C. Maxilla(due to infraorbital nerve injury) D. Nasal fracture
Risk factors for trigeminal nerve injury include Direct maxillofacial trauma with facial bone fractures, injury secondary to local anaesthetic injections, and
during surgical intervention for facial trauma repair. There is an association between trigeminal nerve injury and facial bone fractures. Facial fractures affect
the peripheral branches of the trigeminal nerve, the highest incidence of initial trigeminal nerve impairment was found in midfacial nondisplaced fractures
58. Trigeminal nerve injury is caused by which of the following?
A. Facture temporal bone B. Fracture of maxilla (due to infraorbital nerve injury, br. of maxillary of trigeminal N.) C. Mandible fracture
Maxillary fractures; Lefort fractures II, are associated with significant visual problems (47%), diplopia (21%), and epiphora (37%). Other common
complications include infection, trismus, facial nerve deficits, malunion, and facial asymmetry.
59. A patient came to ER with a fracture to facial bone, while playing football and a ball hit his lower jaw causing mandibular fracture, he
was stable but, suddenly he fainted, with respiratory distress what’s the cause?
A. Hypoglycaemia B. Hypoxia C. Hypotension D. Cervical spine injury
Causes of hypoxia in fractures of maxilla: fractured bone block nasopharynx, posterior tongue falling, haemorrhage, with blood clot or haematoma, swelling
and oedema, associated larynx and trachea trauma or fracture cause displacement of epiglottis, arytenoid cartilages
60. Yor are an emergency surgeon in the ER, of a hospital with only 15 beds capacity, while you are in night shift duty, a mass RTA of
about 30 traumtised patients brought by ambulance cars how to deal with them in the light of priority triage?
A. Deal with all in same care B. Priority for life threatening traumas and states C. Priority for your specialy cases
61. A middle aged boxer presented to the ER with severe trauma to the face and left upper mandible with evident mandibular fracture, he
came with respiratory distress, with naso oral blood clots, the emergency doctor did rapidly jaw thrust, and patient became better, what is
the benefit of use of that maneuvre in emergency situation?
A. Prevent tongue falling back B. Used before nasal intubation C. Used in maxillary fractures D. It is the same as Chin lift
62. A patient with severe facial trauma associated with facial fracture, the anaesthetist can't intubate him, what is best next alternative?
A. Needle cricothyroidotomy B. Surgical cricothyroidotomy C. Tracheostomy
Cricothyroidotomy is an ER procedure indicated in angioedema, upper airway FB, severe oropharyngeal /nasal bleeding. The only absolute contraindication
to surgical cricothyroidotomy is the age of the child. For a paediatric patient, a needle cricothyrotomy with trans-tracheal ventilation should be performed
instead of an incision-based surgical cricothyrotomy. This should then be converted to a formal tracheostomy.
63. An RTA patient brought by ambulance, you found him, shocked and unconscious, with GCS 6-7, how to protect his airway?
A. Endotracheal intubation B. Orotracheal intubation C. Nasotracheal tube D. Face oxygen mask
64. A male victim of gunshot to the thigh, pale and unconscious, BP 90/60 pulse 130, what is next, definitive or best to do?
A. Orotracheal intubation (the 1st. priority as ABCDE approach) B. Blood transfusion C. Shift to OR D. RL infusion
Although, this patient has clear problem that need emergent OR transfer to save his limb, the airway securing his life and life has priority to limb so
intubation to keep him alive is more important to stop bleeding according to ABCDE protocol
65. A patient brought by ambulance after he was involved in RTA, he has face & forehead trauma but no bleeding or fracturs are vesible
also he is haemodynamically stable, breathing well but if left for some time he is snoring, his GCS was 10, how to protect his airway?
A. Orotracheal intubation (if GCS <8) B. Laryngeal mask C. Oropharyngeal airway if GCS > 8 (and to prevent tongue falling back)
Oropharyngeal airway is indicated if the jaw thrust manoeuvre has failed to correct airway obstruction, it is an airway adjunct used to maintain or open
the airway by stopping the tongue from covering the epiglottis, that snoring mostly due to start of tongue falling back if not prevented by OPA will obstruct
his air way and will go comatosed at that time no way have to do orotracheal intubation urgently
66. A patient with a stab wound in anterior neck, he is alert but, after putting pulse oximeter, oxygen saturation is 82%. What to do first?
A. Oxygen mask B. Cricothyroidotomy C. Endotracheal intubation D. Tracheostomy
Oxygen should be given to achieve a target saturation of 94–98% for most acutely ill patients or 88–92% for those at risk of hypercapnic respiratory failure.
67. What is true for the use of Nasotracheal intubation?
A. It is contraindicated for suspected cervical spine injury B. It is contraindicated for apnoeic patients
C. It is usually successful on the first attempt D. It is less tolerated by patients than is endotracheal intubation
Absolute contraindications to nasotracheal intubation are as follows: Suspected epiglottitis, Midface instability, Coagulopathy, Suspected basilar skull
fractures, Apnoea or impending respiratory arrest
Relative contraindications are large nasal polyps, suspected nasal foreign bodies, recent nasal surgery, upper neck haematoma or infection, history of frequent
episodes of epistaxis, prosthetic heart valves (increased risk of bacteraemia during the insertion)
68. A patient involved in the Road Traffic Accident (RTA) with multiple mandibular fracture with severe bleeding, unconscious, no vitals
mentioned, how would your mange his Airway?
A. Laryngeal mask B. Orotracheal C. Nasotracheal D. Cricothyrotomy (unsuitable for intubation)
Cricothyrotomy is indicated for all conditions associated with a difficult airway that may necessitate cricothyrotomy include massive hge, profound emesis,
trismus, obstructing lesions (tumour, polyp), upper airway occlusion (FB, oedema, anaphylaxis), and traumatic and congenital deformities. Can be
converted to formal tracheostomy, the only absolute contraindication is the age as children. Pediatric Advanced Life Support defines the paediatric airway
as age 1 to 8 years, if highly indicated needle cricothyrotomy is preferable to surgical and can be converted to tracheostomy.
69. What is the following is a contraindication for surgical cricothyroidotomy?
A. Laryngeal oedema from burn B. Fracture of larynx C. Unstable fracture mandible
Contraindications to cricothyrotomy include tracheal transection, tracheal or laryngeal trauma, laryngeal fracture, and/or anatomic distortion
70. A middle-aged male has neck trauma came to ER stable after some time he got emphysematous and dyspnoea what to do?
A. Orotracheal intubation B. Cricothyroidotomy C. Tracheostomy D. Oxygen mask 5 liters
71. Newborn baby with sudden SOB, CXR showed right upper hyperlucency with mediastinal shift to the left, what’s the next step?
A. Intubation B. Thoracotomy and lobectomy with local anaesthesia C. Thoracostomy E. Cricothyroidotomy
It is mostly a case of congenital lung emphysema, CLE. Conservative management is preferred in mild to moderate cases and lobectomy is the best option
in severe cases as our case of sudden SOB.
72. Stab injury in zone 1 of the neck, examination shows diffuse emphysema & patient is stable what is next appropriate?
A. CTA neck & chest B. Observation C. Surgical exploration
Unstable or hard signs = Exploration Stable or soft signs: CTA, recently the plan of management is dependant upon haemodynamic status not
zone call non zone management approach
73. A patient with penetrating neck wound in the zone 1 and is bleeding, haemodynamically unstable BP 90/60, P 116 what is the best
management?
A. Open and primary repair B. Endovascular C. Exploration, repair and ligation
If the patient is stable do CTA, if unstable or has hard signs resuscitate according ABCDE protocol, do immediate exploration and manage accordingly
74. A patient who fell from the 4th floor on his legs, came to the ER stable, but he just complains of pain in his ankles, investigations with
labs and imaging were normal, what do you want to do next or initially?
A. Check pulse (as initial a vital sign in ABCDE) B. Chest x ray Head and neck CT D. Abdominal US
E. Pan CT scan (definitive investigation (more than 3 m)
Indications of pan CT scan or WBCT are high speed motor vehicle collision>80km/h, non-trivial motorcycle collision, ejection from the car, death of other
from same car at the scene, fall from height >3 metres, other concerning mechanism of injury, car rollover, abnormal FAST, or trauma chest or pelvis x-
ray, abnormal vital signs as haemodynamically unstable although of fluid resuscitation but firstly must stabilise him.
75. A patient had stab injury in front of right or left ear loop near or below angle of mandible there oozing of blood, on examination
patient is conscious and alert, what is the next step?
A. Surgical exploration B. CT angiogram (stable zone II, no hard signs) C. US D.Wound closure and haemostasis
76. A midline neck stab wound at the level of cricoid cartilage with haematoma management?
A. CT B. Angiography C. Exploration (unstable zone I, hard sign)
77. What is the mainstay of treatment of blunt carotid artery injuries?
A. Surgical exploration and vein patch graft B. Endoluminal stenting C. Catheter thrombolysis D. Anticoagulation
78. What is the most injured cranial nerve during carotid endarterectomy?
A. IX B. X C. XI D. Xll
79. Stab neck below angle of mandible with haematoma, weak carotid pulse drowsiness, management?
A. Exploration (unstable zone II, hard sign) B. CT C. Angiography
Hard signs suggest the presence of a serious injury that needs immediate attention or intervention as there are many important neck structures as vascular
and aerodigestive. Vascular as; rapidly expanding or pulsatile haematoma, severe haemorrhage or difficult to control bleeding, shock refractory to fluid
resuscitation, decreased or absent pulse, vascular bruit or thrill, neurologic deficit. Aerodigestive as: massive haemoptysis, significant haematemesis,
respiratory distress
80. Neck Stab wound 2 cm below the angle of the mandible with progressive haematoma, low BP, and Pulse, ttt?
A. Exploration B. Angiography C. Conservative and follow up
81. An 8-year-old boy presented with stab neck in the middle and lateral side of neck, patient is stable, but has expanding haematoma,
management?
A. Surgical exploration B. Embolisation C. Angiography D. Conservative
82. During a forceps delivery of a baby, the Lt. stylomastoid foramen was injured by the forceps blade, which of the following is going to
occur due to this? Loss of taste sensation of the anterior 2/3 of the tongue, due to injury of the chorda tympani nerve, a branch of the facial nerve
General sensation to the anterior two-thirds of the tongue is by innervation from the lingual nerve, a branch of the mandibular branch of the trigeminal
nerve. Taste perception in the posterior third of the tongue is accomplished through innervation from the glossopharyngeal nerve, which also provides
general sensation to the posterior one-third of the tongue.
83. A patient with neck injury in zone 2, vitally stable, no vascular or aerodigestive injuries, no hoarseness os emphysema what’s next?
A. CT angiography; CTA. B. Neck and chest CT C. Surgical exploration D. Wounds closure and observe E. Discharge
If zone 2 without symptoms as expanding haematoma, airway compromise, dysphagia, subcutaneous emphysema or hoarseness of voice and no transcervical
gunshot wound; go for observation BUT, if any of those problems or if unstable or with severe uncontrolled bleeding go directly for operative exploration.
84. A 34-year-old male after an assault from others he has a penetrating neck injury just below the right mandible with severe bleeding, air
bubbling from the wound, feeble right sided radial pulse, BP 90/60 although 2 L of lactated ringers given, RR is 34 what’s the next best?
A. CTA B. Blood transfusion and IV fluid C. Endotracheal intubation D. Surgical exploration E. Serial examination
85. A patient with stab neck injury at zone 3, he is haemodynamically stable, what is the management?
A. Open exploration B. CT Angiography if positive endovascular embolisation C. Artery ligation D. Primary repair
Zone 3 + Stable = Endovascular embolisation, but first do CTA to diagnose the vascular injury
The management protocol of neck injuries changed from zone approach to non-zone approach, it depends upon if stable or unstable if stable see if he is
symptomatic or asymptomatic will manage here according to affected zone.
86. Cervical sympathectomy what is least likely to improve?
A. Hyperhidrosis B. Scleroderma C. Causalgia D. Frostbite
2. THORACOABDOMINAL DISEASES &TRAUMA SURGERY
1. What is the most common type of congenital diaphragmatic hernia?
A. Morgagni B. Bochdalek C. Hiatal hernia
2. Regarding congenital diaphragmatic hernia what is correct?
A. Requires emergency operation if respiratory distress is resent B. Foramen of Bochdalek hernia is the most common type
C. Foramen of Morgagni hernia presents with respiratory distress D. It is rarely associated with underlying lung pathology
3. A patient has diaphragmatic injury then repaired laparoscopically, after 3 days developed dyspnoea, what is the best most diagnostic
valuable test to assess the condition?
A. Chest x ray Abdomen and chest CT C. ECG D. CTA E. OGD C. ABG
4. A patient with chest stab and diaphragmatic injury, laparoscopic repair done, 3 hours postoperatively; patient is tachycardic, dyspnoeic,
stressed, what is the next investigation?
A. Chest x-ray (unstable can be portable) B. ECG C. ABG D. Chest CT (after being stable & CXR inconclusive
5. What is the main cause of postoperative death in children with chronic diaphragmatic hernia?
A. Increased intra-abdominal pressure B. Persistent lung collapse C. Patent ductus arteriosus
D. Abnormal pulmonary microvasculature
6. What measures the volume of air moved with maximum exhalation after deep inhalation?
A. Vital capacity B. Inspiratory reserve volume C. Tidal volume D. Residual volume E. Total lung volume
7. Middle aged male, with thoracic outlet syndrome, what are the most prominent presentation?
A. Nervous compression manifestation(the commonest) B. Arterial compression(last) C. Venous congestion(2nd.) D. All the above
8. A soldier with pain at Lt., hand, radial pulse palpable and decrease with movement, paraesthesia at ulnar aspect of Rt, arm diagnosis?
A. Cervical spondylitis B. Thoracic outlet syndrome C. Cervical disc bulge
9. An old patient, has neck stiffness, numbness and paraesthesia in little and ring fingers with positive hand elevation test what’s diagnosis?
A. Impingement syndrome B. Ulnar artery thrombosis C. Thoracic outlet syndrome (neurogenic) D. Cervical disc
10. What is the correction for Pectus excavatum?
A. Usually associated with respiratory dysfunction B. Usually associated with dysphagia
C. Often associated with cardiac dysfunction D. Cosmesis is usually the indication for surgery
11. A respiratory quotient (RQ) of 1 indicates that the main source of fuel is?
A. Fatty acid B. Protein C. Carbohydrate D. Ketone E. Glycerol
12. What does the respiratory quotient of 1.2 indicates?
A. Lipogenesis B. Ketogenesis C. Pure fat utilisation D. Carbohydrates are the source of fuel E. Proteins are the source of fuel
13. A 6 yrs. old boy with URTI, 5 days later developed respiratory distress, hypercapnoea, and generalised abdominal pain, what’s the cause?
A. Appendicitis B. Pneumococcal bacterial infection C. Peritonitis
14. Stab chest wound, in unconscious patient associated with pneumothorax, what initially you should do?
A. Chest CT B. Chest wound closure C. Airway D. Blood transfusion
15. A Chest stab wound, eFAST was done and showed fluid in peritoneum, what is the best next step?
A. Thoracostomy B. Pneumonectomy C. Exploration (if unstable) D. Chest CT(if stable)
16. A chest stab wound patient, has abdominal pain, tachycardic& mildly hypotensive what’s the best modality to detect diaphragmatic injury?
A. Thoracoscopy B. Laparoscopy(the best sensitivit 88%, specificity 100%) C. CT(if stable) D. US
17. Stab chest, skin and SC tissue depth is 2 cm, right 8th intercostal space, stable, conscious, eFAST is free, what is the best option?
A. Conservative (as 2 cm with fully stable) B. Thoracotomy C. Thoracostomy D. Chest CT, bronchoscopy & OGD
18. What is the most accurate method to diagnose traumatic aortic arch injury?
A. Upright chest x-ray B. Chest computed tomography C. Magnetic resonance imaging D Transoesophageal echocardiogram
19. A patient with blunt chest trauma, with haematoma, rupture descending thoracic aorta, and mediastinal haematoma, how to manage?
A. Immediate repair B. Delayed repair C. Repair within 6 hours
20. A post chest trauma patient, in ER after chest x-ray showed air in mediastinum best next investigation?
A. Chest CT B. Oesophagoscopy C. Angiography D. Water soluble contrast
21. What is the most common bleeding vessels source of massive haemothorax?
A. Pulmonary tissue from sever contusion B. Intercostal arteries C. Major vasculature in the chest D. Internal thoracic a.
22. What is the cause of massive haemothorax?
A. Subcostal artery (intercostal not subcostal a.) B. Hilar vessel C. Lung parenchyma
Generally, massive haemothorax is caused by injury to the intercostal artery, laceration of the lung, great vessel injury, or diaphragmatic rupture.
23. What is the indication of urgent thoracotomy penetrating chest injury?
A. Initial chest tube output is 500cc B. Initial chest tube output is 1000 cc C. Initial chest tube output is 1500 cc or more than 200 mL/h in 3- 4hr
D. Initial chest tube output is 2000 cc E. Persistent chest tube output of 100 cc/hr.
24. Trauma, tracheal shift, congested neck veins, massive haemothorax, shocked, pale and cold hands what is the first line treatment?
Manage according to ABCDE protocol with rapid Chest tube, ICTD insertion
25. A patient in RTA presented by severe bleeding per nose, disturbed conscious level; disturbed conscious level, O2 saturation is 90%,
FAST +ve, chest x ray query haemothorax, the next step?
A. Exploration B. Stop epistaxis C. Chest tube D. Orotracheal intubation
26. A patient was involved in RTA presented to ER with noisy breathing, and haemothorax, what to do first for life saving?
A. Chest tube B. Assessment of airway(exclude F.B or tongue fall back) C. Blood transfusion
27. If we have a motor vehicle collision; MVC or RTA, which of the following is the first thing to control and save life?
A. Pain B. Bleeding C. Airways D. Disabilities E. Rib fractures
28. A patient came to ER with solid/clotted haemothorax and fracture ribs, ICTD was inserted but no improvement, what should you do?
A. Re-insert another chest tube B. Mechanical ventilation C. Ventilation under pressure D. Support airway + CT+-VATS
29. A post trauma patient with rib fracture and left lower chest bruises, dullness of the left chest, decreased movement of the left chest,
crepitus what is the most likely diagnosis?
A. Flail chest B. Left haemothorax C. Tension pneumothorax D. Cardiac tamponade
30. An RTA patient presented with chest bruises, multiple rib fractures unable to breathe or cough, but his oxygen saturation is 97% on
the room air and severe pain, started to develop hypotension, tachycardia, what is the management?
A. Strong rapid analgesia B. Surgical thoracostomy C. Chest Tube
31. In case of flail chest what is the major concern that make you alert and have to check it as a first priority?
A. Pain B. Impaired or ineffective ventilation C. Number of fractured ribs D. Bleeding chest wound
32. A patient with blunt chest injury resulted in fractures in 3rd., 4th. and 5th. ribs in more than one level, he’s stable. What’s initial step?
A. Intubation B. Assisted ventilation C. IV fluid D. Analgesia
33. A case of diaphragmatic rupture, underwent thoracoscopic repair, after 4 hrs, he developed tachycardia, dyspnoea and cyanosis, what
happened?
A. Pneumothorax (Unequal air entry) B. Air embolism (if equal air entry) C. Recurrent hernia D. ARDS
34. Hypotension and decreased end-tidal CO2 with increased PaCO2 during laparoscopy are likely due to which of the following?
A. Tension pneumothorax B. Inferior vena cava compression C. CO2 embolism D. Anaesthetic overdose
35. A patient came to ER with a stab wound in the neck complaining from SOB and bubbling from the wound, x-ray chest shows air in
mediastinum what is the diagnosis?
A. Ruptured trachea B. Ruptured oesophagus C. Traumatic pneumothorax D. Entrapped external air
36. In the case of Flail chest, which to control first in patient with normal saturation?
A. Airways B. Pain C. Bleeding D. Chest thoaracostomy
37. Trauma to the chest, exam., revealed central trachea, paradoxical chest movement, pO2 85%, BP 80 /60, he is fully conscious?
A. Tube thoracostomy (with 3-sided dressing) B. PEEP C. Epidural analgesia D. Adhesive plaster E. O2 mask
F. Fixation with wire and screw
38. What is the most effective treatment for flail chest, if normal saturation and vitals?
A. Adhesive plaster B. Fixation with wire and screw C. Epidural catheter and analgesia
39. What is the most common complication of epidural analgesia?
A. Hypotension B. Nausea C. Respiratory depression D. Deep vein thrombosis
40. Flail chest, sucking chest wound, fracture 8, 9, 10, 11, initial management?
A. Intubation B. Three side dressing (Heimlich valve maneuver for 3-sided dressing) C. Chest tube through the wound, analgesics.
41. Which of the following is the most emergent in cardiothoracic injuries?
A. Tracheobronchial injury B. Haemothorax C. Tension pneumothorax (obstructive shock and death) D. Aortic rupture
42. A post chest trauma patient he has sucking wound with dyspnoea and oxygen saturation is 87 on room air, what is the cause of
dyspnoea in that sucking wound with that pneumothorax?
A. Hypoxia due to increased plural pressure B. The chest pain is the cause C. Anxiety is the cause D. Chest tube induced pain
43. A policeman came to emergency with gunshot in his chest with occlusive dressing, the patient has pneumothorax, but he is stable,
what is the risk of occlusive dressing in this case?
A. It can hide other injury B. It can cause spread of infection C. It can convert pneumothorax to tension pneumothorax
44. What is the leading cause of death from tension pneumothorax?
A. Decreased venous return (decreased preload and COP) B. Cardiac arrhythmia C. Acute hypoxia D. Acute hypercapnia
45. Chest trauma, distended neck vein, trachea shift to Lt. BP 70\40, no air entry at Rt. side, hyperresonant chest what’s likely diagnosis?
A. Cardiac tamponade B. Tension pneumothorax C. Massive haemothorax
46. Regarding tension pneumothorax, what is the immediate and definitive management plan?
A. IVF B. Needle decompression then thoracostomy C. Tube thoracostomy
47. A patient post motor vehicle collision (MVC) complains of shortness of breath (SOB), while examining the patient there was signs of
respiratory distress and distended neck veins. Chest examination revealed hyper resonance. What is the next step in management?
A. Needle decompression in 2nd intercostal space, if not improved >>ICTD B. Chest tube in intercostal space C. Thoracotomy
D. Tube drainage in intercostal space E. Complete survey
48. A patient after trauma have shortness of breath and decrease air entry in upper left lung lobe and the left lung was hyperresonant to
percussion, the patient is not stable what is the best step of management?
A. Chest tube in left side (for decompression as he’s not stable) B. Needle decompression (only if stable)
C. PA, lateral decubitus chest x ray (initial to DD tension from simple pneumothorax in stable patient) D. AP, Lateral decubitus CXR
49. Traumatic patient presented to ER with profuse bleeding from nose and mouth, cyanosed with decreased breath sound on right side of
the chest, mostly was unstable, but fully conscious which of the following is the most appropriate next step?
A. Right chest thoracostomy (to drain haemothorax) B. Intubation C. IV fluid resuscitation and O type blood transfusion
50. A middle aged male after RTA came to the ER with fracture of 6th. rib with pneumothorax you inserted ICTD in the 5th. space, but no
oscillation, no air or even blood in the ICTD collection set, he is stable O 2 saturation is 98% on room air what is the proper to do next?
A. Insert another tube in the 4th. space B. Insert other in 6th. space C. Give O2 and do chest CT
D. Try to manipulate and manage the tube and recheck ICDT set if all free do CXR
51. What is the most early sign to detect tension pneumothorax? Early signs diminished or absent breath sounds, severe dyspnoea tachycardia
A. Hypotension B. Distended neck veins C. Tracheal shift (late sign) D. Dyspnoea and chest pain E. Tachycardia
52. Scenario of chest trauma, patient presented with trachea shifted to right side, hyperresonance by auscultation, diagnosis?
A. Left tension pneumothorax B. Right tension pneumothorax C. Spontaneous pneumothorax D. Flail chest
53. Trauma patient with multiple injury to axilla and lateral chest wall, fracture 4th,5th ribs, while examination suction sound was found
from lacerated wound on fractured ribs, next step?
A. Chest tube (or Heimlich flutter valve dressing if ICTD is not available) B. Intubation C. Urgent Thoracotomy
54. A middle aged male after fight with other he got a stab chest wound, was haemodynamically stable and admitted for observation, on
the 3rd. days post trauma while doing wound dressing, the doctor found a frothy discharge from the wound site, he has dyspnoea and chest
pain, otherwise all vitals are stable, what is your diagnosis?
A. Septic shock B. Broncho pleural fistula C. Tension pneumothorax D. Delayed pneumothorax
55. Post RTA chest trauma on examination; revealed centralised trachea, paradoxical movement of 6, 7, 8 left ribs, x- ray show: 6, 7, 8 lt.
ribs and blurred costophrenic angle, pO2 is 90%, other parameters are normal, what is the definitive or best management?
A. Intubation and mechanical ventilation B. Tube thoracotomy C. Adhesive strap
Initial is 3 side dressing (Heimlich valve maneuver for 3-sided dressing), Best is intubation and mechanical ventilation
56. A post RTA chest trauma on examination; revealed centralised trachea, paradoxical movement of rib fractures with respiratory rate
34/min, what is the most adverse sequelae can occur for that patient?
A. Hypotension B. Haemorrhage C. Hypoxia with ARDS (12-25%, the most frequent manifestation of post traumatic MOF)
57. Which of the following strategies to avoid air bag injury under 12 yrs. child?
A. Restrain in back seat B. Restrain in front seat C. Rear facing in front seat D. Booster seats in front seat
58. A patient with chest trauma, a chest tube is inserted, but still large amount of air leak associated with increasing pneumomediastinum;
mediastinal emphysema, enlarging subcutaneous emphysema; tracheobronchial tear, dyspnoea and respiratory distress despite chest tube
and large amount of air leakage, what is the best investigation?
A. Bronchoscopy(to R/O bronchopleural fistula or tear) B. Oesophagogram/Oesophagoscopy C. Bronchography D. Chest CT
59. What is the best diagnostic test for emphysema early?
A. Chest x -ray B. HR CT (High resolution CT) C. Bronchoscopy
60. Trauma in middle chest, Lt. sided emphysema, diagnosed as gastric perforation, left sided chest tube, inserted, but no improvement?
A. Oesophagoscopy B. Bronchoscopy
61. What is the most common symptoms of chest emphysema?
A. Productive cough B. Dry cough C. Dyspnoea D. Sweating
62. A post RTA with decreased air entry, cervical emphysema, pneumomediastinum and tympanic chest. What’s the diagnosis?
A. Tracheo-bronchial injury B. Open pneumothorax C. Tension pneumothorax D. Simple pneumothorax
Pneumomediastinum + emphysema = Tracheobronchial injury
63. What is correct site for Lung biopsy taking?
A. Midclavicular 6th intercostal space B. Midaxillary 6th. interspace C. 9th. intercostal space
64. An elderly patient with small lung nodule (<8 mm), he is asymptomatic, not smoker, no family history of cancer, no constitutional
symptoms, your next step is?
A. Follow up with CT scan (if nodule < 5 mm, if unfit, if stable over 4 y discharge, if 5- 8 follow up, if > 8 PET scan) B. Biopsy C. CTH
65. What is the most common solitary lung nodule with a popcorn pattern of calcification?
A. Primary lung cancer B. Metastatic lesion C. An old tuberculosis lesions D. Histoplasmosis E. Hamartoma
66. What is the most common site of impaction of foreign body aspirated by a child?
A. Rt. main bronchus (in line with the trachea) B. Lt. main bronchus
67. An infant has been having episodic coughing for 48 hours. On examination, he is wheezing with decreased aeration of the left chest. A
chest x-ray shows an overinflated left lung, what is the next step in management should be?
A. Insertion of left-side chest tube B. Insertion of right-side chest tube C. Endotracheal intubation
D. Administration of steroid inhaler and observation E. Rigid bronchoscopy (? F.B)
68. Regarding the Lung abscess what is correct?
A. Always excised B. Mostly due to aspiration C. Rarely requires antibiotics if well drained
69. What is the optimal management of aspiration pneumonitis?
A. Endotracheal intubation and mechanical ventilation B. Endotracheal intubation, mechanical ventilation, and bronchial lavage
C. Endotracheal intubation, ventilation, bronchial lavage, and steroids D. Endotracheal intub., ventilation, bronchial lavage, & antibiotic
70. A patient was extubated after thyroid surgery and immediately developed shortness of breath and respiratory distress. Examination
reveals that both vocal cords are in a semi- closed position, how will you secure the airway?
A. Cricothyroidotomy B. Tracheostomy tube C. Re-intubate the patient
71. A case of hydatid cyst in the Rt lobe of the lung, patient has history of SOB, progressive with chest pain, what is the management?
A. Albendazole B. Lung lobectomy (if huge, multiple, ruptured and complicated) C. Aspiration D. PAIR (not for lung)
72. Regarding the intralobar pulmonary sequestration, what is correct?
A. Is supplied by the aorta B. is drained by the azygos venous system C. is commonly associated with diaphragmatic defect
D. Has a separate pleural covering
73. Regarding extra lobar pulmonary sequestration, what is true?
A. Typical presents with repeated pulmonary infections B. is drained by the pulmonary veins C. It is supplied by the Aorta
D. is more common on the right-side E. is commonly connected with the bronchial tree
74. A patient complaining of right sided chest pain, chest x-ray showed right sided lung opacity (cyst), breathing diminished on right side,
diagnosis is hydatid cyst, most appropriate treatment?
A. Albendazole B. Pneumonectomy(mostly for malignancy as NSCLC & mesotheliomas) C. PAIR
D. Cyst excision plus modified capitonnage
75. Lung resection is contraindicated in which condition?
A. Preoperative PO2 is 60% B. Preoperative pCO2 is 50% C. FEVI = 1-liter D. FEVI/VC is 75% E. MBC is 60%
76. A patient confirmed to have plural effusion. Plural tap is obtained, what character suggests that tap is exudative?
A. Pleural/serum protein <5 B. Pleural/serum LDH <6 C. Pleural fluid LDH <1/3 D. Pleural fluid LDH >2/3
77. 60 y.o male with high grade fever, left pleuritic chest pain. O/E early clubbing and left mild pleural effusion, pleural fluid analysis pH
is 7, what is the cause of pleural effusion?
A. Empyema В. ТВ C. Para pneumonic D. Chylothorax
78. How to prevent empyema in a patient with residual haemothorax and a chest tube in place?
A. Intravenous third generation cephalosporins B. Placement of a second chest tube C. Needle thoracentesis D. IV vancomycin
79. What is the most important in case of chest empyema for its eradication among the following options?
A. Close the space of empyema B. Keep chest tube till the end C. Give broad spectrum antibiotics with good drainage/ open or VATS
80. A patient with chronic chest empyema, which is non-TB, what is the best treatment associated with antibiotics?
A. Thoracocentesis B. Decortication(if refractory even to VATS and if symptoms persist 6 mo post resolution) C. Pleurodesis
81. What would make you go for thoracocentesis?
A. Loculated pleural fluid B. pH of pleural fluid C. LDH of pleural fluid
82. If pleural fluid pH >6.5 what that indicates?
A. It is normal B. It indicates oesophageal perforation C. It indicates pleural transudate D. It indicates bacterial infection
83. Analysis of pleural effusion shows red blood cell count of 500/mm3, white blood cell counts of 600/mm3, protein level of 1.5 g/dL, and
specific gravity of 1.010. What is the most likely diagnosis?
A. Congestive heart failure B. Para pneumonic effusion C. Haemothorax D. Bronchogenic carcinoma
84. A patient with bronchogenic cancer presents has massive pleural effusion, pleural tap done several times before, what should you do?
A. Ultrafiltration B. Chemical Pleurodesis C. Diuretic D. ACEI
85. RTA X-ray show widened mediastinum, what is the diagnosis?
A. Aortic injury B. Cardiac contusion
86. Regarding the blunt cardiac injury what is correct?
A. Most commonly affects the left ventricle B. Most commonly results in ventricular arrhythmias
C. Can be reliably excluded if cardiac enzymes are normal D. Should be suspected if admission electrocardiogram is abnormal
87. What is the most frequent manifestation of blunt myocardial contusion?
A. Atrioventricular block C. Premature ventricular contractions (sinus tachy then, PVCs, AF) C. Premature atrial contractions
D. Atrial fibrillation E. Atrial flutter
88. Trauma with low BP, x-ray trachea shifted to the right, expanded lungs and widened mediastinum, what is diagnosis?
A. Massive haemothorax B. Pneumothorax C. Thoracic aorta rupture D. Spontaneous Pneumothorax
89. Regarding spontaneous pneumothorax what is correct?
A. It is more common in young females B. It is typically post exertional C. It is recurrent in at least 30% of cases
D. It often requires thoracotomy in the first episode E. is often associated with severe persistent pain
90. The commonest complication following, coronary artery bypass grafting; CABG?
A. Atelectasis B. Infection C. Arrhythmia (AF in 15- 40%) D. Myocardial infarction (4-5%) E. Bleeding
91. A case with fractured ribs, bilateral severe lung contusion, femur fracture, trauma to descending aorta, mediastinal haematoma,
stable, what is management plan?
A. Aortic repair and clamping B. Aortic repair on femoral bypass C. Monitor vital signs in ICU
92. Two days after coronary artery bypass grafting; CABG nausea, epigastric pain and tenderness are mostly due to?
A. Perforated peptic ulcer B. Acute cholecystitis C. Acute pancreatitis (post op hypoperfusion) D. Acute myocardial infarction
93. Dyspnoea, x ray showed mediastinal shadow, CT showed left upper lung mass and mediastinal LNs, what is the best investigation?
A. Open lung biopsy B. Mediastinoscopy (diagnostic and for biopsy)
94. What is the most common anterior mediastinal tumour?
A. Thymoma B. Retrosternal goiter C. Lymphoma D. Teratoma
95. A 49-year-old male has a right-sided perihilar mass incidentally found on CT scan performed after a motor vehicle trauma 1 month
ago. He has a 30 pack-year smoking history. He reports his clothes fit more loosely. On examination, he has purple striae on his abdomen
and prominent fat on his posterior neck. CT scan confirms a 4-cm irregular mass without enlarged lymph nodes. treatment is most likely
to involve which of the following?
A. Radiation therapy alone B. Combination chemotherapy and radiation C. Surgical resection
D. Mediastinoscopy and if node negative, then surgical resection.
96. Paraneoplastic syndrome is mostly associated with which of the following?
A. Small cell lung cancer (SIADH, hypercalcaemia & cushing) B. Bronchial carcinoid C. Broncho-alveolar carcinoma
97. Regarding small cell lung cancer, or oat cell cancer, what is most commonly associating with it? Cushing with SCLC 1-5% poor prognosis
A. Myasthenia gravis B. Cushing manifestation (lung Carcinoid 30-46/ SCLC 8-20%) C. No weight loss D. Smoking not related risk
98. What is true regarding, Non-small cell lung cancer?
A. Large cell carcinoma is the commonest type B. Adenocarcinoma is diagnosis of exclusion
C. Patient with risk factor should be screened because it can be symptomatic D. 20% of patients are smokers
99. Multicentricity is characteristic of which of the following?
A. Squamous cell carcinoma of the lung B. Small cell lung cancer C. Bronchoalveolar carcinoma
D. Bronchial adenocarcinoma E. Bronchial carcinoid
100. End stage lung cancer patient have only 1 year expected survival, presented with symptoms of PTH related cancer, what to do?
A. Give oral Ca B. Oral Phosphorus C. IV Calcium D. Morphine
101. What is true for the cardiac ejection fraction?
A. It is normally 45% B. It is an accurate indicator of cardiac dysfunction in the elderly C. It is increased in mitral stenosis
D. It is decreased in mitral incompetence E. It is increased in ventricular septal defect
102. A 53-y.o lady came to ER with chest pain and dyspnoea, MI excluded, ECG showed MR, what patient most commonly has history of?
A. MI B. Infective endocarditis C. Congenital heart disease D. Rheumatic fever
103. A prominent "v" wave in the right atrial venous waveform indicates which of the following?
A. Tricuspid valve regurgitation B. Atrial fibrillation C. Pulmonary embolism D. Atrial septal defect E. Atrial flutter
104. Cannon "a" wave on a right atrial wave form tracing indicates which of the followings?
A. Atrial flutter B. Atrial fibrillation C. Tricuspid regurgitation D. Pulmonary embolism E. Atrioventricular block
105. What’s associated with coin lesion indicate malignancy?
A. Calcification B. Stellate like figure
106. A case of blunt abdominal trauma, patient has right upper quadrant and lower chest pain, with pain in the tip of right shoulder, which
organ of the following is most likely to be injured?
A. Liver B. Stomach C. Spleen D. Intestine
107. A blunt abdominal trauma in RTA with right UOQ tenderness x ray normal there is CT image?
A. Liver tear B. Gastric perforation C. Gall bladder rapture
108. A child hit by hoarse in his abdomen mild tenderness & guarding labs normal what is initial investigation?
A. U/S B. DPL C. X-RAY D. CBC
109. A female child with epigastric pain and bruises due to fall with history of neglected fall few days ago, what to do now?
A. CT(as stable) B. US C. Exploration
110. A patient had liver laceration and the patient is haemodynamically unstable, what to do? Liver laceration the treatment is perihepatic packing
A. Right hepatectomy B. Perihepatic packing C. Right hepatic artery ligation D. Ligation of the involved vessel
111. A patient involved in a fight, camewith stab wound in the UOQ abdomen, he is vitally stable, with local mild tenderness management?
A. Admitted & expectant management B. Abdomen CT(stable for CT) C. DPL
112. A patient presented with abdominal stab wound, after wound exploration you found anterior abdominal fascia penetration, (his vitals
were stable) what's your next step'?
A. Abdominal CT (triple contrast) B. Abdominal MRI C. Laparotomy (if laparoscopy failed)
D. Diagnostic laparoscopy (if peritoneal breach/ evisceration)
113. A 36-y old man is brought to ER with an abdominal stab wound, pulse 110 b/m. RR 16/m. BP 112/70. The wound was explored and
there is anterior abdominal fascia penetration, what is the next step? CT abdomen. (Stable patient), don't shift unstable patent to CT or MRI
114. A patient with pelvic fracture is hypotensive and has grossly positive diagnostic peritoneal lavage, what is the next step?
A. Angiography and embolisation B. Computed tomography of the abdomen and pelvis C. Celiotomy (Laparotomy)
D. Application of C-clamp E. Application of pneumatic anti-shock garment

115. A 43-yrs. man is brought to the ER after an abdominal stab wound. On arrival he is alert, pulse is 110, RR 16/m, BP 112/80, exam.,
shows a stab wound about 2 cm long near the umbilicus to the left with eviscerated omentum from the wound, CT is negative what is next?
A. Exploratory laparotomy B. Repeat CT C. Observation D. Close the wound E. Leave it open
116. In trauma case, Caesarean section is indicated if the foetus is viable and what is more?
A. The mother is unstable and cardiac arrest is anticipated B. 5 minutes of CPR were successful
C. If after 5-7 minutes of CPR were unsuccessful D. Cardiopulmonary resuscitation has just been initiated
117. A poly trauma patient with disturbance of conscious level. FAST US showed mild free intra-abdominal fluid, the patient is unstable,
what’s your plan of management?
A. CT B. DPL C. Laparotomy(after resuscitation)
118. A patient with abdominal trauma, presented with generalised abdominal pain, on exam, patient had guarding, ecchymosis in the flanks,
high amylase, CT done showing pancreatic tear but there is no duct injury, what will you do??
A. Observation B. External drainage
119. A farmer came to ER with severe abdominal pain and green discolouration after he was hit by wooden stick to his abdomen, he is
vitally stable, what is most appropriate management?
A. Abdominal CT B. Abdominal US C. Open surgery D. C&S test
120. Another patient was involved in RTA with blunt abdominal trauma with pain, given the initial treatment and abdominal US done
where found free fluid in the abdomen but he is vitally stable, P is 94, BP 110/65, RR is 19, O 2 is 93% RA what is next best option?
A. Diagnostic laparoscopy B. DPL C. Repeat US D. Pelvi abdominal CT scan
N.B. Blunt + Stable = CT ..............Blunt + unstable = eFAST. Penetrating + Stable = CT.......... Penetrating + Unstable = Exploration
Laparotomy: Blunt + Unstable + Positive eFAST, Blunt or penetration + Positive CT (Even if stable), Peritonitis, Evisceration and Rectal/ gastric /
Gastroduodenal bleeding
121. A motorcyclist, blunt abdominal trauma, and 6 hour later he admitted; with abdominal pain and distention, BP is 100/40 and won’t
respond to resuscitation fluids (unstable) given in ER, what is the most suitable initial investigation to do in this case?
A. DPL B. USG (FAST scan) C. Abdominopelvic CT scan D. Exploration E. Plain x-ray
122. A patient had Blunt abdominal trauma, patient is unstable, FAST showed intraperitoneal fluid. Next step?
A. Exploratory laparotomy B. CT (if stable without free fluid or gases)
Intraperitoneal fluid rises the suspicion of hollow viscus injury, if this patient is stable, we need to do CT to assess the internal organs, as the patient is unstable,
go for Exploration
123. In abdominal injuries, what is the most informative initial investigation?
A. CT B. Diagnostic peritoneal lavage C. US D. X-RAY
124. A female patient, during laparoscopic surgery, duodenal perforation happened, the patient is stable how to manage?
A. Exploration B. Laparoscopy C. Conservative
125. A patient with abdominal trauma, rigid abdomen, vitally stable, what is the best initial management?
A. Exploratory laparotomy (If unstable) B. Fluid resuscitation C. US/CT (to diagnose any internal injuries)
126. During laparotomy for hepatic resection something was found in the mesentery, there was a picture showing multiple diverticula at
the mesenteric border of a jejunal loop, management? Jejunal diverticulum
A. Biopsy (If LNs) B. Resection of the loop C. Postoperative antibiotics (if diverticulitis)
127. Which of the following is an age-related change in respiratory functions?
A. Increased total lung capacity (TLC) B. Decreased total lung capacity (TLC) C. Decreased functional residual capacity (FRC)
D. Increased residual volume (RV) E. Increased vital capacity (VC)
128. Regarding COVID19 pandemic disease who has right to tell everything regarding infections, method of management and prevention?
A. Wuhan B. China C. USA D. WHO
129. A confirmed COVID 19 patient required endotracheal intubation for ventilator support. Who is the most appropriate person to
perform intubation? Most experienced person
130. Confirmed COVID 19 patients, developed ARDS and required mechanical ventilation, which of the following is the most appropriate
setting? Low tidal volume ventilation (VT 4-8 ml/kg body weight)
131. What is the most reliable indicator of successful endotracheal intubation?
A. Chest x-ray B. End-tidal CO2 C. Cord visualisation D. Chest auscultation E. Pulse oximetry
132. Positive end-expiratory pressure therapy will result in which of the following?
A. Decrease in extravascular lung water B. Increase in cardiac preload C. Decrease in atrial natriuretic peptide
D. Decrease in functional residual capacity
133. Auto-positive end-expiratory pressure in mechanical ventilation is most likely to develop with which of the following?
A. High rate and prolonged I: E ratio B. High rates and decreased I: E ratio
C. Decreased rate and decreased I: E ratio D. Decreased rate and prolonged I: E ratio
134. Positive end-expiratory pressure will result in which of the following?
A. Increases cardiac output B. Decreases functional residual capacity C. Increases right to left shunting
D. Lowers pCO2 D. Decreases alveolar arterial oxygen gradient
135. What is the action of the lung functional residual capacity?
A. Chest wall exerts inward elastic recoil B. Lungs exert outward elastic recoil C. Alveolar pressure equals pleural pressure
D. Lungs and chest wall exert equal and opposing recoil
136. The use of inverse ratio ventilation will which of the following?
A. Decreases auto-positive end-expiratory pressure B. Improves alveolar ventilation
C. Increase incidence of pneumonia D. Decrease mean airway pressure
137. What is the action of Inverse ratio ventilation?
A. Decreases mean airway pressure and increases intrathoracic pressure B. Decreases mean airway pressure and decreases intrathoracic pressure
C. Increases mean airway pressure and increases intrathoracic pressure
D. Increases mean airway pressure and decreases intrathoracic pressure
138. What is the most reliable indicator of successful ventilatory weaning?
A. PO2 >100 with FiO2 of 40% B. PCO2 <40mmHg C. f/Vt <100 D. negative inspiratory force less (more negative) than 30cm H2O
139. Considering the COVID 19 pandemic, what is the WHO recommendation for washing hands when only using water and soap? 40- 60 sec.
140. A heavily smoker, diabetic and dyslipidaemic male patient, with retrosternal pain, relieved by rest, sweating and vomited diagnosis?
A. Boerhaave’s syndrome B. MI C. Stable angina
141. What is the action of the Intra-aortic balloon pump?
A. Increases pulmonary wedge pressure B. Increases afterload C. Increases duration of systole D. Decreases duration of diastole
E. Increases diastolic pressure (decreases afterload and reduces aortic pressure, with increase diastolic flow coronary blood flow improves)
142. In the elderly, regarding cardiac function, what is correct?
A. The myocardium is oversensitive to the effect of catecholamines B. Systolic function is more impaired than diastolic function
C. Diastolic function is more impaired than systolic function D. Systolic and diastolic functions are equally impaired
143. A patient with fever and chest pain and they mentioned an ECG finding they ask about which part will be affected?
A. Pericardium (Acute Pericarditis) B. Myocardium
144. After an acute myocardial infarction, a drug eluting stent; DES inserted, for how long you should postpone the elective surgery?
A. > I month (Bare-metal or urgent after DES) B. 3 months C. 6 months (if time sensitive or 1 year if elective) D. 1-2 years
145. What is the most important initial management of suspected blunt myocardial injury?
A. Electrocardiogram and troponin1 monitoring, if abnormal >> echocardiography B. Chest computed tomography scan
C. Assessment of cardiac enzymes D. Insertion of pulmonary artery catheter E. Echocardiogram (if abnormal ECG & Troponin 1)
146. An HDU patient, had Aortic surgery, monitored by pulmonary artery catheter, what’s the disadvantage of pulmonary artery catheter?
A. Increase in mortality B. Increase morbidity C. Increase HDU stay D. Increase in Hospital stay.
147. Regarding postoperative myocardial infarction what is correct?
A. It is often associated with chest pain B. Ischaemic injury is more common postoperatively rather than intraoperatively
C. T-wave changes are the most specific finding for acute myocardial infarction
D. It occurs most commonly within the first 48 hours after surgery
3. ORTHOPAEDIC TRAUMAS
1. What is the best way to clear and assess the cervical spine in post-traumatic neck pain?
A. CT B. MRI C. Lateral view x- ray of spine
• In children, a CT scan of the neck is indicated in more severe cases such as neurologic deficits, whereas X-ray is preferable in milder cases, by both USA
and UK guidelines. Swedish guidelines recommend CT rather than X-ray in all children over the age of 5.
• In adults, UK guidelines are largely similar in children. US guidelines, on the other hand, recommend CT in all cases where medical imaging is indicated,

2. An RTA case with lumbar spine pain, bruises and drowsy, what’s the initial most important step before neurosurgery arrive to assess?
A. Pelvic binder B. CT C. Restrict spine mobilisation
3. An elderly female that has back, and she is stooping and bending her back while she walks to relieve the pain. What is the diagnosis?
A. Lumbar spine canal stenosis B. Degenerative Lumbar spine
Lumbar spinal stenosis can cause pain in the low back or buttocks, abnormal sensations, and the absence of sensation (numbness) in the legs, thighs, feet, or
buttocks, or loss of bladder and bowel control. MRI is the preferred method of diagnosing and evaluating spinal stenosis of all areas of the spine
4. A patient with epilepsy, Most common shoulder dislocation?
A. Inferior B. Supra acromion C. Posterior D. Anterior
Most common shoulder dislocation: ANTERIOR Most common shoulder dislocation associated with epilepsy: POSTERIOR
5. A patient came with his shoulder flat, arm is adducted and internally rotated, what is the diagnosis?
Posterior dislocation. (2% - 5% of all shoulder dislocations). N.B Hip dislocations either anterior or posterior, with posterior hip dislocations comprises most
traumatic dislocations reverse of shoulder
6. A 45 y.o patient, has a mass of 5 cm in right upper limb, MRI shows a mass from triceps what to do next?
A. Incisional biopsy B. Excisional biopsy C. Core needle biopsy D. PET Scan
Mass superficial or less than 5cm -> Do an excisional biopsy. Mass 5cm or more -> Do core needle biopsy
• If superficial and less than 2 cm >>> excision biopsy, if 2- 4 cm >> core biopsy. If deep, suspicious; firm/ hard, fixed or firmly attached to muscle or with
gritty sensation on excision or > 4 cm with suspicion refer to cancer centre.
7. Which site Radial nerve can be inured and what is effect of its injury?
A. At axilla with ape deformity B. Humeral groove with humeral fracture; Wrist drop
8. How ulnar nerve can be injured and what’s the effects? Claw hand, Common with elbow injury
9. A 35 years, diabetic female patient, has sudden onset of difficulty to oppose the thumb, what is the involved nerve?
A. Ulnar nerve B. Median nerve C. Radial nerve
10. Injury of forearm, with inability of flexion index and middle finger, which nerve is injured?
A. Claw hand (Ulnar nerve) B. Sensory loss only (also median but usually mixed sensory motor) C. Wrist drop (radial nerve)
D. Median N. injury Median nerve injury; unable to do thumb opposition, common with anterior of flexor side wrist injury
11. A patient fell on his elbow, what will you see in lateral x ray?
A. Radial line anterior to acetabulum B. Some line bisects with acetabulum C. Anterior fat D. Posterior pad of fat
12. A 12- year-old boy injured with supracondylar fracture and distal radial pulse absent, which of the following is the appropriate next
step in management?
A. K wire B. Intramedullary nail C. Surgical exploration D. Closed Reduction
13. What is the fracture that is less likely associated with vascular injury?
A. Mid ulnar bone fracture B. Supracondylar humerus fracture C. Supracondylar femur fracture
D. Post dislocation of knee joint
14. Volkmann's contracture is a complication of which of the following?
A. Humeral head fracture B. Femoral neck fracture C. Posterior dislocation of the knee
D. Supracondylar humeral fracture E. Colles ' fracture
15. What is the most injured nerve under general anaesthesia?
A. Radial nerve B. Ulnar nerve C. Median nerve D. Brachial plexus E. Common peroneal nerve
The ulnar nerve is damaged mostly (0.33% of general anaesthetics); this is three times as common as injury to other nerves. Other nerves at high risk
during general anaesthesia include the brachial plexus, lumbosacral roots, radial, sciatic and common peroneal nerves.
16. Pulp space infection, causing osteomyelitis what is that called? (picture of injured finger)?
A. Felon B. Paronychia C. Tenosynovitis
Felon is a pulp space infection of fingertip with pus collection if neglected can >> Osteomylitis ttt >> drainage + antibiotics. A felon is an abscess between
the specialised fibrous septae in the fingertip pulp. It causes intense pain and may lead to terminal phalangeal osteomyelitis.
17. A picture of dark coloured nail resulted after trauma by heavy blunt object, what is the diagnosis?
A. Subungual haematoma B. Subungual melanoma
18. A patient fell on outstretched hands then he felt severe pain at snuff box that is worse on pinching or grasping what’s problem?
A. Collie’s fracture B. Scaphoid (15% of acute wrist injuries, 60% of carpal fract.)
Scaphoid Fractures are the most common carpal bone fracture, 15% of acute wrist injuries, 60% of all carpal fracture, occurring after a fall onto an
outstretched hand; axial load across a hyper-dorsiflexed, pronated and ulnarly deviated wrist, typically include pain and tenderness in the area just below
the base of the thumb, it may worsen when trying to pinch or grasp something
19. Regarding scaphoid fractures, there is necrosis due to which of the following?
A. There is dislocation fracture B. Blood supply from distal to proximal C. Tendon's cause tension on fractured fragments
20. A patient CO pain along median nerve distribution, and positive Tinel’s sign treatment include casting of the hand in which position?
A. Dorsiflexion B. Palmar flexion C. Wrist extension D. Adduction
21. A 40-yrs. old patient presented to the physician and diagnosed with lumbosacral radiculopathy at S1 sciatica, what do you think the
complaint of the patient was? Pain in the buttock, back of the calf, and side of the foot
22. What is the management of pain caused by Paget’s disease of the bone? Bisphosphonate or Denosumab if failed >> Radiotherapy
23. A patient with pelvic fracture who was resuscitated and is currently unstable, BP 86/50 HR 120 What is the next step?
A. Surgical intervention B. Pelvic binder
24. An open book pelvic fracture, the patient is shocked in class III, what fluid to give for resuscitation? Initial is 2 L Ringers’ lactate, and
the best is Blood
25. First fluid resuscitation after pelvic fracture, in a vitally stable patient? Ringer’s Lactate
26. What is a sports hernia?
The basic definition of sports’ hernia is post physical activity, causing persistent unilateral pain in the groin without demonstrable true hernia.
27. A football player suddenly complained of severe Rt. iliac fossa pain, on examination, no swelling or cough impulse just tenderness dx?
A. Sport hernia B. Inguinal hernia C. Haematoma
Treatment a) local rest following acute injury b) application of ice packs 3-4 times with 10 minutes each time with c) NSAIDs
28. A basketball player C/O sudden severe pain in Rt. lower abdomen, scrotum free?
A. Sport hernia B. Haematoma
29. A 25-year-old football player came with sudden swelling in the right inguinal region, which was tender, scrotum is normal containing
no swellings what is possible cause?
A. Torsion of spermatic cord B. Injury to posterior wall of inguinal canal C. Congenital inguinal hernia sac
D. Rupture of inferior epigastric vessels
Inferior epigastric artery is a branch of the external iliac artery found on the anterior abdominal wall passing superomedially from the inguinal canal
toward the midline, it gives off several branches that supply the skin and muscles of the anterior abdominal wall, deep structures of the abdominal wall,
spermatic cord, it terminates at the level of the umbilicus by anastomosing with superior epigastric artery which is a branch of the internal thoracic a.
30. A young patient with suddenly developed Groin pain and swelling when he was playing football, O/E scrotum and testes are normal.
groin tender swelling. What is the most common cause?
A. Tear in posterior wall of inguinal canal B. Ruptured inferior epigastric artery, IEA. C. Testicular torsion
D. Inguinoscrotal hernia
31. A case of sport hernia, pathology? Tear of the posterior wall of inguinal canal… If pain >>> hernia. if swelling >> haematoma
32. A 24-year-old male, during playing football a groin swelling appeared, it was tender and not reducible, management.
A. Analgesics and physiotherapy B. Surgical evacuation of the haematoma C. Surgical hernia repair
33. What is the best investigation for sport hernia?
A. CT B. MRI C. Aspiration
34. Bilateral unequal limb in child, short femur what is the diagnosis? Perth’s disease
35. Adolescent with Rt. hip pain during activity relieved by rest, 0.5 cm shortening of the Rt. lower limb what is diagnosis?
A. Perth's disease B. Slipped capital femoral epiphysis
Presentation is typically at a younger age than slipped upper femoral epiphysis (SUFE) with peak presentation at 5-6 years, but confidence intervals are as
wide as 2-14 years is a relatively common condition affecting the physis of the proximal femur in adolescents, that leads to slippage of the metaphysis
relative to the epiphysis and is most seen in adolescent obese males.
36. A 10-year-old boy C/O hip pain, that relieved by rest, diagnosis?
A. Slipped capital femoral epiphysis B. Perth’s disease
known as Legg-Calvé-Perthes disease, refers to idiopathic osteonecrosis of the femoral epiphysis seen in children between 4-8 y, with shrter children, and
deformity of the femoral head. Most children are present with atraumatic hip pain or limp. Some children have a coincidental history of trauma.
37. What is the best treatment of the reactive arthritis?
A. Corticosteroids B. Topical steroids C. Rheumatoid arthritis drugs. D. Nonsteroidal anti-inflammatory drugs (NSAIDs)
Prescription of NSAIDs, such as indomethacin (Indocin), can relieve the inflammation and pain of reactive arthritis.
38. A patient with joint swelling and redness diagnosed as Rheumatoid arthritis?
A. MTX B. MTX+ sulpha C. MTX+ Hydroxy D. Methotrexate; MTX+ Hydroxy chloroquine + Prednisone
39. A female with interphalangeal joint pain and swelling no morning no tenderness stiffness lab normal:
A. Hand Osteoarthritis; OA B. Seronegative rheumatoid arthritis RA
40. A child presents with hip and groin pain. An US was done to reveal hip joint effusion. He is afebrile, what is the most likely diagnosis?
A. Toxic synovitis B. Benign acute myositis C. Osteomyelitis D. Septic arthritis
NB. Toxic synovitis = transient synovitis, 70% of cases preceded by URTI Diagnosis mostly clinical diagnosis Management: NSAIDs + rest
41. Pseudogout crystal is mostly due to which calcium salts among the following?
A. Calcium oxalate B. Calcium pyrophosphate C. Calcium urate
42. A patient with foot drop, lost the sensation in web space between 1st. & 2nd. metatarsal bones, which nerve is damaged?
A. Common peroneal nerve B. Deep Peroneal Nerve C. Tibial nerve D. Femoral nerve
43. A patient who had right big toe pain and because of that he was diagnosed with gout and treated with allopurinol. He’s a known case
of psoriasis. He presented now with right big toe pain, right ankle pain and bilateral knee pain and there was no improvement with
allopurinol. What’s the diagnosis? Labs shows high uric acid
A. Active pseudo-gouty arthritis B. Active gout arthritis C. Psoriasis arthritis
44. After reduction of posterior knee dislocation, the patient should undergo?
A. Observation B. Discharge C. Splinting D. Angiogram E. Internal fixation
All reduced knee dislocations without hard signs of arterial injury should be assessed with ABI/API measurements, may need angiography if any weak or
impalpable pulse as may be popliteal artery compression. Any reading of less than 0.90 should prompt further imaging (CT angiography vs duplex US),
which should be decided upon in conjunction with the vascular consult.
45. What can lumber Disc lesion between L4 and L5 lead to?
A. Reduced knee jerk B. Reduced ankle jerk C. Weakness of foot dorsiflexion
D. Reduced sensation on the small toe
L5 radiculopathy and peroneal neuropathy can both present with weakness of the foot dorsiflexors and toe extensors, however, L5 radiculopathy may present
with weakness during foot inversion versus weakness with foot eversion associated with peroneal neuropathy
46. What L5, S1 disc lesion will result in?
A. Weak plantar flexion B. Weak dorsiflexion C. Absent knee jerk D. Lost sensation in the big toe
47. A 35 y.o male came with bilateral severe crush injury of both thighs with femur fracture, what should be in mind for this patient?
A. Haemorrhage B. Myoglobinuria C. Hypotension D. Fracture fixation
48. A 37 year old female, presented to the ER with post traumatic abdominal pain, tenderness, with thigh crush, BP 90/60, P 122 not
responded after IVF and vasopressors labs were, K 6.1, Na 129, what is the appropriate next step?
A. Urine culture B. ECG (peaked T wave, if 8.5 mEq/L >> cardiac arrest) C. Full renal chemistry (if K>7) D. Serum Lactate
49. A case of crushed both lower limbs with hyperkalaemia, what is most concern? Cardiac arrest
50. A 28 -year-old male has a closed head injury, pulmonary contusion, grade III splenic injury, and closed femoral shaft fracture. The ideal
management of his fracture?
A. External fixation B. Skeletal traction C. Intramedullary nailing within 24 hours of injury
D. Intramedullary nailing, I week after the injury E. Use of metal plates and screws
51. A 22-yrs. old male was brought to the emergency due to severe femoral pain after trauma to the leg. X ray was done and showed a
displaced fracture of the femoral shaft with 30 % angulation. What is your next step in management? Open reduction with nail
(interlocking intramedullary nailing)
52. A porter exposed to fall of building over him, patient entrapped and get out after 3 hours, both lower limbs had multiple bruises and
he has oliguria, what is the cause or the pathophysiology of this condition?
A. Reperfusion injury B. Acute limb ischaemia C. Hypovolaemia
53. A middle aged driver involved in MVC with both his thighs were severely crushed, where transferred to hospital where dressing and
washing of his wounds done but after 5 hours, he was oliguric in spite of lot of fluids given, and fasciotomy done, on doing ECG, found
peaked T wave, flattened P wave, prolonged PR interval with ST depression, what is the cause or pathophysiology of his condition?
A. Toxic shock syndrome B. Ischaemic reperfusion with hyperkalaemia C. Refeeding with hyperphosphataemia
D. Hypernataemia E. Hypokalaemia F. Hyponatraemia
54. What is the cause of dark brown urine in crushed limb injury? Myoglobinuria for IVF then urine alkalinisation by IV NaHco3 if acidosis
Crush syndrome may present with rhabdomyolysis, which manifests with hypovolaemia (due to fluid sequestration in muscles) as hypocalcaemia(due
calcium deposition in muscle), hyperkalaemia, hyperphosphataemia, metabolic acidosis, and myoglobinaemia/ myoglubinuria. Don’t perform prophylactic
fasciotomy. Except if confirmed and progressing acute compartment syndrome clinically. Give isotonic saline as one liter/hour at and after extraction,
initiated in the field should be continued with close monitoring of urine output, except if he still under rubble for two hours reduce to 500 cc NaCl/hr, so
within 6 hours from IVF initiation will be from 3-6 liters if he is anuric continue IVF of 0.5 -1 liter/day plus losses of previous days. If he is passing urine see
if older or has history of HF, give 3-6 L/day if not give more than 6 L/day.
55. A case of crush injury, presented in ER with hypotension, red urine then anuria, what is the optimal immediate treatment?
A. IVF if hypovolaemia, glucose insulin (NaCl, 10 U insulin then Dextrose 50 ml of 50%, then dextrose 10% at 50-75 ml/hr to avoid hypoglycaemia)
B. IVF, then urine alkalinisation by IV NaHco3 (50 mEq IV over 5 minutes if acidosis) C. Fasciotomy and debridement
D. IV Ca gluconate (if ECG changes or K >6.5) E. Haemodialysis (in end stage kidney disease or severe impairment)
In patients with severe hyperkalaemia, treatment focuses on immediate stabilisation of the myocardial cell membrane, rapid shifting of potassium to the
intracellular space, and total body potassium elimination. Definitive therapy in severe rise is haemodialysis in patients with kidney failure or when
pharmacologic therapy is not sufficient. Any patient with significantly elevated potassium levels should undergo dialysis; pharmacologic therapy alone is
not likely to bring about adequate reduction of potassium levels in a timely fashion, discontinue any potassium source, enhance its uptake by D50W,
enhance excretion NaCl 0.9% with lasix lastly dialysis if all failed.
56. One patient after an RTA has isolated femur fracture, what is the management?
A. Thomas B. Cast C. Intra-medullary nail (open technique) D. Intramedullary nail (closed technique)
57. A patient had motor bike accident with mid shaft fracture of femur, he is conscious, Oriented, what to do before fixation?
A. Sedation B. Analgesia C. Call operation room D. Examine the pulse
58. A pedestrian hit by car, crushed L.L, x-ray showed bilateral femur fracture, he is hypotensive, what is the major concern?
A. Hypotension B. Haemorrhage (up to 3 L from both thighs) C. Myoglobinuria
59. A male patient came to ER with right femur shaft fracture, he is haemodynamically stable, no other injuries, what is your priority?
A. Pain management B. Control bleeding C. Maintaining function D. Decrease soft tissue trauma
60. A middle aged male patient was involved in an RTA, with polytrauma although he was stable haemodynamically, in ER was examined
by orthopaedic surgeon who decided to to CT for lumbosacral region and pelvis, while they transporting him his BP rapidly decreased to
be 90/60 with 120 pulse rate, the patient was dizzy with some anexity what is the first priority in that situation?
A. Wide bore 2 cannula insertion B. Control bleeding C. Protect airway(ABCDE) D. Do splinting E. Control pain
61. A 2-year-old child refused walking since hitting toy and his right leg was twisted when he fell, what is diagnosis?
A. Spiral fracture of Rt. tibia B. Spiral fracture of Rt. femur C. Soft tissue swelling of Rt. ankle
D. Chip fracture of proximal Rt. tibia
62. Adult fell from height complains of severe heel pain. He is conscious, oriented and has stable vitals. What’s the next step?
A. Pain control B. Lower limb X-ray C. Pulse palpation
63. A male patient presented to the ER after RTA with tibial fracture, and he was stable, what is our priority regarding his management?
A. Control the pain B. Control the bleeding C. Limit the soft tissue injury (nerves, vessels, muscles) D. Thomas splint
64. A patient with AF, embolectomy of femoral artery done for him, then started pain, numbness, swelling of the foot. On exam, tender
calf mass, popliteal artery is felt, what is most likely diagnosis?
A. DVT B. Compartmental syndrome C. Re-embolisation
65. A patient post foot crush injury with heavy object came with excruciating pain and pale cool limb, what is the best next step?
A. X-ray B. CT scan C. Compartment pressure D. Pain killers with IV fluids
If choice of fasciotomy is found it will be the best option but if not mentioned the best alternative is compartment pressure measurement and monitoring to
rule out leg compartment syndrome which if not urgently treated will scarify the limb.
66. A patient had fracture and compartment syndrome, what’s the first alarming sign?
A. Absent pulse B. Toe discolouration C. Paraesthesia
The earliest symptom of compartment syndrome is worsening pain which is typically out of proportion to the severity of the apparent injury and is
exacerbated by passive stretching of the muscles within the compartment, as an example for the anterior leg compartment, by passive ankle plantar flexion
and toe flexion, which stretches the anterior compartment muscles Late manifestations of compartment syndrome include the absence of a distal pulse,
hypoesthesia, and extremity paresis or paralysis
67. A patient with electrical burn of the leg complains of pain on passive movement of the foot, pedal pulses are diminished, what is next
step of management?
A. Escharotomy B. Femoral angiogram C. Leg elevation and intravenous heparin D. Fasciotomy
68. What is the method of follow up to avoid compartment syndrome in limb trauma case with deformity and swelling and intact skin?
A. Doppler US B. Blood pressure measure C. CVP D. Follow up with the hand-held manometer (Stryker device), tonometry
69. After vigorous exercise, if an athlete develops pain on dorsiflexion of the foot and decreased sensation in the first web space,
appropriate action should be?
A. Color duplex scan and immediate heparinisation B. Immediate femoral arteriogram
C. Leg elevation, ice packs, and non-steroidal anti-inflammatory medication
D. Immediate fasciotomy (leg elevation contraindicated in compartmental syndrome)
70. A patient came after limb trauma, with severe pain and paresthesia between his toes and it was pale. X ray shows fractures,
intercompartmental pressure was 35mmhg what to do?
A. Internal fixation B. Closed reduction C. External fixation D. External fixation with fasciotomies
Acute compartment syndrome is a surgical emergency, so prompt diagnosis and treatment are critical. Once the diagnosis is confirmed, immediate surgical
fasciotomy is needed to reduce the intracompartmental pressure. The normal pressure within a compartment is from 0 to 8mmHg. If reaches 30 mmHg or
greater, acute compartment syndrome is present. If greater than 30 mmHg or delta pressures less than 30 mmHg, surgical fasciotomy should be done.
71. Fracture of the leg bones with deformity what to do as a priority of management?
A. Control pain B. Control blood loss C. Save joint
72. Inferior lateral Injury to the knee causing fracture of head of fibula, which nerve could be injured?
A. Tibial nerve B. Common peroneal nerve C. Deep peroneal nerve D. Femoral nerve
73. Almost the same case and the patient is hypotensive, what is the cause of hypotension?
A. Decreased COP B. Haemorrhage C. Peripheral vasodilatation
74. If there is an open fracture, which is the first thing to do?
A. Antibiotics B. Pain control C. Stop bleeding D. Debridement
75. You are working in tertiary hospital, you received 75 cases, one of these case 75 yrs. old male patient, his abdomen was tense and rigid,
fracture pelvis, absent distal pulse, wound in thigh, BP 70/40 RR 40, what triage of this patient?
A. Urgent B. Immediate C. Delayed D. Dead
76. Which of the following treatment options is appropriate for management of osteosarcoma of the distal femur?
A. Above-knee amputation B. Chemotherapy followed by above-knee amputation C. Chemoradiation
D. Chemotherapy alone E. Chemotherapy and limb-sparing surgery
The current standard of care for distal femoral osteosarcoma is neoadjuvant chemotherapy followed by limb salvage for the surgically resectable tumour,
reconstruction of the bone and soft tissue defect, and adjuvant chemotherapy.
77. A case of malignant tumour in the bone associated with onion peel appearance, diagnosis?
A. Ewing sarcoma B. Osteosarcoma
78. Which of the following is the most important determinant of prognosis for a patient with soft tissue sarcoma?
A. Depth of invasion B. Cell type C. Histological grade D. Tumour size E. Tumour site
79. Which of the following sarcomas is most sensitive to chemotherapy?
A. Leiomyosarcoma B. Chondrosarcoma C. Liposarcoma (the least sensitive) D. Synovial sarcoma
80. What is the most important determinant of survival of retroperitoneal sarcoma?
A. The use of adjuvant chemotherapy B. The size of the primary tumour C. The histologic type of the primary
D. The use of intraoperative radiotherapy E. Complete surgical resection
81. What is the most common retroperitoneal sarcoma type?
A. Liposarcoma (large> 20 cm in 50% of cases, worse prognosis > extremity sarcoma) B. Fibrosarcoma C. Leiomyosarcoma
D. Neurosarcoma
82. What is the most common presenting symptom of retroperitoneal sarcoma?
A. Large abdominal mass B. Ureteric obstruction C. Rectal obstruction D. Lower extremity swelling by venous congestion
83. Which of the following is radio resistant tumour?
A. Bronchogenic carcinoma B. Parotid carcinoma C. Osteogenic sarcoma D. Dysgerminoma
84. An adult patient with history of bone cysts, he got another bone fracture what is most likely associated electrolyte abnormality?
A. Hypocalcaemia B. Hypercalcaemia (Osteitis fibrosa cystica in PHPT) C. Hypomagnesaemia D. Hyperphosphataemia
85. What is the most radiosensitive tumour? Lymphoma, Myeloma, Seminoma. Oropharyngeal SCC,Wilm’s,Ewing’s. resist(Renal CC, melanoma, thyroid, CRC)
A. Neuroblastoma B. Nephroblastoma C. Lymphoma D. Melanoma E. Renal cell carcinoma
86. For the Merkel cell carcinoma what is correct?
A. Is highly radiosensitive B. Has the best prognosis when it occurs on the trunk C. Seldom spreads to regional lymph nodes
D. Is less likely than melanoma to recur after local excision

4. UROLOGY AND GENITOURINARY SURGERY


1. What is the arrangement of the anteroposterior anatomic relationship at the renal hilum?
A. Vein, artery, ureter (VAP) B. Artery, vein, ureter C. Ureter, artery, vein D. Ureter, vein, artery
2. A 20 year old patient complaining of lower abdominal pain and tenderness like cystitis typical case, how you will manage
A. No need for treatment B. Empirical treatment C. Wait for culture D. Encourage hydration and Carnaby fruit
3. An elderly has difficulty initiate urination and have lower abdominal pain what most appropriate management?
A. Insert Foley catheter and urine culture (if empiric failed) B. Systemic Antibiotics (Cystitis) C. Urgent prostatectomy
4. UTI in patients with allergy to penicillin, sulpha what is antibiotic can be prescribed?
A. Nitrofurantoin B. Ampicillin C. Aminoglycosides D. Quinolones
5. A child has scrotal pain, on exploration the cord was oedematous and inflamed with red right hemiscrotum, what is the diagnosis?
A. Testicular torsion B. Inguinal hernia C. Testicular appendages torsion D. Epididymo orchitis (mumps -> epididymo-orchitis<2y)
6. A pregnant lady presented with recurrent UTI what to do?
A. X ray B. US C. Ureteroscopy D. Cystoscopy
7. A case with frequency and urgency, what is the most appropriate investigations?
A. Urine analysis B. Urodynamic study C. Uroflowmetry D. KUB CT
8. A case of UTI > 14 days, most probably will cause pyelonephritis in how much percent?
A. 0.05% B. 0.5% C. 5% D. 50%
9. A child with night bed wetting although he was fully trained since he became 6 y/o, labs showed nitrite. what is the diagnosis? UTI
10. A 24 year old female with uncomplicated UTI with fever, flank pain and dysuria, what is the appropriate management?
A. Treat from home with Ciprofloxacin, fluoroquinolones (Uncomplicated UTI non obstructed, no instrumet, confined to lower UT)
B. Treat from home (with irrelevant antibiotic) C. Admit and give carbapenem D. Admit and give ceftriaxone and vancomycin
11. How to detect renal scarring in a child?
A. DMSA scan (dimercapto succinic acid assessing renal morphology, structure and function) B. MCUG (micturating cystourethrogram)= VCUG
12. What is the best imaging modality for vesicoureteral reflux? in children DMSA, in adult VCUG (fluoroscopic voiding cystourethrography)
13. A male patient with renal colic, what is the best diagnostic test for renal colic?
A. KUB B. Renal US C. Abdomen CT-KUB D. Abdomen MRI
14. Which tumour has the least potential of malignant transformation?
A. Renal angiomyolipoma B. Ovarian embryonic carcinoma C. Osteosarcoma D. Mesothelioma
Renal Angiomyolipomas are the most frequent benign renal tumour with a prevalence varying between 0.2% and 0.6% and a strong female predilection
15. A middle-aged male patient with flank pain and urine output 500 cc/24hr., he has history of renal stone, what’s next investigations?
A. CT KUB B. Cystoscopy C. IVP D. Pelviabdominal US E. KUB x- ray
CT abdomen and pelvis without IV contrast has the highest accuracy of the imaging modalities to identify kidney stones.
16. A 31 year old man came to ER with severe colicky Rt lower abd pain, referred to scrotum, groin BP 139\79 CT picture show 2 white
point in front vertebra what the diagnosis?
A. Ureteric stone B. leaking aortic aneurysm C. Acute appendix D. Acute diverticulitis
17. A case of right flank pain haematuria and painful micturition can't recall what is the best next step?
A. CT with oral and IV contrast B. CT without contrast C. US kidneys
In case of ureteric colic with suspicious kidney or ureteric stone; Plain CT is the option, as it is more accurate with no benefits from adding contrast IV/oral
18. A 36-year-old female presents on the ER with right sided loin to groin pain, rigors and a pyrexia of 38.5°C. Bedside testing reveals
microscopic haematuria, pulse 100/m, a BP of 100/60 mmHg with normal RFT. An x-ray of the kidneys, ureter, and bladder (KUB)
revealed an 8 mm opacity at right vesicoureteric junction and US showed a right sided hydronephrosis, what is next best?
A. CT scan with contrast B. Cystoscopy and retrograde ureteric stent C. Antibiotics and discharge
The patient illustrated has an infected obstructed system. The stone is at the vesico-ureteric junction so should be treated by cystoscopy plus stent (+/- stone
extraction) rather than a percutaneous nephrostomy tube. This is less invasive but if this fails then a percutaneous nephrostomy should be performed.
19. To detect haematogenous spread of tumour, all should be done except?
A. Chest x ray B. Cystoscopy C. Abdominal CT D. Bone scan
20. Regarding Ureteric stones how to manage?
A. Open ureterolithotomy B. Ureteroscopic extraction if < 5mm C. Double J stent D. ESWL for all sizes
E. Expectant treatment if < 5mm size with no backpressure will pass spontaneously
Depending on stone size and position, most ureteric stones are managed expectantly, with SWL, or by Ureteroscopic extraction (URS). Most ureteric stones
of size 5 mm or less will pass, and the relatively uncommon ureteric calculus of 20 mm or greater is best managed by ureteroscopy, percutaneous or
laparoscopic means. For those ureteric calculi of dimensions that lie between these extremes, >5 or < 20mm the treatment alternatives are SWL or URS
21. A gynaecologist during emergency hysterectomy due to uncontrollable bleeding after placental retention and uterine atony,
acaccidentally he ligated and cut the right ureter near to at the pelvic brim, what is the best method for its repair?
A. Ureteroureterostomy B. Transureteroureterostomy C. Ureteroneocystostomy D. Ureteropyelostomy
E. Ureterocalycostomy F. Ileal ureter interposition G. Nephrectomy
1. If the injury is grade I-III, consider placing a ureteral stent and allowing the ureter to heal over the stent. If the injury is to the upper or middle ureter,
consider direct ureteroureterostomy (UU) is primary anastomosis of the injured ureter.
2. Transureteroureterostomy TUU; involves anastomosing the injured ureter to the contralateral ureter in an end-to-side or side-to-side fashion
3. Ureteroneocystostomy; Ureteral reimplantation involves reimplanting the ureter directly to the bladder and is useful for pelvic ureteral injuries.
4. Ureteropyelostomy is a procedure in which the ureter is anastomosed to the renal pelvis.
5. Ureterocalycostomy is a procedure in which the lower pole of the kidney is amputated to expose the infundibulum, and the ureter is anastomosed to it
6. Ileal ureter is a procedure in which a segment of ileum is used as ureter replacement. This is useful when there is a long segment of ureter that is destroyed.
22. Trauma patient with extra peritoneal bladder injury, management?
A. Suprapubic catheter B. Urgent exploration and repair C. Catheter repair and assess after 2 weeks
D. Catheter then US after 2 weeks E. Catheter drainage and definitive repair in 14 days.
Suprapubic catheter: in urethral injury (blood from meatus) Urgent exploration repair; in case of Intraperitoneal injury which is less common than Extraperitoneal
23. A one-year child, his mother felt a mass in Rt. lumbar region, so what is the most common solid abdominal tumour in children?
A. Neuroblastoma (<2 years) B. Nephroblastoma C. Lymphoma D. Rhabdomyosarcoma E. Fibrosarcoma
24. A patient with sudden scrotal pain o/e the testis was High riding and tender erythema, what is the diagnosis? Testicular torsion
25. An infant with continuous crying, O/E scrotal swelling, firm, bluish in colour, not fluctuant, no groin or abdominal swelling
A. Congenital hydrocele B. Testicular teratoma C. Testicular torsion D. Strangulated inguinal hernia
26. A 10 year old boy came to the ER with right scrotal pain and swelling, on examination tender right testis, with decreased flow on
Doppler study, what is your diagnosis?
A. Hernia B. Haemocoele C. Testicular torsion D. Orchitis
27. What is the most associated with nephrotic syndrome?
A. Gastritis B. Peritonitis (SBP, without infection source, 75% gram-ve as E. coli-- K. pneumonia in 50% as GIT source) C. Glomerulonephritis
28. What is the pathology that have only single organism as a causative agent in case of no penetration or perforation of surrounding organs?
A. Pelvic inflammatory disease B. Perforated diverticulitis C. Acute cholecystitis D. Primary peritonitis; SBP(G-ve, Ecoli K pneum)
29. In transplant recipients, there is an increased incidence of which cancer type of the following?
A. Colon cancer B. Anal cancer C. Lung cancer D. Breast cancer E. Prostate cancer
30. Post-transplant lymphoproliferative disorders are related to which of the following?
A. Cytomegalovirus CMV B. Epstein-Barr virus (first posttransplant year due to intensive immunosuppressant) C. HIV infection
D. Herpes virus E. Hepatitis B virus
31. Regarding graft rejection, which of the following is correct?
A. Hyperacute rejection is antibody mediated (due to graft ischaemia, necrosis up to 48 hr posttransplant) B. Hyperacute reversed with steroids
C. Acute rejection is B cell mediated (T cell mediated, ABO incomp) D. Acute rejection occurs over month (this for chronic and acute days/ wks)
32. What is correct regarding Chronic allograft rejection?
A. Can be prevented with adequate immunosuppression B. It is the main cause of death after liver transplantation
C. It is more common with liver than heart transplantation D. It is more common with kidney than liver transplantation(HLA mismatch)
33. A complication of tacrolimus therapy is which of the following?
A. New-onset diabetes B. Upper gastrointestinal bleeding C. Thrombocytopenia D. Cardiac arrhythmia
Adverse effects include CVS: Angina pectoris, cardiac arrhythmias, hypertension. CNS: Abnormal dreams, headaches, insomnia, tremors & infection
34. What is the treatment of hyper acute kidney transplant rejection?
A. Administration of OKT-3 B. Administration of massive doses of steroids C. Immediate transplant nephrectomy
D. Observation and dialysis
35. What is the most common cause of death after kidney transplantation?
A. Operative technical complications B. Atherosclerotic complications C. Infection (BK virus,CMV, Epstein-Barr) D. Cancer
36. In therapeutic immunosuppression, Rapamycin; Sirolimus, what is the mode of action?
A. Blocks interleukin-I production B. Blocks interleukin-I action C. Blocks interleukin-2 production D. Blocks interleukin-2 action
In IL-2-stimulated T cells, Rapamycin impedes progression through the G1/S transition of the proliferation cycle, resulting in a mid-to-late G1 arrest.
37. Cyclosporin A inhibits the production of which of the following?
A. Interleukin I B. Interleukin 2 C. Interleukin 6 D. Tumour necrosis factor
The mechanism of action of cyclosporine is as a calcineurin inhibitor, a cytochrome P450 3A4 inhibitor, and a P-glycoprotein inhibitor. Cyclosporin A
(CsA) inhibits the synthesis of interleukins (IL), including IL-2, which is essential for the self-activation of T lymphocytes (LT) and their differentiation
38. What is the absolute contraindication to renal transplantation?
A. Chronic osteomyelitis B. Diabetes mellitus C. Lung cancer treated 10 years ago D. Exertional angina E. Age > 55yr
39. Neurogenic bladder after extensive pelvic surgery is characterised by:
A. Painful urine retention B. Large bladder capacity with overflow incontinence
C. Small residual volume and uninhibited bladder contractions D. Autonomic dysreflexia
40. Neurogenic bladder dysfunction with intact bladder sensation is associated with which of the following?
A. Nerve injury with abdominoperineal resection B. Cauda equina lesion C. Myelomeningocele D. Paraplegia
E. Cerebrovascular accident
41. A case query of Cauda equina lesion with urine incontinence and lower limbs paralysis what to order?
A. MRI spine B. MRI brain C. MRV and MRA brain
42. What is the main motor nerve supply to the urinary bladder?
A. The pelvic splanchnic nerves(from parasympathetic fibres) B. The hypogastric nerve C. The pudendal nerve D. presacral nerve
43. The prognosis of urinary bladder cancer based on?
A. Age B. Haematuria C. Invasion of wall D. Size of mass
44. A 64 year old male with history of painless gross haematuria with dysuria, urgency, frequency of urination bladder, associated with
pelvic pain, no weight loss, LL oedema or metastasis diagnosed as early urinary bladder cancer, what is the best modality of diagnosis?
A. Urine culture and sensitivity B. Pelvi abdominal US C. Urine cytology D. Cystoscopy and biopsy E. Renal function
Clinically painless gross haematuria - Approximately 80-90% of patients; classic presentation. Irritative bladder symptoms (eg, dysuria, urgency,
frequency of urination) - 20-30% of patients. Diagnosis either urine cytology or voided urine cytology, but it has low sensitivity for low-grade and early
stage cancers. Cystoscopy is the primary modality for the diagnosis of bladder carcinoma, it can permits biopsy and resection of papillary tumours.
45. A young patient has recurrent UTI, stones and electrolytes imbalance, on imaging, bilateral kidney enlargement with multiple variable
size thin walled cysts throughout parenchyma. What is the diagnosis?
A. Medullary sponge disease B. Polycystic kidney disease
46. What is the appropriate management of renal cell carcinoma extending into the inferior vena cava?
A. Radiotherapy B. Chemotherapy C. Chemoradiation D. Radical nephrectomy and caval tumour extraction
E. Radical nephrectomy, caval resection, and graft interposition
Renal cell carcinoma extends into the lumen of the inferior vena cava in approximately 4% of patients at the time of diagnosis. Surgical removal of the
intracaval tumour thrombus with radical nephrectomy is the preferred treatment for this malignancy.
47. An old age male patient with history of prostatectomy for prostatic cancer 10 yr. ago, now complains of abd. pain and bleeding per
rectum, colonoscopy showed Colonic cancer, other work up show liver metastasis what about liver metastasis?
A. Synchronous with colon cancer B. Metachronous with prostate cancer C. CEA is elevated with Mets with prostate
48. An old, aged patient with sigmoid colon cancer with multiple metastases in Right liver lobe, what is the resection strategy?
A. 1ry only B. Secondaries only C. Synchronous (as diagnosis) D. Staged 1ry then 2ry
49. Prostatic cancer metastasises to liver after 6-month Metachronous
50. History of spine trauma at T6, paraplegia, what is the cause of urine retention?
A. Detrusor instability B. Detrusor paralysis C. Detrusor dyssynergism
51. An elderly referred to the urology clinic due to painless haematuria, medically free and vitally stable with unremarkable history,
which of the following is the most appropriate for diagnosing his case?
A. Cystoscopy (Bladder cancer?) B. Abdominopelvic US C. Abdomen CT D. Intravenous pyelogram
52. A 55-year-old man presents with fever and pain in the perineal region. Upon further questioning he also complains of frequency,
urgency, dysuria, and a decreased urinary stream. On physical examination his abdomen is soft, nondistended, and nontender. Digital
rectal examination demonstrates sever tenderness on the anterior aspect. Laboratory examination reveals leukocytosis and findings on
urinalysis are consistent with a bacterial infection. Which of the following is the most likely diagnosis?
A. Urinary tract infection B. Benign prostatic hyperplasia C. Prostatitis D. Pyelonephritis
53. A 62 year old male with Benign Prostatic Hyperplasia (BPH), His BMI: 41, what is the risk of BPH in this case?
A. Age B. Obesity C. Smoking
54. An old age male patient with symptoms of urine drippling and back pain, labs showed high PSA, what is the diagnosis?
A. Prostatic cancer B. BPH
55. A patient has prostatic cancer, colonic cancer, he developed liver mets, what is the best prognostic marker to search for follow up?
A. CEA B. APC C. PSA D. CA19-9 E. Alpha fetoprotein
56. A 65 year male comes to the clinic with a mild intermittent urinary flow reduction PR examination, urinalysis and prostate specific
antigen studies are normal. Ultrasound prostate: Enlarged median lobe. Which of the following is the best way to investigate?
A. Annual renal function monitoring B. Periodic PSA measurement C. Beta-blocker therapy D. Cystoscopy
57. What is the most useful serum marker for Prostatic cancer screening?
A. Prostate specific antigen; PSA B. CA 19.9 C. Alpha-fetoprotein D. Carcinoembryonic antigen
58. What is the common primary cancer causing metastatic osteosclerotic bone lesions seen in plain X-ray?
A. Prostate B. UB C. Lung D. Liver
N.B. 1. Osteosclerotics mostly prostate, breast. Osteolytic tumours > sclerotic, as; renal, thyroid cancers and lung (typically lytic but in 15% mixed)
Mixed tumours Carcinoma of cervix, testicular cancers and ganglioneuroblastoma in paediatric patients
59. A patient was planned for TURP, came comatosed, with full bladder, what is next at time of operation?
A. Foley’s catheter B. Suprapubic catheter C. Indwelling catheter
60. A post-operative hypotension + drain is empty, suprapubic fullness and no urine output, management?
A. Diuretics B. Catheter flush C. Dopamine D. IV fluids
61. What is the most common malignancy at age of 30 years?
A. Osteosarcoma (10-14 and >60) B. Wilm’s tumour (3 to 4 rarely after 5) C. Seminoma (15-34)
62. What is the marker that would distinguish non-seminoma from seminoma?
A. Lactate dehydrogenase B. Alkaline phosphatases C. Human chorionic gonadotropin D. Alpha fetoprotein (50-75% in non seminomatous)
63. What is the most common presentation of testicular cancer?
A. Painless scrotal mass B. Acute testicular pain C. A secondary hydrocele D. Gynecomastia (5% in germ cell tumours)
E. Retroperitoneal lymphadenopathy
Primary testicular tumours are the most common solid malignant tumor in men 20 to 35 years of age in the United States. Germ cell cancers account for
more than 90% of all testicular cancers. In patients with localised disease, painless swelling or a nodule in one testicle is the most common presenting sign.
A dull ache or heavy sensation in the lower abdomen could be the presenting symptom.
64. A 25-year-old male with a painless scrotal mass, what is the initial step in management?
A. Fine-needle aspiration and cytology B. Abdominal computed tomography scan D. Scrotal ultrasound
C. Observation and repeat examination in 2 weeks E. Orchiectomy
65. A 32-year-old man presents with an asymptomatic mass in his right testicle, on examination, the mass cannot be trans illuminated
ultrasound showed a solid mass in the right testicle. Which of the following is the most accurate method in obtaining a diagnosis of
testicular cancer?
A. Serum levels alpha fetoprotein and beta human chorionic gonadotrophin. B. Radical inguinal orchiectomy
C. Percutaneous biopsy of the testicular mass D. Incisional biopsy of the testicular mass through a scrotal incision
Radical inguinal orchiectomy with high ligation of the spermatic cord at the level of the internal ring is the procedure of choice for testicular cancer.
66. A patient underwent a right inguinal hernia repair one month ago, now presenting with a shrunken right testis, what is the cause of his
presentation?
A. Mesh prolapsed into the scrotum B. Thrombosis of the pampiniform plexus C. Tight external ring of the inguinal canal
D. Testicular artery ligation (Atrophy without ischaemic orchitis and atrophy is rare)
Ischaemic orchitis is mostly due to thrombosis of pampiniform plexus of veins within the spermatic cord after hernia repair that can lead to testicular
atrophy. The ischaemia and necrosis occur after thrombosis of pampiniform plexus but not with arterial injury or intended ligation where atrophy occurs
without ischaemia or necrosis due to rich collaterals from inferior epigastric, vesical, prostatic, and scrotal arteries.
67. Pelvic Fracture with injury to membranous urethra, blood seen in external meatus. What’s the most appropriate action?
A. Cystoscopy B. Folly’s Catheter C. Suprapubic catheter D. Pelvic CT
Note: Suprapubic catheter: in case of urethral injury (blood from meatus). Urgent exploration repair: in case of Intraperitoneal injury
Extraperitoneal injury is more common than Intraperitoneal injury
68. Victim of RTA came with pelvic fracture and there’s blood in the meatus, next step?
A. Foley catheter B. Retrograde urethrogram C. Ureteroscopic urethrogram
69. A victim in RTA who did Ureteroscopic urethrogram and found injury in the bulbar urethra, what do you want to do next?
A. Suprapubic catheter B. Foley catheter
Posterior urethral injuries are in the membranous and prostatic urethra. These injuries are most commonly related to major blunt trauma such as motor
vehicle collisions and major falls, and most of such cases are accompanied by pelvic fractures. Injuries to the anterior urethra are located distal to the
membranous urethra. Most anterior urethral injuries are caused by blunt trauma to the perineum (straddle injuries) mostly for suprapubic catheter.
70. A 17-year-old male presented with penile urethral injury. the urine extravasated to?
A. Anterior abdominal wall B. Femoral triangle C. Femoral canal D. Perineal region
71. What is the most effective method of treating hyperkalemia?
A. Intravenous calcium gluconate B. Intravenous sodium bicarbonate C. Haemodialysis
D. Cation-exchange resin E. Intravenous glucose-insulin
72. A patient needs haemodialysis as soon as possible, what you’ll choose?
A. Haemodialysis with graft B. Haemodialysis with fistula C. Central venous catheter (Permacath) can be used in ESRD
Gynaecologic and genitourinary diseases
1. A 17 year old female medically free, she developed breasts later and never menstruated, on examination she is tanner stage 5 but no
menstruation, what is the diagnosis?
A. Hypothalamic hypogonadism B. Imperforate hymen C. Gonadal agenesis D. Testicular feminisation
Tanner staging is utilised in paediatric and adolescent deal with Sexual Maturity Rating; pubic hair in both, breast in female and male ext. genitalia
each one is five degrees that can classify both regarding full maturity or not
2. A healthy female with regular cycle, never had sex, history of bilateral breast pain 3 days before her cycle, no family history of cancers.
She said she wants to get pregnant within 2 years, what will you screen her for?
A. Breast US B. Pap Smear C. Human papilloma virus D. Gram stain for streptococcus
3. An 18 year old unmarried girl asked about when she can do screening for PAP smear what to tell her?
A. At 21-year B. When sexually active C. Come in one year
4. Woman complains of pain before menses and resolves in 3rd day, how do you diagnose by pelvic examination and clinical symptoms
5. Woman with urine incontinence while coughing and sneezing, no other symptoms what is the cause?
A. Enterocele B. Rectocele C. Paravaginal defect D. Hypermobile urethra
6. Woman in delivery bleeding didn’t stop, she wants to conceive in the future which structure you should ligate?
A. Uterosacral ligament B. External iliac artery C. Internal iliac artery D. Something vein
7. A 34 year old lady G4, P2, C/O of abd. pain, nausea, vomiting, during her 1st trimester of pregnancy, HCG > 100, pelvic U/S shows
snowstorm, which of the following is true: Risk of malignancy 20%
8. A female patient while doing C/S for her, she developed massive uterine bleeding, where is main source of bleeding?
A. IMA B. Aorta (give internal iliac>>uterine a.) C. Splenic artery D. Perforated peptic ulcer
9. Newly married female went to check up, what to check?
A. General appearance B. Pelvic examination C. Abdominal examination
10. A newly married lady with regular menstrual cycle after 3 weeks of last period got morning gastric upset, nausea, belching and retching
with frequently going to bathroom for urination, no UTI or previous medical diseases, what the best to do for her?
A. CECT B. Pelvic US (after 6 wks) C. Urine test D. Pregnancy test (hCG can be detected 8 days after ovulation, +ve preg. before next cycle)
11. What is the gold standard for the diagnosis of pelvic inflammatory disease?
A. Vaginal microbiology swab B. Pelvic US C. Laparoscopy D. Pelvic CT E. Endometrial biopsy

12. A female patient after treatment of breast cancer, she wants to get pregnant she can get pregnant after how long?
A. One-month B. Three months C. 9 months D. 1 year
13. A patient post hernia repair, she is one year married without kids, when can she get pregnant?
A. 3 months B. 6 months C. 9 months D. 12 months (1y. post H repair & last chemoth dose, 2 yrs after last CRTH dose)
14. A 70 yrs. old woman came for evaluation of unusual bleeding for the past 2 weeks. Endometrial biopsy from the body of the uterus
shows endometrial adenocarcinoma, which of the following is the most likely commonest site of metastasis due to endometrial carcinoma?
A. Bone B. Brain C. Small bowel D. Vagina and Cervix
Typical sites of metastasis of endometrial carcinoma are pelvic and para-aortic lymph nodes, vagina, peritoneum, and lungs.
Body of the uterus >> External iliac NB: Middle third of the vagina & cervix > internal iliac. Ovaries >para aortic.
15. Ovarian cyst surgery, mild wound pain and discharge, fever 38, WBCs 12, what is suitable analgesic?
A. Epidural B. NSAID C. IV analgesic D. Oral paracetamol
16. A 24 y.o lady, after normal delivery with episiotomy, she developed retroperitoneal haematoma, was bluish painful coloured skin, what
is the plan of management?
A. Packing B. Aspiration C. Surgical evacuation D. Blood transfusion if HB < 10gm/dl
Haemodynamically stable; conservative approach is recommended after receipt of a blood transfusion, and in whom the haematoma is not expanding, and
symptoms of pressure are absent.
Haemodynamically unstable without extensive haematoma-->>> vascular embolisation by interventional radiologist have good outcomes.
Finally, laparotomy is indicated as a last resort in clinically unstable patients with extensive haematoma, due to risk of haemorrhagic shock, DIC& sepsis
17. A newly married female 28 yrs. old presented with acute abdominal pain, vaginal bleeding, and hypotension 80/50, P 112, TLC 8.5, Hb
9.4, last menstrual period was 1.5-month, what is most probably diagnosis?
A. Ruptured ectopic pregnancy B. Acute appendicitis C. PID D. Ruptured ovarian cyst D. Bleeding PUD
18. A 25 yrs. female localised pain at Rt. side US not conclusive, total leucocytic count is only 6, what is possible diagnosis?
A. Acute appendicitis B. Ruptured ectopic pregnancy C. Rupture ovarian cyst
19. A 58-year-old female with a tuboovarian abscess should be treated with which of the following?
A. Antibiotics and laparoscopic drainage B. Antibiotics with hysterectomy and bilateral salpingo-oophorectomy
C. Antibiotics and observation D. Antibiotics and unilateral salpingo oophorectomy
E. Antibiotics and computed tomography guided drainage
20. A 30 - years old female presented as hypotensive with severe abdominal pain and last period 2 months ago what to do?
A. Methotrexate B. Immediate surgery (tubal ectopic pregnancy) C. Refer to gynaecologist
21. A female patient presented with continuous vomiting of 3 days and then developed abdominal pain, diagnosis? Ovarian torsion
22. A female patient presented with nausea and vomiting with suprapubic and left iliac fossa pain and tenderness, Pregnancy test was
negative, urine dipstick normal, Normal WBC counts, what is initial management?
A. Abdominal X-Ray B. Abdominal CT C. Diagnostic laparoscopy D. Analgesia & Discharge
The best modality to diagnose adnexal torsion is by US, and as pain is constant and tenderness with nausea and vomiting even if no fever or leucocytosis
must do emergency exploratory laparoscopy as the definitive management even if it was not confirmed by US
23. A 36-year-old lady, smoker with family history of type 2 DM, what is the highest risk on infertility?
A. Age B. Smoking C. Family history
Age is the single biggest factor affecting a woman’s chance to conceive and have a healthy baby. A woman’s fertility starts to reduce in her early 30s, and
more so after the age of 35. The risks of pregnancy complications increase as women age.
24. The intrauterine foetal death during pregnancy is most commonly caused by which of the following?
A. Abruptio placentae B. Subarachnoid haemorrhage C. Penetrating foetal injury D. Maternal demise
25. Oestrogen therapy for postmenopausal women is associated with which of the following?
A. Increased incidence of thrombophlebitis B. Increased incidence of hepatic adenoma
C. Increased incidence of endometrial cancer D. Increased incidence of breast cancer
26. A 90-yr. old female, HTN and DM, admitted to the hospital as a case of HF and was treated with medication, there was incidental
finding of uterine procidentia, when taking history from the patient, she said it has been for 10 years and never bothered her, she used to
push her uterus back when urinate and pass stool, what is your management?
A. Vaginal pessary B. Vaginal hysterectomy C. Radical hysterectomy D. Lap hysterectomy with Sacrospinous suspension
27. An old lady follows up, incidentally found grade 3 uterine prolapse and she has no complaint what to do?
A. Surgical repair B. Ring (Vaginal pessary support) C. No need for intervention
28. An old lady with stress urinary incontinence what is optimal management plan for her?
A. Vaginal pessary B. Urethral ring No kugel exercise
1) Behavioural Pelvic muscle exercises such as Kegel exercises, bladder retraining, Biofeedback methods, Electrostimulation as acupunctures and if not
succeeded insert vaginal ring & Gellhorn Pessaries. 2) Drugs as anticholinergic as Oxybutynin, Tricyclic anti-depressants with alpha adrenergic effects
Perioperative care
1. The adverse effects of steroids on wound healing delay, can be reversed with which of the following?
A. Vitamin C B. Vitamin A C. Copper D. Vitamin D E. Vitamin E
2. Malnourished patient, what element should be normal if patient is for surgery?
A. Vit C B. Copper C. Folic acid D. Calcium
3. What is the amino acid important for nutrition of mucosal cells?
A. Arginine B. Glutamine C. Valine D. Citrulline E. Arginine
4. What is the cause of eczematoid rash in intertriginous areas with prolonged total parenteral nutrition?
A. Zinc deficiency (acrodermatitis enteropathica) B. Niacin deficiency C. Copper deficiency D. Magnesium deficiency
Zinc deficiency in a patient on long-term TPN, manifests as acrodermatitis enteropathica with symmetric, intertriginous dermatitis, specifically, around
the perineum, along with treatment-resistant seborrhoeic dermatitis, and diarrhoea.
5. Alopecia with prolonged total parenteral nutrition may be caused by which of the following?
A. Zinc deficiency B. Magnesium deficiency C. Vitamin A intoxication D. Essential fatty acids deficiency
6. What is the cause of dry, scaly, pruritic rash on the trunk and extremities of a patient receiving total parenteral nutrition?
A. Zinc deficiency B. Vitamin A deficiency C. Vitamin C deficiency D. Free fatty acid deficiency
7. What is the substrate for nitric oxide synthase?
A. Glutamine B. Alanine C. L-arginine D. Valine E. L-Citrulline
8. During prolonged starvation, what is the brain's main fuel?
A. Glucose B. Amino acids C. Ketones D. Short-chain fatty acids
9. What is the optimal infusion rate giving the maximum efficiency of glucose use in total parenteral nutrition?
A. 4mg/kg/min B. 5mg/kg/min C. 6mg/kg/min D. 7mg/kg/min (this in TPN but as IVF 0.5 g/kg/h not exceeding 0.8g/kg/h)
Glucose utilisation maximum rate in TPN is 5 to 7 mg/kg/min. Maximal glucose infusion rate in TPN; should not exceed 1.2 g/kg per hour, (20 mg/kg per
min. Glucose IV infusions may be administered intravenously to healthy individuals at a rate of 0.5g/kg per hour without producing glycosuria; the
maximum infusion rate should not exceed 0.8g/kg per hour
10. In the absence of sepsis, glucose intolerance with total parenteral nutrition may indicate which of the following?
A. Copper deficiency B. Zinc deficiency C. Magnesium deficiency D. Chromium deficiency
11. A medically free patient, need inguinal hernioplasty next week? He is normal healthy patient, so he is grade ASA 1
A. ASA I (normal healthy patient, not smoker or minimal alcohol) B. ASA II C. ASA III D. ASA IV
12. A patient with controlled diabetes mellitus, what’s his ASA grade?
A. ASA II (well controlled mild disease as DM/HTN, pregnancy, currently smoker, mild lung disease, social alcoholic, BMI 30 - <40)
B. ASA IV (Severe systemic disease with constant threat to life as recent (<3 months) MI, CVA, TIA or CAD/stents, severe valve dis., shock, sepsis)
C. ASA I D. ASA III E. ASA VI (Declared brain-dead patient whose organs removed for donation purposes)
13. A 30-year-old man with no past medical history undergoes emergency hernia repair. He has a 10-packs per-year smoking history,
what is his American Society of Anaesthesiologists? (ASA) physical status classification?
A. ASA class I B. ASA class II C. ASA class III
D. ASA class V (moribund, not expected to survive without surgery as ruptured abdominal/thoracic aneurysm, massive trauma, intracranial hge e mass)
14. Hypertensive patient on ACE inhibitor & B-blocker, also he is diabetic on oral hypoglycemic drugs, he changed it to insulin, ASA?
A. ASA I B. ASA II C. ASA IV
D. ASA III (severe systemic disease, poorly controlled DM/ HTN, COPD, BMI ≥40, (>3 ms) MI, CVA, TIA or CAD/stents implanted pacemaker)
15. Most risky preoperative risk factor?
A. Aortic stenosis B. Bronchitis
16. A patient with BP 170/90, T3, T4, TSH are normal, Hb is normal, platelet is 100000, WBC is normal, to control BP should send the
patient to which of the following specialty physician?
A. Cardiology B. Pulmonology C. Endocrinology D. Haematology
17. After abdominal surgery there is surgical site infection, how to prevent and decrease the risk of this infection?
A. Antiseptic before operation B. Shaving at night before surgery C. Antibiotic within 12 hrs. D. Prophylactic LMWH
E. Prevent perioperative hypothermia
18. Which of the following is not a risk factor for wound infection?
A. Prolonged operative time B. Prolonged preoperative hospitalisation C. Shaving the skin, the night before surgery
D. Patient's having upper respiratory tract infection E. Surgeon's hand scrub for 5 instead of 10 minutes
19. Which of the following is correct regarding sterilisation?
A. Chemical sterilisation is the commonest type B. Scrub up team must keep hands above waist
C. Disinfection is enough for surgical instruments D. Suspicion of organism’s spread during operation is not harmful
20. How to avoid MRSA infection from an infected inpatient case?
A. Isolation B. Vancomycin C. Nasal swab N.B he asked about avoidance or protection not treatment
21. Postoperative cardiac events are most likely if preoperative electrocardiogram showed what?
A. ST-T wave changes B. Bundle branch block C. Left ventricular hypertrophy D. Q-wave (indicate current or prior MI)
22. The effectiveness of prophylactic antibiotics in surgery is mostly related to which of the following?
A. Use of broad-spectrum agents B. Continuation of antibiotics for 24 hours after surgery C. Timing of initial administration
D. Use of two synergistic agents E. Use of bactericidal agents
23. Wound infection in uncontrolled DM, what to do to decrease wound infection?
A. Control high blood sugar post op B. Shaving C. Antibiotics within 30 - 60 min. before skin incision D. Increase hypothermia
24. A patient presented by femoral fracture will be managed under spinal anaesthesia, what is the contraindication of neuraxial anaesthesia?
A. Sarcoidosis B. Malignant hyperthermia C. Anticoagulation
25. A postoperative patient in HDU, developed acute atrial fibrillation how to manage?
A. Digoxin B. Blockers C. Calcium channel blocker D. Cardioversion
26. Which of the following is an indication of nutritional support?
A. Anorexia nervosa B. Intestinal fistula C. Malignancy D. All of the above (impaired GIT funct, inadequate intake hypercatabolic states)
27. What is the most accurate measure of adequacy of nutritional support?
A. Serum albumin level B. Body weight C. Triceps skin fold measurement D. Serum pre albumin level
28. What is the most reliable means of preoperative nutritional assessment?
A. Clinical history of weight loss B. Serum albumin level C. Impaired cell-mediated immunity
D. Triceps skin fold measurement
29. Which of the following is known contraindication of Succinylcholine use as depolarising muscle relaxant for rapid sequence induction?
A. Patient with hepatitis B. Parotidectomy C. Thyroidectomy
D. Celiotomy (laparotomy) 3 days to 9 months after spinal cord injury >> hyperkalaemia >>> life-threatening cardiac arrhythmias
30. Appropriate treatment of malignant hypothermia is intravenous administration of which of the following?
A. Morphine sulfates B. Dantrolene C. Benzodiazepines D. KCI E. Calcium gluconate
Succinylcholine can cause malignant hyperthermia if occurred give Dantrolene IV is the treatment of choice (2.5 mg/kg every 5 minutes) until symptoms
subside). Malignant hyperpyrexia is life threatening clinical syndrome can lead to increased CO2 production, metabolic and respiratory acidosis, accelerated
oxygen consumption, heat production, hyperkalaemia, DIC, multiple organ dysfunction and failure.
31. Which of the following is correct for Fentanyl?
A. It is normally found in the body B. 100 times more potent than morphine
C. It has twice as long a duration of action as morphine D. It results in hypotension because of histamine release
32. What is the preferred neuromuscular-blocking agent in a liver failure patient?
A. Vecuronium B. Atracurium C. Pancuronium D. Pipecuronium
33. Tachycardia is the main side effect of which of the following?
A. Fentanyl (confusion, hypotension, hypoventilation) B. Succinylcholine (hyperkalaemia, cardiac arrest & Rhabdomyolysis)
C. Morphine (constipation CNS & respiratory depression) D. Pancuronium (hypotension, tachycardia & bronchospasm)
34. Which of the following is a powerful vasoconstrictor?
A. Procaine B. Bupivacaine C. Lidocaine D. Cocaine
35. What is the most common complications that enteral and parenteral feeding can cause?
A. Increased incidence of sepsis B. Intestinal villous atrophy C. Elevated liver transaminases
D. Hyperosmolar non-ketotic coma E. Diarrhoea
36. What is the optimum calorie/nitrogen ratio for protein synthesis?
A. 25-50: 1 B. 50-75: 1 C. 75-100 D. 100-150: 1
37. What is the most common cause of death related to a central venous catheter insertion?
A. Air embolism B. Central vein perforation; mostly internal jugular vein C. Tension pneumothorax
D. Catheter embolism E. Catheter-related sepsis
38. Rightward shift of Oxyhaemoglobin dissociation curve occurs with which of the following?
A. Hypothermia B. Decrease in pH (acidosis) C. Hypocapnoia D. Methaemoglobinaemia
The causes of shift to right can be remembered using the mnemonic, "CADET, face Right! for CO2, Acid, 2,3-DPG, Exercise and Temperature (high).
Hypocapnoia, hypothermia, low 2,3-DPG, HbF, methaemoglobin, carboxyhaemoglobin all of them shift to left of the oxyhaemoglobin dissociation curve.
39. Decreased haemoglobin affinity to oxygen at the tissue level is caused by which of the following? Shift to right
A. Increased body temperature (fever) B. Decreased 2,3-diphosphoglycerate C. Decreased pCO2 D. Increased pH
40. What is the earliest and most specific sign of malignant hyperthermia?
A. High fever B. Hypotension C. Increases in end tidal CO2 D. Tachycardia E. Hypoxia
41. What is true regarding malignant hyperthermia?
A. It is triggered by the stress of surgery or anesthesia only B. It is a contraindication to future general anaesthesia
C. It can be prevented by perioperative calcium channel blockers D. It is associated with intraoperative rise of end tidal CO2
42. What is correct regarding malignant hyperthermia?
A. It is rare in children B. It is an autosomal dominant disorder C. It results in respiratory alkalosis
D. It is prevented by intravenous calcium gluconate
43. Which of the following amino ester?
A. Lidocaine B. Tétracaine (Pontocaine HCL) C. Bupivacaine D. Mepivacaine
44. What is correct regarding malignant hyperthermia?
A. Can be induced by local anaesthetics B. Can be induced by non depolarising muscle relaxants
C. Can be induced by nitrous oxide D. Is related to disordered K+ metabolism E. Is more common in children than adults
45. What is the local anaesthetic that can be safely administered with tetracaine allergy or toxicity?
A. Lidocaine B. Cocaine C. Procaine D. Chloroprocaine
Early symptoms are circumoral numbness, tongue paresthesia, and dizziness. Sensory complaints may include tinnitus and blurred vision. Excitatory signs,
such as restlessness, agitation, nervousness, or paranoia, may progress to muscle twitches and seizures.
46. Which of the following is an early feature of Lidocaine toxicity?
A. Arrhythmia B. Muscle twitching C. Respiratory depression D. Hypotension
47. A man weighing 125 kg, known to have COPD, with history of snoring and sleep apnoea, planned for operation, under GA, what not
to give post-operatively?
A. NSAIDs B. Sedatives (may cause respiratory depression due to airway obstruction and ventilatory arrest due to relaxant action)
48. A patient before shifting to theater was anxious and irritable, what is the best to give him as a premedication?
A. Opioid B. NSAIDs C. Tramadol D. Benzodiazepines
49. Inadvertent tissue extravasation of intravenous dopamine is best managed with which of the following?
A. Topical steroids and elevation B. Local ice packs and elevation C. Local infiltration of nitroglycerin
D. Local infiltration of phentolamine E. Local infiltration of 1% lidocaine
50. Which of the following is a dopamine antagonist?
A. Phentolamine B. Propofol C. Haloperidol (1st. gen. antipsychotic, block D2 receptors after 72% blocked--> max effect ) D. Clonidine
51. Which of the following is an analgesic?
A. Sodium thiopental B. Ketamine C. Etomidate D. Propofol
52. What is the contraindication to the use of ketamine?
A. Hypotension B. Head injury C. Asthma D. Hypoventilation
53. In wound healing what is correct?
A. Remodeling phase may last up to 2 years B. Prolonged continuous pressure decreases healing time
C. Foetal wounds heal with primary intension D. Type I collagen is present mainly in healing wounds
54. In inflammation, early "rolling" of neutrophils on endothelium is a function of which of the following?
A. Selectin B. N-Cadherins C. Immunoglobulin superfamily D. Integrins E. Complement activation
The selectins are physiologically important in inflammation, lymphocyte homing, immunological responses, and homing of bone marrow stem cells. They
play a role in atherosclerosis, ischaemia-reperfusion injury, inflammatory diseases, and metastatic spreading of some cancers.
55. A male patient, with history of COPD underwent anterior resection, epidural catheter was inserted for pain management, 8 hrs. post-
operative developed fever, what do you think, the cause of fever?
A. Atelectasis B. Body reaction to surgery C. Pneumonia D. Epidural abscess
56. A postoperative patient complaining about 1-week intermittent fever on examination abdomen, chest and investigation are normal and
the wound is clean. What's the most appropriate step?
A. Reassure (may be drug allergy as penecill/cephalosporin after 7-10 d) B. Abdominal CT (if chest, abdomen or investigatory abnormalities)
D. Exploration
57. An old aged patient with caecal cancer, hemicolectomy done, and after 24h post operatively, presented with fever what’s cause of fever?
A. Atelectasis B. Leak C. UTI
58. A 37 year old lady morbidly obese, she underwent a Sleeve gastrectomy, after 24h, she complained of dyspnoea with decreased breath
sounds on left chest side what is the best to do for her?
A. Chest x ray (to R/O atelectasis) B. Bag to blow it or Spirometer C. Chest Physiotherapy
59. A post gastrectomy patient day 3 left hypochondrium pain, fever 39.5, auscultation of Lt. basal lung > crepitations, with decreased air
entry what is cause of fever?
A. Pneumonia B. Atelectasis C. Left subphrenic collection D. Catheter related infection
60. On third postoperative day after abdominal surgery, fever, decreased air entry on the left lower lung base, diagnosis?
A. Pneumonia (2nd. - 3rd. day) + haematoma or tissue necrosis) B. Atelectasis (day 1 post op.) C. Sub phrenic abscess
61. A patient presented on the 7th day post op. with chest pain with decrease air entry on lower part of left chest wall, T 39, diagnosis?
A. Atelectasis B. Pneumonia C. DVT, PE (6th.,7th. days PE & DVT) D. UTI (3-5 days) bacterial cystitis or catheter related)
Dyspnoea, chest pain and cough are the most frequents symptoms of pulmonary embolism, while fever, tackycardia, abnormal pulmonary signs and
peripheral vascular collapse are the most common physical findings. Cyanosis, haemoptysis are less commonly observed. The lower lobes are more
frequently affected than the upper lobes with bilateral involvement being common, hence the patient may has bilateral lower chest pain.
62. On POD l, a patient develops a temperature of 104 0 F (40 C) and foul-smelling wound drainage, what is the most likely isolate?
A. Gram-negative rod B. Gram-positive rods C. Gram-negative cocci D. Gram-positive coccus
63. In wound healing the maximum amount of collagen occurs at?
A. 10 days B. 21 days C. 6 months D. Less than 1 week
The inflammatory phase occurs immediately following the injury and lasts approximately 6 days. The fibroblastic phase occurs after inflammatory phase
and can last up to 4 weeks. The collagen deposition in normal wound healing reaches a peak by the third week after the wound is created. Scar maturation
begins at the fourth week and can last for years. Collagen strength will reach approximately 20% of its tensile strength after three weeks, increasing to
80% by the 12th week. The maximum scar strength is 80% of that of unwounded skin.
64. RTA post laparotomy dark brownish urine, management?
A. Urine analysis B. Na bicarbonate C. IV fluids (hypovolaemia-->> concentrated urine)
It mainly due to dehydration so, must give plenty of IV fluids till urine colour change to Ambar yellow/ clear colour
65. What is the best resuscitation fluid for trauma patient?
A. Blood B. Isotonic fluid C. Hartman’s fluid (Ringers’ lactate, crystalloids) D. Hypotonic fluid
66. What is the correct statement regarding the three phases of wound healing?
A. Inflammation, epithelisation, contracture B. Inflammation, fibroplasia, contracture C. Fibroplasia, epithelisation, contracture
67. Which of the following is an inhibitor of wound contraction?
A. Glucocorticoids B. Penicillamine D (immunosuppressant) C. Colchicine D. Aspirin
68. What epidermal growth factor stimulates?
A. Angiogenesis B. Wound contraction C. Fibroblast proliferation D. Epithelialisation
69. Which of the following is more important in wound healing?
A. Antibiotic B. Wound debridement C. Good hydration
70. Healing of the donor site for a split thickness skin graft is accelerated by which of the following?
A. Transforming growth factor B. Recombinant human growth hormone C. Epidermal growth factor D. Platelet-derived growth factor
71. What is the advantage of full thickness over split thickness skin graft?
A. Less wound contraction B. Better take C. More resistance to infection D. Better sensory function
72. Keloid formation has been associated with an increased amount of which of the following?
A. Transforming growth factor B. Platelet-derived growth factor C. Epidermal growth factor D. Tumour necrosis factor
73. In contrast to a keloid, what is true for a hypertrophic scar?
A. Is more likely to be familial B. May subsides spontaneously
C. May develop in delayed fashion years after initial injury D. Often extends beyond the limits of the original wound
74. What will occur if delayed primary wound closure happened?
A. Result in increased angiogenesis B. Results in decreased wound strength C. Results in lower collagen content
D. Results in a wider scar
75. Apoptosis is defined as which of the following?
A. Is an energy-dependent cell death B. Results in cell swelling C. Is associated with an inflammatory response
D. Is usually toxin induced E. Is indistinguishable from necrosis
76. Collagen synthesis in the actively healing wound is best assessed by which of the following?
A. Glutamine content B. Arginine content C. Hydroxyproline content D. Alanine content
77. What is the characteristic for Foetal wound healing?
A. Increased angiogenesis B. Increased hyaluronic acid synthesis C. Increased inflammatory response D. Decreased collagen
78. In contrast to adults, what is true for foetal wound healing?
A. Has a higher content of type III collagen B. Has a higher level of transforming growth factor
C. Has an exaggerated inflammatory phase D. Has much less hyaluronic acid content
Both foetal and adult granulation tissues have elevated type III collagen content, but normal fetal tissue has a much higher content of type III than does
normal adult tissue.
79. In wound healing which is important to harden the healing?
A. Tensile B. Elastin C. Collagen
80. In the light of melanocytes role in wound healing with maximum amount of collagen occurs for wound to heal well what is true?
A. Melanocytes inhibit fibroblast proliferation B. Collgen deposition maximally up to 3 months C. Tensile strength up to 50% by 4 months
Melanocytes stimulate the growth and proliferation of fibroblasts, increase collagen synthesis and extracellular matrix deposition, activate the TGF-β
signaling pathway. Peak collagen deposition rates are found 7 to 14 days after injury. Collagen may take around 12 weeks to show its effects on skin. Scar
maturation begins at the fourth week and can last for years. Maximal tensile strength of the wound is achieved by the 12th week, and the ultimate resultant
scar has only 80% of the tensile strength of the original skin that it has replaced
81. Which of the following cell types is essential for wound healing?
A. Neutrophil B. Macrophage C. Fibroblast D. Lymphocyte E. Endothelial
Macrophages enter the wound and participate in the phagocytic process. In addition, macrophages release growth factors and cytokines that help bring in
the proliferative phase of healing
82. Among the following which one is the main fuel for most cancer cells?
A. Butyrate B. Glutamine (primary fuel to live) C. Glucose (main fuel 10 times than normal cells) D. Ketones
83. Which of the following is true for Glutamine?
A. It is supplied in total parenteral nutrition B. Increases intestinal cellularity C. It is an essential amino acid
D. It is a substrate for gluconeogenesis
84. Which of the following provides the main energy source during critical illness/ acute injury?
A. Skeletal muscle B. Liver C. Adipose tissue / Fat D. Kidney E. Glycogen D. Amino acids and protein
85. How much the daily needed protein for adults? (0.8 g/kg/day this is the minimum) better up to 1.2 gm/kg/day for active; higher: up to 2.4 g,
86. Optimal daily protein intake for infants and children in grams per kilogram of body weight? infant 3g./kg/d and children > 1.5g/kg/d
87. 41 y.o lady undergoes complex repair of a deep laceration in her hand, when removing the dressing on postoperative day 2, a large clot
with mild surrounding erythema is found, which of the following regarding the inflammatory phase of wound healing is correct?
A. The complement component C5a and platelet factor attract neutrophils to the wound
B. The presence of neutrophils in the wound is essential for normal healing C. It lasts up to 24 hrs., after injury
88. Potassium level more in or what is the highest body fluid with K?
A. Saliva B. Bile C. Pancreatic juice D. Intestine
89. What the colon secretes?
A. Water B. Sodium C. Chloride D. Potassium
90. A patient post RTA with massive bleeding in the nose and mouth with leg fracture, decreased BP, patient can take his breath well, no
airway problem, but he is afraid of what happened, what is the next step?
A. Intubate with stretching neck B. Packing nasal bleeding C. IV fluid (just elevate his BP and reassure him) D. Blood transfusion
91. What do baroreceptors affect?
A. Affect COP B. Increase vascular resistance
In hypotension; decreases in mean arterial pressure, resulting in decreased nerve firing and reduced stimulation of the nucleus tractus solitarius, attenuating
inhibition and increasing sympathetic outflow to peripheral vasculature and vasoconstriction >>> restore the BP again to normal & increases cardiac output
92. Stimulation of baroreceptors in the aortic arch leads to?
A. Decrease heart rate B. Central sympathetic stimulation C. Vasoconstriction of skeletal blood vessels
Impulses originating in the aortic arch travel along >> afferent fibers of the vagus nerve to synapse at the nucleus tractus solitarius; NTS. The NTS
tonically provides sympathetic outflow to peripheral vasculature, decreased peripheral vascular resistance and increases parasympathetic activity to the
heart, >>> bradycardia >> restores blood pressure back to its original level.
93. Regarding the carotid body, what is correct?
A. is a pressure receptor B. is an osmoreceptor C. is a subintimal structure D. It is found in the adventitia E. In the media
The carotid body is a 2 to 6 mm, round bilateral sensory organ in the peripheral nervous system located in the adventitia of the bifurcation of the common
carotid artery.
94. Which of the following hormones is secreted when stimulation or activation of baroreceptors occurs?
A. Thyroxin B. Catecholamine C. Cortisol
95. Which of the following ligands bind to cell surface receptors?
A. Steroids B. Catecholamines C. Retinoids D. Thyroid hormones E. Vitamin D
96. In case of hypovolaemic shock, in which organ autoregulation of its own blood supply? Brain, Myocardium then Renal, then skeletal ms & bowel
A. Kidney (after brain & coronary) B. Brain (also renal and coronary) C. Intestine (moderate) D. Muscle (moderate)
97. Regarding serum osmolarity what is deficient to do other than BUN, Serum Na, serum urea?
A. Serum glucose B. Serum chloride
Serum osmolality is affected by the concentration of blood chemicals like chloride, sodium (Na), proteins, bicarbonate, and glucose serum osmolality.
It is primarily determined by sodium and its corresponding anions (chloride and bicarbonate), glucose, and urea;BUN.
Serum osmolality = 2x (Na mEq/L) + glucose (mg/dL) /18 + BUN (mg/dL) /1.8. The normal serum osmolality for adult/elderly: 285-295 mOsm/kg H2O or
285-295 mmol/kg, and for children is: 275-290 mOsm/kg H2O
98. Which system is less affected by ischaemia?
A. Cardiovascular B. GIT C. Liver D. Brain
The most affected organ is the brain due to many causes followed by myocardial muscles, then kidneys the next most susceptible organ system to reductions
in their blood supply, with permanent damage not appearing until the duration of ischaemia exceeds 30 min in humans. The liver is the organ that is less
affected by ischaemia and the ischaemia times exceeding 120 min are well tolerated using vascular inflow occlusion by repeated intermittent hepatic artery
and portal vein occlusion by compressing the hepatoduodenal ligament to control blood loss in trauma patients with a liver laceration.
99. During which cell cycle does DNA replication occurs?
A. Gl phase B. G2 phase C. S phase D. M phase
100. Arrest of cell cycle in the metaphase is the action of which of the following?
A. Cyclophosphamide B. Methotrexate C. Doxorubicin D. Vincristine
101. Peripheral neuropathy is the main side effect of which one of the following chemotherapeutic agents?
A. Cyclophosphamide B. Vincristine C. Methotrexate D. Mithramycin E. Adriamycin
Vincristine is a vinca alkaloid agent; antineoplastic agent used as a treatment for various cancers including breast cancer, Hodgkin's disease, Kaposi's
sarcoma, ALL, AML, CML, NHL, neuroblastoma, sarcomas, small cell lung cancer, Wilms' tumour, brain tumours and testicular cancer. It results in
sensory motor neuropathy with long-term use, severe constipation and paralytic ileus, sensory changes with paraesthesia and loss of deep tendon reflexes.
102. Severe peripheral neuropathy is a complication of which of the following?
A. Vinca alkaloids B. Cyclophosphamide C. Cytarabine D. Cisplatin
103. Which of the following chemotherapeutic drugs is DNA alkylating agent?
A. Cyclophosphamide B. Vincristine C. Methotrexate D. Doxorubicin
Alkylating agents three groups are almost always considered classical. Nitrogen mustards that include Cyclophosphamide; the most widely used alkylating
agent of modern times, Chlormethine also known as mechlorethamine or mustine, Uramustine or uracil mustard, Melphalan, Chlorambucil
104. What is true regarding Doxorubicin?
A. It is an alkylating agent B. It is an antimetabolite C. It is an alkaloid D. It is an anthracycline antibiotic
Doxorubicin is a part of the anthracycline group of chemotherapeutic agents, also daunorubicin, idarubicin, and epirubicin. Commonly, doxorubicin is an
agent used in the treatment of solid tumours in adult and paediatric patients. Doxorubicin may be used to treat soft tissue and bone sarcomas and cancers
of the breast, ovary, bladder, and thyroid. It is also used to treat acute lymphoblastic leukaemia, acute myeloblastic leukaemia.
105. Pulmonary fibrosis is a complication of which one of the following chemotherapeutics?
A. Bleomycin B. Cyclophosphamide C. Tamoxifen D. Vincristine E. Methotrexate
106. Spontaneous antitumour activity is a function of which of the following?
A. Macrophages B. B lymphocytes C. Cytotoxic T cells D. Helper T cells E. Natural killer cells
NK cell-mediated killing of tumour cells depends on the balance between stimulatory and inhibitory signaling, human NK cell is defined as
CD3−16+CD56+ lymphocytes. They comprise 10-15% of all circulating lymphocytes and are also found in lymph nodes, spleen, and peripheral tissues.
107. Regarding cytokines, what is correct?
A. Serum level is not related to the severity of illness B. Are stored intracellularly as preformed molecules
C. Are produced by a limited number of specific cells D. Most commonly function in an endocrine fashion
E. Low serum levels are normally detected in healthy individuals
108. The cytokine directly responsible for hepatic acute phase response is which of the following?
A. Interleukin-I B. Interleukin-2 C. Interleukin-6 D. Tumour necrosis factor alpha
109. What is the most potent inhibitor of T cell proliferation?
A. Transforming growth factor β B. Platelet-derived growth factor C. Epidermal growth factor
D. Basic fibroblast growth factor
Sepsis and surgical infections
1. A male patient came with scalp open wound, 6 hours after assault, what is the optimal wound management?
A. Secondary closure B. Debridement with primary closure C. Debridement with granulation D. Leave it for granulation
2. Intestinal obstruction, laparotomy, resection anastomosis, no peritonitis but with spillage, what is the wound type?
A. Clean B. Clean contaminated (if planned elective, prepared) C. Contaminated (emergency, not prepared) D. Dirty(abscess)
Wounds are classified according to sterility and contamination to 4 types, clean, clean contaminated, contaminated and dirty
1. Clean as mastectomy, hernia repair, thyroidectomy, laparotomy of non infected abdominal surgery, total knee or hip arthroplasty and vascular surgery.
2. Clean contaminated in which there is controlled intentional entering the GIT, genitourinary, or respiratory tracts as small bowel resection anastomosis
after good preparation, lung lobectomy, cholecystectomy without GB perforation or bile spillage catarrhal non perforated appendix with very minimal
controlled spillage, TURP, Whipple pancreaticoduodenectomy and hysterectomy
3. Contaminated as acute non purulent inflammation as acutely inflamed appendicitis, bile spillage after cholecystectomy, diverticulitis
4. Dirty or infected as abscess drainage, frank peritonitis, perforated diverticulitis, repair of perforated bowels or emergency non prepared colonic surgery
3. What is the percentage of surgical site infection in a wound after laparotomy with resection anastomosis and gross faecal spillage?
A. 1-3% (clean) B. 5- 8% (clean contaminated) C. 20-25% (contaminated) D. 30- 40% (for dirty) E. More than 50%
The wound classification system categorises all surgeries into clean, clean/contaminated, contaminated, and dirty, with estimated postoperative rates of
surgical site infection for clean wounds: 1%-3%, for clean contaminated: 5%-8%, for contaminated: 20%-25%, and for dirty wounds : 30-40%, (Amboss).
Based on the most recent estimates from the Centers for Disease Control and Prevention (CDC), 22% of all hospital acquired infections are SSIs, and 15%
of these are associated with colorectal procedures. No spillage, no entry of GIT, respiratory or genitourinary= clean.
Minimal spillage = clean contaminated (5-8%), Gross spillage = contaminated (20-25%). Abscess or perforated bowel with pus =dirty (30-40%)
4. Which organs have resistant microorganisms to antibiotics?
A. GIT B. Blood C. Pulmonary D. Genitourinary
5. What is the most common finding in contamination of hyperalimentation subclavian catheter in an immunocompromised patient on
broad spectrum antibiotic?
A. Pneumocystis carinii pneumonia B. Pseudomonas aeruginosa C. Candida albicans
6. What is the most common cause of acute suppurative otitis media; ASOM in children?
A. Mostly Moraxella catarrh.(16%) B. viral infection 77% (33%) C. Mostly bacterial infection (33% strept.) D. H influenza 55% (33%)
7. What is the most common causative agent in nosocomial sinusitis in the ICU?
A. Pseudomonas aeruginosa B. Staphylococcus aureus C. Staphylococcus epidermidis D. Streptococcus pneumoniae
8. A young boy presents with a chronic productive cough, recurrent chest infections and sinusitis, he has poor growth. Bronchoscopic
examination of the sputum reveals atypical organisms. What is the most likely diagnosis?
A. Tear of aortic intima B. Acute myocardial infarction C. Community acquired Chlamydophila pneumoniae
D. COPD with Bronchiectasis E. Congenital pulmonary cystic fibrosis
Congenital cystic diseases of the lung are a rare but significant cause of morbidity in children and young adults presenting with respiratory distress and
repeated chest infections. It can be complicated by chest infection with atypical organism as Bulkholderia cepacia complex.
9. A patient with generalised lymphadenopathy, axilla, neck, suffer from intermittent attacks of low-grade fever, and has cystic lesion in
the spleen, diagnosis?
A. Kala azar B. Malaria C. TB
10. An image of a big swelling in the back, it is cystic, with low-grade fever for 3 months and weight loss, history of spinal pain, mostly;
cold abscess what is the optimal management?
A. Incision and drainage B. Aspiration (if failed medical ATT, if spinal compressive manifestation in zigzag line) C. Anti-TB drugs
The treatment of TB as recommended by the WHO includes 6–9 months of chemotherapy with a combination of drugs given as an initial intensive phase
of 2 months and a subsequent continuation phase. The first-line drugs include isoniazid, rifampicin, ethambutol and pyrazinamide, of which the first
two drugs are not withdrawn during the continuation phase.
11. A middle aged lady with very big tumifaction (swelling) in her back about 20 cm, she has this swelling for 3 months associated with
anorexia, night sweating, and loss of weight, query TB with cold abscess, what is the first next step?
A. Incision and drainage C. US guided pigtail drain D. Aspiration only E. Anti TB 6-9 m if failed do aspiration if disc compression
Medical treatment is the mainstay for 6-9 months whatever the size is except if disc or spine compression or neurological manfistation can do decompression
by aspiration or even surgical debridement
12. What is the treatment for vertebral cold abscess?
A. Incision B. Aspiration C. Anti TB drugs (for early disease with type 1A without spine or disc compression or destruction)
13. A patient hit by horn or his foot injuried, came with complete body bent or arch shaped body and fits, what is most likely diagnosis?
A. Tetanus (spasm, arched or bent no lymphangitis) B. Epileptic fits C. Spinal cord injury D. Malingering
1. Any wound, in completely unvaccinated patient, have to be given tetanus toxoid and immunoglobulin within 21 days of injury
2. Minor or clean wound in vaccinated < 3 doses or unknown to be given tetanus without immunoglobulin. Contaminated wound < 3 doses or unknown give
both tetanus and immunoglobulin.
3. If > 3 doses if minor clean wound, no need to be given tetanus toxoid, except if last dose more than 10 years, if dirty wound dont give except if last dose more
than 5 years, in all cases no need for immunoglobulin. Wounds considered contaminated if dirt, faeces, soil, or saliva; puncture wounds; avulsions; or wounds
resulting from missiles, crushing, burns, or frostbite
14. A patient with wound due to falling on dirty place, with lacerated wound, received tetanus 15ys ago, what to do?
A. Tetanus alone B. Tetanus with immunoglobulins (dirty infected wound last dose >15 y) C. Immunoglobulins
15. A patient with hand injury by a barbed wire received anti-tetanus vaccine and completed 12 yrs. ago, since last dose what is best?
A. Tetanus toxoid (not dirty object) B. Immunoglobulin C. Tetanus + toxoid+ immunoglobulin D. Antiserum toxin
16. A patient with foot injury by a piece of clean wood, while he arrange, his office, last dose of tetanus vaccine 12 years back, what will
you give him?
A. Tetanus toxoid (minor, clean, last > 10y) B. Tetanus immunoglobulin C. Tetanus antiserum D. Tetanus toxoid + IG
17. A patient with clean minor cut wound, tetanus vaccine 6 ys back or child had tetanus vaccine 6 yrs. ago, injured with screw?
A. TT B. Immunoglobin C. No vaccines needed (nothing less than 10 y only advise him to take next dose at 10 yrs. after last dose)
18. Through what the protective immune function of immunoglobulin A is mediated?
A. Inhibition of bacterial adherence to epithelial cells B. Activation of complement C. Opsonisation
D. Direct destruction of microorganisms
19. What type of immunoglobulin provides mucosal defense?
A. IGA B. IGG C. IGM D. IGD E. IGE
Immunoglobulin A (IgA) is the most abundant type of antibody in the body, comprising most of the immunoglobulin in secretions and a significant amount
of circulating immunoglobulin. In secretions, it serves to protect the mucosal tissues from microbial invasion and maintain immune homeostasis
20. What is the causative organism leading to infection caused by dog and cat bites?
A. Pasteurella species B. Mycobacterium C. Staphylococcus aureus D. Actinomyces E. Candida
Pasteurella multocida is the most common cause of infection following a bite or scratch from domestic pets as cats and dogs. Exposure can lead to rapidly
progressing soft tissue, respiratory, or other serious invasive infections.
21. An infected dog bit a pedestrian guy was walking beside its home, what is optimal drug treatment for his wound beside vaccination?
A. Gentamicin B. Amoxicillin-clavulanate C. Vancomycin D. Clindamycin
Antibiotic treatment for infected wound with high risk as if immunocompromised, hand, face wound, delayed presentation or gaping. If the wound is
infected on presentation, a course of 10 days or longer is recommended with rabies, vaccine and tetanus if las > 5 years. The first-line oral therapy is
amoxicillin-clavulanate. For higher-risk infections, a first dose of antibiotic may be given intravenously (ie, ampicillin-sulbactam, ticarcillin-clavulanate,
piperacillin-tazobactam, or a carbapenem). Alternatively; cefuroxime plus clindamycin or metronidazole, a fluoroquinolone plus clindamycin.
22. What is the causative organism of Hand infection due to a human bite?
A. Staphylococcus aureus B. Clostridium difficile C. Herpes simplex D. Eikenella corrodens E. Candida species
The predominant organisms in human bite wounds consist of both aerobic bacteria and anaerobic bacteria, including streptococci, Staphylococcus aureus,
Eikenella, Fusobacterium, Pepto streptococcus, Prevotella, and Porphyromonas spp
23. Which of the following organisms is slime producing?
A. Candida albicans B. Staphylococcus aureus C. Staphylococcus epidermidis D. Klebsiella pneumoniae
24. A patient is presented with hand cellulitis and red streaks in the hand and tender axillary lymphadenopathy. This condition is more
likely to be associated with?
A. Malignancy B. Pyoderma C. Neuropathy D. Lymphangitis
25. A boy with foot wound by a piece of wood with swollen leg, red streaks, fever and normal TLC, what is the possible causative organism?
A. Clostridia Tetani B. Streptococcus Pyogenes (lymphangitis) C. Staphylococcus Aureus D. Clostridia perfringens
Lymphangitis is an inflammation of lymphatic channels and most commonly develops after cutaneous inoculation of microorganisms; Streptococcus
pyogenes, Staphylococcus aureus, and gram-negative organisms. Animal exposure can confer risk for lymphangitis due to Pasteurella, Erysipelothrix, and
anthrax. The clinical manifestations of lymphangitis are variable and may be characterised by erythematous streaks with pain and rapid spread or by
nodular swellings along the course of the lymphatic vessels
26. An ICU elderly patient developed sacral ulcer with discharge and necrotic area & exposed fat what’s management?
A. Primary closure B. Debridement with 1ry. closure C. Debridement with skin graft D. Debridement with VAC
27. What is the most common organ involved in graft-versus host reaction?
A. Lungs B. Kidneys C. Heart D. Skin
28. Diabetic patient with pseudo epitheliomatous hyperplasia in situ, what you should do?
A. Amputate toe B. Ulcer Debridement C. Follow up D. Repeat biopsy
Pseudo epitheliomatous hyperplasia is a benign condition characterised by hyperplasia of the epidermis and adnexal epithelium must be differentiated from
SCC, as the treatment and prognosis for the patient will change, it can be managed with surgical excision with adequate margin and antibiotics
29. An elderly patient in ICU has bed sores with exposed necrotic skin and subcutaneous tissue. What is your management?
A. Debridement with a skin graft B. Debridement with assisted vacuum C. Debridement with primary closure
D. Debridement & Dressing with secondary closure
30. Risk of irreversible tissue damage in pressure sores is highest with which case?
A. Constant pressure of 50 mm Hg for 2 hours B. Constant pressure of 70 mm Hg. for 2 hours
C. Constant pressure of 100 mm Hg for 30 minutes D. Constant pressure of 150 mm Hg for 20 minutes
31. A diabetic patient not controlling his disease with foul smelly ulcer, patch over the dorsum of left foot and crepitus extending to the
mid-thigh what is best treatment?
A. Debridement, drainage, CS & broad spectrum Abs. B. Advanced wound care C. Vacuum assist D. Antibiotics
32. The circulating level of which cytokine can be used as prognostic marker in sepsis?
A. Interleukin-I B. Interleukin-2 C. Interleukin-6 D. Interleukin-8 E. Tumour necrosis factor
33. Which of the following distinguishes adrenal insufficiency from sepsis?
A. Hypotension B. Fever C. Tachycardia D. Altered mental status E. Hypoglycaemia
34. What is the main source of fuel in sepsis?
A. Glucose B. Fatty acids (acute starvation < 5 days) C. Ketones (if starvation > 5 days) D. Amino acids
Lipids are the primary source of energy in patients with infections. The oxidation of fatty acids is a key source of energy in conditions of increased demand,
including sepsis which is associated with life-threatening organ dysfunction resulting from a dysregulated host response to an infection, this response is
generally characterised by an acute and massive release of stress hormones, leading to an overwhelming production of energy substrates in the form of
glucose, fatty acids, amino acids and lactate.
35. Anterior resection, post op day 7, abdominal pain, generalised tenderness, distension. fever 39 C, what is the cause of fever?
A. Sub phrenic collection B. Pulmonary embolism C. Suspected leak (if other surgery without anastomosis can choose PE/DVT)
36. A case of SIRS P > 90/min, Temp >38 or <36, RR >20 or pCO2 <32, WBCs >12000 or <4000 or 10% band cells
N.B. Body temperature is measured as core or central body temperature, not peripheral, a tricky point telling peripheral body temperature
37. Which one of the following diagnoses is determined clinically without pathology?
A. SIRS B. Septicaemia C. Septic Shock D. Haemorrhagic shock
SIRS is a collection of physiological and immune mediated reactions to infectious and non-infectious insults as acute inflammatory process or trauma
38. After 5 days of Roux en Y lap gastric bypass, middle aged female developed abdominal pain and fever, pulse: 95, BP: 110/70, WBC:
12000, the drain output was: 60 ml of non-clear fluids, although the drain was empty since surgery, what the pathophysiology?
A. SIRS B. Sepsis (SIRS+ leak; infection source, no hypotension) C. Severe sepsis D. Septic shock
39. What is the most common complication of otitis media in children?
A. Labrynthitis B. Meningitis C. Encephalitis D. Mastoiditis (the most common)
Complications of mastoiditis are related to the spread of infection or inflammation from the middle ear or mastoid to contiguous structures, subperiosteal
abscess occurred in an average of 58 % of cases of acute mastoiditis, Infection in the mastoid may spread to other parts of the skull, resulting in osteomyelitis
40. A male patient with injury of foot, took antibiotic, then developed sepsis, what is the pathology? Increased capillary permeability
41. A patient is present with bleeding after 5 days of tonsillectomy, what is the cause of bleeding?
A. Sepsis B. Incomplete removal C. Coagulopathy D. FB
42. A patient present with colitis and temperature 39, pulse 95/min, RR 22/min, BP 90/60, diagnosis?
A. SIRS B. Sepsis (infection source + SIRS) C. Severe sepsis (Sepsis+MAP >65 with end organ damage, fluid responsive)
D. Septic shock (if MAP <65 with fluid irresponsive and vasopressors need)
43. Which electrolyte disturbance is a sign of early sepsis?
A. Metabolic alkalosis B. Metabolic acidosis (lactic acidosis) C. Respiratory alkalosis D. Respiratory acidosis
Hyponatraemia is the common electrolye disturbance in sepsis. Patients with severe sepsis and septic shock exhibit a complex metabolic acidosis at intensive
care unit admission, caused predominantly by hyperchloraemic acidosis,which was more pronounced in nonsurvivors. In the early phase of shock, the most
common metabolic disturbance is metabolic acidosis. This occurs due to tissue hypoperfusion and inadequate oxygen supply, leading to the accumulation of
lactic acid. Metabolic acidosis is usually accompanied by an elevated lactate level in the blood. In sepsis, the most common electrolyte abnormality in the
early stage is hyponatraemia. Sepsis can cause an excessive release of antidiuretic hormone (ADH) or vasopressin, leading to water retention and dilutional
hyponatraemia, also result in increased capillary permeability and fluid shifts, which may further contribute to electrolyte imbalances
44. A patient with BP 120/80, creatinine and urea were high, diagnosis?
A. SIRS B. Sepsis C. Severe sepsis (organ dysfunction; renal) D. Septic shock
45. A patient in ICU, SIRS, elevated BUN, and Creatinine, what is the case?
A. Severe sepsis B. SIRS C. Septic shock D. Sepsis
46. A patient after surgery develop fever urine become + of E. coli, normal BP, with tachycardia 100, what is the diagnosis?
A. SIRS B. Sepsis (SIRS+ source of infection) C. Bacteraemia
47. Death from postoperative renal failure is most commonly due to which of the following?
A. Myocardial infarction B. Bleeding C. Sepsis D. Liver failure
48. A sudden onset of glucose intolerance in patients receiving total parenteral nutrition often indicates which of the following?
A. Diabetes mellitus B. Sepsis C. Hypophosphataemia D. Adrenal insufficiency E. Zinc insufficiency
The risk predictors for hyperglycaemia are BMI >25 kg/m2, a previous history of DM2, the use of vasoactive amines, the use of systemic steroids, and the
presence of infection or sepsis.
49. A patient with hypoperfusion, sepsis what can be given?
A. Meropenem IV B. Vancomycin C. Ciprofloxacin
50. A 23 year old girl came to surgical OPD, with history of insect bite (mosquito) one week ago she has now small bulla 1x 0.5 cm in her
forearm, no redness, hotness, throbbing pain or fever, just some itching, what is the suitable action?
A. Antibiotics (if infected) B. Antihistaminics C. Incision and drainage D. Debridement (if necrotising fasciitis)
51. A Mosquito or insect bite followed by erythema after 2 days, swelling, crepitus detected under the skin, leucocytosis, with increased
creatine kinase, what to do, another scenario for a lady with cannula site infection with signs of gas gangrene, WBC 18000, T39C with
normal neurovascular status no compartment what’s the optimal management?
A. Debridement as there is sc. gas B. IV antibiotics (after debridement and drainage) C. Antihistaminic D. Observation
52. A 25 y. o patient presented with a papule at calf muscle for 1 day, spread via S.C. struck to groin, the most causative organism is?
A. E coli B. Streptococcus Pyogenes C. Clostridia Difficle D. C.W. (Clostridium welchii)
53. A case with red streak in the right thigh (lymphangitis) what is the management?
A. Oxacillin (IV reserved for serious infections) B. Tetracycline C. Dicloxacillin (oral as acid stable best for lymphangitis)
Lymphangitis is mostly caused by group A beta haemolytic streptococci and staph aureus infection which can be treated with Dicloxacillin, cefalexin,
cefazolin, cefuroxime, ceftriaxone, and Clindamycin
54. Haemorrhagic dermal bullae are characteristic of wound infection caused by which microbial agent?
A. coagulase-negative Staphylococcus B. Streptococcus pyogenes C. Clostridium perfringens D. Clostridium tetani
56. A child has wound in right foot with wood prick, red, hot and tender, has bent or arched body what is the most likely organism?
A. Clostridium tetani B. Clostridium perfringens C. Streptococcus pyogenes
57. A 30 ys male. underwent hernioplasty >>24 hrs. after crepitus, discharge. H&E>>rod with gram-positive spore-forming bacilli
terminal spore? (Like tetanus?) the most important ttt is? terminal spore? (Like tetanus?)
A. Surgical debridement & large dose of penicillin G (Cl. perfringens can cause gas gangrene) B. Large dose of penicillin G
C. Anti-tetanus D. Surgical debridement
58. A patient with history of Littré's hernia where Meckles was necrotic and diverticulectomy done with herniorrhaphy only done, after
about 5 days he got fever, localised wound pain, oedema and redness of suture line, some sutures open with gush of smelly grayish
discharge with some areas with criptus subcutaneously, mostly necrotising fasciitis what is the best management?
A. Debridement B. Antibiotics only C. Debridement and antibiotics D. Just open sutures with twice dressing
Firstly, must see the patient’s general condition if he is stable go the next step of debridement under cover of broad spectrum IV antibiotics, but if he is
hypovolaemic, shocked or toxic have to fully resuscitate him. Surgical debridement is the primary treatment under cover of broad spectrum antibiotics
59. An elderly known case of poorly controlled DM comes with ulcers on tip of three of his toes, diminished dorsalis pedis bilaterally and
popliteal artery what’s the initial management?
A. Amputation B. Negative pressure dressing C. Surgical intervention D. Debridement
60. What is the proper antibiotic for treatment of infected bullous dermatitis of the buttock with gram +ve bacteria associated by
subcutaneous crepitus, most likely, necrotising fasciitis?
A. Clindamycin B. Penicillin G C. Ciprofloxacin D. Clindamycin plus Penecillin
Combination therapy with penicillin and clindamycin should be continued until patients are clinically and haemodynamically stable for at least 48 to 72
hours; thereafter, penicillin monotherapy may be administered. Penicillin (4 million units IV every four hours in adults >60 kg with normal renal function
or 300,000 units/kg per day divided every six hours in children) plus Clindamycin (600 to 900 mg IV every 8 hours in adults or 40 mg/kg per day divided
every 8 hours in neonates and children). Other acceptable empirical antibiotics include carbapenem as meropenem or imipenem 1 g IV every 6 to 8 hours
or Piperacillin-tazobactam 4.5 g every 8 hours, plus vancomycin IV: 15 to 20 mg/kg/dose every 8 to 12 hours initially (for MRSA) plus Clindamycin.
61. Diabetic foot patient, ulcer between 1st. and 2nd. toe, there is fever, oedema, and hyperaemia, high WBCS, High RBS management?
A. Antibiotics B. Debridement and injectable antibiotics, sliding scale for DM C. Fasciotomy D. Amputation
62. I.V addict, severe infection at arm, normal vessels and sensation what is best treatment option?
A. Amputation B. Antibiotic C. Aggressive debridement
63. A patient injured his finger with knife, self-dressing, after 4 days the GP gave him antibiotics and he stop it after 2 days, now erythema
of the finger, crepitus in the arm, drowsy, management? Debridement, its pathophysiology due to? Increased capillary permeability
Surgical exploration is the only way to establish the diagnosis of necrotizing infection. Findings on direct examination include swollen and dull-gray
appearance of the fascia, thin exudate without clear purulence, and easy separation of tissue planes by blunt dissection
64. Which of the following is a feature of soft tissue necrosis (necrotising fasciitis) what you will find on examination?
A. Warm hand with diffuse swelling B. Wound+ fever > 40 C. Non-pitting oedema D. Wound with pus discharge
The best initial radiographic imaging exam is computed tomography (CT) scan. The most useful finding is presence of gas in soft tissues, which is seen most
frequently in the setting of clostridial infection or polymicrobial (type I) necrotising fasciitis; this finding is highly specific for NSTI and should prompt
immediate surgical intervention. Other radiographic findings may include fluid collections, absence or heterogeneity of tissue enhancement with
intravenous contrast, and inflammatory changes beneath the fascia. Surgical exploration is the only way to establish the diagnosis of necrotising infection.
Findings on direct examination include swollen and dull-gray appearance of the fascia, thin exudate without clear purulence, and easy separation of tissue
planes by blunt dissection
65. What is the most sure sign of necrotising soft tissue infection?
A. Grayish brown discharge from the wound B. Fever + soft tissue infection C. Redness, pain, pitting oedema with fever
Clinical manifestations of necrotising infection include Erythema (without sharp margins; 72 %), oedema that extends beyond the visible erythema (75%)
Severe pain (out of proportion to exam findings in some cases; 72 %), fever (60 %), crepitus (50 %), Skin bullae, necrosis, or ecchymosis (38 %).
Hypotension may be present initially or develop with progressive infection. Other symptoms include malaise, myalgias, diarrhoea, and anorexia. The
subcutaneous tissue may be firm and indurated, such that the underlying muscle groups cannot be palpated distinctly. Marked oedema may produce a
compartment syndrome with complicating myonecrosis requiring urgent fasciotomy. In the setting of surgical wound infection, NSTI is characterised by
copious drainage, dusky and friable subcutaneous tissue, and pale, devitalised fascia.
66. An addict patient with multiple pricks on forearm subcutaneous gas (gas gangrene) very bad condition metabolic acidosis 7.12,
hypoxia, what is the optimal management?
A. ICU admission B. Fluid replacement C. Massive debridement D. Antibiotics
67. A case of necrotising fasciitis/gas gangrene, C/S showed gram positive rods, what’s the best antibiotic for it?
A. Penicillin G B. Tetracycline C. Clindamycin D. Combination of Clindamycin and Penecillin
68. A case of foot injury with wood piece, causing lymphangitis, what is the causative organism?
A. Clostridium tetani B. Streptococcus pyogenes (in normal immunity patient) C. Clostridium perfringens (food poisoning & gas gangrene)
69. A patient underwent right hemicolectomy, admitted to ICU, deteriorated, preoperatively he was treated from bronchiectasis with
metronidazole and Ceftriaxone, Sputum culture showed gram negative bacterium, which antibiotics you will prescribe?
A. Gentamycin B. Meropenem (gram -ve k. pneumonia) C. Ciprofloxacin D. Ampicillin
Meropenem is a broad-spectrum carbapenem antibiotic. It is active against Gram-positive and Gram-negative bacteria. Meropenem exerts its action by
penetrating bacterial cells readily and interfering with the synthesis of vital cell wall components, which leads to cell death
70. Endophthalmitis is characteristic of which of the following?
A. Escherichia coli sepsis B. Toxic shock syndrome C. Systemic candidiasis D. Facial necrotising fasciitis
71. A painful red swelling in the left side nose what most dangerous complication?
A. Face oedema B. Sinusitis C. Cavernous sinus thrombosis (dangerous triangle)
72. Multiple pilonidal sinus openings of abscess are seen laterally in a picture, what is the pathology?
A. Abnormal hair growth/ Malgrowth of hair follicles B. Infected congenital sinuses C. Bad hygiene (with abscess)
73. An image of Sacral PNS with multiple openings over the sacral area, patient complains of pain & discharge, diagnosis?
A. Pilonidal sinus B. Anal fistula
74. A pilonidal sinus image and scenario, treatment?
A. Warm paths B. Incision and drainage of the abscess C. Excision of the sinus and all openings or tracks
75. Truck driver with severely painful intergluteal; para midline swelling, with tenderness and infrequent discharge management?
A. Aspiration B. Antibiotic treatment C. Observation D. Incision and drainage (Pilonidal abscess)
76. An old, aged ICU patient was found to have a large wound in the sacral area. On exam, there is discharge and devitalised skin and
fatty tissue, what is the management?
A. Debridement with primary closure D. Debridement with Vacuum assisted closure (VAC) C. Debridement with graft
77. Distinction between toxic epidermal necrolysis and staphylococcal scalding skin syndrome is based on which of the following?
A. Degree of erythema B. Bullae formation C. Level of exfoliation D. Response to steroids
78. Toxic epidermal necrolysis in children is most related to which of the following?
A. Pseudomonas aeruginosa B. Escherichia coli C. Staphylococcus aureus D. Streptococcus pneumoniae
79. What is the most serious problem to give parenteral nutrition in central line?
A. Infection B. Electrolyte imbalance C. Intestinal atrophy
80. Common organism isolated from infected central catheter in (femoral vein)
A. Staph aureus B. Bacteroides C. Staph epidermidis D. Candida’s
Various studies report Gram - positive bacteria, Staphylococcus aureus (S. aureus) as the most common aetiological agent
81. A male patient with neglected foot injury then, received antibiotics, but developed sepsis, what is the pathology? Increased capillary
permeability
82. Ventilator associated pneumonia (VAP) was encountered in several patients in the medical intensive care unit, which of the following is
the most effective and the most straightforward method to prevent transmission of microbes?
A. Wash hands between the patients or before and after every case exam B.Wear gown before entering dressing room
C. Wear shoe cover D. Wear mask and overhead cover E. Do dressing with betadine and hydrogen peroxide
83. Organism that was known skin commensal but found to be pathogenic; colonise catheters and cause bacteraemia and endocarditis?
A. Staphylococcus epidermidis B. Pseudomonas aeruginosa
84. A 63y admitted for an elective colon resection for recurrent attacks of sigmoid diverticulitis, you want to give prophylactic antibiotics
in choosing a regimen you should be aware that the most common organism found in the colon of normal individuals?
A. E. coli B. Clostridium difficile C. Pseudomonas species D. Bacteroides species
Bacteroides fragilis is an obligate anaerobic, Gram-negative bacillus and a part of the human gastrointestinal microbiota but can cause opportunistic
infections in human. The human colon has the greatest population of bacteria in the body (over ten organisms per gram of wet weight), and the largest part
of these organisms are anaerobes; of these, approximately 25% are species of Bacteroides.
85. The most common virus transmitted by blood transfusion?
A. Hepatitis A B. Hepatitis B C. Hepatitis C D. HIV
It is estimated that 240 million individuals are chronically infected with hepatitis B virus. Of these cases, more than 680,000 individuals die every year due
to complications of hepatitis B, including cirrhosis and liver cancer. Approximately 150 million individuals worldwide have chronic hepatitis C infection,
and nearly 700,000 of them die each year from liver disease. The three most common bloodborne pathogens are human immunodeficiency virus (HIV),
hepatitis B virus (HBV), and hepatitis C virus (HCV) CDC). CMV will affects the children more and is the most common infectious cause of birth defects.
86. Compared with hepatitis B virus, hepatitis C virus?
A. Is an RNA virus B. Less commonly results in persistent viremia C. Can be prevented with effective vaccination
D. Can be transmitted via the fecal-oral route
87. Standard screening tests on blood donor include all except
A. Hepatitis C B. Hepatitis B C. Rubella D. Syphilis
SHOCKS
1. What is the earliest sign of hypovolaemic shock?
A. Tachycardia (with increased diastolic BP, narrow PP) B. Wide pulse pressure C. Hypotension D. Disturbed conscious level
2. What is the best indicator of resuscitation in case of hypovolaemic shock?
A. CVP B. Increased urine output; UOP (The single most indicator of successful resuscitation) C. Peripheral oxygen saturation
3. RTA, pelvic fracture and thigh wound with profuse bleeding; and hypovolaemic shock parameters, what latest sign of shock to occur?
A. Increase venous return B. Increased cortisone level C. Increased arteriolar resistance and decrease COP D. Tachycardia
4. A PO2 of 90 torr, PCO2 of 28 torr, and pH of 7.16 on room air are indicative of which of the following?
A. Hypovolaemic shock B. Alveolar hypoventilation Prolonged nasogastric suctioning Hyperventilation
5. A 25-year-old male patient who was involved in motor vehicle collision, brought by ambulance to ER, he has increased JVP, faint heart
sounds, BP 90/56, P 116 with tachycardia with chest and abdominal bruises, fully resuscitated and investigated, what’s shock type?
A. Cardiac tamponade B. Cardiogenic shock C. Hypovolaemic shock (loss of blood in thoracic cavity—tricky Q) D. Spinal shock
6. A middle aged patient came to casualty post RTA in hypovolaemic shock how much initial fluid resuscitation to be given as a bolus?
A. 10-15 ml/kg B. 20-30 ml/kg C. 50 ml/kg D. 50-70 ml/kg E. 5-10mg/kg
The routine maintenance as follow normal requirements are 25-35 ml/kg/day water plus 1 mmol/kg/day of each sodium, potassium and chlorides plus 50-
100 gm glucose/day (glucose 5% contains 5 gm/100 ml i.e., 500-1000 glucose 5% /day). Initially check losses and start to give 500 ml as a bolus then re assess
vitals if still in need give up to two liters as a bolus each 500 ml with 15 minutes if more than 2 litres given and still in need ask for expert help
7. A 27 y.o guy after MVC he has deep wounds with severe bleeding that make him shocked in ER he was fully resuscitated, how much
the optimal MAP and systolic blood pressure to be reached for his general condition to be stable and quiet?
A. MAP 90-100 or systolic 90-100 B. MAP 100-110 or SP 110 C. MAP 80-90 or SP >90mmHg. D. MAP 110-120 SP 115
MAP= diastolic BP + 1/3 (systolic- diastolic) or diastolic x 1/3 PP so, = diastolic + 0.33 (S-D) the normal MAP is 70-100 so the optimal to be here 80-90mmHg.
8. Another trauma patient with shock, capillary refill just 2 sec, cause of shock? Hypovolaemia
9. What is the acceptable minimal urine output in case of hypovolaemic shock to be considered not oliguria?
A. 0.5mL/kg/hour B. 1mL/kg/hour C. 1.5 mL/kg/hour D. 2mL/kg/hour E.2.5 mL/kg/hour
Normal urine output in normal people is 0.5-1.5 ml/kg/hour and patient should urinate at least every six hours, oliguria is < 400 ml/ day urinary output of
or less than 20 ml/hour and is one of the earliest signs of impaired renal function, in children if less than 0.5cc/kg/hr., in infants if <1 cc/kg/hr., and anuria if
no or minimal urine output usually < 100ml/day in adults
10. A patient involved in RTA, urine output 25 ml/ h, cold clammy sweating, mild anxious, RR= 25, BP= normal or 110/65, pulse= 110,
what is the volume of blood loss?
A. < 15% B. 15-30% C. 30-40% D. > 40%
11. A post RTA patient, his blood pressure is 80/60, HR 110, RR 20, what is his grade of shock?
A. Second stage B. Third stage C. Fourth stage D. First stage
12. A patient with hypovolaemic shock, mildly anxious, BP 100/70, P: 110, what’s the stage of shock?
A. Stage I B. Stage II C. Stage III D. Stage IV
13. A shocked patient anxious, restless, confused, BP 80/40 which fluid
A. Ringer’s lactate B. Saline 1000 mL C. Crossed matched blood D. Non-cross matched O negative blood
14. Scenario of hypovolaemic shock after RTA with BP is 90/60, HR 130, RR 28; clear signs of grade III hypovolaemic shock what is the
best fluid for resuscitation?
A. Ringer’s lactate or Crystalloids (initial) B. Blood C. Colloids
15. Which of the following indicates compensated shock?
A. Anuria B. Confusion C. Hypotension D. Pale peripheries
16. An RTA with hypovolaemic shock signs, HB low, what to give initially?
A. Crystalloid (Ringer lactate) B. Packed RBC C. Whole blood transfusion
17. A patient with RTA came to ER with deep bleeding scalp wound, BP 70/50, P 130, RR 35, what initially will be given?
A. Colloid B. Crystalloids C. Blood of his group D. O-ve non cross matched blood (definitive if no response to 2L of IVF)
18. A patient was involved in RTA, came to ER in hypovolaemic shock, P; 110, BP;110/70 what is the best bore cannula?
A.16 (3rd- 4th.) B. 18 (2nd degree) C. 12 (4th degree up to CVP line with 9.5 French cannula) D.14 (3rd. degree)
19. Trauma patient, unconscious, oxygen sat. is 95%, shocked, distended abdomen, fracture femur, what is the initial management?
A. Maintain airway (ABCDE for intubation as comatosed) B. Wide bore cannula and IV fluid
20. Chest trauma, stab wound at epigastrium and cause cardiac tamponade, what is the site injured at the heart?
A. Right ventricle B. Left ventricle C. Right atrium D. Left atrium
21. A case of chest stab wound, the patient has congested neck veins, muffled heart sounds, diagnosis? Cardiac tamponade
The classic physical findings in cardiac tamponade included in Beck's triad are hypotension, jugular venous distension, and muffled heart sounds
22. A patient had a chest stab wound, another scenario; post RTA with chest trauma, came with hypotension, weak thready pulse,
distended jugular veins or raised JVP, normal respiration with equal bilateral air entry what is diagnosis?
A. Haemothorax B. Pneumothorax C. Cardiac tamponade D. Pulmonary contusion (chest ecchymosis)
Due to bilaterally equal air entry; not pneumothorax, and no chest wall ecchymosis or contusion so, it is not pulmonary contusion
23. Post RTA patient found shocked with congested neck veins, decrease of heart sounds with increased JVP low COP, high SVR?
A. Cardiogenic B. Distributive C. Obstructive (cardiac tamponade) D. Hypovolaemic
24. A patient with chest trauma, congested neck veins, muffled heart sounds, average blood pressure, tachycardia, what’s the shock type?
A. Neurogenic B. Hypovolaemic B. Cardiogenic D. Cardiac tamponade
25. A chest stab wound, with symptoms and signs of cardiac tamponade what’s the type of shock?
A. Cardiogenic shock B. Neurogenic shock C. Hypovolemic shock D. Obstructive shock
26. A post RTA patient who was stable in the ambulance, after some time sudden deterioration occurred with increased JVP and weak
thready pulse, what cause of shock?
A. Cardiac tamponade (all SVR, CVP and PCWP increase but PCWP <15 mmHg with low COP)
B. Cardiogenic shock (all SVR, CVP and PCWP increase but PCWP >15 mmHg, with low COP)
C. Hypovolaemic shock (CVP, PCWP and COP decrease, but SVR high) D. Neurogenic shock (CVP, PCWP, CO, and SVR decrease)
E. Septic shock (SVR, CVP, PCWP decrease but COP increase at start and decrease late)
27. A patient after RTA dyspnoea, limited chest movement with hyper resonance on the percussion of the Rt side, there is a penetrating
wound of the abdomen, sat 92%, RR 40, HR, 140, BP 90/40, what to do next?
A. Laparotomy B. Observation C. CT D. Needle thoracostomy
28. Distinction between haemorrhagic and cardiogenic shock can be based on which of the following?
A. Level of urinary sodium B Ventricular filling pressures C. System vascular resistance
D. Serum lactate level E. Mixed venous oxygen saturation
29. A stab wound developed cardiac tamponade in right ventricle which will be seen regarding cardiac output and load?
A. Increased preload B. Decreased Rt. ventricular preload C. Increased afterload D. Decreased afterload
The preload is the force that stretches the cardiac muscle in diastolic phase prior to contraction, it decreases in Cardiac tamponade, as haemopericardium
impede the heart to receive the blood coming via superior and inferior venae cavae
The afterload is the amount of pressure that the heart needs to exert to eject the blood during ventricular contraction (systolic BP) afterload will be
increased in Cardiogenic shock, cardiomyopathy, MI and cardiac muscle dysfunction.
30. What is the effect of Dobutamine use in cardiogenic shock and heart failure?
A. Has a major chronotropic action B. Decreases cardiac filling pressure
C. Increases systemic vascular resistance D. Directly increases renal blood flow
31. Truck driver presented with raised JVP and decreased BP and distant heart sounds, what kind is the shock?
A. Anaphylactic B. Septic C. Cardiogenic (Cardiac tamponade is under cardiogenic) B. Haemorrhagic
32. What is the type of shock having the following characteristics, increased JVP, low COP, low BP, high SVR, and low O2?
A. Cardiogenic B. Hypovolaemic C. Distributive D. Obstructive
The increased SVR present in cardiogenic, haemorrhagic, and obstructive shock is the body's attempt to maintain blood pressure (perfusion pressure) by
increasing arteriolar tone
33. A patient post trauma has distended neck vein that does not move with respiration his cardiac output 2.5 liter, what’s type of shock?
A. Cardiogenic B. Hypovolaemic C. Obstructive
Normal COP is 5-6 liter/minute at rest and can reach up to 35 liter /minute in elite athletes during exercise. Low Cardiac output + distended neck vein;
both goes with cardiogenic shock and cardiac tamponade but mostly here it is cardiac tamponade due to trauma which may cause pericardial space
bleeding leading to decrease cardiac preload and restricts the ventricular relaxation to accommodate the coming venous return from SVC and IVC.
34. A mechanically ventilated patient with post RTA multiple traumas and cardiogenic shock and ARDS the intensivist doctor decided to
continue vasoactive medications for him, what is the most common adverse effects of vasoactive medication?
A. Hypotension and bradycardia B. Arrhythmias and myocardial ischaemia C. Cerebral haemorrhage D. Bleeding PR
35. What is the type of shock having the following characteristics, increased JVP, low COP, low BP, low SVR, and low O2?
A. Cardiogenic B. Hypovolaemic C. Distributive (septic shock) D. Obstructive
36. A middle aged male patient brought from forest in shocked state with shortness of breath, chest wheezes, abdominal wall puffiness and
tachycardia what is the type of shock?
A. Septic shock B. Hypovolaemic shock C. Anaphylactic shock D. Cardiogenic E. Obstructive shock
Promptly and simultaneously, give: IM epinephrine (1 mg/mL preparation): Give epinephrine 0.3 to 0.5 mg intramuscularly, preferably in the mid-outer
thigh. The first and most important treatment in anaphylaxis is epinephrine. There are NO absolute contraindications to epinephrine in the setting of
anaphylaxis, can repeat every 5 to 15 minutes (or more frequently), as needed.
37. An RTA case with injury to thoracic vertebra (T9) presented, with hypotension, bradycardia with paraplegia, bladder and bowel
incontinence, breathing completely normal, what’s the best and definitive step?
A. Vasopressors and inotropics (Spinal shock, for norepinephrine +/- Desmopressin / phenylephrine; as below T6) B. OT intubation
C. IV fluids (initially if no response give vasopressors)
38. A 33 y.o guy, was involved in MVC, then brought by ambulance team, presented with hypotension, bradycardia, hypothermia, seen by
orthopedic and spine surgeons where found T4 fracture, with normal respiration, bowel and bladder functions, what’s the best option?
A. About 2-3 liters saline B. Vasopressors and inotropics (Neurogenic shock for Norepinephrine or Dopamine) C. Protect airway
Neurogenic shock, ocuurs mostly due to injury of spinal cord above the level of T6, mostly presented with haemodynamic changes if the injured cord was
above T6, classically associated with hypotension, bradycardia, hypothermia and flushed warm skin, no affection of bowel or bladder functions
The primary management for neurogenic shock involves the administration of vasopressors and inotropic agents.Vasopressors will increase peripheral
vascular resistance, and inotropic agents will increase heart rate. These changes will result in a more evenly distributed blood volume within the circulatory
system. Common medications used are dopamine (inotropic), vasopressin, norepinephrine, and atropine. Above T6 consider norepinephrine or dopamine
and below T6 give phenylphrine, but some consider also norepinepherine as its effects better than phenylphrine. Profound bradycardia atropine 0.5-1mg
IV can be repeated after 5 minutes with maximum 2mg IV in one hour.
39. A post RTA patient has neurogenic shock post spinal cord trauma presented with paraplegia, what is indicating a neurogenic shock?
A. Bradycardia (with bladder and bowel incontinence) B. Hypotension C. Tachycardia D. Trauma to spinal cord
40. RTA with query cervical spine fractures and spinal cord injury he was thrown away; he can’t move his lower limbs which are warm,
what type of shock?
A. Hypovolaemic B. Cardiogenic C. Neurogenic D. Obstructive
41. What is the best indicator of adequate resuscitation in septic shock?
A. Normal blood pressure B. Normal pulse C. Adequate urine output D. Improved mental status E. Decreased lactate level
42. A 25-year-old driver who sustained a car accident came to ER with flaccid paralysis, bradycardia and hypotension, what is the most
likely diagnosis?
A. Neurogenic shock B. Cardiogenic C. Hypovolaemic
43. A patient with neurogenic shock (warm, pink extremities) with hypotension and bradycardia (HR: 45), he received 1 L of crystalloid
and did not improve, what is the most appropriate management?
A. I.V Crystalloid fluid infusion B. I.V mannitol C. Blood transfusion
D. Vasopressors and inotrpes (Dopamine, vasopressin, norepinephrine/atropine)
The first line of treatment of hypotension to give 2 litres of IV fluid resuscitation that would compensate for vasogenic dilatation if hypotension not improved
after IVF despite of euvolaemia must give vasopressors and inotropes as second line. The primary management of neurogenic shock is by giving vasopressors
44. What is the correct definition of the mean arterial pressure?
A. Diastolic pressure + 1/2 pulse pressure B. Systolic pressure + pulse pressure C. Systolic pressure + 1/3 pulse pressure
D. Diastolic pressure + 1/3 pulse pressure
MAP = DP + 1/3(SP – DP) or MAP = DP + 1/3 (PP) It is expressed: MAP=DBP+(0.412\times PP). Pulse pressure PP= systolic BP - diastolic BP
45. Scenario of shock after chest stab wound admitted in hospital after few days' saturation↓. HR↑ what is most likely diagnosis?
A. Hypovolaemic shock B. Cardiogenic shock C. Septic shock
46. What is the type of shock that needs vasopressin or vasopressor?
A. Hypovolaemic B. Cardiogenic C. Distributive shock N.B. Distributive shock = septic, neurogenic and anaphylactic
47. Stab chest, mild amount of pleural effusion, was stable, admitted for observation, after 4 days of admission, the patient developed
severe hypotension, high grade fever, and disturbed conscious levels, intact airway, normal breath sounds, the wound in the chest was
dirty and smelly, what is the possible cause of that?
A. Massive haemothorax B. Haemorrhagic shock C. Septic shock D. Missed intra-abdominal injury
48. An old age patient has deep wound in the hand, not properly treated; doctor prescribed antibiotic for him, patient received it only 2-3
days then stopped by his own for one week, then he came to ER presented by inflamed hand inflamed (red, tender, hot and swollen), fever,
hypotension, BP 80\50 not responding to IV fluids and leucocytosis what’s likely diagnosis?
A. Tetanus infection B. Septic shock C. Wound infection D. Severe sepsis
Incubation period of Tetanus about 10 days+ rigidity and spasticity of jaw muscles also chest, neck and back with fever, headache and irritability
49. Which of the following is true regarding septic shock?
A. It is characterised by poor perfusion of end organs. B. Maintaining haemoglobin level greater than 10 g/dL is recommended.
C. In early septic shock, whole body oxygen consumption is decreased. D. Positive fluid balance is associated with increased mortality.
E. The liver can serve as a continued source of inflammatory products.
Intravascular hypovolaemia is typical and may be severe in sepsis. Rapid, large volume infusions of IVF (30 mL/kg) are indicated as initial therapy for
severe sepsis or septic shock, unless there is convincing evidence of significant pulmonary oedema, the mortality can increase in high volume in patients
with cardiorespiratory problems as CHF or pulmonary oedema.
50. A case with parameters of septic shock, which of these means septic shock?
A. Sepsis + organ dysfunction B. SIRS + infection C. Severe sepsis + hypotension D. Pulse > 90 + WBCs > 12000
Simply to differentiate between severe sepsis ans sepsis see if MAP is > 65=severe sepsis if MAP < 65 = septic shock and the main goal to increase it >65
51. A patient with septic shock, which HB value considered for blood transfusion?
A. 6-7gm/dL (in septic and critically ill) B. 7-8gm/dL (in non-septic) C. 8-9 gm/dL D. 10 gm/dL
Based on clinical experience, randomised studies, and guidelines on transfusion of blood products in critically ill patients, it is typically reserved red blood
cell transfusion for patient with haemoglobin level < 7 g/dL. Exceptions include suspicion of concurrent haemorrhagic shock or myocardial infarction.
Stable medical and surgical patients, we recommend using a restrictive transfusion strategy (giving less blood, transfusing at a lower HB level (7 to 8 g/dL).
52. A male patient presented 10 days ago; with diarrhoea, vomiting, now he is hypotensive, altered conscious level, severe myalgia, with
skin rash, and desquamation. labs: total bilirubin 50 µmol/L, and AST 100 U/L, ALT 110 U/L, BUN 21 mmol/L CR-S 215, dark urine
what is your diagnosis? Toxic shock syndrome
Toxic shock syndrome acute-onset illness characterised by fever, hypotension (systolic blood pressure < 90 mm Hg, orthostatic drop in diastolic blood
pressure < 15 mm Hg, orthostatic syncope, and dizziness), sunburn-like rash, and end-organ damage mostly due to toxin producing- toxigenic strain of
Staph. aureus or Streptococcus pyogenes, with renal impairment creatinine > double normal, platelets < 100.000 or DIC, ALT, AST, and total bilirubin
more than double upper normal, ARDS, necrotising fasciitis.
53. What is the characteristic feature of toxic shock syndrome in children with burns?
A. Purulent wound drainage B. Leucopenia C. Hypothermia D. Bradycardia
54. A patient with tachycardia, fever, hypotension, what is the cause or pathophysiology of hypotension?
A. Low cardiac output B. Increased systemic vascular resistance C. Decreased arteriolar resistance
55. Eight hours after treatment for a scald injury, an infant has a temperature of 40 0C and a white blood count of 5,000/mm3. The burn
wound is clean, what is the most likely diagnosis?
A. Cytomegalovirus infection B. Clostridial wound infection C. Toxic shock syndrome
D. Pseudomonas wound infection E. Pneumocystis pneumoniae
The five categories are (1) fever; (2) rash (diffuse macular erythroderma); (3) desquamation (1-2 wk after illness onset, involving palms and soles); (4)
hypotension (systolic blood pressure < 90 mm Hg., Orthostatic drop in diastolic blood pressure < 15 mm Hg, Orthostatic syncope, and dizziness); and (5)
evidence of multisystem involvement
56. A patient with history of parietal abscess drainage, packing the wound with gauze soaked with saline, patient came with dark urine
hypotension, fever, renal impairment, rash over her abdomen, what is the most likely diagnosis?
A. Drug allergy B. Septic shock C. Toxic shock syndrome
57. What is the pathophysiology of septic shock? ↑ Increased Capillary permeability+peripheral vasodilatation>> decreased VR>> decreased COP
58. RTA, exploration done, wounds got infected and complicated with septic shock, what’s cause of poor peripheral tissue perfusion?
A. Low cardiac output (anaerobic metabolism>> lactic acidosis >> increased capillary permeability&VD >> pooling of blood peripherally>>
decreased VR & COP late in shock >> poor & ineffective perfusion) B. Increase resistance C. Ineffective peripheral tissue perfusion
D. Decrease glomerular filtration rate
59. A 65 y.o patient is intubated in the ICU because of septic shock, which of the following will best indicate adequate systemic perfusion?
A. Cardiac index B. CVP C. Mixed venous oxygen saturation D. Pulmonary capillary wedge pressure
Normal mixed venous oxygen saturation levels are 70 –75%. Levels below 50% indicate inadequate oxygen delivery and increased oxygen extraction by the
cells. This is consistent with hypovolaemic or cardiogenic shock. High mixed venous saturations (>75%) are seen in sepsis and forms of distributive shock.
60. What are the electrolyte disturbances in the late stage of septic shock?
A. Hypercalcaemia B. Hyponatraemia C. Hypokalaemia D. Hypochloraemia with alkalosis
Hyponatraemia and hyperkalaemia were apparent in the late stages of sepsis, these alterations reached statistically significant levels in the shock period.
The electrolyte derangements associated with sepsis and septic shock could not be related to energy depletion. The presence and severity of dysnatremia is
associated with poor outcome and prognosis in a graded fashion. The most dreaded complication in a patient with symptomatic hyponatraemia is acute
cerebral oedema. Hypomagnesaemia is associated with concomitant electrolyte disturbances such as hypokalemia and hypocalcaemia.
61. A patient with necrotising pancreatitis, you did necrosectomy, patient developed septic shock, not responding to fluid resuscitation,
and vasopressor were given, the most accurate indicator of improvement of patient condition? Increase of mean arterial blood pressure
62. A patient post pancreaticoduodenectomy for severe trauma, POD 5, hypotension, anuria & afebrile, what is the cause of hypotension?
A. Decrease peripheral vascular resistance B. Decrease COP (late septic) C. Decrease GFR
63. A case post pancreatic surgery developed hypotension, what is the cause? Low cardiac output due to sepsis in late stages or leak early
Chronic alcohol use is the single most common cause of chronic pancreatitis, resulting in ~40% to 70% of all cases, and increases an individual's risk of
developing pancreatic cancer. Alcohol-induced pancreatitis occurs in prolonged, chronic alcohol use. Acute pain, guarding or normal, radiates to the back.
64. A middle aged uncontrolled diabetic patient with peritonitis secondary to perforated diverticulitis, was presented by septic shock, with
leucopenia, increased urea, and creatinine with change of the level of consciousness BP is 70/40, pulse is 110, what’s first step of resuscitation?
A. Intravenous fluids B. Broad spectrum IV antibiotics C. Urgent laparotomy D. Vasopressors
The essential management of the septic patient includes early recognition, broad spectrum IV antibiotics, pressors (norepinephrine first, then vasopressin)
and fluid resuscitation. The cornerstone of initial resuscitation is the rapid restoration of perfusion and the early administration of antibiotics.
65. A post liver transplant patient presented with shock after trauma, after control of bleeding and IV fluids patient couldn’t maintain BP,
cardiac examination is normal, how to maintain BP?
A. Vasopressors (it is septic shock) B. Inotropes C. IV Methylprednisolone
HERNIAS AND ABDOMINAL WALL
1. A patient presented with pus and discharge from umbilicus, what is the best investigation? Sinogram, CT or MRI and US in children to rule
out congenital umbilical lesions as patent urachus, urachal cyst, omphalitis, and persistent omphalomesenteric duct.

2. What is the best for umbilical sinus diagnosis?


A. MRI sinogram B. Abdominal x-ray erect C. Abdominal US
3. A middle-aged male patient with an inguinal swelling passing beyond the pubic tubercle down to the base of scrotum, on examination
deep ring occlusion test is negative, which type this swelling?
A. Incomplete indirect inguinal hernia B. Complete indirect hernia C. Direct inguinal hernia D. Femoral hernia
4. A 76 years old male with reducible direct inguinal hernia and mild pain and discomfort, what’s the treatment option?
A. Open mesh repair B. Non-surgical procedure (Conservative treatment) C. Simple repair D. Lap. Repair
Not all hernias should be repaired, hernias with no or minimal symptoms can be safely observed especially direct one as it has extremely low risk of
strangulation so, minimal symptomatic or asymptomatic requires no treatment
5. Which type of hernia is above and medial to pubic tubercle?
A. Inguinal B. Direct C. Femoral D. Obturator
N.B. Above and medial = inguinal hernia. ----- Below and lateral = Femoral hernia
Medial to inferior epigastric vessels = Direct inguinal hernia ------ Lateral to inferior epigastric vessels = Indirect inguinal hernia
6. What is the most common hernia in women?
A. Femoral hernia B. Obturator hernia C. Inguinal hernia D. Umbilical hernia E. Spigelian hernia
7. Male patient with an inguinal hernia, reached to the (half) of his scrotum, what is the type of that hernia?
A. Complete indirect B. Incomplete indirect C. Complete direct D. Sliding hernia
8. A man, with oblong swelling on top of scrotum increase in size with Valsalva maneuver what’s diagnosis?
A. Direct inguinal hernia B. Complete indirect inguinal hernia C. Varicocele D. Femoral hernia
9. Persistant patent processus vaginalis has an association with which of the following?
A. Direct inguinal hernia B. Indirect inguinal hernia C. Incisional hernia D. Lumbar hernia
10. A patient with inguinal swelling, hernioplasty done without opening hernial sac, what is the type of that hernia?
A. Direct B. Indirect C. Femoral D. Ventral
11. The Cremaster muscle is derived from which of the following?
A. The external oblique muscle B. The internal oblique muscle C. The transversus abdominis muscle
D. The transversalis fascia
The lateral cremaster muscle originates from the internal oblique muscle, just superior to the inguinal canal, and the middle of the inguinal ligament.
The medial cremaster muscle originates from the pubic tubercle and sometimes the lateral pubic crest. Both insert into the tunica vaginalis underneath the
testis. It is supplied by the cremasteric artery, a branch of the inferior epigastric artery, along with anastomotic flow from the other arteries supplying the
scrotum. It is innervated from the sensory and motor fibers of the genitofemoral nerve that originates from the L1, L2 spinal nerve nuclei.
12. An Indirect inguinoscrotal hernia what is specific sign for differentiating it from other inguinoscrotal swellings?
A. Cannot palpate the vas B. +ve transillumination test C. Irreducible
The indirect hernia sac lies anterolateral to the cord structures and is visualised by dividing the cremaster muscle longitudinally. The cremaster muscle
should not be divided transversely or excised, because doing so may result in low-lying testes and dysejaculation.
13. What is the landmark to differentiate between indirect and direct hernia?
A. Scrotum B. Vas deferens C. Pubic tubercle D. Internal ring
14. Clinically suspicious inguinal hernia how to confirm the diagnosis as a hernia?
A. Clinical examination (initial) B. Abdomen CT(if MRI is not available) C. Abdominal US D. MRI (if US inconclusive)
In the majority of cases, a diagnosis of inguinal or femoral hernias can be made based upon history and physical examination sensitivity of 75 % and
specificity of 96. In the absence of suspected intra-abdominal complications, we suggest groin ultrasound as the initial diagnostic modality. Pelvic US is
noninvasive and inexpensive and overall has a high sensitivity and specificity for hernia, can increases the sensitivity of detecting an occult hernia from 80 %
with physical examination alone to 96%. US is the best initial imaging modality for identifying occult inguinal hernia in patients with suggestive symptoms but
no detectable hernia on physical examination, according to current guideline have to confirm with MRI or CT before deciding surgery. MRI can also diagnose
other conditions that could cause groin pain, such as adductor tendonitis, pubic osteitis, hip arthrosis, bursitis iliopectineal, and endometriosis. CT with
Valsalva is another option when MRI is not available (uptodate).
15. What is the medial boundary of the triangle of DOOM? Vas deferens (spermatic vessels laterally and peritoneal fold inferiorly)
The "triangle of doom" is bound laterally by the gonadal vessels, and medially by the vas deferens in the male, or the round ligament of the uterus in the
female. Within the boundaries of this area, you can find the external iliac artery and vein, and this is the significance of this triangle not like triangle of pain
in which the main caution is avoidance of nerve injuries; femoral, genitofemoral nerve and lateral femoral cutaneous nerves.
N.B. Both triangles sharing same apex but caution in DOOM mainly for vessels and triangle of pain caution for nerves its apex is called Circle of death
16. What is the posteromedial boundary of triangle of pain?
Triangle of pain is an inverted "V" shaped area with its apex at the internal inguinal ring. It is bound anteriorly by the iliopubic tract / inguinal ligament,
posteromedially by the testicular (spermatic) vessels and no defined posterolateral boundary, so it is not complete triangle
Deep to this triangle in endo abdominopelvic fascia that contain important nerves if caught or entrapped during mesh fixation will cause severe pain which
are: Lateral femoral cutaneous nerve, Femoral nerve and Genitofemoral nerve. Those three nerves can suffer damage or entrapment when performing a
laparoscopic herniorrhaphy and cause pain (hence the name of the area) as well as motor and sensory disorder, those three nerves are:
(1) Lateral femoral cutaneous nerve L2 and L3, sensory innervation to the anterior skin of the thigh
(2) Femoral nerve: ventral rami of L2, L3, and L4, give motor and sensory innervation to the anterior thigh compartment sensory branches to the hip joint
(3) Genitofemoral nerve: ventral rami of L1 and L2, this nerve divides anterior to the psoas major muscle into two branches. The genital branch of the
genitofemoral nerve enters the inguinal canal and provides sensory and motor innervation to the scrotum and cremaster muscle, as well as the labia majora
and mons pubis. The femoral branch of the genitofemoral nerve enters the "triangle of pain" region and passes inferior to the inguinal ligament to provide
sensory cutaneous innervation to the superior aspect of the thigh.
17. What is the medial boundary of space of Bogros?
The space of Bogros is located lateral to the space of Retzius and is bound anteriorly by the superficial transverse fascia, medially by the inferior epigastric
vessels, laterally by the pelvic wall, and posteriorly by the psoas muscle, the external iliac vessels and the femoral nerve.
18. Long case of umbilical hernia, firm and not reducible no distension patient is not in intestinal obstruction diagnosis?
A. Strangulated umbilical hernia content viscus B. Irreducible umbilical hernia contains omentum
C. Obstructed umbilical hernia D. Strangulated umbilical hernia with omentum
20. Hernia with crescentic shaped umbilicus is?
A. Umbilical B. Epigastric C. Spigelian hernia D. Paraumbilical hernia (if bulge lemon shape it is umbilical)
21. What is the name of sign when hernia not seen while lying but appears while standing?
A. Colle’s sign B. Grey turner sign C. Carnett’s sign
22. Inguinal hernia reaching the neck of scrotum, negative internal occlusion ring test, what is the type of hernia?
A. Direct inguinal hernia B. Femoral hernia C. Indirect inguinal hernia
23. A 40 year old patient is diagnosed as having a reducible unilateral inguinal hernia, on further examination, nothing else is detected.
What should be done?
A. Urgent repair B. Exploratory laparotomy C. Full standing up exam., valsalva’s maneuvre, CT, and for elective hernia repair.
24. A 65-year-old asthmatic man with COPD, presented with irreducible inguinal hernia, pain with skin over the hernia is red looks
inflamed what type of repair for his hernia?
A. Laparoscopic repair B. Lockwood repair C. Lichtenstein open repair D. Herniotomy E. Reduction
The rule for hernia repair is open; Lichtenstein repair especially in old if complicated by strangulation, if comorbidity as COPD and the insufflation will
increase his intra-abdominal pressure with more cardio respiratory affection. If the incarceration is of longer duration or there are signs of local
inflammation suggestive of strangulation, an open procedure is safest and most expeditious.
25. A 68- year-old man has been advised to undergo surgical treatment for a left reducible inguinal hernia, what is the treatment?
A. Open Mesh Repair B. Laparoscopic repair C. Conservative watch and wait
All adult hernia cases -> Mesh repair Hernioplasty except (PUH <2 cm and infected strangulated inguinal hernia managed by Herniorrhaphy)
Open vs laparoscopic? All cases are managed by an open approach except in case of bilateral hernia or recurrent hernia, femoral hernia or if patient
requested are managed laparoscopically. Children only herniotomy
26. Cholecystectomy 10 months ago, 2 months swelling 5 cm below xiphisternum, defect 1.5 cm in linea alba no impulse on cough, not
tender what’s the diagnosis?
A. Fatty hernia B. Port site hernia C. Seroma/Haematoma
27. After lap cholecystectomy there is an irreducible mass at epigastrium below xiphisternum, no redness, hotness or tenderness no
impulse on cough what’s that mass might be?
A. Gossypiboma B. Incisional hernia C. Fatty hernia D. Irreducible port site hernia
A gossypiboma, also called textiloma or cottonoid, refers to a foreign object, such as a mass of cotton matrix or a sponge that is left behind in a body cavity
during surgery. It is an uncommon surgical complication.
28. How to manage a port site hernia?
A. Laparotomy B. Laparoscopy C. Abdominal CT
29. Irreducible inguinal hernia, pain, no fever, type of repair?
A. Lockwood repair B. Herniotomy C. Liechtenstein repair
30. An old age patient asthmatic with acute asthmatic attack for 2 days, now he is presented with tender, irreducible inguinal hernia ttt?
A. Lap repair B. Liechtenstein open repair C. Herniotomy D. Lockwood
31. A football player patient with history of lifting heavy weight, has tenderness, abdominal irreducible swelling, with negative cough test,
what is the diagnosis and management?
A.Ventral hernia B. Inguinal hernia C. Rectus sheath haematoma (supportive if small and stable otherwise exploration)
Treatment mostly supportive with haematoma compression, ice bags, NSAIDs, and rest very rarely if expanding with instability to explore and ligate bleeder
32. Same scenario of rectus sheath haematoma, what is the cause?
A. Tear in the posterior wall B. Tear in the inferior epigastric artery
33. In a patient post mastectomy, they do for her reconstruction from rectus muscle, what is the vessels to be injured or effected?
A. Inferior epigastric a. (Already it is harvested and cut) B. Superior epigastric artery C. Internal mammary a D. Ext iliac a
34. A male patient underwent sleeve gastrectomy more than 10 yrs. ago, presented with weight loss and features of intestinal obstruction,
there is pain radiated to right thigh and knee, what to do? (Obturator hernia)?
A. Exploration B. Laparoscopy
35. A female, abdominal pain, radiate to medial aspect of the thigh till the knee, hypotension, WBCs 18, management?
A. MRI B. CT C. Laparotomy (strangulated obturator hernia)
36. Obturator hernia, after reduction of the intestinal loop, small segment was necrotic at the site of constricting ring, what type of necrosis?
A. Haemorrhagic B. Liquefactive C. Coagulative necrosis
37. What is true for Obturator hernia?
A. It is most common in older women with cachexia B. It is associated with pain in the sciatic nerve distribution
C. It is rarely strangulated D. It is repaired through a transverse incision in the upper medial thigh
Obturator hernias occur almost exclusively in older women and are more common on the right side. Obturator hernias commonly cause lower abdominal
pain. Up to 88% of patients present with symptoms of small bowel obstruction. Because the hernia orifice is small, Richter's hernia and strangulation are
common, and 50% of patients are present with bowel necrosis.
38. An 83-year-old female presents with vomiting and abdominal distension, she complains of pain in the medial aspect of the right thigh,
and a palpable lump can be felt on the right side on rectal examination. What is the appropriate action?
A. Nasogastric tube, intravenous fluids, and observation B. Flexible sigmoidoscopy and drainage of the rectal mass
C. Urgent laparotomy (Obturator H) D. Gastrografin study with small bowel follow-through E. Groin exploration and hernia repair
Mostly it is an obstructed obturator hernia which usually occurs in women and on right side so it can be palpable rectally as it is deep in the pelvis
39. A case of weight loss after bariatric surgery 10 yr. ago with significant weight loss he complained of severe abdominal pain, distension
and constipation with pain radiate to right thigh and knee, what is the proper decision?
A. Laparotomy B. Laparoscopy (Obturator hernia) C. Conservative
Given the location of this hernia deep within the pelvis, a laparoscopic approach allows for clear visualisation of the contents of the hernia and subsequent
treatment. Almost all case reports published have described a transabdominal preperitoneal (TAPP) approach.
40. Howship-Romberg sign is characteristic of which of the following?
A. Femoral hernia B. Spigelian hernia C. Obturator hernia D. Lumbar hernia E. Epigastric hernia
The Howship-Romberg sign is an indication of obturator nerve irritation resulting in inner thigh pain that may extend to the knee on internal rotation of
the hip. The Howship-Romberg sign is a test used to confirm the diagnosis of obturator hernia. A positive result occurs when pain extends down the medial
aspect of the thigh with abduction, extension, or internal rotation of the knee.
41. A child, after he played football got a sudden inguinal swelling and pain, the scrotum is normal, what is most likely diagnosis?
A. Complicated inguinal hernia B. Sports hernia C. Haematoma for conservative ttt
Conservative treatment consists of a) local rest following acute injury b) application of ice packs three to four times per day for approximately 10 minutes is
beneficial, c) ibuprofen or another non-steroidal anti-inflammatory medication
42. Same scenario, in middle aged patient, what is the most accurate diagnostic investigation?
A. Abdominal x-ray erect B. MRI C. Abdominal CT D. Abdominal US
43. A patient with a tense, tender right inguinal mass with distension, anorexia and vomiting, what is the management for his condition?
A. Lap hernia repair B. Open hernia repair C. Lockwood repair
The definitive treatment for all hernias, regardless of origin or type, is open surgical repair. Many treatment options available:
1. Observation and watchful waiting with modifying risk factors, including smoking cessation, medical optimisation (eg, diabetes), and weight loss. It can be
indicated for males with asymptomatic or minimally symptomatic inguinal hernias and pregnant patients with uncomplicated inguinal hernias. All other
patients should be considered for surgical open repair
2. Laparoscopic TAPP or TEP repair for uncomplicated unilateral primary inguinal hernias in male, we suggest a laparoscopic totally extraperitoneal;
TEP or transabdominal preperitoneal TAPP when feasible (Grade 2B). Laparoscopic repair has faster recovery, reduced pain both immediately after
surgery as inguinodynia and up to 5 years later
44. An image showing adhesions and gangrenous patch of intestine during laparoscopic exploration, how to manage?
A. Laparotomy B. Laparoscopic repair (if can manage lap otherwise convert open) C. Ileostomy
Indications of TAPP suitable for direct, indirect, femoral and recurrent hernias some are doing for irreducible & large inguinoscrotal hernia not TEP
• Bilateral hernias, recurrent hernia after open repairs, large hernia defects, and no scarring from previous lower abdominal surgery
• Hernias in obese patients. Hernias in subjects with intense physical activity (sport, strenuous working).
Contraindications
• The only absolute contraindication is patient with high anaesthesia risk, intolerance to capnoperitoneum (severe cardiorespiratory problem)
• Relative contraindications are Extensive intra-abdominal adhesion after previous surgery that require significant adhesiolysis as post radical prostatectomy,
CS, hysterectomy or midline incision surgery, strangulated or perforated hernia with intercurrent sepsis, severe ascites, recurrence after laparoscopic repair,
• TEP repair is preferred over TAPP repair as it is less invasive and avoids entry to the peritoneal cavity

45. A patient presented by tender umbilical mass with fever, redness and normally audible intestinal sounds, what is the diagnosis?
A. Strangulated omentum B. Strangulated hernia with loops C. Obstructed hernia with loops
46. A picture of huge para-umbilical hernia, and the hernia was tender, erythema upon skin, no signs of bowel obstruction, and the patient
feverish, what is the hernia content?
A. Strangulated intestine B. Incarcerated intestine C. Strangulated omentum D. Richter hernia
47. For the same patient diagnosis, how to confirm the content it is strangulated omentum not bowel?
A. US B. CT C. Intra-operative D. Clinical assessment is enough
48. What is the diagnosis of the lesion in the image (umbilical mass with discoloration, I think it is umbilical hernia)
A. Ventral hernia B. Paraumbilical hernia C. Spigelian hernia D. Inguinal hernia
49. What is a characteristic of Spigelian hernia?
A. Common in elderly(can occur rarely in infants with Cryptorchidism B. Common in females C. Occur near arcuate line
Spigelian hernias are the hernias that occur along the arcuate line., inside the Spigelian hernia belt which is the region from just below the level of the
umbilicus to the interspinal plane (a horizontal line between the right and left anterior superior iliac spine). They affect men and women equally and can
occur at any age, but are most common in elderly people, most patients are diagnosed in the fifth or sixth decade of life. These hernias are rarely described
in children. When diagnosed in the paediatric population, they may be associated with other anomalies such as undescended testes.
The diagnosis should be suspected because of the location of the symptoms and is confirmed by CT. US can be performed in the upright patient.
Surgery is recommended because the narrow and fibrous neck predisposes to strangulation. Surgery can be open or laparoscopic. At open surgery a skin
crease is made over the hernia, but no abnormality will be seen until the external oblique is opened.
50. Ventral hernia in female with +ve cough impulse, disappear after lying supine, it appears near a slit above umbilicus (slit means
incision? if means incision so will be incisional hernia)
A. Umbilical B. Paraumbilical(if meant umbilical slit) C. Epigastric D. Incisional
51. What is the best mesh for hernia repair?
A. Non-absorbable B. Delayed absorbable C. Inelastic
52. A patient with history of post open cholecystectomy 2 months ago presented by reducible swelling or bulge with palpable edge between
operation scar and swelling?
A. Dehiscence B. Seroma C. Incisional hernia D. Divarication of recti E. Port site hernia
53. A patient with midline incisional hernia of defect 5 cm, repair done by suturing mesh in the defect, what is the name?
A. Sublay (below rectus abdominis, superficial to posterior sheath) B. Onlay (above anterior rectus sheath and muscles in sc space)
C. Inlay (within the defect edges) D. Preperitoneal (below posterior sheath and above peritoneum) E. Intraperitoneal/ underlay
54. A female patient underwent lap chole 10 months back, presented with irreducible epigastric mass 2x3 cm. about 5 cm below xiphoid
process with discomfort and no impulse on cough, what is the most appropriate to do?
A. CT (to rule out port site hernia) B. US C. Laparoscopy D. Aspiration
55. A 70 y.o, smoking 2 packs per day for 50 yrs., he has nocturia, dysuria, not on treatment for that, he has an inguinal hernia, what is the
strongest risk factor for his hernia?
A. Old age B. Smoking C. BPH D. Male gender
56. A 73 year old patient, with symptomatic inguinal hernia for 10 years, he smokes 2 packs per day for 50 years, he has dysuria and
difficult micturition, residual urine 120 mL, prostate size 55 gm what is the initial optimal management?
A. Hernioplasty B. Herniotomy C. TURP then hernioplasty D. Alpha-blocker then hernioplasty
57. An old patient, with symptomatic inguinal hernia for 10 years, he smokes 2 packs per day for 40 years, dysuria and difficult micturition, what
is the appropriate investigation?
A. Trans rectal US; TRUS for prostate B. Urine culture C. PSA
BPH is extremely prevalent, affecting 20% of men aged 30 to 79, with 80% of men being affected by age 70. There are many indications for trans rectal
prostate ultrasound, including:Abnormal PSA (>4 ng/mL), Abnormality palpated on DRE or Suspected infection (prostatitis or prostatic abscess)
58. A scenario of 50 years old smoking patient, with history of Prostatism presented with hernia, what is the most important risk factor?
A. Smoking B. Prostatism C. Constipation
59. Same scenario, for previous patient as a preoperative investigation which is the best?
A. Endorectal US B. CXR and ECG
60. What is the best investigation to differentiate between inguinal hernia, femoral hernia and Saphena varix?
A. MRI B. CT C. Duplex US
61. A 50-year-old male complained of right iliac fossa dull aching pain. The exam showed that he had right iliac fossa mass with positive cough
impulse. The examining doctor found a bluish tinge on the mass surface and the percussion tab was positive, what is the most likely diagnosis?
A. Right inguinal hernia B. Right femoral hernia C. Right vaginal Hydrocele D. Cyst of Morgagni E. Saphena varix
Saphena Varix is dilated saccular varicose swelling, arises from the proximal end of the long saphenous vein. It presents as a reducible swelling in the groin
situated in the femoral canal region. A cough impulse may be elicited, and the swelling disappears when the patient lies down
62. Boundaries of Hesselbach’s triangle include all the following except?
A. Lateral border of rectus muscle B. Inferior epigastric artery C. External iliac artery D. Inguinal ligament
63. A female patient with irreducible, non-tender umbilical hernia, what is the treatment?
A. Expectant treatment B. Elective repair
64. A case with irreducible painful tender mass vitally stable, otherwise, exam. is normal except tender periumbilical lump what’s next?
A. Observation B. Urgent surgery C. Abdominal CT with oral contrast (to detect if strangulated or gangrenous contents) D. US
65. A 45 year old patient, came with history reducible hernia in inguinal area it pop out every two days with mild pain and he can manual
reducible it by his finger, then he suddenly developed severe constant pain in early morning in that area and came to hospital, on exam. of
the inguinal area and scrotum examination and he is free from both them and pain is disappear, Now just have persistent nausea and
vomiting, what is the next step ?
A. Pelvic and Abdomen CT B. Hernia repair today after 2-hour C. Reassurance and discharge D. Hernia repair after 2 days
For patients who present with nausea, vomiting, and abdominal distention associated with a history of groin pain or mass, bowel obstruction due to bowel
incarceration or strangulation should be suspected. For patients with clinical features of bowel obstruction CT is generally more useful than ultrasound.
66. During TEP, what is the commonest complication during dissection of peritoneum plane?
A. Urinary bladder injury B. Inferior epigastric artery injury C. Internal iliac vessel injury D. Peritoneal penetration
67. A patient underwent hernia repair with mesh, one month ago now he came complaining of pain extending to the thigh associated with
numbness and paresthesia, what to do?
A. Remove the mesh B. Remove mesh with neurectomy C. Nerve block (if persists for >3 months) D. NSAIDs (2-4 wks post op.)
Depends on the time of the presentation; If the patient present within 2-4 weeks post-surgery: NSAID, if persists for 3 months, this indicates neuralgia, and
treated by nerve block
68. A 70 y.o patient with recurrent inguino scrotal hernia, only irreducible, not obstructed or strangulated with history of retropubic
prostatectomy, what is the best treatment?
A. Lap TAPP safe and feasible, less recurrence and less pain B. Open hernioplasty C. Lap TEP
69. A patient with previous bilateral hernia repair, the best procedure for management recurrent hernia? Laparoscopic repair with TAPP
70. A patient for elective inguinal hernia repair post recurrence after open repair, what is the best operation?
A. TAPP B. TEP C. Open
71. A 30-year-old patient with total extraperitoneal repair of hernia, the pressure must be?
A. More than 10 mmHg B. 11-15 mmHg C. 16-20 mmHg.
The insufflation tubing is attached to the Hasson's Cannula and insufflation is begun with pressure setting at 12 mmHg.
72. An adult patient with right sided big inguinoscrotal swelling, signed consent for laparoscopic hernia repair how much optimal
insufflation pressure during his laparoscopic repair?
A. Less than 10 B. More than 15mmHg. C. From 12 - 15mmHg. D. Less than 8mmHg.
73. During obstructed umbilical hernia repair, toxic fluid came out during dissection, what type of repair you should do?
A. Mayo repair B. Anatomical repair C. Repair with mesh (if no bowel opening, spillage or strangulation) D. Tension suture
The use of polypropylene mesh in acute incarcerated and /or strangulated hernia cases was safe and advisable. Saline 0.9% can be used in irrigation after
hernia reduction and even some content excision especially in the presence of toxic fluid or haematoma in the hernia sac. There are some studies detected
safely use of prosthetic meshes in strangulated acute hernias with acceptable surgical wound infection (Topcu et al., 2013 and Pandey et al., 2018).
2017 update of the WSES guidelines for emergency repair of complicated abdominal wall hernias summarised that:
1- For patients with intestinal incarceration and no signs of intestinal strangulation or concurrent bowel resection, the surgical field is presumed clean and
the infectious risk for synthetic mesh is low. The absence of intestinal wall ischaemia makes patients less prone to bacterial translocation
2- Prosthetic repair with a synthetic mesh is recommended for patients with intestinal incarceration and no signs of intestinal strangulation or concurrent
bowel resection (clean surgical field) (grade 1A recommendation).
3- For patients having a complicated hernia with intestinal strangulation and/or concomitant need of bowel resection without gross enteric spillage (clean
contaminated surgical field, CDC wound class II), emergent prosthetic repair with a synthetic mesh can be performed (without any increase in 30-day
wound-related morbidity) and is associated with a significant lower risk of recurrence, regardless the size of hernia defect (grade 1A recommendation). In
the cases of incarcerated or strangulated hernia repair, the infection rate may reach over 10%.
74. You have one patient who has a PUH small defect 2 cm, what is the best option for repair technique?
A. Local tissue repair B. Primary repair C. Lap repair D. Mesh repair
1. Small hernia defects of less than 1 cm, for primary sutured repair without mesh, no good evidence for mesh hernia repair if less than 1 cm.
Open repair if less than 2 cm. Laparoscopic mesh repair should be considered for defects between 2 and 6 cm.
2. There is high level evidence for mesh hernia repair of diameter 1-4 cm, for prevention of recurrence and complications.
3. Mesh versus suture repair - Small (<1 cm) primary ventral (eg, mostly umbilical, some epigastric) hernias can be repaired with sutures with or without
mesh reinforcement. More than 2 cm mesh repair is highly recommended. In a systematic review and meta-analysis of 350,000 ventral hernia repairs (75 %
umbilical or epigastric hernias without specifying sizes), the recurrence rates were 8.2 % after suture repair and 2.7 % after mesh repair.
4. Despite the strong evidence supporting mesh repair of primary ventral hernias, many surgeons believe simple suture repair still be appropriate for the
repair of small primary ventral hernias (<1 cm), provided there is no residual tension. The most common symptom of umbilical hernias is pain at the
umbilicus (44% of cases), pressure (20%) and nausea and vomiting (9%). Complications such as irreducibility, obstruction, strangulation, skin ulceration,
and rupture are more common in paraumbilical hernias than in other abdominal hernias Indications for operative repair are pain, incarceration,
strangulation, defect larger than 1 cm, skin ulceration, hernia rupture.
75. During right inguinal hernia repair, you find a compressible tubular structure 5x1.5 cm, in the hernial sac, mostly was Meckel’s
diverticulum, 5 cm x base 1.5 cm, how to manage or what is the best treatment of Littre’s hernia?
A. Diverticulectomy only B. Hernia repair with mesh C. Hernia repair without mesh
D. Resection anastomosis with hernia repair E. Diverticulectomy plus hernia repair without mesh (herniorrhaphy)
76. In a patient with incisional hernia, during adhesiolysis perforation of transverse colon resulted with gross faecal contamination,
primary closure was done, what is next best?
A. Repair and mesh C. Delayed hernia repair (after 4-7 days IV antibiotics) B. Closure in mass with tension sutures
77. During incisional hernia, injury to colon, with evident gross spillage, colon then repaired primarily, what to do with the hernia?
A. Primary closure B. Mesh repair C. Tension suture D. Delay > 4 days, then proceed to mesh repair
78. On exploration for incarcerated hernia you found suspicious loop, which is dark violet or light pink colour, what is the best recent
method to confirm bowel viability?
A. Serosal lusterness (first initial clinically) B. Contraction on percussion C. Pulsatile loop D. Bleeding if pricked
E. Laser Doppler flowmetry with Doppler US and fluorescein (the best and most sure)
Currently, flowmetry with Laser Doppler ultrasonography and fluorescein dye are the preferred methods employed in the operating room, with fluorescein
flowmetry viewed as the gold standard due to efficacy and ease of use. Intestinal microcirculation and viability is usually estimated from the color of the
serosal surface, presence of bowel peristalsis, pulsation and bleeding from the marginal arteries. This is subjective and based on the surgeon’s experience.
Visible light spectrophotometry and Laser Doppler ultrasonography have been the most frequently used in humans over the last decade
79. During exploration for incarcerated paraumbilical hernia with dusky skin colour over it, the surgeon found a pert of the loop looks
necrosed, after 10 minutes hot fomentation, and increased oxygen flow with pinching it no change, what is the surest sign of necrosis?
A. Dusky coloured loop B. Not bleeding on pricking C. Absent arterial pulsation D. Not shiny serosa
80. A patient with 10-year history of reducible PUH, become irreducible for 8 hours, during surgery the sac contained serosanguinous
fluid, the loop is purple, what is the management?
A. Warm packs for 10 minutes B. Resection anastomosis C. Wedge resection D. Reduce the loop into the abdomen
If in doubt, the bowel should be wrapped in hot packs for 10 minutes and then reassessed. The state of the mesenteric vessels and pulsation in adjacent
arcades should be sought. Viability is also confirmed by colour, sheen and peristalsis. If, at the end of this period, there is still uncertainty about bowel
viability, it should be resected unless there is concern that the extent of resection may lead to short bowel syndrome.
81. An image taken during laparotomy surgery of intestinal loop with constriction ring showing strangulation, dusky red or purple and
necrotic patch, what is the initial pathophysiology or cause of that condition?
A. Venous congestion/ obstruction B. Arterial occlusion C. Part of inflammatory process D. Iatrogenic during dissection
82. A patient presented with incarcerated / or obstructed hernia, with vomiting and absolute constipation, with erect abdominal x-ray showing
dilated small bowel loop, what if you found in the erect x-ray indicates urgent laparotomy without delay?
A. Air under diaphragm (indicate ischaemia & perforation) B. Dilated bowel C. Air fluid level D. Valvulae conniventes
83. A patient with history of inguinal or PUH swelling, presented with mildly tender irreducible inguinal swelling, abdominal distension,
vomiting and constipation for 1-day, no fever, diagnosis?
A. Irreducible hernia B. Incarcerated hernia C. Obstructed hernia D. Strangulated hernia
84. A history of long-standing reducible hernia, the hernia become irreducible 1 week ago, no pain, with no signs of obstruction, the hernia
is soft and not tender what is the complication?
A. Irreducible hernia B. Strangulated hernia C. Incarcerated hernia
85. An inguinal swelling was reducible now became irreducible with soft and lobulated content what’s diagnosis?
A. Obstructed hernia B. Incarcerated hernia C. Irreducible hernia Strangulated hernia E. Irreducible omental contents
86. A patient with inguino-scrotal hernia presented with abdominal distention, vomiting and severe pain, not tender most likely diagnosis?
A. Obstructed B. Strangulated C. Incarcerated D. Irreducible E. Inflamed
87. A case of irreducible inguinal hernia with sluggishly audible bowel sounds, what the best tool to confirm if obstructed or not?
A. Erect abdominal x-ray B. Abdominal US C. Abdomen CECT D. Abdomen MRI E. Clinical evaluation
Abdominal and/or pelvic computed tomography is the preferred imaging study to diagnose all abdominal wall hernias. can help detect many elusive
hernias by demonstrating extracoelomic location of the bowel, bladder, or female internal reproductive organs. CT may be indicated in the diagnosis of a
spigelian or obturator hernia and in cases where the patient’s body habitus makes it difficult for the physician to perform a good physical examination.
88. A case of paraumbilical hernia with only tender swelling with no signs of obstruction and the abdomen is soft and lax what is that
hernia complication?
A. Obstructed hernia B. Incarcerated bowel content C. Incarcerated omentum
D. Strangulated hernia E. Irreducible hernia
89. An image with umbilical mass, was dusky red, painful, no fever or other complaints what about this mass is it?
A. Irreducible UH B. Strangulated UH (if there is fever and leucocytosis) C. Obstructed UH D. Incarcerated UH
90. A 41-year-old female with known femoral hernia scheduled for operation came to ER with severe pain over hernia and fever, on
examination, hernia was tense, tender and negative impulse on cough, what is the most likely diagnosis?
A. Inflamed hernia B. Strangulated C. Obstructed D. Incarcerated.
91. A female patient, with abdominal and groin pain referred to thigh, signs, symptoms of obstruction? Strangulated femoral hernia
92. A picture of umbilical hernia, on auscultation audible intestinal sounds, not reducible, what’s hernia type?
A. Strangulated umbilical hernia B. Obstructed umbilical hernia C. Irreducible umbilical hernia
93. What is the most common hernia vulnerable for strangulation?
A. Femoral B. Umbilical C. Inguinal D. Epigastric E. Paraumbilical F. Spigelian G. Incisional
94. A 65 year old female, with asymptomatic femoral hernia, generally fit, has no medical diseases, what’s the most appropriate manag?
A. Observation B. Open repair with mesh C. Laparoscopic repair (femoral, bilateral, recurrence after open)
Asymptomatic non-complicated femoral hernia in good health and fit patient without comorbidities can undergoes laparoscopic hernia repair.
Laparoscopic hernia repair if no contraindications is indicated for patients with; Recurrent or Bilateral inguinal hernia, femoral hernia
95. An adult soldier with bilateral inguinal swelling with positive cough impulse what is the most appropriate management?
A. Laparoscopic repair B. Herniotomy C. Open hernia repair and mesh D. Observation
Laparoscopic repair for bilateral, or recurrent after open repair, or femoral provided no contraindications as lower abdominal surgeries like previous lap repair,
prostatectomy, hysterectomy, C. Section, ascites, incarcerated, strangulated or large inguinoscrotal hernia, patient cannot tolerate GA, all of those for open repair
96. A recurrent and irreducible inguinoscrotal hernia, not obstructed, not strangulated with history of prostatectomy, what’s management?
A. Open and repair with mesh B. Laparoscopic repair
97. A 30 year old male had open hernia repair few years ago, presenting now with reducible inguinal hernia on one side extending to
hemiscrotum, what is the appropriate thing to now?
A. Laparoscopic repair with mesh (lap for recurrence after open and vice versa) B. Open repair C. Pelvi abdominal CT D. US
98. Richter hernia is most common in which of the followings?
A. Inguinal hernia B. Femoral hernia C. Umbilical hernia D. Epigastric hernia
99. A 45-year-old lady with groin hernia complicated by intestinal obstruction with long history of reduction but shortly became tender
irreducible, inflamed she passed some stool while she was in ER and part of small bowel remains, and difficult to palpate! type of hernia?
A. Richter hernia B. Amyand hernia C. Littre’s hernia D. Obturator hernia
Richter hernia is a herniation of the anti-mesenteric portion of the bowel through a fascial defect, typically in elderly 60 - 80 years old, females more as the
femoral ring is the most common site for a Richter hernia and femoral hernia occur more commonly in females than males. The most common location for
this pathology to occur is in the femoral canal (36 - 88%), followed by the inguinal canal (12 - 36%) and abdominal wall incisional hernias (4 - 25%).
N.B Richter hernia= femoral with circumference part of small bowel. Amyand hernia = hernia with appendix. Littre’s hernia = Meckle’s diverticulum in a
hernia inguinal (50%), umbilical (20%), and femoral (20%). Maydl's hernia = M or W hernia 2 adjacent loops mostly closed loop, incarcerated can be
necrotic. Spigelian hernia is a rare, abnormal protrusion of abdominal contents or peritoneum through a defect (spigelian fascia), which is comprised of the
transversus abdominis and the internal oblique aponeuroses. Obturator hernia occurs when abdominal contents traverse the obturator foramen
100. For femoral hernia, what’s the best treatment?
A. Lockwood procedure (low, femoral approach) B. Lichtenstein procedure C. Laparoscopic surgery D. Herniotomy
All cases are managed by open repair except in case of bilateral hernia, recurrent hernia, femoral or patient request are managed laparoscopically
101. What is the medial boundary of the femoral canal?
A. Femoral vein B. Femoral artery C. Lacunar ligament D. Inguinal ligament E. Femoral nerve
102. What is the lateral boundary of a femoral hernia?
A. The femoral nerve B. The femoral artery C. The femoral vein D. The lacunar ligament
103. A 40-year man with a 2-year history of abdominal hernia presented with symptoms of bowel obstruction. The skin over hernia is red
in colour, it is tense and tender, what complication has the patient developed?
A. Strangulation (tense, tender, fever or leucocytosis) B. Incarceration (not tense, or tender no fever or leucocytosis) C. Obstruction
104. An image of umbilical swelling then operative image showing intestinal loop, what is the sure sign of intestinal necrosis in that loop?
A. Absent arterial pulse B. Increased serosal lusterness (for viability) C. Increased peristalsis D. Venous congestion.
105. A case of strangulated inguinal hernia, intraoperatively you found a changed loop colour how to deal with query ischaemic loop?
A. Hot fomentation (10 minutes and 100% oxygen) B. Resection and ileostomy C. Resection anastomosis
106. An irreducible umbilical hernia, surgery started, operative findings, loop of bowel was dark blue congested, and surrounded by
hyperaemia, what is the best initial action?
A. Resection and Anastomosis B. Limited resection
C. Hot Fomentation and wait for 10 minutes then if returned to normal color nothing to do, or resection if not
107. Epigastric pain or periumbilical pain referred to right iliac fossa + history of inguinal hernia, by examination there is irreducible
mass in the right iliac fossa, Diagnosis?
A. Maydl’s hernia = W hernia B. Amyand’s hernia (appendix in hernial sac) C. Richter’s hernia
N.B it is a case of acute appendicitis, so treatment is appendectomy through a herniotomy hernia repair without mesh
108. Which hernia that is never completely reduced?
A. Epigastric hernia B. Littre’s hernia C. Sliding inguinal hernia D. Pantaloon hernia
109. What is the most common early complication following hernia repair? Urinary retention
110. A female with abdominal wall hernia after caesarian section planned for laparoscopic hernia repair, most common complication?
A. Pain (by tackers or N. entrapment) B. Infection (rare with lap) C. Adhesion (not specific for repair) D. Cosmetic
111. A 30-year-old man was admitted as an emergency with a 24 hr. history of a sudden onset of painful irreducible right inguinal swelling
following a meal. The groin swelling had been present for over a year associated with intermittent pain but had suddenly becomes bigger.
The pain at presentation was associated with vomiting, progressive abdominal bloating, and constipation with inaudible bowel sounds. On
examination he was distressed with a BP 100/60 mmHg, pulse 90/min, respiratory rate 28/min, and a temperature of 37.9°C. There was
abdominal tenderness with tense, and irreducible groin swelling of > 5 cm in diameter, after abdominal CT two bowel loops found in a
narrow defect, the diagnosis of a strangulated inguinal hernia was made, what is the best option to do for him?
A. Have to reduce hernia B. Observe, IVF and antibiotics C. Urgent exploration
D. Give painkillers and laxative E. Do laparoscopic appendectomy
It is mostly a case of Maydl’s hernia which is a rare type of incarcerated hernia with presence of two small bowel loops within a single hernial sac with tight
neck, that is, there are two efferent and two afferent loops of bowel, forming a "W" shape. It is more prone to strangulation and necrosis as the intervening
bowel loops are deprived from blood supply and is also at risk of a closed loop obstruction and or strangulation, so for urgent exploration
112. A female patient with 4 CS, she has big intra-abdominal mass, post laparoscopic most common complication? (2 pictures abdominal
mass bulged, and flat abdomen)
A. Infection B. Wound failure (recurrence late) C. Adhesion D. Bowel and vascular injury (early 0 to 14%)
The most feared but rare complications after laparoscopic hernia repair are bowel lesions, that are more often connected to laparoscopic than to open
repair and are associated with marked morbidity and mortality. Less than 1 % of patients develop mesh infection following laparoscopic ventral hernia
repair. Hernia recurrence from 1.5 to 2.5%. Postoperative pain after laparoscopic ventral hernia repair is common but mostly resolves by four to 6 weeks.
The incidence of persistent pain beyond six weeks ranges from 1.6 to 28%
113. A patient underwent a hernia repair 12 years ago, now presents with signs of intestinal obstruction. Which of the following is the
most likely cause?
A. Improper Repair B. Late Onset Crohn's C. Adhesions D. The cause is irrelevant to the prior surgery
114. A female with 4 times CS presented with incisional hernia, what is the most appropriate factor for management plan?
A. Size of the defect B. Location C. Content of the hernia
115. Same scenario, what is the most appropriate investigation for incisional hernia?
A. US B. Laparoscopy C. CT
116. An incisional hernia in multigravida patient, what’s the most common complication post trans peritoneal repair?
A. Infection B. Adhesion C. Recurrence(late) D. Perforation (early intraoperative) E. Pain
117. Two images of a female patient with a large lower abdominal incisional hernia that appears on standing, the patient had 4 times CS,
hernia was repaired using the laparoscope, the most common late complication?
A. Adhesions B. Fistula(very rare over 20y.) C. Mesh migration(rare) D. Wound failure (due to weaknes of the abdominal wall)
118. An inguinal hernia repaired, but after 10 years the hernia occurred again in same site, what is the type of that hernia?
A. Recurrent (1-5 years) B. Incisional (in 3-6 months) C. Inguinal
119. What is the time that hernias usually recur? Usually within the first 2 years after repair
120. Which type of hernia is found above umbilicus and you can feel the linea alba between the swelling and the umbilicus?
A. Umbilical B. Para umbilical C. Epigastric (fatty hernia of linea alba)
121. What is the most common cause of recurrent epigastric hernia?
A. Using absorbable sutures in the repair B. Sport C. Missed defect D. Increased intraabdominal pressure
122. Collection after lap-chole due to mostly cystic duct leakage with no fever or CBD dilatation, how to do next?
A. ERCP (best) B. US aspiration (initial)
123. A female patient presented with abdominal swelling 10x10 cm post umbilical hernia repair, afebrile, not expansile on cough, diagnosis?
Post op seroma
124. A post lap chole patient 10 days ago came now for clip removal you foud in the epigastric port site soft compressible swelling but
negative impulse on cough, no severe pain, no fever or leucocytosis, what is most likely diagnosis?
A. Absecss B. Portsite hernia C. Seroma(due to extensive dissection,big dead space; 7-10 days post op.) D. Gossypiboma
Seromas can form in any surgical wound where extensive soft tissue dissection and dead space creation occur. Patients often present 7 to 10 days following
wound closure or wound drain removal with a fluctuant collection near the operative site
125. A patient underwent repair of incisional hernia, after 14 days she presented with mass 10 x 10 cm not reducible, not tender, all
parameters are normal, (mostly Seroma) what is next?
A. Antibiotics B. Hot fomentation C. US guided Aspiration D. Laparoscopy E. CT guided Aspiration
Seromas that are minimal in volume and painless, no functional deficit, or strain on the wound closure can be managed conservatively with observation. In
most cases, these will reabsorb without the need for further intervention. US aspiration for big sized, symptomatising, failed conservative or if infected.
126. A scenario of one week post-operative Seroma, no fever or leucocytosis but after one month of conservative treatment and
compressive dressing still same size with discomfort and pain , what is the best option in management? Percutaneous drainage if big,
symptomatic, infected or not resolved conservatively
A. Conservative B. US guided aspiration C. Antibiotics IV then oral(if inflamed) D. Explore with drainage (if infected & big)
Large volume seromas > 75 to 100 sympatomatic as pain, infection, and reduced function of the associated area, such as shoulder movement in axillary
seromas can be aspirated. If recollection occurs, multiple aspirations may be required. US-guided aspiration is indicated in patients with periprosthetic
seroma, such as in patients with breast implants, due to the risk of rupture. Open surgical drainage only for chronic encapsulated or infected pseudocyst
Contraindications to seroma are small volume seromas not causing pain, functional deficit, or compromise to wound closure and in presence of a pulsatile mass
127. After how long the patient post big ventral hernia repair can return to labor work in which he is carrying and lifting heavy objects?
A. Post laparoscopic one week B. Post open two weeks C. After 4-6 and up to 8 weeks D. Maximally up to 3 months
In big hernia with open methods, strict bed rest for 3 weeks followed by convalescence of 9 weeks can be advised to decrease the recurrence rates. Most
patients can return work 2 weeks post surgery, if minimal lifting work for 2-3 weeks but, if job require heavy lifting or sternuous activity, up to 6 weeks.
FLUIDS AND ELECTROLYTES
1. How much IV fluid resuscitation maintenance for a 70 kg weight person? Adult 1st. 10=100, next 10-20 =50 , after 50 kg wt=20ml= 104(110)
A. 20 ml/kg B. 50 ml/kg C. 100 -110 ml/kg (4/2/1 rule) D. 130-150 ml/kg
2. How much IV fluid resuscitation as a bolus for 70 kg weight person? Bolus 500 ml in < 15 min. or 20ml/kg as a bolus with Na 130–154 mmol/L
A. 20 ml/kg B. 50 ml/kg C. 100 ml/kg
3. What is the most common complication after massive blood transfusion?
A. Coagulopathy hypocalcaemia hyperkalaemia hyperthermia B. Coagulopathy hypocalcaemia hyperkalaemia hyperthermia
C. Coagulopathy hypocalcaemia hyperkalaemia hypothermia
4. What is the most frequent major complication of blood transfusion? Major = serious. common as febrile nonhemolytic reactions and Chill-rigor reactions
A. Volume overload B. Haemolytic reaction(the most serious) C. HIV virus infection D. Hepatitis C
5. A patient with history of 15 years duration of duodenal ulcer presented with frequent vomiting, what is the best I.V. fluid?
A. Ringer lactate (it cause metabolic alkalosis) B. Normal saline C. 3% saline D. Hetastarch
6. A case of GOO, young man with ulcer healed with stricture vomiting frequently pH. 7.5, what is the best fluid replacement?
A. Ringer lactate (not allowed can lead to metabolic alkalosis and aggravate the condition) B. Normal saline
7. What is the best fluid replacement in case of gastric outlet obstruction?
A. Normal saline B. Ringer C. Glucose
8. A case scenario of post intestinal resection anastomosis leak, fever, tachycardia what’s IV fluid to be given?
A. Ringer's Lactate (30mL/kg max 5 liters/24 hours, vasopressors within 3 hr) B. NS C. D5 1/2NS D. Dextrose 10%
9. Severe diarrhoea in a patient with disturbed conscious level, Na 162, K 4.9, what is the best fluid replacement?
A. Saline B. Ringer C. D5 dextrose 5% (in 3-6 ml/kg/hr. up to maximum 666 ml/hr + losses, till Na <145 then 1ml/kg/hr) D. DNS
10. A patient underwent closure or reversal of ileostomy, no oral intake, which fluid can be used for maintenance?
A. Dextrose saline B. Saline C. Ringer D. TPN E. Hetastarch
11. A patient prepared for stoma closure, (reversal) what is the proper IV fluid, during fasting time? HOF=high output fistula
A. Ringer Lactate (trauma, ileostomy diarrhoea, post op, pancreatitis, burn, blast) B. Dextrose saline C.Normal saline (GOO, CHPS, ileostomy, HOF)
D. Dextrose (diarrhoea+hypernatraemia) E. Salt free albumin (in ascites of liver cirrhosis)
12. What is the best replacement fluid in case of Ileostomy? Normal saline or D5, 0.45 NS
13. A child underwent ileostomy and now has ileostomy diarrhoea, what is the proper or best fluid to give him?
A. Ringer lactate B. Saline C. Hartmann’s D. Dextrose
14. What is the fluid maintenance for Ileostomy 3 months after sigmoid anastomosis, with normal electrolytes?
A. Normal saline B. DNS C. Ringer
15. What is the evident sign of chronic dehydration?
A. Muscle wasting (only muscle cramps and weakness) B. Sunken eyes
16. A paediatric patient came to ER with projectile vomiting, what is the initial management to give this case?
A. Normal saline B. Ringer’s lactate C. D5NS D. 1/2 NS
17. A 19 year old boy after eating outdoor meal complained of about 10 times vomiting no bleeding labs, some dehydration and decrease
K, otherwise he is well how to resuscitate him?
A. K+ fluid B. Normal saline until urine output become > 40 ml/h then give K
18. Normal anion gap acidosis is associated with which of the following?Anion gap = Na- (Cl+HCO3) high by MUD PILES mentho, uremia, lactic Acidosis
A. Ketoacidosis B. Lactic acidosis C. Salicylate poisoning D. Severe diarrhoea E. Uraemic acidosis
19. A non-anion gap hyperchloraemic metabolic acidosis is associated with which of the following?
A. Methane intoxication B. Large amount of saline resuscitation C. Diabetic ketoacidosis D. Cardiogenic shock
20. A non-anion gap metabolic acidosis is associated with which of the following?
A. Diabetic ketoacidosis B. Haemorrhagic shock C. Excessive sodium chloride administration E. Ethylene glycol Ingestion
21. What is correct regarding electrolyte changes in gastric outlet obstruction among the following?
A. Hypokalaemia B. Hyperkalaemia C. Hyponatraemia D. Hypernatraemia E. Hypercalcaemia
22. A case of gastric outlet obstruction, + ve succussion splash what is the electrolyte abnormality?
A. Metabolic alkalosis B. Hypochloraemic metabolic alkalosis C. Hypokalaemia D. Hypochloraemic metabolic acidosis
23. A child with picture of gastric outlet obstruction, where NGT tube inserted, with output more than 1.5 L/day, low K, low Na, low Cl,
pH 7.50, normal urineoutput with rehydration, what’s most likely electrolyte disturbance?
A. Metabolic acidosis B. Metabolic alkalosis C. Hypochloraemic alkalosis D. Hypokalaemic metabolic acidosis
24. An old aged male with chronic peptic ulcer he has upper abdominal distension with succussion splash, with high pH, low, chloride,
low, sodium, low calcium, what is the abnormal electrolyte disturbance?
A. Hypokalaemic metabolic alkalosis with alkaluria B. Hypochloraemic metabolic acidosis with aciduria
C. Hypokalaemic metabolic alkalosis with paradoxical aciduria D. Hyponatremic, hyperkalaemic metabolic alkalosis with aciduria
25. What is the electrolyte imbalance case of GOO?
A. Hypokalaemia, hyponatraemia, metabolic alkalosis with paradoxical aciduria
B. Hypokalaemia, hypernatraemia metabolic alkalosis C. Metabolic acidosis
26. In case of GOO, there is an association with hypokalaemic metabolic alkalosis, what is the hypokalaemic changes in ECG?
A. Prolonged QT segment B. Prominent U wave C. Widened QRS complex
27. Which of the following is an ECG feature of gastric outlet obstruction, with average urine output and normal kidney function?
A. Tented T wave B. Depressed ST segment C. Prominent P wave D. Sinus tachycardia
28. Appearance of a U-wave on electrocardiogram occurs in which of the following electrolyte disturbances?
A. Hyperkalaemia B. Hypokalaemia C. Hypermagnesaemia D. Hypomagnesaemia E. Hypercalcaemia
29. Which is early ECG changes in a case with non-bilious vomiting, due to gastric outlet obstruction; GOO with hypokalaemia?
A. Depressed ST segment (+ prolonged PR, prom U, flat T, VF, V tack) B. Elevated T wave C. P wave prominent D. T wave sagging
30. A patient with long time chronic vomiting, what are the changes in his ECG?
A. Peaked T wave B. Prolonged QT interval C. ST segment depression (due to hypokalaemia) D. Sinus rhythm
31. You examined a middle-aged female with history of arthritis with long use of NSAIDs with two months history of severe epigastric
pain relieved with high dose PPI with one-time haematemesis and frequent vomiting, on visiting emergency room you found a succussion
splash on abdominal examination what is the suspected electrolyte imbalance in this patient.
A. Hyperkalaemia, hypernatraemia, metabolic acidosis B. Hypokalaemia, hyperchloraemia, metabolic alkalosis
C. Hypokalaemic, hypochloraemic metabolic alkalosis with paradoxical aciduria
D. Hypokalaemia, hypochloraemia, metabolic acidosis
32. A patient vomiting 3 times daily, investigation K: 1.8, what’s the suspected ECG? Prominent U, flat T waves, depressed ST
33. A patient came to ER with 4 days repeated vomiting, what's the most common electrolyte disturbance?
A. Hyperkalaemia B. Hypercalcaemia C. Hypokalaemia D. Hypocalcaemia
34. A male patient with duodenal ulcer has vomiting for 2 weeks, what metabolic disturbance will be found?
A. Hypochloraemic, hypokalaemic metabolic acidosis B. Hypochloraemic, hyperkalaemic metabolic acidosis
C. Hypochloraemic, hypokalaemic metabolic alkalosis
35. A barium swallow study done for a patient known with Gastric ulcer, presented with projectile vomiting, contain old food, offensive,
frequently follow meals, for long time, what is the metabolic abnormality? Hypokalaemic hypochloraemic metabolic alkalosis
36. An old age patient with Linitis plastica with gastric outlet obstruction and succussion splash, what is metabolic finding?
A. Metabolic alkalosis B. Hypochloraemic alkalosis C. Hypokalaemia
D. Hypokalaemic hypernatraemic metabolic alkalosis with hypocalcaemia hypernatraemia if delayed ttt, hypocalcaemia if neglected
37. A patient has history of vomiting for 2 weeks, labs ↓K, ↓Na, ↓Cl, metabolic alkalosis, concentrated urine, what to infuse?
A. Dextrose B. Hartmann’s RL (will cause alkalosis) C. Bolus saline + K D. Sodium chloride till improve UOP then add K
38. A 70 yrs. old patient with sepsis has a pH: 7.18. Which of the following is true regarding to his metabolic acidosis?
A. The most common cause of excess acid is prolonged nasogastric suction
B. Metabolic acidosis results from the loss of bicarbonate or the gain of an acid
39. An old, aged male with urinary bladder cancer, the surgeon after radical cystectomy did urinary diversion by uretero- sigmoidal
anastomosis as a continent diversion what is the common complication of that diversion?
A. Hypochloraemic metabolic alkalosis B. Hypokalaemic, hypocalcaemic hyperchloraemic, hypomagnesaemic metabolic acidosis
C. Hypochloraemic hypomagnesaemic, hypercalcaemic metabolic acidosis D. Hypokalaemic, hypocalcaemic metabolic alkalosis
40. You reviewed a laboratory investigation for a patient who has low pH, low bicarbonate, normal pCO2 what is the diagnosis?
A. Compensated respiratory acidosis B. Uncompensated respiratory acidosis
C. Compensated metabolic acidosis D. Uncompensated metabolic acidosis
41. Abdominal compartment syndrome, low blood pressure, low Hb, low PO2, high creatinine, K 6.5, what is most concerning?
Cardiac arrest, Ventricular fibrillation or asystole follows if not hyperkalaemia treated
42. After massive blood transfusion of cold blood what is the most adverse effect that can occur?
A. Hypocalcaemia & alkalosis (citrate chelating of serum Ca) B. Febrile non-haemolytic reaction C. Haemolytic reaction
The lethal triad of acidosis, hypothermia, and coagulopathy associated with massive transfusion is associated with a high mortality rate.
43. What is true for hypothermic coagulopathy?
A. It is associated with clotting factor depletion B. It can be corrected with fresh-frozen plasma transfusion
C. It is associated with prolonged prothrombin time and partial thromboplastin time D. It is a complication of massive transfusion
44. Hypothermic coagulopathy is characterised by which of the following?
A. Normal prothrombin time and normal partial thromboplastin time
B. Normal prothrombin time, prolonged partial thromboplastin time C. Prolonged prothrombin time, normal partial thromboplastin time
D. Depletion of factor VII E. Depletion of factor VIII
45. Which of the following coagulation factors are deficient in stored blood?
A. Factor II B. Factor VII C. Factor VIII Factor IX Factor XI Stored blood is often deficient in factors; V and VIII
46. What is the known change occurring in stored blood?
A. Increased H+ B. Increased 2, 3-diphosphoglycerate C. Decreased red cell fragility D. Decreased K+ E. Increased Ca2
47. Hypocalcaemia is a complication of which chemotherapy?
A. Cyclophosphamide B. Vincristine C. Methotrexate D. Mithramycin E. Adriamycin
48. Scenario, a case of massive blood transfusion and the patient had cardiac arrest. What is the cause?
A. Hyperkalaemia B. Hypercalcaemia
49. For a patient with a serum potassium level of 7 mEq/dL and an absent P-wave on electrocardiogram, what is the initial management?
A. Intravenous Lasix B. Intravenous glucose/insulin C. Kayexalate enema
D. Intravenous sodium bicarbonate E. Intravenous calcium gluconate
50. What tented or peaked T wave in ECG means? Hyperkalaemia
51. What are disturbances in tumour lysis syndrome? Hyperkalaemia, hyperphosphataemia, & hypocalcaemia
52. A patient with non-Hodgkin lymphoma on treatment developed tumour lysis syndrome, what that indicates?
A. Hypokalemia hypocalcemia B. Hyperkalaemia hypocalcaemia C. Hypokalaemia hypercalcemia D. Hyperkalemia hypercalcemia
53. For whom the greatest amount of maintenance intravenous fluids is required?
A. 21 -year-old male athlete B. 55-year-old obese male office worker C. 21 -year-old housewife
D.75-year-old female with recent weight loss
54. Case with symptoms and signs of dehydration after severe bouts of gastroenteritis, what about serum Sodium level?
A. Low (perspiration, diuretics & water overdrinking, ttt of hypernat.) B. Normal(if compensation of losses) C. High (vomiting, diarrhoea, thirst)
55. A patient with multiple small bowel resections for Crohn’s disease presents with an acute flare. Due to his preexisting short length of
bowel and current profuse diarrhoea, he is suspected of having functionally short gut syndrome and is started on TPN, after time the
patient complains from muscle weakness, diarrhoea, arrhythmia and hypotension, refeeding syndrome is characterised by which of the
following electrolyte abnormalities?
A. Hyponatraemia, hypokalaemia, and hypercalcaemia B. Hyperphosphataemia, hypokalaemia, and hypocalcaemia
C. Hypophosphataemia, hypokalaemia and hypomagnesaemia D. Hypocalcaemia, hyponatraemia, and hypomagnaesemia
56. A polytrauma patient in ICU, no brain issues but after 14 days develop subarachnoid Hge., what’s the associated electrolyte imbalance?
A. Hypernatraemia B. Hyponatraemia C. Hyperkalaemia D. Hypercalcaemia
57. Severe hypernatraemia, what is the best resuscitation fluids treatment?
A. Hypotonic fluid (D5W in acute hypernatraemia or 0.45%NaCl in chronic) B. Isotonic saline C. Starch
58. Correction of fluid in patient with hypernatraemia is? avoid rapid over correction of hypernatraemia must do over 48 hours
59. Over or rapid correction of which of the following can cause; headache, vomiting, and seizure?
A. Hyponatraemia B. Hypernatraemia C. Hypokalaemia D. Hyperkalaemia E. Hypercalcaemia
60. An 80-year-old male is confused, lethargic, and has tonic spasms. He has been receiving intravenous 5% dextrose in water in a half
normal saline for 3 days after right hemicolectomy, what is the most likely cause?
A. Hypokalaemia B. Hyperkalaemia C. Hypocalcaemia D. Hypernatraemia E. Hyponatraemia
61. Renin angiotensin system is activated by which of the following?
A Hyponatraemia B. Hypernatraemia C. Hypokalaemia D. Hypocalcaemia E. Hypercalcaemia
62. A patient in the ICU, with hypotension, bradycardia, with low serum sodium, high potassium, high cholride and high urea, what is the
cause of these changes?
A. Hyperkalaemia B. Hyponatraemia (low BP if compared with >135 mmol) C. Hypercholraemia D.Uraemia
63. Another scenario with hypernatraemia diarrhoea, best resuscitated with? Hypotonic solution as D5W, 0.25 NaCl, 2.5% dextrose
64. What is the most common cause of hypophosphataemia in hospitalised patients? <0.48mmol/l. Aet surgery e fasting, IV glucose/ G-ve septicaemia
A. Renal failure B. Sepsis C. Glucose overload D. Diarrhoea
65. Hypophosphataemia is associated with which of the following?
A. Increased haemoglobin p50, respiratory failure, and encephalopathy B. Decreased haemoglobin p50, haemolysis, and respiratory failure
C. Respiratory failure, hypothyroidism, and haemolysis D. Ataxia, cardiomyopathy, and hypothyroidism
66. Regarding of Magnesium blood level what is true?
A. Deficiency can be accurately diagnosed with serum level measurement B. Hypomagnesaemia is associated with hyperkalaemia
C. Hypomagnesaemia is associated with neuromuscular excitability D. Hypomagnesemia is a complication of renal failure
67. What is the electrolyte disturbance that occurs due to chronic diarrhoea? as Cholera, VIPomas villous adenomas
A. Hypercalcaemia B. Hypernatraemia C. Hypokalaemia, hyponatraemia metabolic acidosis D. Metabolic alkalosis
68. Refractory hypokalaemia can be caused by which of the following?
A. Hypocalcaemia B. Hyponatraemia C. Hypophosphataemia D. Hypomagnesaemia
69. Regarding Magnesium, what is the correct choice of the following?
A. It is an intracellular ion B. Its deficiency leads to hyporeflexia C. Its depletion is common in shock states
D. It is excreted mostly in stool
70. A patient receiving 1,800 cal/day in total parenteral nutrition will require how much of the following?
A. 125 g protein/day B. 150 g protein/day C. 200 g protein/day D. 250 g protein/day E. 300 g protein/day
71. How much is the protein loss equivalent to 100 g of negative nitrogen balance?
A. 75 g B. 150 g C. 375 g D. 525 g E. 625 g
72. Hypermagnesaemia is a complication of which of the following?
A. Extensive burns B. Acute pancreatitis C. Oliguric renal failure D. Terminal ileal resection E. Diuretic therapy
73. A case of metabolic acidosis, low HCO3, pH is 7.2 with intestinal fistula, what is the cause of this condition?
A. Intestinal fistula B. Hypovolaemic shock C. Hyperchloraemic metabolic acidosis
74. How to differentiate between pre renal and renal azotemia?
A. Urinary sodium level (>40 mEq/L in renal and < 20 mEq/L in pre renal) B. Serum sodium level C. Urine creatinine level
D. Serum creatinine level urine microscopy
75. The serum sodium level in a 60-year-old man who weighs 70 kg is 125mEq/L. How much is his sodium deficit?
A. 130mEq B. 210 mEq C. 360 mEq D. 420 mEq E. 850 mEq N.B. Na deficit= target Na level(135)- current Na 125 x 0.6 x body wt.
76. Which of the following is characteristic of prerenal azotemia?
A. Abnormal urine sediment B. Fractional excretion of sodium value < 1% C. Urine sodium level <40mEq/L
D. Blood urea nitrogen/serum creatinine level >10
Vascular surgery
1. A 40-yrs. female presented with long time of bilateral lower limb oedema and skin change. On examination, the distal pulses were
intact, only dilation of greater saphenous vein’s tributaries was seen, which of the following is the best investigation of choice?
A. Venous duplex ultrasound B. Venography C. ABPI
2. A 56 year old woman presents to the clinic with a non-healing ulcer over her right lateral malleolus, she is hypertensive, pulse is normal,
and her local exam shows dark discoloration of the skin around the ulcer and viable ulcer bed, what is the best next step?
A. CT angiogram B. Venous duplex US C. Arterial Doppler US D. Conventional angiography
3. A female patient with long time of bilateral leg swelling and skin changes, what is the best diagnostic modality?
A. Doppler U/S B. Venography C. ABPI D. Duplex US
4. A patient with medial side left leg ulcer, what is the highest risk factor?
A. DM B. Age C. Buerger’s disease D. Venous Hypertension
5. Which of the following is characteristic of Buerger's disease?
A. Superficial migratory thrombophlebitis B. Female predominance C. Severe involvement of the aortoiliac segment
D. The disease is restricted to the lower extremities
6. A 40 years old female patient, with long standing progressive non-pitting oedema of both lower limbs, with skin changes and hardening,
with intact pulses of both sides, no pain or tenderness, no history of insect bite, what is the possible diagnosis?
A. lymphoedema precox(2-35y) B. Lymphoedema tarda(>35 y if > 50 ? malignancy) C. Elephantiasis D. Lipodermatosclerosis
7. A patient with bilateral non-pitting oedema with whitish discoloration at the ankle, with history of living in some endemic area diagnosis?
A. Chronic venous insufficiency B. Filariasis(pain, disfigure,nonpitting, endemic) C. Lipodermatosclerosis D. Lymphoedema
8. A 40 year old female with 12-year history of varicosities, complaining of left limb oedema and heaviness when standing, distal pulses
are intact and there is bilateral varicose in the great saphenous vein territory, what is your management?
A. Sclerotherapy B. Endovenous laser C. Thermal ablation (Percutaneous endovenous ablation; RFA, LASER, via US sclerotherapy)
9. Platelet aggregation induced by Adenosine diphosphate is inhibited by which of the following?
A. Aspirin B. Heparin C. Dipyridamole D. Coumadin
10. Which of the following is an inhibitor of platelet aggregation?
A. Prostacyclin I B. Thromboxane A2 C. Adenosine diphosphate D. Serotonin E. von Willebrand' s factor
11. What is the action of Nitric oxide?
A. Is a potent vasoconstrictor B. Prevents platelets aggregation
C. Requires cyclic adenosine monophosphate for its actions D. Is normally stored in endothelial cells
12. A patient with cancer tongue, modified neck dissection level 1 on right side, modified neck dissection level 2 on left side, patient developed
vomiting, drowsy and confused, headache, duplex show thrombosis of left IJV what will you do?
A. Neurosurgery consultation B. CT C. Re exploration and Thrombectomy D. Anticoagulants(the 1st. choice ttt in JVT)
13. A case of old male patient with cancer tongue, during surgical excision of the tumour and radical dissection of the neck done on right
side with ligation of the IJV, on the left side modified neck dissection only, patient developed cerebral oedema and manifestation of brain
oedema, due to thrombosis of the right IJV, how to manage?
A. Thrombectomy B. Anticoagulant (thrombolytic therapy alteplase in 3-4.5 hr, 0.9 mg/kg max 90mg, 10% in 1min 90% in 1hr) C. LMWH
14. What is the optimal therapy for effort thrombosis of the axillary vein? Paget-Schroetter syndrome
A. Therapeutic heparin followed by Coumadin for 3 months B. Therapeutic heparin followed by Coumadin for 6 months
C. Thrombolysis, anticoagulation, and possible first rib resection D. Thrombolysis, anticoagulation, and balloon angioplasty
E. Thrombectomy, anticoagulation, and stent placement
15. A patient underwent hip replacement, postoperative medications were IV fluids & analgesics & Proton pump inhibitors, 7 days later
she developed dyspnoea, what could prevent this complication?
A. Prophylactic LMWH B. Prophylactic aspirin C. Warfarin therapeutic dose D. IVC filter
16. Post hip replacement, he developed swelling of both LL, tender on touch, duplex US; venous stasis with compression by probe what next?
A. Thrombolytic therapy and anticoagulants B. Antibiotics C. Elastic stocking (contraindicated)
17. A patient had pelvic operation, the best way for prophylaxis against DVT?
A. High dose heparin B. Prophylactic LMWH C. Low dose heparin (usually for prophylactic low dose but for therapeutic high dose)
18. A patient who underwent a major pelvic operation as colorectal surgery, how to prevent DVT?
A. Mechanical prophylaxis B. Enoxaparin (LMWH) C. Heparin D. IVC filter (if frail with Contraindicat. Give anticoagulants)
19. An old patient after rectal surgery is not doing well, after the recovery he started to have leg pain and developed DVT from the popliteal
to the femoral, what is the management?
A. Enoxaparin (therapeutic dose 1mg/kg/12h) B. Heparin C. Warfarin D. IVC filter
20. A female patient developed DVT, 10 days after Caesarean section, what to give?
A. Thrombolytics B. LMWH (5-10 days, 1mg/kg/12h then oral as warfarin or DOACs) C. IVC filter D. Embolectomy
21. A patient post gastric sleeve 10 days ago presented with DVT how to treat?
A. Heparin LMWH B. Warfarin C. IVC Filter D. Thrombolysis
22. How to manage deep vein thrombosis during pregnancy?
A. 10-day intravenous heparin and then Coumadin until term B. 10-day intravenous heparin and then Coumadin for 6 months
C. 10-day intravenous heparin and then prophylactic subcutaneous heparin until term
D. 10-day intravenous heparin and then therapeutic subcutaneous LMW heparin until term
23. A 30 years old postpartum developed DVT and was started on enoxaparin 80 mg BD. On the same admission she developed sudden
onset of shortness of breath and right pleuritic chest pain (PE). BB 125/70 HR 100 RR 22 temperature 36.6 oxygen saturation 95% CT
shows thrombus in right lower pulmonary artery. What is most appropriate?
A. Change enoxaparin to sodium heparin B. Thrombolytic therapy D. Thrombectomy
D. Do combination of LMWH /Warfarin (for 6 weeks then shift to warfarin only for 6 months)
24. Which of the following is correct regarding use of direct oral anticoagulants?
A. Can be direct oral Xa or thrombin inhibitors B. I & D of perineal collection C. Analgesic and antipyretic
25. Immediately after intravenous (IV) injection of 5,000 U of heparin, an adverse effect occurred how can be reverse its effect?
A. 20 mg protamine sulfate B. 30 mg protamine sulfate C. 40 mg protamine sulfate D. 50 mg protamine sulfate
26. What is the cause of bleeding after adequate heparin reversal with protamine sulphate?
A. Protamine toxicity B. Heparin rebound C. Hypothermia D. Thrombocytopenia E. Factor VIll depletion
27. What is the most common complication of heparin reversal with protamine sulfate?
A. Bradycardia B. Hypotension C. Thrombotic crisis D. Thrombocytopenia E. Leucopenia
28. The development of thrombocytopenia and arterial thrombosis with heparin requires which of the following?
A. Continuation of heparin and platelet transfusion B. Continuation of heparin and thrombolysis
C. Doubling the heparin dosage D. Changing the route of heparin administration E. Discontinuation of heparin then can give Argatroban
29. Which of the following drugs is specific therapy for heparin-induced thrombocytopenia?
A. Low-molecular-weight heparin B. Coumadin C. Dextran-60 D. Lepirudin (direct irreversible thrombin inhibitor)
30. A 56-year-old patient with history of DVT was kept on LMWH preoperatively, he underwent lap. Chole. 5 hours postop. His drain
showed blood collection denoting there is bed site oozing CBC and INR within normal what is the next to do?
A. Protamine sulfate B. Fresh frozen plasma C. Blood transfusion D. Tranexamic acid E. Antihemophilic factor
31. A 34 years old nulliparaous woman, with previous history of DVT, what drugs will give this patient?
A. Enoxaparin B. Heparin C. Warfarin D. No need treatment (as now is free)
32. A patient post-bloody surgery developed left lower limb painful oedema up to the thigh, which thromboprophylaxis is suitable for him?
A. UFH/LMWH B. Alteplase C. Mechanical pneumatic compression (intermittent have no risk of bleeding)
33. One week after Coumadin therapy, a patient developed severe right leg pain in with skin necrosis, what’s best action?
A. Transfusion of fresh frozen plasma B. Intravenous Vit. K B. Intravenous Vit. K
C. Stop Coumadin and start oral rivaroxaban or heparin
34. What is the cause of Coumadin-induced skin necrosis?
A. Protein S deficiency B. Protein C deficiency C. Antithrombin Ill deficiency D. Disseminated intravascular coagulation
35. A child will go for circumcision, what is the best investigation to be done before the operation?
A. BT (bleeding time) B. Prothrombin time PT or aPTT B. Factor VII C. Factor V
36. Abnormal bleeding with normal prothrombin time occurs with all of the following diseases except?
A. Heparin overdose B. Cirrhosis C. Haemophilia D. von Willebrand’s disease
37. Abnormal bleeding with prolonged partial thromboplastin time with normal platelet count and PT occurs with which of the following?
A. Heparin overdose B. Cirrhosis C. Haemophilia A &B D. von Willebrand’s disease
38. For von Willebrand's disease which of the following is true?
A. Is an autosomal dominant disorder(affect 1%, F:M2:1 due to deficiency of vWF) B. Results in prolonged prothrombin time
C. Is associated with normal bleeding time D. Is due to decreased hepatic synthesis of von Willebrand’s factor
E. It is typically associated with joint bleeding F. Affects males only
39. In the treatment of Coagulopathy, what is the true?
A. Calcium should be routinely infused with massive transfusion B. Desmopressin (DDAVP) stimulates the release of factor VIII
C. von Willebrand's disease can be treated with factor VIII concentrate D. The effect of aspirin can be reversed by fresh frozen plasma
40. A 10-year-old boy comes from a family of "bleeders, ' his coagulation profile shows: prolonged bleeding time, normal prothrombin
time, and prolonged partial thromboplastin time. His platelet count is 150,000/mm 3, what is the most likely diagnosis?
A. Idiopathic thrombocytopenic purpura B. Thrombotic thrombocytopenic purpura C. Haemophilia
D. Christmas disease E. von Willebrand's disease
41. Clotting factor correlating with Vit. K is? (2, 7, 9, 10)
A. VII B. XI C. X (vit k dependant associted only with bleeding) II, VII, IX deficiency associated with bleeding & clotting, Protein C&S with clotting
42. Which of the following is correct for Vitamin K?
A. Is water soluble B. Is absorbed in proximal small bowel D. Is required for factor V synthesis
C. Fat soluble vitamin requires bile salts for absorption
43. Which clotting factor is mainly of the endothelial cell origin?
A. Factor II B. Factor V(only liver) C. Factor VII(hepatic & endothelial sources) D. Factor VIII (originally from endothelial cells)
44. What is the mechanism of action of heparin?
A. Potentiation of antithrombin III factor (block conversion of fibrinogen to fibrin) B. Inhibition of cyclooxygenase C. Direct thrombin inhib.
45. After chest trauma, weak femoral pulse and CXR revealed mediastinal air with wide mediastinum, what is most likely diagnosis?
A. Thoracic Aortic rupture B. Flail chest C. Tension pneumothorax
46. A patient with right foot gangrene up to above his knee, CT angiogram done and showed left sided occlusion of external iliac vessels,
what is proper treatment?
A. Stent (if inappropriate IR revascularisation or bypass) B. Embolectomy C. Bypass (Aorto bifemoral) D. Endovascular intervention
47. Which of the following is an aetiological factor in occlusive vascular diseases?
A. Arginine B. Glutamine C. Methionine D. Homocysteine E. Xanthine
48. An old age lady with uncontrolled DM, came to ER with blackish discoloration at Lt. foot 2 weeks duration, no palpable anterior and
posterior tibial artery, leucocytosis, CTA, revealed severe stenosis at distal superficial femoral artery, complete occlusion popliteal and
infra popliteal artery, no collaterals, no distal run off, what is the best management?
A. Bypass graft B. Below knee amputation C. Above knee amputation
49. A case of ischaemic limb for below knee amputation, what you will check and examine before doing amputation, what’s your decision?
A. Femoral artery B. Dorsalis pedis artery C. Popliteal artery D. Profunda femoris artery
Pre-amputation popliteal artery must be pulseless or severe stenosis with no distal run off and ischemic necrosis of tissues, to take decision of amputation, as if
there is pulse may be other less aggressive options for limb salvage as bypass or stenting.
50. Which artery ligation or complete block with a thrombus among the following will result in lower limb tissue loss?
A. Popliteal artery B. Common femoral artery C. Superficial femoral artery D. Posterior tibial artery
51. What is the outcome of Popliteal artery entrapment?
A. It is more common in females than males B. It is diagnosed by passive dorsiflexion of the foot
C. It results from compression by the medial head of the gastrocnemius D. It requires bilateral exploration in most cases
52. An old aged heavily smoking patient presented with critical limb ischaemia, failed embolectomy, completely obliterated popliteal artery,
only patent superficial femoral artery, what is the optimal management?
A. AKA; Above knee amputation B. BKA C. Hip Disarticulation
53. An old age diabetic patient with gangrene in the forefoot, will undergo for amputation, which type?
A. Syme’s amputation (at level of ankle preserving the heal pad) B. Transmetatarsal amputation C. Below knee amputation
54. A diabetic patient with inflammatory swelling with fluctuation, over the dorsum of foot, Doppler US showed that all vessels are triphasic
except dorsalis pedis artery is monophasic, what is your plan of management?
A. Debridement (initially with good dressing and IV antibiotics) B. Amputation (Metatarsal ray amputation if no improvement/ worsening)
55. An image for diabetic patient, with history of amputation of middle toe with delayed healing, duplex intact dorsalis pedis pulse, intact
femoral with 3 cm constriction in distal part of it, what is best for the patient?
A. Femorofemoral bypass B. Femoropopliteal bypass C. Profundoplasty D. Femorotibial bypass
56. An elderly smoker man known case of poorly controlled DM came with ulcers on tip of three of his toes, diminished dorsalis pedis
bilaterally, however, intact popliteal pulse, what’s the initial management?
A. Amputation B. Long term anticoagulation C. Immediate surgical intervention
D. Lifestyle modification, healthy diet and sugar control
57. A 70 years old with HTN and Cardiac disease, complains of claudication after walking 100 yards in left lower limb, while examine the
peripheral pulse you noticed that the inferior popliteal pulse is weak, dorsalis pedis not felt, next step?
A. Vascular Doppler B. Embolectomy C. Conventional angiography D. ABI measurement
58. An elderly male with leg pain after waking 200 meters, relieved by sitting, what best treatment option for him?
A. Non-supervised exercise program B. Supervised exercise program C. Aerobics exercise D. No treatment needed
59. What is the golden standard investigation to be done for a patient who has critical limb ischaemia?
A. CT angiography B. Conventional angiography
60. A male diabetic smoking patient presented with right big toe gangrene, admitted with right lower limb, pain, numbness and paralysis
for 12 hours with thromboembolic insult, CTA showing occlusion of the lower anterior tibial and dorsalis pedis artery, what is the best
choice of the following?
A. Embolectomy (as acute embolic ischaemia) B. Angioplasty C. Femoropopliteal bypass (if chronic) D. Femorodistal bypass
61. An old aged chronically smoker, with DM, HTN, and atherosclerosis with no complaints before of lower limb problem, suddenly he
complined of severe leg pain with signs of acute ischaemia which of the following we can do to reach to the main origin of the disease?
A. ECG (gold standard initial) B. Angiography C. Echocardiography (for structural valve disease that can be origin of showering)
D. Chest x ray E. Dupplex US
62. A scenario of acute limb ischaemia, what is the best investigation?
A. ECHO B. Doppler US C. CT Angiography
63. What is the best way to diagnose peripheral arterial disease?
A. US doppler (initial) B. CT angiography CTA (the best) C. Clinical assessment D. Ankle brachial index ()
64. A 62-year-old female smoker has type 2 diabetes mellitus, morbid obesity, and a recent diagnosis of symptomatic peripheral arterial
disease, you have started her on atorvastatin (Lipitor), offered a supervised exercise program, and discussed smoking cessation and
interventions, which one of the following should be recommended to prevent cardiovascular events in this patient?
A. Aspirin (anti-platelet) B. Cilostazol (direct arterial vasodilator) C. Enoxaparin
65. An old, diabetic, chronic smoker man with history of lower limb claudications, he got TIA attack, Duplex and contrast enhanced MRA
showed right carotid stenosis > 70%, the patient is unfit for endarterectomy what is the best beside lifestyle modification?
A. Clopidogrel with aspirin B. Aspirin alone C. Clopidogrel alone D. Clexane therapeutic dose
66. A 65 y.o man with DM, HTN, IHD with history of claudication on clopidogrel and 5- ASA for old MI now he has lateral lower part leg
ulcer and pain increase with walk and decrease with rest what type of that ulcer?
A. Venous ulcer B. Arterial or ischaemic ulcer C. Neuropathic ulcer
67. Diabetic foot patient with uncontrolled DM, he has ulcer 3x4 cm reaching to subcutaneous layer with frequent smelly discharge, with
little sensation, what ulcer type?
A. Ischaemic B. Neuropathic C. Marjolin ulcer
68. Regarding the diabetic foot what is true?
A. Atherosclerosis often involves the pedal arteries B. Foot sepsis is often polymicrobial
C. Ankle brachial index accurately measures the degree of ischaemia D. Diabetic neuropathy involves only sensory nerves
69. In diabetic foot ulcers what is correct?
A. Neuropathy is restricted to somatic nerves B. The heel is the most common location
C. Motor neuropathy mostly affects toe flexors D. Cold, dry foot is a classic finding
E. the ankle brachial index is an accurate measure of the degree of ischaemia
70. What is the difference between chronic or acute ischaemic limb in a patient who is heavily smoker, diabetic with chronic leg pain?
A. History of intermittent claudication (in chronic but sudden in acute embolic) B. Pulseless left leg C. Leg discolouration
71. A female patient with big toe gangrene of bluish discoloration, severe claudication and rest pain, tissue loss, how much is the ABPI?
A. >1 B. 0.8 – 0.9 C. 0.5 – 0.6 D. < 0.3
72. An old aged heavily smoker diabetic patient with history of chronic lower limb ischaemia and claudication pain that progressed to rest
pain that is not relieved except by narcotics which of the following makes the intervention is an urgent action?
A. Rest pain B. Peripheral neuropathic pain C. Superficial femoral artery occlusion D. Leg colour changes
73. A female patient with leg oedema, need to be treated by gradual pneumatic compression and garment compression, but it is a
contraindicaed if ABI?
A. > 1 B. 0.8 – 0.9 C. < 0.5
74. Lymph Node swelling and was clear lymphatic fluid in this swelling. What next step or initial?
A. Surgery(definitive treatment ) B. Sclerotherapy (cystic hygroma) C. Radiotherapy D. Chemotherapy
75. An Infant with lateral neck mass, +ve transillumination increase size with crying compressible, US showed it multicystic diagnosis?
A. Branchial cyst B. Cystic Hygroma
76. Which can compromise respiration during delivery to baby?
A. Cystic hygroma B. Torticollis
77. A 60-year-old diabetic and smoking lady with picture of ischaemic limb with mottled and cold skin what is the most common diagnosis?
A. Acute limb ischaemia B. Chronic limb ischaemia C. Trench foot
78. Which of the following, is not related to atherosclerosis, smoking or metabolic disease neuropathy?
A. Acute ischaemia B. Chronic ischaemia C. Diabetic foot D. Trench foot
79. An adult patient was involved in motor vehicle collision with severely crush injured limb and exposed muscles with no bleeding, and
absent posterior tibial and dorsalis pedis arteries pulsations he is shocked with BP 60/40, Pulse 120, what is the best initial to be done?
A. Below knee amputation B. Extensive debridement C. CT Angiography D. Handheld Doppler
80. A patient with signs of acute limb ischaemia, peripheral pulses; dorsalis, popliteal are absent in right leg and in left leg, dorsalis pulse
only is absent, what is the best management option?
A. Heparin and observe B. Femoral thrombectomy C. Catheter thrombolysis
81. A victim with thigh stab wound, with active profuse femoral artery bleeding, no associated injuries, patient is fully conscious,
saturation 97% on room air, blood pressure 100/70, pulse 80, what is the initial management?
A. Direct pressure above the oozing point to control bleeding or tourniquet around the wound B. Fluid resuscitation
C. Airway securing D. Push the patient to operative room, OR
82. A diabetic patient with varicosities of the left leg, presented by bleeding from a vessel beside medial malleolus, 1st plan for him?
A. Tourniquet B. Tight bandage C. Urgent stripping D. Ligation of offending vessel
83. A patient after fight has a thigh puncture, lot of blood loss, compression done, bleeding doesn't stop, shocked, cold pulseless limb what
is the best treatment option?
A. Duplex then surgery B. CTA then surgery C. Surgery urgently (he is critical, unstable with hard sign)
84. A post traumatic paraplegic patient of one-year history of trauma, he came with SOB and cough diagnosed as PE and DVT, he is unstable
haemodynamically what’s treatment?
A. Enoxaparin B. Tissue plasminogen activator (Thrombolytics) C. IVC filter
85. A Case of chronic limb ischaemia what the gold standard for diagnosis?
A. Conventional Angiography B. CT Angiography (confirmatory) C. Doppler (initial)
86. A patient with chronic limb ischaemia, presented with sudden leg pain, diminished popliteal and distal pulses in right leg, and diminished
distal pulse with intact popliteal in the left, what's the appropriate next action?
A. Heparin (the 1st step in acute limb ischaemia) B. CT Angiogram C. US D. Conventional Angiography
87. What is the most common visceral aneurysm?
A. Coeliac artery (4%) B. Superior mesenteric artery (6%) C. Splenic artery (60%) D. Hepatic artery (20-50%)
88. What is true for splenic artery aneurysm?
A. Can present with double rupture phenomenon(rupture into lesser omentum the peritoneal cavity) B. Is more common in males C. seldom multiple
89. What is correct regarding Splenic artery aneurysm?
A. is usually asymptomatic B. is usually a pseudoaneurysm C. is more common in young males D. Rupture is seldom fatal
90. A splenic aneurysm 1.7 cm no HTN, DM or medical problems by CTA, no calcification or sacculation what is the management?
A. Observation and follow up(as <3cm, asymp, no growth) B. Ligation of the aneurysm C. Splenectomy with removal of the affected a.
91. A 56-yr. patient has history of acute pancreatitis show splenic aneurysm 2.5cm with calcification, what is the best management option?
A. Observation and follow up B. Ligation of aneurysm C. Splenectomy with removal of affected artery
92. A 45-yr-old with history of pancreatitis, he has Splenomegaly, normal portal pressure, but splenic vein thrombosis then developed
frequent haematemesis, after resuscitation and injection of sclerotherapy he became stable, which of the following is the best management?
A. Splenectomy B. Splenorenal shunt C. Veno caval shunt or portocaval shunt D. None
93. An elderly patient presented with constant abdominal pain radiating to right groin and scrotum, he looked ill BP is normal, CT
showed the following, what’s your diagnosis?
A. Leaking AAA B. Ureteric colic C. Diverticulitis D. Appendicitis
94. Same previous patient who is elderly patient presented with constant abdominal pain radiating to right groin and scrotum, he looked
ill BP normal, CT showed AAA, what is the plan of his treatment according to your previous diagnosis?
A. Endovascular repair B. Ureteroscopy +/- stent C. Appendicectomy D. IV antibiotics
95. An elderly patient presented with constant left lower abdominal pain, pain in left lower back radiating to right groin, scrotum and
testes, he looked ill, blood pressure is average, CT axial view, showed aortic aneurysm, what is most likely diagnosis?
A. Diverticulitis B. Appendicitis C. Leaking aortic aneurysm
96. An image for aortic aneurysm mainly to the left side with right lower abdominal pain, radiated to the right testis, diagnosis?
A. Appendicitis B. Diverticulitis C. Aneurysmal dissection
97. A patient presented with tearing sensation, abdominal pain radiating to the back, pulsatile abdomen. He was unstable, what would you
do next?
A. Exploratory laparotomy (leaking AAA+ unstable) B. CT C. Laparoscopic exploration D. IV fluid resuscitation
98. A 50 y.o lady with epigastric pain, with dizziness and collapsed at home. In the ER she has mild low back pain and her BP 90/60, P
about 130/m what’s the most likely diagnosis?
A. Mesenteric ischaemia B. Leaking/ruptured aortic aneurysm C. Perforated duodenal ulcer D. GORD
E. Gastric ulcer F. Chronic pancreatitis
99. A patient with sudden lumbar back pain radiating to flanks, he gave history of the pain not radiating 2 days ago, his pulse: 80, BP 110/70,
temp. 37, after 1 hour: pulse 106, BP 80/40, temp. 37, what’s the management?
A. Abdomen CT B. Laparotomy (leaking, ruptured AAA) C. Endoscopic repair for the aneurysm
100. A patient has HTN came with pulsatile abdominal swelling what could be?
A. Aortic aneurysm B. Renal cause
101. On the second day after AAA repair, the patient passes grossly bloody stool, you suspect colonic ischaemia, what’s the next?
A. Mesenteric angiography B. Sigmoidoscopy C. Upper GIT endoscopy
D. Immediate exploratory laparotomy E. Abdomen CT with contrast F. Barium enema
102. Repair of aortic aneurysm, what is the most dangerous complication?
A. Aortoenteric fistula (can occur 4 months to 5 years) B. Renal failure C. Aneurysm
103. A 60-year-old smoker male with history of symptomatic AAA, EVAR done with vascular surgeon team, after surgery by about three
years the patient was presented with abdominal pain, vomiting, shock then after resuscitation presented with melena, what diagnosis?
A. Colonic diverticulosis B. Angiodysplasia C. Peptic ulcer disease D. Aortoenteric fistula(3rd.- 4th duodenum, after EVAR)
104. What is the most common strong anticoagulant secreted by medicinal leeches?
A. Heparin B. Warfarin C. Hirudin (Lepirudin)
The most potent known natural thrombin inhibitor from blood-sucking leeches (Hirudo medicinalis), Hirudin has served as a standard for designing the
natural coagulation inhibitors as an anticoagulant drug.
105. An old aged, HTN, diabetic, known he has AAA, came with pulsatile abdominal mass, hypotension (80/40), what is the best option?
A. US then OR B. CTA then OR C. Urgent exploration in operative room emergently D. Fluid resuscitation then reassess.
106. A Case of Abdominal Aortic Aneurysm (AAA) and the patient is haemodynamic unstable, present after he ate food with severe
abdominal pain despite taking analgesic. He became confused and unconscious later in the hospital, examination revealed: tender and
pulsatile mass in the abdomen, BP low. What is your most appropriate management?
A. US (as unstable cannot transfer to CT) B. CT (if stable) C. Exploratory laparotomy (if known or radiologically confirmed AAA)
107. All the following can complicate the abdominal aortic aneurysm repair, except?
A. Paraplegia B. Renal failure C. Hepatic failure D. Leg ischaemia
108. What is the most common cause of early postoperative death in elective abdominal aortic aneurysm repair?
A. Acute renal tubular necrosis B. Ischaemic stroke C. Acute myocardial infarction D. Respiratory failure
E. Exsanguination from anastomotic disruption
109. A 70-yrs. old man presented with abdominal pain radiating to his right flank, he has hypertension and coronary heart disease.
He has smoked a pack of cigarettes daily for 30 years, pulse 96 b/m. BP 125/80. O 2 saturation 95%, examination showed a painful
pulsatile abdominal mass, what is the appropriate next step in management? CT angiography (as it is a suspected Symptomatic AAA)
NB >>> if you suspected a symptomatic AAA >> do Abdominal U/S, if symptomatic >> CTA
The most common sign for the aortic aneurysm is painless pulsatile abdominal mass mostly supraumbilical.
110. Which of the following can be presentation of an inflammatory aortic aneurysm?
A. Repair is associated with a higher incidence of graft infection B. It is more likely to rupture than is a non-inflammatory aneurysm
C. Leads to circumferential thickening of the aorta D. May present with abdominal pain in the absence of rupture
111. What is the indication for preoperative angiography for elective abdominal aortic aneurysm surgery?
A. Suspected contained rupture B. Suspected inflammatory aneurysm C. Aneurysm > 7 cm D. History of claudication
112. Prosthetic graft infection is most commonly due to which causative organism?
A. Staphylococcus epidermidis B. Staphylococcus aureus C. Escherichia coli D. Streptococcus faecalis
113. Regarding the Popliteal artery aneurysms which of the following is correct?
A. Less common than femoral aneurysm B. Often bilateral in 50% of cases C. More common in females
D. Seldom cause limb ischaemia
114. What is true for popliteal artery aneurysms?
A. Commonly present with rupture B. Seldom result in limb ischaemia C. Are most commonly false aneurysms
D. Are the most common peripheral arterial aneurysms E. Require operation only if they result in embolisation
115. A 60-year-old man is noted to have a common iliac artery aneurysm. It is decided to pursue a policy of surveillance. At which of the
sizes listed below should surveillance cease, and surgery be performed?
A. 1.5cm B. 2.5cm C. 3cm D. 2cm E. 3.5 cm (for surgery otherwise rupture if left till > 4cm)
The risk/ benefit ratio would favour 4cm as the rate of spontaneous rupture is very low below this diameter. Most would intervene between 3.5 and 4cm.
116. A 66-year-old man presents with pain in his left leg. On examination he has a dusky forefoot and a palpable popliteal artery aneurysm.
What is the most appropriate initial management?
A. Systemic heparinisation alone B. Above knee amputation C. Intra-arterial thrombolysis followed by early surgical bypass
D. Popliteal embolectomy, exclusion of the aneurysm and bypass E. Trans femoral embolectomy and femoro-distal bypass
117. After 3 years, vein graft failure in the lower extremities is mostly caused by which of the following?
A. Technical complications B. Anastomotic aneurysm C. Atherosclerosis of the graft D. Intimal thickening
118. A case of Gunshot injury in the leg with paraesthesia and weak peripheral pulsation, management?
A. Exploration & repair B. Expectant treatment C. CT Angiography
119. Trauma with thigh haematoma and weak pulses distal to it. Next step in ER until vascular surgeon comes?
A. CTA B. Observe and analgesia C. Explore the wound
120. A chronic smoker man, presented by, chest pain, haemoptysis, cavity in the left upper lobe of lung on chest x ray, what’s best next?
A. Chest CT B. Rigid bronchoscope under anaesthesia C. Noradrenaline injection D. Conservative
121. A 75 y.o male patient, no comorbidities, suspected pulmonary embolism after 5 days of prolonged time intraoperatively for lower
anterior resection, he received anticoagulants (Enoxaparin) for three days upon the preoperative time, and then it stopped 12 hours before
the surgery (as physician instructions to the nurse) and the anticoagulants were resumed after 12 hours of the surgery, what is the possible
cause of developing PE?
A. Long time surgery major surgery (GA more than 45 min. = 2 points) B. Immobilisation after surgery
C. Using non-proper anticoagulants D. Old age as by Caprini score age > 75 = 3 points
122. In the previous PE scenario, what is the initial investigation, to reach the diagnosis?
A. ECG B. ABG C. X ray D. D-dimer
123. An elderly patient with COPD with severe asthma, has fracture of femur neck, pulmonary consultation was done, and the doctor
prescribed aggressive physiotherapy and cortisone treatment for 4 days, patient developed pulmonary embolism, what is the cause?
A. Immobilisation (if no physiotherapy or ambulation) B. NSAIDs C. Recent and current Steroid use (for asthma and COPD)
124. Post femoral neck surgery, what medication will be given as VTE prophylaxis?
A. Aspirin B. Low molecular weight heparin C. Full dose of heparin D. Clopidogrel
125. A young female with chest pain started after she arrived by flight journey what is management?
A. Thoracostomy tube B. Thoracotomy C. Observation D. CT Angiography (mostly PE)
E. ECG and cardiac enzymes
126. A female patient developed pulmonary embolism in the postpartum period, what is the best investigation or the best diagnostic tool?
A. ECG B. X ray C. ABG D. D dimer E. CT Pulmonary angiography; CTPA
Computed tomography angiography (CTA) is the initial imaging modality of choice for stable patients with suspected pulmonary embolism. The American
College of Radiology (ACR) considers chest CTA to be the current standard of care for the detection of pulmonary embolism.
127. What is the best diagnostic investigation in PE from the following?
A. ECG B. Ventilation-Perfusion VQ scan(if CTA unavailable or contrast allergy) C. D dimer D. Lower limb dupplex US
128. A patient admitted to hospital, mostly for fracture few days later, he developed retrosternal chest pain, diaphoresis, and confusion,
what is the diagnosis?
A. MI B. PE (due to long immobilisation) C. PUD
129. What is the most frequent presentation of pulmonary embolism?
A. Haemoptysis B. Dyspnoea C. Syncope D. Chest pain
130. A pulmonary embolus is associated with which of the following?
A. Decrease in pulmonary capillary wedge pressure B. Decrease in mean pulmonary artery pressure
C. Decrease in central venous pressure D. Decrease in dead space/tidal volume ratio
131. An elderly patient admitted for treatment of pulmonary embolism, received multiple injections all day and night, patient suffered
from difficulty of sleeping and become agitated, and the staff loss control, what is the cause?
A. Hospital delirium B. Intracranial hge.
132. A young female presented to ER she has been involved in motor vehicle accident; MVA, sustained tibial and femoral fracture, the
patient is stabilised and admitted then developed sudden respiratory distress, what she is suffering from now? Fat embolism
133. An RTA patient with a femur fracture only, C/O dyspnoea, petechiae in upper chest and neck, diagnosis?
A. Fat embolism B. Tension pneumothorax
134. After 5 days post orthopaedic surgery had sudden dyspnoea and confusion on examination shows rash on neck and on CXR bilateral
lower lobe infiltrates, what is the cause or diagnosis?
A. Fat embolism B. PE C. Pneumonia D. Pneumothorax E. Adult respiratory distress syndrome
135. How to diagnose fat embolism in laboratory?
A. Fat in urine B. Fat in Sputum C. Biopsy of petechial hge.
136. A patient presented to the ER with right thigh swelling and hotness, past medical history revealed that the patient had Atrial
Fibrillation, what is the best definitive management?
A. Femoral embolectomy B. Thrombolysis C. Surgical exploration
137. An old, aged patient in ICU for severe pneumonia with atypical organism, he has history of pelvic malignancy, complicated by DVT,
which was well treated 4 years age, no complaints after, recently with examination ICU doctor felt loss of left femoral pulse, what’s best?
A. IVC filter B. IV tPA B. Urgent left femoral embolectomy D. Full heparin only E. IVF and ambulation
138. A diabetic patient, with embolus occluding a segment from common iliac artery down to common femoral artery, with dry gangrene in the
leg up to above knee, how to improve the prognosis and decrease level of amputation?
A. Endovascular stent B. Embolectomy (Acute saddle embolus) C. Axillofemoral bypass D. Aortobifemoral bypass
Paediatric Surgery
1. What is the dose of maintenance fluid resuscitation, in a child after 20 kg of his total body weight? Or 4-2-1
A. 4ml/kg/h B. 2ml/kg/h C. 1mL/kg/h Maintenance 100 ml/kg/ day for the first 0 to 10 kg, 50 ml/kg/ day for next 20kg, then 20ml/kg/d >20
2. How much fluid resuscitation for infant weighing 20 kg as an initial bolus? 20 ml/kg bolus and maintenance is 60-65ml/kg/day
A. 50-60 mL/kg B. 60-65 mL/kg (Maintenance) C. 65-70mL/kg D. 70-80 mL/kg
3. A 2 month aged boy came with fever and history of circumcision, you take urine culture, and it was 80000 E coli and sensitive to Sulfa
you gave him antibiotics for 10 days and discharged after 2 days mother said his fever improved, what is next?
A. Complete course for 10 days and no need for further action B. Do US C. Repeat urine C/S
4. A newborn with persistent vomiting x-ray shows no air in GIT what type of anomaly? Oesophageal atresia with no TEF
5. In trachea-oesophageal fistula, what do you see in CXR?
A. Coiled nasogastric tube B. Stomach bubbles C. Gasless stomach
6. A baby 6 weeks old coming today with bilious vomiting and not gaining weight, and he passed stool before. abdomen examination is
unremarkable. What’s the best tool of benefit?
A. Endoscopy B. Barium meal C. Upper GI Barium series (Malrotated bowel/ duodenal atresia >> go for barium)
D. Lower GIT with contrast E. Abdomen X ray
7. A neonate has bilious vomiting and a double bubble sign on plain x-ray, the most appropriate operation?double bubble sign= duodenal obstuct.
A. Division of annular pancreas B. Gastroenterostomy C. Duodeno-duodenostomy (Annular pancreas) D. Duodenal resection
8. What is the most important element in the history of an infant with vomiting?
A. The frequency of vomiting B. The amount of vomiting C. The presence of fever
D. If vomiting is projectile E. If vomiting is bile stained (duodenal atresia or annular pancreas)
9. Paediatric patient with low grade fever and non-bilious vomiting? Above 2nd part of the duodenum >> GOO
A. Pyloromyotomy(Ramstedt operation as IHPS) B. Surgical reconstruction
10. Regarding the duodenal atresia what is true?
A. is caused by intrauterine mesenteric vascular accident B. Commonly exhibits normal muscular wall with a mucosal web
C. is seldom associated with normal passage of meconium at birth D. is rarely associated with other congenital anomalies
11. A 4 y.o child brought by his mom complaining of constipation since birth, his mom started, high fibre food and stool softeners 1 year
ago. He has one motion every week. No diarrhoea or any symptoms of soiling. Doing PR examination - empty rectum with good anal tone,
on withdrawing the finger there was gush of stool. How would you manage this case?
A. Increase stool softener dose B. Refer to surgery (mostly Hirschsprung disease) C. Refer to oncology D. Increase water & high fibres
12. A paediatric patient suddenly developed SOB, x- ray showed coiled nasogastric tube in the oesophagus, the baby will have a great risk
developing which of the following?
A. GORD B. Chylothorax C. Phrenic nerve injury D. Aspiration
13. The fluid of Chylothorax is composed of which of the following?
A. Pure fat B. Fat and neutrophils C. Fat and lymphocytes D. Fat and macrophages
14. A new born baby with right inguinal reducible hernia, high raising right testis and no left inguinal hernia, what is the management?
A. Observation B. Repair with mesh now C. Wait till 6 years D. Urgent referral to a paediatric surgeon if < 3months
15. A newborn with antenatal history of polyhydramnios cannot feed, drooling and bubbling of his nose when he feeds, imaging shows
nasogastric tube curled/coiled at oesophageal pouch, what is the common complication of the repair procedure?
A. Phrenic nerve injury B. GORD (tracheo oesophageal fistula type C, 84%) C. Chylothorax D. Malignancy
16. Underweight infant presented with dripping of saliva with abdominal distention and cyanosis, dyspnoea (Ryle tube fill with air) X-ray
shows Ryle in chest, what is the initial management? Infants with EA type A and B will not have stomach gas EA types C-E >>> gastric gases
A. Continues cleaning and suction of oesophagus (type, B, D or E) B. Repair of oesophageal fistula C. Intubation
D. Gastrostomy to decompress distended stomach
17. Tracheo Oesophageal fistula, distressed, NGT arrested in chest, what is the optimal? Continuous suction
18. What is the most common congenital cardiac defect?
A. Atrial septal defect B. Ventricular septal defect C. Transposition of great vessels D. Aortic coarticulation
19. What is the most common cardiac anomaly found in adults?
A. Atrial septal defect B. Ventricular septal defect C. Transposition of great vessels D. Coarctation of the aorta
20. Which of the following is cyanotic heart disease?
A. Coarctation of aorta B. Tetralogy of Fallot C. Ventricular septal defect D. Pulmonary stenosis
21. A 10 days of age infant with shortness of breath, absent femoral pulse what is the diagnosis and management?
A. Surgical repair B. Coarctation of the aorta (Prostaglandins E1 infusion; 0.05 to 0.1 mcg/kg/minute to reopen the ductus arteriosus)
22. What is the most common posterior mediastinal mass in children?
A. Neuroblastoma B. Teratoma C. Lymphoma (13% of chilhood cancers; 60% non Hodgkin, 40% Hodgkin) D. Pheochromocytoma
23. A 4 week old child with clear, non bilious vomiting, what’s the best investigation? US to R/O CHPS
24. A 3 week old infant with frequent non-bilious vomiting, what is most likely diagnosis?
A. Duodenal atresia B. Pyloric hypertrophy C. Annular pancreas D. Malrotation
25. An infant with continuous clear vomiting; not bile stained what the cause, tool to diagnose and metabolic disturbances will be found?
A. CT enterography B. Small bowel series (when US is not diagnostic) C. Abdominal USS (target sign) D. Barium enema
26. A 2-months infant bilious vomiting after each meal, he is in 50 centiles, he passed meconium early and stool, what is his diagnosis?
A. Midgut volvulus B. Meconium ileus (not passing early meconium and stool)
C. Hirschsprung disease (not passing early meconium and stool)
27. A baby 3-7 days presented with non-bilious vomiting, decrease oral intake, this happened after introducing milk formula, he passed meconium after
birth and after that yellow stool with mucous and blood with distended tense abdomen, fever and leucopenia what’s the diagnosis?
A. Hirschsprung B. Milk allergy C. Midgut volvulus D. Necrotising enterocolitis
28. A child case with an image of intra operative showing intussusception, what is maximum pressure of pneumatic reduction?
A. 80 mmHg B. 80-100 mmHg (KSA protocol) C. Less than120 mmHg.(uptodate & many sources) D. 200 mmHg
29. A paediatric case presented with abdominal pain, jelly like stool, right quadrant mass? Intussusception, what’s the best diagnostic test?
A. Abdominal x-ray B. Abdominal CT C. Abdominal U/S (doughnut sign/target) D. Barium enema
30. A neonate boy, 16 days old with abdominal distension, bilious vomiting, abdominal tenderness, tachycardic, WBCS = 8900 diagnosis?
A. Enterocolitis B. Volvulus(sudden onset bilious vomiting, distension, tenderness & haemodynamic deterioration) C. Intussusception
31. A premature infant (30-week gestation) presents with distended and tense abdomen, passing blood and mucus per rectum, signs of
sepsis, what is the explanation of that picture?
A. Viral gastroenteritis B. Necrotising enterocolitis C. Meconium ileus D. Intestinal volvulus E. Ileal atresia
32. Pneumatosis intestinalis is pathognomonic, for which of the following?
A. Hirschsprung's disease B. Acute necrotising enterocolitis (NEC) C. Pseudomembranous enterocolitis D. Neonatal UC
33. Which of the following is a clear indication of surgery for necrotising enterocolitis?
A. Generalised gas distension of the intestine B. Bleeding per rectum C. Pneumatosis intestinalis
D. Gas in the portal vein E. Pneumoperitoneum (indicating perforation and peritonitis)
34. What is the indication for laparotomy in neonatal necrotising enterocolitis?
A. Distended bowel loops B. Thickened bowel wall C. Peritonitis & perforation(radiological & clinical) D. Pneumatosis intestinalis
35. An image of an 8 yr. old boy with intussusception, what is the most common cause of this?
A. Payers patches enlargement B. Benign polyp C. Cancer D. Congenital band
36. What is true for paediatric Ileocolic intussusception?
A. It presents with rectal bleeding in 90% of cases B. It is commonly caused by an underlying pathology
C. It is ideally treated by operative reduction D. It is the most common cause of intestinal obstruction before the age of 3 years
37. A child with anaphylactoid purpura develops an acute colicky abdominal pain and bleeding per rectum, what is the diagnosis?
A. Perforated duodenal ulcer B. Bleeding Meckel’s diverticulum C. Enterocolitis with perforation D. Intussusception
38. What is the best description for paediatric intussusception?
A. Non bilious vomiting +50% current jelly early symptom (late and rare) B. Gastroenteritis and history of respiratory infection
C. Commonly due to Henoch-Schönlein purpura D. Child has continuous severe abdominal pain E. Mostly surgically treatable
39. What is the best investigation to diagnosis CHPS?
A. Endoscopy B. US will show target sign (due to hypertrophied hypoechoic muscle surrounding hyperechoic mucosa)
40. Two weeks age neonate, presented by jaundice, US done and showed, biliary atresia and contracted gallbladder, invisible extrahepatic
bile ducts, what is the best time of management or time limit for surgery to prevent irreversible damage and after that time will be poor
prognosis and surgery will be poor results?
A. 8 weeks B. 12 weeks C. 14 wks. D. 18 wks.
41. A 5 month old child, his mother noticed that; his right testicle is in the inguinal canal, not in scrotum, but the left one in his scrotum
since birth when to decide surgical intervention?
A. Surgery at diagnosis time B. Better from 6-12 months (maximally before 2 yrs) C. Before 6 years old D. After 2-3 years
42. A young male baby came to well-baby clinic, upon examination his right testis was palpable in the inguinal canal and small size and
easily moved to scrotum, the left is normal, what is the cause?
A. Ectopic testis B. Undescended testis C. Testicular torsion D. Retractile testis (hyperreflex cremastric m. if easy mobile no intervention)
43. A child about one and half years age brought by his mother to the paediatric clinic surgeon as she noticed a perineal swelling, she
noticed since birth he has an right scrotum testis only, some one told her don’t worry everything is normal, on examination you found
really an empty left scrotum what to do next (initial)?
A. Perineal US (initial) B. Abdominal MRI (best imaging) C. Laparoscopy (the most sure) D. Excision and histopathology
44. A 2 y.o boy brought by his mother, O/E; testes not found in scrotum or inguinal region, the best investigation to confirm diagnosis is?
A. CT B. MRI C. Laparoscopy N.B. if bilateral, do karyotyping for gender if male, do laparoscopy
45. What is the most appropriate option for assessment of a 2 yrs. child with bilateral undescended testes? Laparoscopy
46. One and half yrs. old boy with unilateral undescended testis just at the deep inguinal ring what the rational of surgical intervention?
A. To the risk of malignancy (after age of 2) B. Fear of trauma (if in pubic region) C. To decrease risk of infertility
47. What is the most common congenital anomaly associated to gastroschisis? Intestinal atresia.
Down’s syndrome is associated with annular pancreas, duodenal atresia, imperforate anus and Meckel's diverticulum
48. Regarding hypertrophic pyloric stenosis, what is correct?
A. More common in males B. Most commonly presents in the first week of life C. Vomit is typically bile stained
C. Diagnosis should be confirmed with upper gastrointestinal contrast study
49. A neonate with round or olive like epigastric mass & projective vomiting will treated by?
A. Ramstedt procedure CHPS; congenital hypertrophic pyloric stenosis B. Kasi C. Ladd D. Sistrunk
Normally US pyloric ms <3mm, length <14 mm, pyloric width < 7mm. Incision along stricture intact mucosa diverting ms, suture the muscle edges
50. What the most common anomaly which is associated with Gastroschisis?
A. Intestinal atresia B. Malrotation (duodenal stenosis or atresia, oesophageal atresia) C. Annular pancreas D. Meckel's diverticulum
51. What is correct for Gastroschisis?
A. It is usually associated with other anomalies B. It is usually associated with chromosomal disorders
C. It is located on the left of the umbilical cord D. Repair is followed by prolonged ileus
52. A 22-year-old medical student is seen by the student health service prior to beginning school, routine labs are drawn, and the medical
student immunised against hepatitis B in childhood will have which hepatitis profile?
A. Hbs Ab +ve, Hbs Ag +ve, Hbc Ab +ve B. Hbs Ab +ve, Hbs Ag +ve, Hbc Ab -ve
C. Hbs Ab -ve, Hbs Ag -ve, Hbc Ab -ve D. Hbs Ab +ve, Hbs Ag -ve, Hbc Ab -ve
53. A newborn with congenital hernia, parents asked medical advice for their infant in a clinic, what’s ttt?
A. Expeditious elective surgery (Urgent) B. Elective surgery at age of 3 mo C. Wait until body weight is 10 kg D. After 2 yrs.
54. An infant with soft reducible right inguinal swelling reaching down to base of scrotum what is the name of this swelling?
A. Patent process vaginalis B. Direct hernia C. Indirect inguinal hernia
55. A 10-month boy with inguinal swelling down to scrotum increases in size with crying, what is the type of that swelling?
A. Direct inguinal B. infantile indirect inguinal hernia C. Encysted hydrocele
56. A 2 yrs. old baby with bilateral undescended testis, most diagnostic test?
A. Laparoscopy (if normal karyotyping) B. Karyotyping (initially)
57. A loud and continuous crying infant, with firm hot tender bluish inguinal mass, normal groin what is possible diagnosis?
A. Appendigeal torsion (blue-dot sign) B. Irreducible inguinal hernia C. Testicular tortion (lost cremast. reflex, high riding, transverse, red tender)
58. A 5-year-old boy with one testicle which undescended, what is your management?
A. Orchidectomy B. Orchidopexy C. Wait till puberty
59. A 6-year-old boy has a right undescended testicle, what his parents should be informed and asked to consent for that?
A. Right orchiectomy should be performed B. A course of endocrine treatment is advisable
C. Orchiopexy is performed to improve spermatogenesis and prevent cancer
D. Orchidopexy is performed but with no effect on spermatogenesis or cancer prevention
E. The right testicle is probably atrophic and can be left undisturbed
60. A 12 yrs., with severe testicular pain that increased by elevation of scrotum, clockwise twist, doppler diminished blood supply, what is
the management?
A. Bilateral orchidopexy B. Unilateral orchidopexy C. Orchidectomy D. Analgesia
61. An infant with ventral hernia bulges out when he cries, and reduced when he is sleeping. what is most the optimal management?
A. Observation (till 4-5 years not more unless complicated) B. Herniotomy C. Mesh repair D. Laparoscopic
62. A 5 year old child has asymptomatic umbilical hernia, what is the most appropriate action to do?
A. Reassurance B. Cauterisation C. Surgical correction (after age of 5 very rare to close spontaneously)
SKIN AND SOFT TISSUE
1. A lesion of the big toe, contain HPV by some sort of biopsy, what diagnosis of that biopsy? Verruca vulgaris (common wart)
2. What is the mechanism of tissue expansion while doing tissue expanders to cover a bald area?
A. Mitosis B. Connective tissue disruption N.B Mitotic activity in the epidermis is increased during expansion
3. Picture of dark coloured nail resulted after trauma by heavy blunt object, what is the diagnosis?
A. Subungual haematoma B. Malignant melanoma subungual type
4. The finger like projection of connective tissue core that lined with epithelium, what is that called?
A. Fibroma B. Papilloma C. Desmoid tumour D. Ganglion
5. A painless swelling about 2cm away from outer aspect of orbit, or lateral to outer eye canthus smooth not transilluminating, management?
A. Aspiration B. Incision and drainage C. Excision (dermoid cyst)
6. A back swelling 2x2 cm., with applying little pressure upon it a foul smell material, with punctum on top, what is the diagnosis?
A. Lipoma B. Sebaceous cyst C. Abscess
7. A 31 y.o female with bilateral axillary swellings, with some puncta and painful, red with 5x3 cc pus evacuated, C/S bacteria, where
showed Porphyromonas and Prevotella with staph aureus species, what is the treatment plan?
A. Extended Abs B. Excision primary closure. C. Excision with flap (after wound management) hidradenitis suppurativa
D. Healing by 2ry intention (after initial I&D)
8. What is the minimal safety margin for skin SCC excision? if < 2cm ->> 4mm safety margin. If >2cm ->>1cm
A. 2mm B. 3 mm C. 4 mm (low risk features 4-5, high >6 in high risk area as eye, lip, nose, 1cm in moderate risk area) D. 5mm E. 1cm
9. What is the origin of SCC? Keratinising cell layer of the epidermis or its appendages. It arises from the stratum basalis of the epidermis
10. What is the most suitable investigation for cutaneous Bowen disease?
A. Tumour cells in Bowen disease typically stain positive for p53. B. HPV, and high molecular weight cytokeratin.
C. Negative cytokeratin 10 immunohistochemical staining in nests of atypical cells found in seborrheic keratosis.
D. Hyperkeratosis, parakeratosis, and acanthosis E. Pagetoid cells F. All of the above.
11. What is the best diagnostic tool for Bowen’s disease?
A. High Resolution Microscopy; HRM B. MRI C. CT
12. A patient has chemical burn all back area what to do in ER?
A. Sweep chemical powder B. Wash by clean water for 30 min (if liquid)
13. What is the chemical mediator in a spilled hot water burn?
A. Serotonin B. Bradykinins C. Thromboxane D. Prostaglandins E. Leukotrienes
14. What is the most common organ affected from burn?
A. Skin B. Muscles C. Internal organs D. Brain tissue
15. Which of the following is complicated with hyponatraemia after its topical application?
A. Silver nitrate B. Povidone iodine C. Sulfamylon D. Bacitracin
16. Metabolic acidosis is a complication of topical application of which of the following?
A. Sodium mafenide; severe burn topical ttt (sulphonamide) B. Silver nitrate C. Silver sulfadiazine D. Betadine/ Bacitracin
17. What is the most common source of infection in burn patients?
A. Burn wound B. Urinary tract infection C. Pneumonia D. Thrombophlebitis E. Endocarditis
18. What is the most common fatal infection in burn victims?
A. Pneumonia B. Venous line related sepsis C. Burn wounds sepsis D. Urinary tract infection
19. Which of the following is a stage of third degree burn?
A. Stasis (middle zone with tissue hypoperfusion goal not to convert to coagulation) B. Hyperaemia (1st. degree) C. Coagulation (center of damage)
20. A patient of 17 years with picture of burn of whole Rt. upper limb, what is percentage?
A. 9% B. 18% C. 4.5% head front & back 9%, trunk 18% front 18% back, each arm 9%, each LL 18%, perineum 1%. infant head 18, each LL 13.5
21. How capacitive coupling, cause of thermal injury?
A. Electric current transfer to organ touched by the metal torcher B. Sandwiching C. Wrong energy D. Failure of insulation
22. A patient caught an electrical wire, what decrease zone of stasis?
A. Peripheral VD B. Control temperature
23. All of the following are true except?
A. Burn of low temperature but long contact produces mild injury B. Epithelium is intact with erythema
C. Every burned patient must receive tetanus vaccination D. Any burn affecting face should be referred to a burn unit
24. What is the early complication of burn?
A. Dyspigmentation B. Sepsis (infection) C. Hypertrophic scar D. Contractures
25. What is correct choice regarding deep partial thickness dermal burn? Superficial partial thickness, painful, red weep blanch on pressure
A. Heal with granulation tissue & fibrosis B. Painful to pressure only, full thickness painless B. Have intact hair follicle (Superficial 1st. degree)
D. Blanch on pressure (Superficial partial thickness)
26. Which of the following is most reliably can exclude burn-associated smoke inhalation?
A. Absence of carbonaceous sputum B. Normal flexible bronchoscopy C. Normal carboxyhaemoglobin level D. Normal xenon-133 scan
A. The chest x-ray on admission is normal C. The arterial oxygen saturation is over 90% D. The FEVI/FVC is normal
27. In burn victims, what is the most indicative finding of inhalation injury?
A. Singed nasal hair B. Soot around the mouth C. Dyspnoea D. Carbonaceous sputum (longterm brochiectasis, short ARDS, pneumonia)
28. A young male patient was rescued from a burning building, when he arrived at the hospital, he was fine apart from mild drowsiness and had
singed facial and nasal hair. What’s the most appropriate thing to do?
A. Elective intubation (if just perinasal, burnt nasal hair perioral burn) B. ICU admissions and Observation C. Discharge
29. An image of burnt patient with 3rd. degree burn with burn of face, chest and upper arm he is in need for intubation, how to intubate him?
A. Orotracheal intubation (oral, nasal is intact) B. Thoracostomy C. Surgical Cricothyroidotomy D. Needle cricothyroidotomy
30. A patient developed circumferential right forearm burn. O2 saturation in right digit is 86%, what is the appropriate action?
A. Bedside escharotomy B. Measure intra-compartmental pressure
31. A patient with massive trunkal burn, what is the IV route for emergent resuscitation?
A. Unburned peripheral limb (is the most efficient if unavailable >> CVP, intraosseous) B. Venous cut down C. Central venous line
32. How much fluid resuscitation required for a 70-kg male with 50% body surface area second degree burn? 4mL x weight x TBSA=14000/24h
A. 7,000 mL of lactated Ringer's solution over the first 6 hours B. 7, 000 mL of lactated Ringer's solution over the first 8 hours (1/2 of total 1st. 8h)
C. 8,000 mL of lactated Ringer's solution over the first 8 hours D. 10,000 mL of lactated Ringer's solution over the first 8 hours
33. A patient in blast injury have pneumothorax, burn 40%, femur fracture and shrapnel injury, what is the primary impact?
A. Femur fracture (Tertiary) B. Burn (,smoke; Quaternary) C. Shrapnel (Secondary) D. Tension Pneumothorax (Primary) GIT, ear
34. A 24 yrs. Guy exposed to an explosive injury, with abdominal injury and internal haemorrhage, what is the injury orientation?
A. Burn (smoke, HTN, angina, hyperglycaemia, athma, COPD, all Quaternary) B. Shrapnel (2ry) C. Primary (drum, pleura, bowel)
35. What is the degree of high voltage electrical burn on a boy 15 years old?
A. 1st degree B. 2nd. degree C. 3rd. degree (epidermis, dermis and deepest structures) D. 4th.
36. What is true regarding the Electrical burn injury?
A. Can be estimated by the extent of skin damage B. Typically affects the trunk more than the extremities
C. Requires close cardiac monitoring D. Is inversely related to tissue resistance
37. A case of major burn admitted to the burn ICU unit, he developed massive rectal bleeding, what’s initial or next after fluid resuscitation?
A. NGT B. Laparoscopy C. Upper endoscopy (if NGT aspirate is bloody) D. Colonoscopy E. Gastroscopy
38. A patient was referred from the burn unit to the ER for massive lower GI bleeding; there was no blood in the NGT?
A. Upper GIT endoscopy (if negative >> colonoscopy >> if -ve >>> CTA) B. Angiography (if negative OGD & colonoscopy)
39. A flame burns in closed space (suspected inhalation injury) manipulation? Endotracheal intubation
40. An adult male from firemen team while extracting some victims from fire scene he got face, neck and upper chest burns, brought to
ER in bad condition, with airway tube, where he has inhalation injury, airway obstruction, Spo2 is less than 90% otherwise he is stable
what is the route of securing his airway?
A. Nasotracheal intubation B. Orotracheal intubation (inhalat. burn, airway obstr, O2<90%, GCS<8, C arrest) C. Surgical cricothyroidotomy
D. Tracheostomy
41. What is the first fluid resuscitation for burn? Ringer’s Lactate
42. How to assess the tissue perfusion in a burn of 40%? Urine output
43. How much total body surface area percent in patient with burns of the entire back, scalp, 50% of head, neck and posterior thighs?
A. 40% B. 28% C. 20% D. 32% (18+4.5+9)
44. According Rule of 9 for burn, how mush percentage of left upper limb + partial of chest?
A. 4.5 B. 9 C. 13.5 D. 18
45. A patient 17 years with pic burn all Rt. upper limb, what percentage
A. 9% B. 18% C. 4.5%
46. A patient with 25 % 2nd. and 3rd. degree burn, his condition improved and burn wounds healed, and patient is for discharge home,
what is your advice to him?
A. Tell him burned area may be erythematous and itchy B. Apply pressure garment all day and take it off at night
C. If skin fissuring occurs, do sterile dressing, and report your doctor D. Apply skin emollient
47. Picture of electrical burn. Ask for stasis zone where?
A. Upper chest B. Arm C. Forearm (the msot common contact: hands, forarm and skull, most ground exit: heel & foot)
48. An image of electric burn with forearm, arm, up to axilla and big area of the abdomen and one black point at the lower abdomen,
what is the exit point of that burn?
A. Other hand B. Abdomen C. Foot (travel from hand to body to exit from feet as ground) D. Back
49. A 29 y.o man was involved in a fire after some flame in his sleeproom, where transported to the hospital with major burns of the limbs,
chest hack with face blisters and black colour, he is agitated with hypotension and tachycardia, what is the optimal for him?
A. Antibiotics and painkillers B. H2 antagonists C. Oxygen with intubation if needed D. Just IVF and dressing
50. A 35 year old female patient with picture of ulcer at posterior lateral aspect of the knee, history of burn at same site 20 years back with
scar, no LNS, what’s the proper management?
A. Wide local excision B. Wide local excision with SLN biopsy C. Wide local excision with LN dissection
D. Above knee amputation
51. Isosulfan blue injection for sentinel node biopsy may result in which of the following?
A. Cardiac arrhythmia B. Inaccurate pulse oximetry C. Malignant hyperthermia D. Hypercapnia E. Skin sloughing
52. A 57 y.o male patient with history of thermal burn 20 years ago at posterolateral side of the knee with chronic non-healed indurated
ulcer with rolled elevated margins, no detected lymphadenopathy what to do for him?
A. Wide local excision with flap reconstruction B. Excision with sentinel LN (if US showed LNs with query of SCC after Marjolin’s ulcer)
C. Excision with prophylactic inguinal lymphadenectomy (if positive sentinel LN indicating SCC )
53. A male patient 55 years, worker, has a small nodule lateral to the nose, with pigmentation of the nodule and lateral ulceration, and no
lymph node detected, what is the origin of this type of cells?
A. Keratinising cell B. Melanocytes C. Non-keratinising cell D. Pluripotent cell (in the basal layer of the epidermis or the follicle; BCC)
54. What is the pathogenesis or pathophysiology of BCC?
A. Exposure to Ultraviolet sun light (causes DNA damage and immunosuppression) B. Genetics
55. An old age male patient with lesion lateral to eye, it shows pigmentation and lateral ulceration, what is the diagnosis?
A. SCC B. BCC C. Melanoma D. Marjolin’s ulcer
56. A 65 year old female with face skin lesion, beside the nose; cheek for excision basal cell carcinoma, safety margin?
A. 0.5 mm B. 1 mm C. 2 mm D. 10 mm. Safety 5mm, small facial 2-3 mm 2mm if low risk, 3mm if high risk feature is better 99% cure
57. An old aged female patient with chest skin lesion (like BCC), skin biopsy margins are?
A. 5 mm B.1mm C. 2 mm D. 10 mm for face 3 mm is better and least is 2mm and for others than face >> 5 mm
58. An ulcer in cheek, no LN palpable what is the most likely diagnosis?
A. Melanoma B. BCC C. SCC
59. What is the most fretting or worrying complication after BCC cauterisation therapy?
A. Infection B. Recurrent due to margin loss (will be R1 or R2) C. Cosmetic discharge. recurrence moh’s 0.1%, excision 10.1% caut 7.7, RTH 8.7
60. An image of 3rd. degree burn at posterolateral aspect of knee with ulcer and contracture, what is the method of coverage?
A. 1ry. intension B. 2ry C. Skin graft D. Flap reconstruction
61. A 27 year old female was exposed to flame burn with skin loss, she underwent long time treatment then her surgeon decided to
transplant a full thickness skin graft what this kind of graft depends upon for good take state and survival?
A. Arterial B. Capillaries (Plasmatic imbibition) C. Venous D. Type if full or partial thickness
62. A big post burn Marjolin’s ulcer on the lateral aspect of the knee, what indicates bad prognosis? Inguinal LNs involvement
63. A patient in a blast injury, what is the best IVF for resuscitation?
A. Normal saline B. Ringer lactate C. 3% Saline
64. What is the optimal management of face BCC on the nose, Mohs micrographic surgery; MMC; 2mm safety margin in face or 3mm if high risk
65. What is the best management plan for case of BCC?
A. Conventional excision B. Moh’s surgery C. Radiotherapy
66. An image for skin lesion and history of change in colour and size, what is the optimal for tissue diagnosis?
A. Incision biopsy B. Excision biopsy
67. A patient with melanoma, depth > 2 mm, how much free margin of this lesion? In situ=0.5-1cm, <1mm=1cm, 1.1-2 mm=1-2cm, if >2->4= 2cm
A. < 0.5 cm B. 1-2 cm (for >2mm to >4mm =2cm only) C. 2-4 cm D. > 4 cm
68. Melanoma lower back of male 70 years, and depth (more than 4 mm), what is the minimum safety margin:
A. 20 mm (2cm) B. 30 mm C. 40 mm D. 50 mm
69. Melanoma in lateral side of leg, biopsy done that showed it as Clark II level, what is optimal management?
A. Wide local resection with safety margin B. Resection with inguinal LN dissection
70. An old aged male has right arm melanoma, with positive SLN, which excised with lymphadenectomy, which’s the level of LN dissection?
A. Level I axillary LN B. LNs posterior to pectoralis minor C. LNs medial to pectoralis minor D. Palpable LNs
71. What is the most important prognostic variable for melanoma?
A. Gender B. Age C. Clark's level D. Breslow’s thickness
72. How much percentage of risk regional node metastasis in 0.70 mm thick melanoma?
A. Less than 5% B. 10% C. 20% D. 50%
In a study found even when lesions were ≥ 0.76mm, mitogenic, of level IV-V, patients > 65 years old had a 0% rate of SLN positivity
73. Melanoma 4mm, free margin excision? 20 mm (2mm to > 4mm = 2cm safety margin)
74. Melanoma 2 mm thickness safety margin?
A. 0.5 cm B. 1cm C. 1.5 cm D. 2 cm
N.B. For Melanoma (1) if in situ = 0.5-1 cm. (2) if < 1 mm =1 cm. (3) if 1 -2 mm = 1-2 cm. (4) if more than 2 mm to >4mm = 2 cm only
75. A picture of a lesion in the forearm and there are axillary lymph nodes, free margin is detected by?
A. Depth of the lesion (melanoma) B. Positive lymph nodes C. Size of the lesion
76. What determines the excision margin? Depth of dermal invasion
77. An image showing arm melanoma, what indicates wider radical excision?
A. Positive LNs B. More thickness of the lesion C. Wider lesion D. Age
78. Kidney transplant recipients are at increased risk for which of the following?
A. Non-melanoma skin cancer B. Lung cancer C. Colon cancer D. Breast cancer E. Uterine cancer
79. Excessive sweeting in the hands and soles, what is the first line of treatment?
A. Botulinum toxin injection B. Sympathectomy
80. What is the best diagnostic for soft tissue Sarcoma?
A. Incisional B. Excisional C. Core needle D. Aspiration
MEDICAL ETHICS, PROFESSIONALISM AND PATIENT SAFETY
1. A 60-year-old diabetic was admitted to the hospital unconscious, in a coma due to diabetic ketoacidosis and a gangrenous foot.
The doctors decided to amputate his foot as soon as stabilised enough to withstand anaesthesia. The patient’s sons, daughters and most of
his family refused the operation, even after explanations that the gangrene would spread and result in fatal septicaemia. They reasoned
that it was better for him to die and be buried with all parts of his body than to live with an amputated limb. What should the doctor do?
A. Respect their wish and decision B. Court of law C. Report to the authorities D. Meet ethical committee
E. Do amputation because its life saving F. Ask another surgeon for amputation then proceed
2. An old patient was prepared for non-urgent surgery, he refused it, but his family want to do it, what to do? Respond to the patient wishes
3. A patient threatened you for complaining if you don’t do his surgery, what is your action toward that patient?
A. Consult physician B. Call social workers C. Do it D. Ethical committee E. Referring him to other hospital or surgeon
4. An elderly with hernia came for your evaluation based on a physician recommended repair, he is not complaining of any symptoms, you
saw no need, but he insisted on doing the surgery in threatened you if you don’t do it, he will write a complaint to the hospital management,
what you will do?
A. Perform the surgery B. Refer the patient C. Call social worker D. Call hospital administration.
5. Woman brought by the police with bruises, how to deal with her condition? Call the social service of the hospital(they can solve the issue)
6. A female patient, came with multiple injury, on examination you detected that she was hit by her husband what is the optimal action?
A. Inform ethical committee B. Inform the authorities C. Call violence and write a report
7. A pregnant lady, now she is in labour. On examination, foetal distress was confirmed, she was counseled for CS, but she refused bluntly,
what appropriate action should be taken?
A. Allow vaginal delivery (if choice to refer to other dr is correct) B. Her husband decision C. Ethical committee D. don’t care do CS
8. A doctor wants to examine female patient and he called his male nurse to attend with him, which is most related to ethics?
A. Non-maleficence B. Privacy (not allowed to examine her while male nurse can see her) C. Maleficence
9. A patient underwent haemorrhoidectomy after 2 days he came to SOPD for follow up, he was with his son, doctor asked the nurse to
nurse the patient, which one of the following exceeds the privacy of the patient?
A. Examination in the presence of nurse (she can stand behind the curtain) B. Examine without his son C. Examine without white sheet
10. Warfarin is not stopped by the patient regardless of clear instructions by the nurse, in OR they noticed that the patients didn't stop it
as instructed and informed the surgeon, So the surgery postponed what’s that called?
A. Near miss B. Adverse event C. Medical negligence
11. During finishing of an operation, the nurse told the surgeon that, there is a missed pack, and we should do x-ray before closure, the
surgeon refused and said he is a 20- years experience, no need to waste time and money, after 2 weeks patient came back with abdominal
pain and discovered there is missed pack?
A. Surgeon must be blamed for refusing x ray B. Nurse must insists for x ray C. Operation team is responsible for the mistake
12. You attended the surgery theater as internship trainee, in a renal surgery and the surgeon was preparing to do the surgery on the right
side, but you knew from the history that the diseased kidney is the left one, what should you do?
A. Call the chef intern B. Tell the surgeon (if you are sure, it is unethical to hide an advice for patient safety) C. Do nothing
13. A Cardiothoracic consultant will do an operation for a patient with a success rate of 10%, the operation is very useful for the patient if
not done the patient’s condition will deteriorates, surgeon is worried to tell patient as he may refuse to sign the consent what he can do?
A. Do operation without consent as you know his benefit than him B. Do operation with consent from relatives
C. Discuss with the patient his situation and the importance of surgery/ tell the patient and let him to take decision and sign consent
D. Don’t tell the patient all information so that he may sign the consent E. Ask another colleague to do the surgery
14. A similar scenario but the surgeon wants to do surgery for the patient that can be complicated by amputation but if the surgery was
not done, the disease also will progress more, and amputation will be a must, what is the optimal act?
A. Tell the patient and let him take the decision C. Don’t tell the patient so that he may sign consent
B. Take consent from a relative D. Do operation without consent because you know what is good for him
15. A cardiac surgeon, his patient needs cardiac surgery, the patient doesn’t know the procedure and complications, what should be done?
A. Start to work without patient knowledge B. Abandoning the operation
C. You should inform the patient of all the details of the process
16. A patient planned to do surgery, he agreed for risky operation, during preoperative discussion with treating doctors he refused to be
informed about all details and asked the consent paper and signed it before doctors completed his explanation, what is the action?
A. Involve the family B. Involve the ethical committee C. Leave this dilemma to the attending surgeon
D. Insist to inform patient with all the details to proceed for surgery (if refused or shout involve ethical committee)
17. A patient undergo surgery for an abdominal neoplasm, he discusses it with his surgeon, and surgeon described the procedure to him in
detail, what is still necessary for the patient to be able to give informed consent to this procedure?
A. A medical screen including ECG and coagulation panel B. An explanation of alternatives to the surgery
C. Consultation with a family member of the patient coagulation panel D. A psychiatric assessment
18. A child came to ER without his parents with a bleeding wound in his face, he is stable how to deal with him?
A. Suture without parent consent B. Suture with relative come with him C. Apply a gauze till his parents come
19. A child presented to the emergency with wound in his head, his brother and nursemaid were with him, but father and mother were in
their way starting to come to hospital, what is the optimal action to take?
A. Start to stitch the wound B. Cover the wound and wait for the mother or father (it is not lifesaving)
20. A recently married woman wants to take oral contraceptive pills; OCP, and her husband is refusing to take OCP, what to do?
A. Empathy B. Follow wife she has right to choose C. Follow both wife and husband D. Follow husband’s wish
21. A medical student came to you in the clinic wants you to give him sick leave to his final exam tomorrow because he was with his sick
mother in hospital last night and couldn’t study for the test, what is your action?
A. Refuse and tell him he was supposed to study earlier B. Refer him to another colleague who will accept this
C. Give him a sick leave and save your future colleague
D. Call the hospital and make sure he’s saying the truth and his mother is sleeping there
22. A middle aged woman underwent a pelviabdominal surgery, she got fever with some pus came out from the wound, after that she
visited another surgeon for checking up for that issue, who did abdominal x-ray and US and found metallic object mostly forgotten during
operation, what is the best act to do?
A. Call and inform the first surgeon B. Call attorney and asked about legal action C. Tell her what you found
D. Tell her that is one of the possible complications of that operation
23. While doing CPR on a patient, the nurse noticed that the doctor is going to give wrong medication, and she told him, but he ignored?
A. Negligence B. Authority gradient C. Call hospital manager D. Just write incident report
24. What stages of change for smoking cessation?
Precontemplation: current not planning on quitting smoking in the next 6 months
Contemplation: current smokers who are planning to quit in next 6 months but not have made quit attempt in the past year
Preparation: planning to quit within next 30 days and have made a quit attempt in the past year
Action: individuals who are not currently smoking stopped smoking within the past 6 months
Maintenance: not currently smokers and have stopped smoking for longer than 6 months but < 5 years
25. Which one is not correct in the four principles of Beauchamp & Childress?
A. Beneficence B. Non-maleficence C. Respect for persons D. Justice
The four principles of Beauchamp and Childress - Autonomy, beneficence, non-maleficence, and justice
26. A comatosed infant, he needs amputation not in bleeding or severe sepsis, from whom you take consent?
A. His parents B. Waiting to take consent from the patient C. Do operation without consent
27. A child came after car accident with isolated head injury with subdural haematoma, needs urgent exploration and all his family with
him but his father, is coming on the way to hospital, what is the proper action?
A. Wait for his father arrival and take consent from him B. Take consent from the most senior family member (if refused do without consent)
C. Inform the father by phone and take verbal consent D. Go without consent as it is lifesaving procedure
28. A young girl was involved in MVC, came to ER with a compromised vascularity to the lower limb, you should take her to surgery
which among its complications is amputation, her father is not present in the hospital, but her grandfather is there, what should you do?
A. Take her immediately to surgery B. Taking her father’s consent on the phone
C. Taking her grandfather’s consent (he can replace father)
29. A 15 year old child, brought to the clinic by his parents with fever and neck stiffeness, after was seen by neurologist who ordered a
lumbar puncture, some discussion happened to take consent from who, what is the correct regarding who should give the consent?
A. Take consent from parents only B. Take patient consent and parents assent (< 15 y = parent’s consent, 15-18= both, > 18= only patient)
C. No need for consent D. Only patient consent without parents assent
30. A newly diagnosed diabetic patient, arrived at time of his appointment to the clinic, the doctor was busy in some emergency cases in
ER, so he came late, the patient was furious and highly angry which of the following is the most appropriate next step?
A. Explore the cause of anger B. Explain the cause he was treating an emergency cases in ER C. Ignore the cause of his anger
D. Shout with him am not your servant E. Dismis this as just anger due to newly diabetic
31. While you are in your clinic, a liberal lady, wearing very revealing cloths and talking in a naughty seductive way with inspiration and
loughing loudly, and unprofessional behaviour, looks like non straight one which of the following is appropriate in this situation?
A. Report her to hospital administrator B. Shout to her C. Report to authorities
D. Ask a female nurse to talk and deal with her (dont examine her solitudely) E. Take her number for friend relation
32. During your clinic work, a foreigner lady came to you asking for some health services, she is speaking in unknown language to you,
you dont understand what she needs and also, she is not understanding your language what to do in that situation?
A. Exile her out from clinic B. Call hospital authority C. Refer to doctor of same nationality (can do if no nurse and if was same tongue)
D. Ask a nurse or any worker from same nationality & tongue for help
33. Who is responsible for solving ethical problems in the hospital? Ethical committee
34. A patient with diabetic foot, orthopedic surgeons and general surgeons are arguing where to admit the patient, how to prevent this
problem? Instructions from the medical director >>> follow hospital policy
35. You found a certain drug’s lot with specific patch number causes a specific side effect in your patient, whom to tell? Hospital
pharmaceutical administrator if not responeded inform hospital adminstrator
36. You discovered a problem in an energy device; same problem was also in other hospitals also to whom you should report?
A. Ministry of health B. Manufacturing company C. Maintenence administrator if no response then hospital manager
37. How to train the junior resident?
A. Prepare curriculum B. Round meetings C. Video coferences
38. A couple came asking about surrogate pregnancy, you don’t know much about this topic, what should you do? Give a follow up
appointment and study the case further
39. As a doctor, if you see a patient and you faced difficulty to get an accurate and real information from him, but he is not going directly
with you and you are unable to know the truth, what is the best tactic to get information from him?
A. Threaten him to tell police B. Ask him a direct question C. Ask an open question D. Offer a gift if told truth
40. What is the most powerful way to communicate?
A. Verbal B. Written C. Paraphrase D. Body language
41. A primigravida patient, was trying to conceive for 10 years. She finally conceived and reach 10 weeks but later, a complete abortion
has developed. This trauma emotionally affected her and her husband a lot. The doctor tried to bring the couple together by addressing
their issues and emotions. What communication skills did the doctor practice here? Empathetic and sympathetic response.
42. A couple came to clinic complaining of trying to conceive for 3 years, after completing the fertility investigations the wife was found to
have blocked tubes. Regarding the patient’s privacy which of the following is the best approach?
A. Inform husband only B. Inform wife only C. inform both husband and wife D. Consult ethics committee before informing
43. A 30 y.o woman and her husband present to clinic at 9 weeks of gestation with mild spotting after intercourse, she has history of
abortion, the couples are blaming themselves and showed strong emotion, what of the following is the best action?
A. Identify their emotion and calm them down B. Explore and validate their emotions with an empathetic response
C. Empathetic response is enough to calm them D. Understand their emotions without showing them
44. An intellectually disabled pregnant woman, with IQ 80, presents to the clinic with her mother, the mother wants the patient to have
abortion, but the patient wants to continue her pregnancy, which of the following is appropriate?
A. Refuse to perform abortion B. Do abortion as patient is not competent C. Obtain court order to do D. Call child protective survice
45. A policeman came with a guy to ER, you are the physician asking you for alcohol screening, but the guy refused, what do you do?
A. Respect confidence of the patient B. Respond to policeman request C. Call a supervision D. None of the above
46. A patient came with his son; he doesn’t want the surgery, but his son insisted to do, what is the best action?
A. Don’t do without patient consent C. If his son is mature can consent C. Consult a colleague D. Refer to ethical committee
47. A patient gave you a gift and you refused, next day patient attempted suicide as she felt rejected, what you should do in this situation?
A. Accept the gift and give it to patient charity B. Write paper on OPD door that gifts are unacceptable
C. Explain for the patient that it’s unethical to accept gift then refuse D. Accept the gift for yourself
48. A urologist, before surgery for renal transplantation knew that the donor received money from the patient, what is optimal action?
A. Cancel the procedure it must be free B. Proceed and inform authority C. proceed and keep the secret
D. Ask patients to hide this information from other hospital staff
49. A businessman came to hospital, and you consult him then he appreciates you and need to make friend relationship with you what you
will do from ethical point for you?
A. Do relationship with him B. Refuse any relationship or escape any relationship that harm your professional attitude
C. As no patient harm it’s OK D. Keep the number and call him later E. Avoid meeting at hospital building.
50. An 85 year old widow diabetic, smoker man, he has dry gangrenous left leg due to diabetes related vascular insult the surgeon advised
him and explained all issues and asked for a consent to amputate his leg, but he refused, what is the best action toward his decision?
A. Repeat informed consent in presence of family B. Request psychiatric for competence evaluation C. Ask him to explain his decision
D. Seek a court order to amputate E. Let him to sign DAMA and prescribe antibiotics and morphin
51. A 25 y.o female patient you are resuscitating her with CPR, you want to examine her genitalia, what to do regarding consent?
A. Shout you want to do exam B. Examine without consent or senior agreement C. Wait for family D. Call a female doctor
52. An ER doctor finished his duty, his colleague has not arrived, will come after 15 min there is a patient in the ER what to do?
A. Assess and stabilise the patient till his colleague comes B. Leave the patient and ER as your shift is finished
C. Call his colleague to come within 10 minutes. D. Endorse the case the nearest doctor or head nurse staff
53. Which one of the following is a medical error?
A. Mistaken diagnosis or prescription B. Negligence C. Management plan not completed as intended D. All of the above
54. An anaesthetist put two different medications of same shape and colour on drug preparation shelf, what we call this if caused an error?
A. Medication error B. Latent error C. Anaesthesia error
55. An old lady has end stage cancer, with good intellectual power, her son asked not to tell his mom about her condition, what to do?
A. Tell her any way B. Ask the son why not to tell her C. Don’t tell her
56. An old female with dementia, diagnosed with metastatic breast cancer, her son asked you not to tell her?
A. Tell her B. Do not tell her (she is dementia not aware, no benefit to tell) C. Ask him why he wants you not to tell her
57. A 63-year-old lady who had colonic cancer 12 years ago, was well treated in some cancer centres, today she came with core biopsy
result with breast cancer, what is the reason that make you not talking about her present diagnosis?
A. Her family asked don’t tell her B. Her request does not tell and keep secret C. For psychological problem
58. 61-year-old lady who had breast cancer 13 years ago, she was well treated and recovered now presented with colonic cancer, what is
the cause prevent you from telling her about the current new disease?
A. If telling her will affect her physical fitness B. If telling will affect her psychological condition C. If her family requesting not tell
PATIENT SAFETY
1. While you were in SOPD, one patient came for oesophagectomy and asked you to do but you are, not competent to do it what to do?
A. Refer the patient to another competent surgeon
B. Take consent from them and describe the complication and proceed you want to improve your skills
2. A female patient wants or needs a weight reduction or bariatric surgery, but she is diagnosed as having a mental illness, she is hesitated
and incompetent to give consent what should be the management?
A. Psychiatric consultation B. Husband signs the consent C. Do surgery with a colleague
D. Take another surgeon’s opinion
3. A female patient wants to have a cosmetic procedure in her face, but she is diagnosed with a psychiatric illness, what should you do?
A. Husband consent B. Psychiatric consultation
4. A case of Bipolar depression on medication, she needs metabolic surgery to control his DM, from who you can take the consent?
A. The patient himself B. Her husband C. Doctor and physiatrist D. Two family members
5. You received in ER, husband and wife, the wife was complaining of mild epigastric pain, then during bed examination; you discovered
scattered bruises and abrasions in both arms and back, you asked about violence, and both denied, and they asked you: don’t tell anybody
about these marks, and wife refused to call police or inform violence authorities, what is the most appropriate action?
A. Obey and respect her wish B. Tell the violence authority C. Call the police D. Educate her about violence
6. A middle-aged lady have a baby with bruises over his body, she looks angry toward him and shouting with threating words what to do?
A. Report to ethical responsible B. Report to your supervisor C. Mother have right with her baby
7. A young boy brought by his mother to examine him, you found some bruises and scratches over his hands and face how to behave?
A. Ask him in front of his mother B. Inform the authority C. Ask the child away privately D. Write an incidental report
8. A Girl came to ER with her mother, the father out of the town, diagnosis was appendicitis how to proceed?
A. Prepare for surgery B. Take verbal consent from the mother C. Wait till the father come back
D. Explain the procedure to the girl and take the mother signature
9. An 11 y.o child who was clinically as suspected appendicitis you planned for surgery; his mother refused to do what you should do?
A. Tell her he will die if not done B. Do appendectomy and don’t care C. Refer him to other hospital
D. Refer after mother signs DAMA
10. A 13 y old child, his parents brought him to the ER, he needs urgent intervention, but his parents refused to sign the consent?
A. Take consent from other family member B. Do not do the intervention
C. Do the emergency maneuvre or intervention if lifesaving
11. A child with respiratory distress, critical case and need urgent procedure, parents refused the procedure, what to do?
A. Respect their wishes B. You will do the procedure as it is lifesaving C. Try to convince the parents
12. An unconscious patient came with his son to ER with post traumatic head injury, the emergency surgeon suggested an operation for
him but with some complication risk, but son refused to proceed to the surgery what to do in that case?
A. Take consent from patient B. Do emergency procedure C. Try to convince the son D. Don’t do operation
13. A 12-y. o girl presents with severe right lower-quadrant abdominal pain and marked tenderness and is found to have perforated acute
appendicitis, with peritonitis she is toxic, the child is at a sleep away camp. You are not able to locate her parents. They are not at home,
and you cannot reach them on the cell phone. The camp counselor and the director of the camp bring in the child. what should you do?
A. Do not do the surgery without parents’ consent (if suspicious, mild, not toxic not life threatening) B. Consent from camp counselor or director
C. Ask the patient for consent D. Perform the appendectomy (lifesaving)
14. A case of pregnant lady undergoing CS, prior to the operation she told the nurse she doesn’t like to get blood transfusion even if bled
massively, later when they deliverd, got placental retension with profuse bleeding till faint and shocked, what you will do next?
A. Call her husband for his permission B. Ignore her request and urgently transfuse blood (it is urgent, and transfusion is life saving )
C. Call hospital ethics committee D . Follow her request and don’t transfuse blood
15. A scenario of surgeon booked a case from OPD and marked the surgery side in the ward was the left foot that indicated for amputation,
but the nurse by mistake sterilised, scrubbed and prepared the right one, the surgeon discovered that she did prepare it wrongly and
remembered the correct side from previous marking, and he did not amputate the wrong one but did for the correct one, what is this called?
A. Near miss (problem not occurred) B. Never event (already occurred, avoidable problem as FB, wrong side, wrong patient, wrong procedure)
C. Medication error
16. Two patients of same name, were admitted to the ward for below knee amputation due to ischaemic gangrenous limbs, one was for left
side amputation and the other was for right side, that one for Right side signed DAMA or abscond. At morning the patient for left side
shifted to operative room as both of same name the ward nurse wrongly endorsed him as that one for right side so the OR sister scrubbed
and sterilised right side the surgeon came and amputated that wrong side after surgery, he discovered the problem what to do?
A. Write case report to director B. Write incident report C. Fight and harm ward sister D. Complain against OR nurse
17. Wrong side of surgery, patient throw himself from his room (suicide attempt), what is the cause?
A. Close call B. Never event C. Absence of risk management protocol
18. A case scenario, surgeon did something wrong, his assistant told him surgeon changed the action and no harm happened to patient
good medical and surgical practice include the following? OVR means Occurrence Variance Report
A. The event must be recorded in operative notes (OVR) B. The assistant tells the patient only C. Nothing to be done
19. A 32 y. old highly experienced surgeon, during appendectomy, nurse told him there is missed gauze, he refused to check according to
his experience and told no need to do x ray, no toil lost, whose to be wrong?
A. Surgeon B. Nurse C. Hold countable
20. In the intra operative room, nurse give wrongly scissor instead of artery forceps to let doctor to cut a vessel?
A. Commission (negligence, neglected LMWH, antibiotice preop) B. Omission (doing wrong intentioally or carelessly, as complication, mismanag.)
21. A surgeon after doing wrong side nephrectomy, he injured intestine and peritoneum but back up repair intestine, patient left hospital
without complication after one-week patient knows what happened, so he can
A. Sue the surgeon B. Sue the hospital C. Demand money (compensation) D. Post on social media
22. What is the name of the event in previous question; wrong side nephrectomy with intestinal injury?
A. Sentinel event B. Near miss
23. A lady for an elective laparoscopic cholecystectomy and resident talk to her and let her to be a part of the decision what is that?
A. Patient safety B. Hospital care C. Safety committee
25. A medical mistake has occurred, and you want to teach the students, what is the appropriate method for learning?
Make a meeting about mortality and morbidity B. Write in social media about that doctor C. Go to court and complain

26. The surgical intervention is indicated in which medical case among the following patients?
A. If condition is not responding or patient refused medical treatment
B. Responders C. Unconscious D. Transient responders
27. A child came alone with crushed limb with severe bleeding and should be amputated what to do?
A. Amputate (lifesaving) B. Wait parents
28. A child post RTA, has huge haematoma on his thigh and profuse bleeding, needs urgent intervention, his father on the way to the
hospital, what to do?
A. Wait for his father to sign the consent
B. Proceed with operation without consent if lifesaving (but if limb saving stop bleeding if you can and wait his father)
29. A patient for right cochlear implant shifted to operative room instead of another with bilateral cochlear implants & bilateral cochlear
implants were inserted, how to prevent this error? WHO safety check list (time out)
30. A patient was booked from SOPD, for left lower limb amputation, but due to a fault from the admission nurse, he was recorded for
right lower limb amputation, during surgery after the right lower limb was almost amputated wrongly, and already his left leg is indicated
for amputation, the error was discovered and bilateral amputation was done, how to prevent this error? WHO safety check list (time out)
31. Which of the following prevent confusion to the side and site of surgery?
A. Time out by operative team B. Operative technician does side marking C. Senior resident does side marking
32. A surgeon marked a limb to amputate it, before surgery, wrong side draped, surgeon discovered the problem and corrected it how to
prevent its occurrence?
A. Marked with permanent pen B. Take patient information before surgery C. Time out before skin incision
33. There is gangrene on Lt. foot and patient in OR nurse prepare Rt one, surgeon came and amputate Rt, instead of Lt. how to avoid
amputation of wrong sided foot? Marking, (and following safety checklist as sign in and time out)
34. Your colleague in the hospital has HBV positive, he wish to ignore that and no action to do what will you do from patient safety view?
A. Ask him to tell his patients B. Write confidential report C. Don’t refer patient to him
35. A female patient 10 days post failed vaginal delivery with episiotomy that completed with CS, then she complained of dysuria where
her obstetrition diagnosed her as UTI but no improvement on prescribed antibiotics, she went for another doctor where he found a missed
infected and smily vaginal swab, what is the cause of this problem?
A. The first doctor lets midwife to assist him B. The first doctor failed to follow surgical protocol in OR
C. No communication between first and second doctors D. Genuine difference assessment or diagnosis of the patient’s condition
36. Doctor is doing intra-articular injection for patients with osteoarthritis in his clinic without optimum sterilisation, as a registrar who
wants to document the incidents what he must do?
A. Write an incident report B. Forget the issue, no harm upon you C. Try to advice him that is against rules
37. A nurse with accidental needle prick injury, while dealing with a suspicious patient, you asked to report, she refused to report the
infection control, what will you do?
A. Report the hospital B. Leave here go home C. Write incident report if C not mentioned in choices >> Choose Report the hospital
38. Parents refused to vaccinate their child, what will you do?
A. Social service B. Ethics department C. Vaccinate him D. Tell them advantage of the vaccine
39. A scenario of list of two cases for cochlear implant, one will do left side cochlear implant and the other will do bilateral, the turn now
for the case of bilateral, but the patient was cancelled for some reasons and no body informed the surgeon and they pushed the one for
unilateral left side and the surgeon opened on the right side, then they discovered this mistake intraoperatively, where is the defect?
A. Booking defect (and defect in sign in and time out for patient identity, side, consent, procedure) B. Marking defect
40. A 22-week newborn with severe congenital anomalies, you want to intubate him, and his mother is your colleague and she refused to
intubate, what will you do?
A. Respect her wishes B. Intubate C. Refer the patient to another doctor
41. A 24-week preterm baby delivered, the mother is a doctor and she asked not to resuscitate her baby, what you will do?
A. Ignore the mother and resuscitate (>24 w resuscitate, 22-24 convince < 22 DNR) B. Tell social services C. Respect her wishes
42. A scenario of a child with subdural Hge, need to enter the OR, what will you do? If lifesaving proceed if they refused to give consent
A. Take consent from father by phone B. Take consent from the nearest relative C. Take another advise from your colleague
43. A neighbour of a 14-year-old boy brought him to the emergency room after sustaining slight scalp laceration due to head trauma, you
evaluated him and determined that scalp sutures are necessary, he is stable and not bleeding, what to do?
A. He is mature can give consent B. The neighbour can sign consent C. Wait consent from one parent
D. Both parents must sign consent E. It is a minor case no need for consent
44. A 19 ys male, RTA, unconscious, trauma to one thigh with profuse continuous bleeding distal pulse is not palpable, planned for
amputation, father on his way to the hospital, his relatives are many around you?
A. Wait the father B. Amputate without consent it is life saving (19y is mature) C. Choose the most senior from his relative
45. While doing appendectomy in 12 yrs. female you discovered simple ovarian cyst and you can do it efficiently without increase in
operation time, and for prevention of later surgery for her what to do?
A. Leave the cyst as you didn't have consent B. Go and get a written consent from her father
C. Ask another colleague’s opinion D. Do the proper interference without consent to prevent another laparotomy
E. Do the best for the patient (by your paternity) F. Remove mass and preserve ovary
G. Take a biopsy and sent to histopathologist N.B all these choices collected from many scenarios
46. During open appendectomy of acute appendicitis, you found it inflamed, and you took it out, then you found (simple ovarian cyst), you
want to excise it to use the benefits of an open procedure, the ovary was normal, no signs of malignancies, what will you do?
A. Excise it B. Obtain a quick consent from the relatives outside OR C. Call your gynae colleague (if gyn & obst not available leave it)
47. During lap exploration of presumed appendicitis of middle-aged female, CT was not done before the surgery, you found ugly ovarian
mass, and the appendix looked free, what will you do?
A. Take the appendix out, and terminate the surgery and write detailed operative notes
B. Appendectomy and oophorectomy C. Leave the appendix, and terminate the surgery, and record the finding
D. Take the appendix out, and biopsy from the mass E. Make intra-operative gynecological consultation
F. Leave the appendix, and terminate the surgery, and record the finding
N.B if CT done pre-op and appendix was free so must consult Gynaecologist and ask him to be in OR with him.
48. At an operation for appendicitis, the appendix is found to be normal, and the fallopian tube is found to be thickened with surrounding
purulent exudate. What the operative management should be?
A. Appendectomy only B. Appendectomy and salpingectomy C. Salpingectomy D. No operative intervention
49. A business group wants to open a health care facility and want clearance and license to finish, what’s type of service to be provided there?
A. Predict ulcer and manage B. Can prevent fall from bed C. Do surgery under GA
D. Can prevent wrong patient, wrong medication and wrong procedure
50. A junior resident surgeon, first time to do appendectomy his senior consultant assisted him, but consultant was angry because he was
late, so he did operation by himself after closure of fascia, senior resident came to OR and said in loud voice: this is wrong patient what
about this situation? the priority for patient safety (mistake of not signing in and time out to confirm patient identity that was not done)
51. You were in operative room, assisting your senior in appendectomy during that, ER team called your senior, he went and left you
alone in OR assissting in appendectomy you find tumour over appendix or Caecal mass, what will you do?
A. Call senior and wait B. Make only appendectomy and close abdomen
C. Take rapid concent from family and proceed D. Tell the relatives it is unresectable
D. Struggle and try to remove E. Remove it as you watch the procedure in YouTube
52. A post-operative patient, while in airport, the warning device used for body surveillance beeps, what to do in operative room to avoid
that? sign out
WHO safety checklist; Sign in= Before induction of anaesthesia, Time out = Before skin incision, Sign out= Before patient leaves OR out
53. A patient with forgotten sponge or gauze in his abdomen after abdominal closure after surgery finished, how to avoid that error?
A. Time out & Sponge counting B. Time in C. Sign out
54. From WHO safety checklist point of view, how to identify patient or how to define patient identity?
A. Sign in (first time by sign in revision by time out) B. Sign out
WHO safety checklist includes 3 stages with 19 points must be fulfilled the stages start from patient shifting from surgical ward till receiving him again
either in his ward or ICU without fulfilling all these the WHO safety checklist is not complete
1- Sign in before induction of anaesthesia 2- Time out before skin incision 3- Sign out before patient leaving operative room
55. Regarding the WHO surgical safety checklist, at which stage the patient identity or identification revision will be done? Means for 2nd time
A. Sign in (the initial) B. Sign out C. Time out (identity revision before incision) D. Time in
56. In Surgery safety checklist what is correct?
A. Before entering OR before anaesthesia B. Before entering OR before anaesthesia and post op care
C. After induction of Anaesthesia and post-operative care
57. The World Health Organisation (WHO) published the WHO Surgical Safety Checklist in 2008 to increase the safety of patients
undergoing surgery. The Checklist consists of three phases of surgical procedure?
A. Before admission, before skin incision, at discharge from hospital. B. Before admission, before induction of anaesthesia, at hospital discharge
C. Before induction of anaesthesia, before skin incision, before patient leaves operating room.
D. Before admission, before induction of anaesthesia, before patient leaves operating room.
58. The surgeon, nurse and anaesthesia professional review aloud the key concerns for the recovery and care of the patient, what is used to
prevent errors in hospitals?
A. Efficiency B. Equivalent C. Patient safety
59. What is the priorities and basics of patient medical services?
A. Patient’s safety (whatever his kind, nationality or ethnic) B. Patient centered care
60. Regarding error prevention in hospitals, what is meant by sex equality?
A. Efficacy B. Equivalancy C. Patient safety (discrimination is illegal for medical service, advice and management)
61. A resident surgeon wants to do a procedure in the clinic that he never did it before, what is the optimal act to perform?
A. Till the patient he cannot do it B. Do it C. Wait the consultant
62. A consultant transects CBD for the third time, what to do: Report in the file
63. A scenario of surgeon ligate CBD 3 ligatures during lap chole, mistaken as cystic duct what to do?
Write an operative note what happened
64. Some medical mistake occurred during your patient’s care, what to do, and whom to inform?
A. Tell patient and the senior B. Tell patient alone C. Tell senior alone
D. Avoid telling anyone and avoid repeating that mistake.
65. A scenario of a case, doctor prescribed Clindamycin for patient, later culture shows resistant to the antibiotic, doctor was not informed
about the result he continued same drug and patient died from sepsis, how to minimise this problem?
A. Drug code B. Reco C. Record result have to be seen on system (and to be informed soon it is ready)
66. What is the cause of this problem? Poor system communication
The nurses must inform and update the doctor’s information that related to the patient benefits and safety during all his stay in the hospital
67. A 27 y.o, mechanic male, C/O fever, SOB, he is worried about his job when they know that he is a COVID case, he wants his manger
not to know about him that you will do?
A. Report as suspected COVID case & wait 24 hr B. Do not say to his manager C. Report as case positive
D. Report he is not COVID
68. A newly diagnosed HIV patient worries about telling his wife and ask not to tell her, what will you do?
A. Tell his wife’s mother and ask her to tell her daughter C. Protect the patient confidentiality
D. Reassure him and ask your assistant to tell his wife B. Tell his wife
69. A patient 84 yrs. old, his wife 29 yrs. old, he came to ER with vomiting, nausea, abdominal pain, distended and tympanic abdomen,
rebound tenderness, temp. 38.5, RR 20, HR 101, H/O 2 MI & CVA 5 years ago, after CVA he has trouble in speaking, he doesn’t have power
of autonomy, patient needs operation, his wife refused, his son demands that everything possible must be done to save his father’s life, what
level of care is appropriate?
A. Attempt intervention by local district judge court B. Initiation of hospice care
C. Medical and supportive care only D. Urgent surgical exploration
70. A patient with seizure taking drug to decrease attention, he is a bus driver, he asked dont tell, otherwise will suspend him your action?
A. Respect confidentiality and don’t tell anyone B. Tell him not to drive C. Tell his supervisor (his life and passengers saving)
71. An ophthalmologist in a peripheral district, no other alternative in same specialty, after some ocular surgery he had a seizure and fits.
What to do from medicolegal point of view toward him
A. Suspend his license forever B. Suspend it for one-year
C. Suspend for 1 year till be treated then regain D. Let him work as no alternative for him
72. A study conducted to evaluate medication among depressed patients, they were not taking their medications regularly?
A. Unacceptable study because it affects drug efficacy B. Acceptable because patients signed the consent voluntarily
C. Unacceptable because we can’t take consent from depressed patients (mental instability)
73. A patient involved in RTA, he has a fractured leg, needs foot amputation, after you explained the risk and benefits of procedure, he
agreed because he hears voices telling him to do it, what do you do?
A. Find another decision maker because patient is incompetent (mentally and psychologically, it limb not lifesaving)
B. Involve another surgeon to confirm C. Proceed to the lifesaving surgery (since it’s an emergency intervention)
74. A child presented with crushed limb with severe bleeding and need amputation, his parents away from hospital, what you will do?
A. Amputate (as it is lifesaving can die from bleeding and crush syndrome) B. Wait the parents
75. During some surgical operation, a mobile rang, some people told it is not allowed in theater to have mobile even if silent, but the
surgeon insisted it is not forbidden even if not silent, what is the correct choice from medicolegal aspect of view?
A. Mobile is forbidden in OR B. Mobile is allowed but if silent C. Should be kept >1 m away low tone and far from sterile field
76. A 24 y.o patient post-accident, he was comatosed and kept on ventilator, in private hospital, his family cannot afford that hospital as it
is highly costy for them and they are unable to pay, what to do?
A. Refer the patient B. Give full care (un ethical illegal to remove vent. because of unable topay) C. Stop management if dosen’t pay
77. A child in ICU, family couldn’t afford hospital stay because of the cost, what you will do? keep him will full health services
78. A patient in hospital and on ICU then relative can’t pay for hospital what will u dofrom ethical point of veiw?
A. Discharge patient B. Referred to other hospital C. Keep patient on ventilator without any compromise
79. During lap. Chole. You injured the CBD and told the patient post-operatively, what does this mean?
A. Disclosing an apparent complication B. Disclosing complication C. Disclosing error D. Disclosing an apparent error
80. An elderly frail patient with advanced, inoperable, and metastatic colorectal cancer with severe bone pain that require all time
morphia even sometimes not relieving his pain, he asked you to let him to die to get rid off that suffer, what is decision?
A. Better to do euthanasia to help him B. The patient has right to consent for DNR C. The doctor has right to choose drugs to die
Medical Statistics
1. The government has decided to set up campaigns to help increase the awareness of hypertension among the public, as well as educate
them about the risk factors of hypertension and encourage low salt diets. What kind of prevention is this?
A. Primordial (health education campaign) B. Primary (risk reduction) C. Secondary (disease manag.) D. Tertiary(reduce severity)
2. Camping in community to educate people about health determents is considered as?
A. Health education (primordial prevention) B. Active surveillance
3. The MOH is organising camping, workshops, for disease prevention and risk reduction in the public about the dangers of obesity and
its associated complications and to decrease risk of hypertension. What kind of prevention is this?
A. Tertiary B. Secondary C. Primary D. Primordial
4. Camping in community to decrease risk of hypertension is considered as what?
A. Primary B. Secondary C. Tertiary
5. Anaesthetist try many sizes of mask on one patient. Standardisation
6. A new screening test for Diabetes has a sensitivity of 90% and specificity of 80% which is the best interpretation?
A. 80% of non-diabetic has positive results B. 80% of diabetes had positive results
C. The test was positive for 90% of diabetic patients D. For each 100 positive test results 90% were diabetic
7. The sensitivity of the test is defined as?
A. Proportion of patients with the disease who have a positive test B. Proportion of patients without the disease who have a negative test
C. Proportion of patients with a positive test who have the disease D. Proportion of patients with a negative test who do not have the disease
8. The specificity of a test is defined as what?
A. Proportion of patients with the disease who have a positive test B. Proportion of patients with a positive test who have the disease
C. Proportion of patients without the disease who have a negative test D. Proportion of patients with a negative test who do not have disease
9. When you order the test for cancer screening you will choose?
A. Highly specific B. Highly sensitive C. Quick to perform D. Cheap
Highly sensitive tests will lead to positive findings for patients with a disease, whereas highly specific tests will show patients without a finding having no disease
10. What does the positive predictive value of a test mean?
A. The proportion of patients with the disease who have a positive test B. Proportion of persons without the disease who have a negative test
C. The proportion of persons with a positive test who have the disease D. Proportion of persons with a negative test who do not have disease
11. The negative predictive value of a test is defined as?
A. the proportion of patients with the disease who have a positive test B. The proportion of patients with a positive test who have the disease
C. the proportion of patients without the disease who have a negative test D. Proportion of patients without the disease who have a positive test
E. the proportions of patients with negative test who do not have the disease
12. Which test is appropriate to study 5 different types of exercise given to obese ladies over a period and follow there BMI?
A. Cohort B. Cross sectional C. Case control D. Clinical trial
13. What is the highest level of evidence to determine exercise effect on quality-of-life?
A. Cohort B. Case control C. Clinical trial D. Observational study
14. A diagram of surgical resident and non- surgical resident followed for 4 years outcome of depression study, what is the type of study:
A. Cohort (can be retrospective or prospective) B. Case series C. Control study D. Navy tape test
15. Study on effect of dairy products on group with colonic cancer and group with no colonic cancer, what is the type of study:
A. Cohort study B. Control study C. Section study D. Correlation
16. A study conducted to 30/500 babies who were exposed to Di-ethyl-stilbestrol in utero were followed up for 20 years and the occurrence
of cancer in this group was compared with that of 1500 babies not exposed to DES, over the same period, what is the study design?
A. Cohort B. Case control C. Cross sectional D. Correlation study
17. A study conducts to see the effect and relationship of earphone Bluetooth and brain cancer, what is the best study to describe that?
A. Cross sectional B. Cohort (Retrospective study) C. Ecological D. Case control E. Correlation
If study conducted to see effect of risk factor started with exposure and searched for an outcome or disease it will be Cohort study If started with a disease
(case) and asked about what is relation of risk factors or exposure to it > Case control
18. A senior resident ortho noticed that rate of infection is high, on reviewing the OPD files he noticed that the consultant doesn’t take the
proper anti-septic measures, what’s the type of this study?
A. Prospective B. Retrospective
19. Diabetes mellitus among health care workers is 30 out of 100 doctors were diabetic as compared to 50 out of 200 nurses, remember
(doctors-nurses) (diabetic-non-diabetic), which is the most appropriate statistical test?
A. T-test B. ANOVA C. Chi-square D. Correlation
20. What is the mean of cholesterol in Arabic area, American and European people?
A. Chi-square B. Student t-test C. ANOVA D. Correlation
21. What is the parameters to compare by Chi-square?
A. Age and BP B. Smoking and BP C. Gender and smoking D. Heart-disease and smoking

You might also like